Lab diagnosis

अब Quizwiz के साथ अपने होमवर्क और परीक्षाओं को एस करें!

A 45-year-old woman is evaluated in the emergency department for progressive shortness of breath and fatigue for the past 6 weeks. She also has a 5-year history of diffuse cutaneous systemic sclerosis. Medications are nifedipine, lisinopril, omeprazole, and aspirin. On physical examination, the patient is alert but short of breath. Temperature is 37.2 °C (99.0 °F), blood pressure is 126/92 mm Hg, pulse rate is 124/min, and respiration rate is 26/min. BMI is 25. Oxygen saturation is 98% on 2 L of oxygen. Cardiac examination is normal. Velcro-like crackles are heard throughout the chest. Diffuse skin thickening of the face, anterior chest, arms stopping at the elbows, and legs is present; sclerodactyly of the fingers is also noted. There is no rash. Pedal edema is present. Chest radiograph shows bilateral reticulonodular infiltrates and ground glass opacities with normal cardiac silhouette. High-resolution CT scan is consistent with active nonspecific interstitial pneumonitis. An open lung biopsy confirms the diagnosis of nonspecific interstitial pneumonitis. Which of the following is the most appropriate treatment? A. Cyclophosphamide B. D-penicillamine C. Infliximab D. Methotrexate

A. Cyclophosphamide Educational Objective: Treat a patient who has interstitial lung disease associated with diffuse cutaneous systemic sclerosis. Cyclophosphamide has been shown to have some benefit in patients who have interstitial lung disease associated with diffuse cutaneous systemic sclerosis. Cyclophosphamide is appropriate for this patient who has interstitial lung disease (ILD) associated with diffuse cutaneous systemic sclerosis (DcSSc). This patient with DcSSc presents with dyspnea, decreased exercise tolerance, and characteristic Velcro-like crackles. These clinical findings are strongly suggestive of ILD and are supported by the imaging studies and confirmed by open lung biopsy. Patients with systemic sclerosis who have active inflammatory lung disease may be treated with immunosuppressive agents. Cyclophosphamide is the only treatment shown to have some benefit in patients with ILD associated with DcSSc. Cyclophosphamide given orally or intravenously for 1 year provides modest benefit. Although it has shown limited clinical improvement, it is the only evidence-based therapy and is therefore appropriate. High-dose glucocorticoids are frequently used in these patients but are of unclear benefit and may precipitate scleroderma renal crisis; therefore, if used, low doses are typically recommended by experts. Azathioprine may have a role as maintenance therapy.

A patient with a BMI of 29 and GERD is best treated with: A. Diet modification B. Diet modification and medical weight management program C. Gastric banding D. Gastric bypass E. Nothing ​

A. Diet modification.

A 68-year-old woman presents with an upper GI hemorrhage. She has a history of ulcer disease and has recently completed a treatment for H. pylori. Upper endoscopy reveals brisk arterial bleeding from a duodenal ulcer located on the posterior wall. Despite numerous attempts to control the bleeding endoscopically, the ulcer continues to bleed. The patient has received 4 units of blood. Her hematocrit is 25%, her blood pressure is 110/60 mm Hg, and her heart rate is 120 beats per minute. Which of the following is the best management option? A. Duodenotomy, oversewing the ulcer, truncal vagotomy, and pyloroplasty B. Duodenotomy and oversewing the ulcer C. Truncal vagotomy and antrectomy with Billroth I reconstruction D. Truncal vagotomy and antrectomy with Billroth II reconstruction E. Highly selective vagotomy ​

A. Duodenotomy, oversewing the ulcer, truncal vagotomy, and pyloroplasty Bleeding from duodenal ulcers can be controlled endoscopically in the majority of patients; thus surgery is rarely indicated. Predictors of failure of endoscopic management include the presence of shock or a large ulcer (>2 cm). Even when bleeding recurs after having been controlled endoscopically, endoscopic treatment can again be attempted with a high rate of success, thus avoiding surgery. The bleeding is usually from a posterior ulcer that has eroded into the gastroduodenal artery (remember anterior ulcers cause a free perforation and peritonitis, posterior ulcers penetrate and bleed). Surgical management decisions should be based on the hemodynamic stability of the patient, the patient's overall medical condition, and whether the patient has a history of ulcer disease that has been treated for H. pylori. In the patient who is actively bleeding, the duodenum should be opened across the pylorus as is used in a pyloroplasty. The ulcer bed should be oversewn with multiple figure-of-eight sutures. If the patient has a history of ulcers that have been treated for H. pylori and is stable in the operating room, an ulcer operation should be performed. The best option in this type of emergent setting is to perform a truncal vagotomy and to close the longitudinal duodenotomy in a transverse fashion as with a pyloroplasty. If the patient is a high surgical risk and unstable, another option would be to simply perform a smaller duodenotomy, oversew the ulcer, simply close the duodenotomy, and treat postoperatively for H. pylori (B). Although vagotomy and antrectomy are another option, they would seldom be used in the emergent setting because of the higher associated morbidity rate (D, E). An HSV (C) would not address the actively bleeding ulcer.

A 30-year-old patient presents with an episode of jaundice associated with feeling tired. On examination there is anaemia, splenomegaly and jaundice. He has had several such episodes in the past. The blood smear shows reticulocytosis and the red cells demonstrate increased osmotic fragility. Which one of the following is the most likely diagnosis? A. Hereditary spherocytosis B. Paroxysmal nocturnal haemoglobinuria C. Pyruvate kinase deficiency D. Sickle-celI disease E. Thalassaemia ​

A. Hereditary spherocytosis This is a young patient who has symptoms of anaemia and jaundice. The combination of anaemia and jaundice should always suggest haemolytic anaemia until proved otherwise. The reticulocytosis suggests that this is not a problem with the bone marrow. Anaemia with the presence of increased reticulocytes suggests that blood cells are being lost or destroyed outside the bone marrow. Therefore, anaemia and a raised reticulocyte count usually indicates either bleeding or haemolysis. The fact that we are told that the red cells demonstrate increased osmotic fragility is diagnostic of the condition hereditary spherocytosis. ​

Which one of the following features is MOST suggestive of megaloblastic anaemia? A. Hypersegmented neutrop hils in the peripheral blood film B. Atrophic gastritis C. Pancyt openia D. Low reticulocyte count E. Raised LDH ​

A. Hypersegmented neucrophils in the peripheral blood film This is a very helpful finding and a reliable feature of megaloblastic anaemia. ​

A 66-year-old patient with COPD has repeated exacerbations (at least two episodes each year for the past 3 years) and has an FEV1 45% predicted. She is currently being treated with salmeterol, two puffs twice per day, and albuterol, two puffs every 6 hours as needed. Which one of the following should be added to her treatment regimen? A. Inhaled corticosteroid B. Home oxygen therapy C. Prophylactic antibiotic therapy D. Long-term low-dose oral corticosteroid E. Long-term theophylline therapy ​

A. Inhaled corticosteroid The patient has stage 3 (severe) COPD with frequent exacerbations. Inhaled corticosteroids need to be added.

A 4-month-old infant is anemic with a hemoglobin level of 9.5 g/dL and MCV 77 fL. The baby looks well, the height and weight growth parameters are normal, and she is breast-feeding well. Which of the following is the most likely cause for her anemia? A. inadequate dietary iron B. hemolysis C. late clamping of cord D. iron malabsorption E. folate deficiency

A. inadequate dietary iron Iron deficiency is the most common cause of anemia in infancy. Sixty percent of body iron concentration at birth is contained in circulating Hb. Milk is a poor source of iron, so the most common cause of iron deficiency in infancy is prolonged breast or bottle feeding. Cereals are high in iron content. Hemolysis and folate deficiency as the cause of this baby's anemia would result in a normocytic or macrocytic anemia, and it is unlikely malabsorption is the cause since the baby is gaining weight and doing well clinically. ​

In a maximal expiration, the total volume expired is A. tidal volume (VT) B. vital capacity (VC) C. expiratory reserve volume (ERV) D. residual volume (RV) E. functional residual capacity (FRC) F. inspiratory capacity ​

B. vital capacity (VC) The volume expired in a forced maximal expiration is forced vital capacity, or vital capacity (VC).

The above patient (age 30) undergoes laparoscopic gastric bypass and on POD #5 has significant irritability. Her vital signs are: BP 110/60, HR 130, RR 40. She also complains of vague shoulder discomfort What is the most appropriate next step: A. Abdominal CT B. Upper GI series C. Re-exploration D. Heparin E. WBC scan ​

C. Re-exploration for leak. Tachycardia may be the only finding of a leak. Other Sx's - tachypnea, irritability, left shoulder or flank pain; late oliguria and fever (usually do not have abdominal pain or tenderness). ​

Because of safety concerns, which one of the following asthma medications should be used only as additive therapy and not as monotherapy? A. Inhaled corticosteroids B. Leukotriene-receptor antagonists C. Short-acting β-agonists D. Long-acting β-agonists E. Mast-cell stabilizers ​

D. Long-acting β-agonists Because of the risk of asthma exacerbation or asthma related death, the Food and Drug Administration (FDA) has added a warning against the use of long-acting β-agonists as mono-therapy. Inhaled corticosteroids, leukotriene-receptor antagonists, short-acting β-agonists, and mast-cell stabilizers are approved and accepted for both monotherapy and combination therapy in the management of asthma.

A 50-year-old man with a persistent cough and difficulty breathing is referred by his family physician for pulmonary function tests. The test results show that the forced vital capacity (FVC), forced expired volume in 1s (FEV1), and functional residual capacity (FRC) are all significantly below normal. Which of the following diagnosis is consistent with these pulmonary function test results? A. Asthma B. Chronic bronchitis C. Emphysema D. Pulmonary fibrosis ​

D. Pulmonary fibrosis FVC and FEV1 are expected to be reduced in all these conditions. The coexistence of low FRC indicates reduced resting lung volume and is consistent with the low static lung compliance found in patients with pulmonary fibrosis.

A 30-year-old woman is evaluated for amenorrhea. She and her husband are interested in pregnancy in the next year, and they are concerned that they will not be able to conceive. Her menses became irregular about 2 years ago. In the past 12 months, she has had menses twice. Her last menstrual period was 4 months ago. She also notes low libido and dyspareunia. She has not had weight changes, constipation, hair loss or hirsutism, or skin changes. Her medical history is significant for primary hypothyroidism and bipolar disorder. Medications are levothyroxine, lithium, and risperidone. She reports that she has been stable on these medications for a few years and feels well. She plans to discuss her medications with her psychiatrist prior to pregnancy. On physical examination, blood pressure is 118/72 mm Hg and pulse rate is 82/min. BMI is 24. The thyroid is normal. Visual fields are intact. Laboratory studies: Follicle-stimulating hormone 1.3 mU/mL (1.3 U/L) Luteinizing hormone 2.0 mU/mL (2.0 U/L) Prolactin 102 ng/mL (102 µg/L) Thyroid-stimulating hormone 1.1 µU/mL (1.1 mU/L) Which of the following is the most likely cause of her hyperprolactinemia? A. Hypothyroidism B. Lithium C. Pituitary adenoma D. Risperidone

D. Risperidone Educational Objective: Diagnose hyperprolactinemia caused by an antipsychotic agent. Key Point: Antipsychotic agents block dopamine and decrease inhibition of prolactin release at the pituitary, causing hyperprolactinemia. Hyperprolactinemia is a known side effect of antipsychotic agents, and this patient's hyperprolactinemia is likely due to risperidone. Antipsychotics block dopamine and decrease inhibition of prolactin release at the pituitary, causing hyperprolactinemia. Stopping the medication can reverse the hyperprolactinemia but should only be done under the supervision of a psychiatrist. The patient's risperidone should not be discontinued unless her psychiatrist is consulted. Although the patient has a likely explanation for her hyperprolactinemia, a prolactinoma is still possible. Risperidone can cause prolactin levels above 200 ng/mL (200 µg/L), so her level of 102 ng/mL (102 µg/L) is not unreasonable. Remeasuring the prolactin level following discontinuation of the drug is recommended before further evaluation for a pituitary adenoma; the patient's psychiatrist should be consulted before withholding the risperidone for testing. A patient with hyperprolactinemia without a clear secondary or drug-induced cause should be assessed by an imaging study (preferably, MRI of the pituitary gland) to a exclude pituitary lesion.

The patient above undergoes resection but intra-op U/S and biopsy shows diffuse liver metastases. The next appropriate Tx is: A. Radiation therapy B. 5 fluorouracil and cisplatin C. Imatinib (Gleevec) D. Do nothing ​

C. Imatinib (Gleevec) Surgical excision with clear margins is the best Tx for GIST tumors (75 % benign, 25% malignant). Imatinib (Gleevec) a tyrosine kinase inhibitor, has been extremely effective for malignant GIST tumors.

Which combination of features below is most likely to be relevant to a diagnosis of axSpA? A. Achilles tendon enthesitis, anterior uveitis and pubic symphysitis B. An aunt who has psoriasis, rheumatoid factor and fatigue C. Back pain, joint swelling and stiffness, scleritis, fatigue and positive ACPA D. High ESR, anterior uveitis, ankle swelling, raised serum angiotensin-converting enzyme (ACE) E. Low back pain, rosacea, prostatism and diarrhoea ​

A. Achilles tendon enthesitis, anterior uveitis and pubic symphysitis Scleritis and ACPAs are features of RA. Uveitis, ankle swelling and raised ACE are typical of sarcoid. Fatigue is a feature of all autoinflammatory and autoimmune conditions. A small minority of PsA patients may have a positive rheumatoid factor. Achilles insertional tendonitis (enthesitis) and symphysitis are recognised axSpA lesions. ​

A 72-year-old man takes large quantities of nonsteroidal anti-inflammatory drugs (NSAIDs) because of chronic degenerative arthritis of the hips and knees. Over the past 2 weeks, he has had epigastric pain with nausea and vomiting and an episode of hematemesis. On physical examination, there are no remarkable findings. A gastric biopsy specimen is most likely to show which of the following lesions? A. Acute gastritis B. Adenocarcinoma C. Epithelial dysplasia D. Helicobacter pylori infection E. Hyperplastic polyp ​

A. Acute gastritis Prolonged use of nonsteroidal anti-inflammatory drugs (NSAIDs) is an important cause of acute gastritis. NSAIDs inhibit cyclooxygenase-dependent synthesis of prostaglandins E2 and I2, which stimulate nearly all defense mechanisms. Excessive alcohol consumption and smoking also are possible causes. Acute gastritis tends to be diffuse and, when severe, can lead to significant mucosal hemorrhage that is difficult to control. Epithelial dysplasia may occur at the site of chronic gastritis. It is a forerunner of gastric cancer. Infection with Helicobacter pylori is not associated with acute gastritis. Hyperplastic polyps of the stomach do not result from acute gastritis, but may arise in association with chronic gastritis. Acute gastritis does not increase the risk of gastric adenocarcinoma.

A 35-year-old woman is evaluated in follow-up for worsening asthma. Medical history is significant for environmental and food allergies, allergic rhinitis, and asthma diagnosed at age 10 years. Although her asthma had been previously well controlled, her symptoms have worsened over the past year with increased wheezing and a cough productive of dark-colored mucus. She was admitted to the hospital 2 weeks ago for her asthma symptoms and was diagnosed with pneumonia. She was treated with antibiotics and a tapering course of glucocorticoids. Her respiratory symptoms have recurred following completion of therapy. Medical history is otherwise unremarkable. Medications are fluticasone/salmeterol and as-needed albuterol metered-dose inhalers. She works as a school teacher. On physical examination, temperature is 37.1 °C (98.7 °F), blood pressure is 110/70 mm Hg, pulse rate is 92/min, and respiration rate is 16/min; BMI is 23. Diffuse wheezing is noted on expiration with diminished airflow across the upper right lung field. The remainder of the examination is unremarkable. Laboratory studies show a leukocyte count of 10,500/µL (10.5 x 109/L) with 15% eosinophils. Chest radiograph shows a right upper lobe infiltrate and diffusely increased lung markings. Which of the following is the most likely diagnosis? A. Allergic bronchopulmonary aspergillosis B. Cystic fibrosis C. Eosinophilic granulomatosis with polyangiitis (Churg-Strauss syndrome) D. Hypersensitivity pneumonitis

A. Allergic bronchopulmonary aspergillosis Educational Objective: Diagnose allergic bronchopulmonary aspergillosis. Key Point: Allergic bronchopulmonary aspergillosis is a chronic hypersensitivity reaction that occurs in response to colonization of the lower airways with Aspergillus species; it occurs most commonly in patients with atopic asthma or cystic fibrosis. The most likely diagnosis is allergic bronchopulmonary aspergillosis (ABPA). ABPA is a chronic hypersensitivity reaction that occurs in response to colonization of the lower airways with Aspergillus species. It occurs most commonly in patients with atopic asthma or cystic fibrosis (CF). The resulting inflammation causes difficult-to-control asthma, impaired mucociliary clearance, destruction of pulmonary parenchyma, and bronchiectasis. Symptoms typically include severe and uncontrolled asthma, cough with expectoration of mucus plugs, and systemic symptoms such as low-grade fever and fatigue. Radiologic findings may be normal or may show recurrent infiltrates or evidence of centrilobular bronchiectasis. Patients will have immediate skin test reactivity to Aspergillus antigens; this is often used as an initial diagnostic test. Laboratory studies show peripheral eosinophilia (usually greater than 1000/µL [1 x 109/L]), serum IgE levels greater than 1000 U/mL (1000 kU/L), and precipitating serum antibodies to Aspergillus. Treatment consists of standard asthma therapies and the lowest dose of systemic glucocorticoids that effectively controls symptoms. Antifungal agents may be warranted to reduce fungal colonization, and omalizumab may also have a role in therapy. Cystic fibrosis (CF) is an autosomal recessive disorder causing abnormal chloride transport resulting in thick, viscous secretions in the lungs, pancreas, liver, intestine, and reproductive tract with associated multiorgan dysfunction, with the most prominent disease occurring in the lungs. Although an increasing number of cases of CF are diagnosed in adulthood, this patient has a long history of allergies and asthma, but not chronic respiratory infections or other evidence of multisystem disease suggestive of CF. This lack of consistent symptoms and her older age make CF an unlikely diagnosis in this patient. Eosinophilic granulomatosis with polyangiitis (formerly known as Churg- Strauss syndrome) is an autoimmune small-vessel vasculitis that presents with peripheral eosinophilia and characteristically involves the lungs as asthma. However, it typically has additional manifestations such as purpura on the hands and legs and sensory or motor neuropathy. This patient does not demonstrate these findings. Hypersensitivity pneumonitis is an immunologic reaction within the pulmonary parenchyma to a wide variety of inhaled agents. It may be acute, intermittent, or chronic and typically presents with cough, shortness of breath, and fatigue. However, this patient has no clear exposure history, and hypersensitivity pneumonitis is not usually associated with peripheral eosinophilia or wheezing.

A 50-year-old man is brought to the emergency department due to a severe, sudden-onset headache that started an hour ago. The patient reports that he has had mild headaches and decreased libido over the past 3 months. He has no other medical conditions and takes no medications. Physical examination reveals bilateral deficits involving the temporal visual fields and impaired extraocular eye movements. Shortly after being admitted to the hospital, he becomes acutely hypotensive and loses consciousness. The patient dies despite aggressive resuscitation efforts. Which of the following is most likely to be found on autopsy? A. Acute hemorrhage in the pituitary gland B. Bleeding within the putamen C. Dissection of the internal carotid artery D. Ischemic necrosis of the pituitary gland E. Ruptured intracranial aneurysm

A. Acute hemorrhage in the pituitary gland This patient had typical features of acute pituitary hemorrhage (pituitary apoplexy), including a severe headache bitemporal hemianopsia (due to compression of the optic chiasm), and ophthalmoplegia (due to compression of the oculomotor nerve [CN III]). Pituitary apoplexy usually occurs in a preexisting pituitary adenoma, as in this patient with chronic headaches and low libido (likely due to hyperprolactinemia). The diagnosis can be confirmed on cross-sectional imaging (CT or MRI scan). Patients with pituitary apoplexy can develop cardiovascular collapse due to ACTH deficiency and subsequent adrenocortical insufficiency Pituitary apoplexy is a medical emergency that requires urgent neurosurgical consultation and treatment with glucocorticoids. Educational objective: Acute pituitary hemorrhage (pituitary apoplexy) is characterized by severe headaches, bitemporal hemianopsia (compression of the optic chiasm), and ophthalmoplegia (compression of the oculomotor nerve). It usually occurs in a preexisting pituitary adenoma. Pituitary apoplexy is a medical emergency that requires urgent treatment with glucocorticoids to prevent acute adrenal crisis and circulatory collapse.

A 30-year-old woman is evaluated in follow-up for wheezing and cough that started 6 weeks ago after a viral respiratory illness. At that time she had daily wheezing and cough with nocturnal symptoms that would disturb her sleep. An as-needed β2-agonist was prescribed for symptomatic relief. She reports feeling much better. Her daytime symptoms of wheezing and cough have resolved, but she notes that she awakens from sleep with shortness of breath approximately twice a week. Medical history is notable for mild asthma as a child that was treated for several years; however, she has not required treatment since age 15 years. Her only medication is as-needed inhaled albuterol. She is a never-smoker. On physical examination, vital signs are normal. No wheezing is noted on pulmonary examination. The remainder of the examination is normal. Spirometry demonstrates an FEV1/FVC ratio of 76%, with an improvement of 15% in the FEV1 following treatment with a bronchodilator. Which of the following is the most appropriate management? A. Add a glucocorticoid inhaler C. Continue current therapy and reevaluate in 4 weeks C. Discontinue inhaler medication D. Switch to a long-acting β2-agonist inhaler

A. Add a glucocorticoid inhaler Educational Objective: Treat inadequately controlled asthma by stepping up therapy. Key Point: Viral respiratory infections, frequently due to rhinovirus, may cause airway hyperresponsiveness and obstruction through nonallergic mechanisms in patients without a history of asthma; they may also exacerbate underlying asthma and require step-up therapy. The most appropriate management is to step up therapy for this patient's asthma by adding a low-dose glucocorticoid inhaler. This patient has a history of asthma as a child, which has been quiescent for many years and has not required treatment as an adult. She recently experienced a viral respiratory tract infection with associated wheezing and cough. Viral respiratory infections, most commonly with rhinoviruses, may induce bronchospasm in patients without preexisting reactive airways disease and may require acute treatment for symptom relief. However, viral respiratory infections may also exacerbate underlying asthma and require a step-up in treatment. Although this patient has not required therapy for asthma for many years, she remains persistently symptomatic with evidence of ongoing obstruction beyond the time frame when virally induced bronchospasm in patients without asthma typically resolves, which is usually by 6 to 8 weeks after a respiratory infection. Her symptoms and spirometry results are consistent with mild persistent asthma; therefore, the addition of an inhaled glucocorticoid, consistent with guideline-based treatment recommendations, is indicated to control her asthma. Because this patient remains symptomatic beyond the time frame in which patients without asthma would be expected to have symptom resolution, continuing either an as-needed or long-acting β2-agonist would not address the underlying mechanism of obstruction and would not provide optimal control of her asthma. Therefore, neither of these interventions is indicated. Similarly, discontinuing the inhaler medication would not be appropriate in this patient with asthma who remains symptomatic.

A 28-year-old woman is evaluated because of symptoms of asthma that have worsened since she became pregnant 2 months ago. She has frequent daytime symptoms and increased night time awakening because of the symptoms. She has used her albuterol inhaler several times per week to achieve symptomatic relief. History includes mild persistent asthma that was well controlled before her pregnancy with an as-needed short-acting β2-agonist and medium-dose inhaled glucocorticoids. On physical examination, vital signs are normal. The lungs are clear. Cardiac examination shows normal S1 and S2 with no gallops or murmurs. No leg edema is noted. Spirometry shows forced expiratory volume in 1 second (FEV1) of 85% of predicted and an FEV1/forced vital capacity ratio of 78%. Laboratory studies show a hemoglobin level of 11.5 g/dL (115 g/L). Which of the following is the most appropriate next step in management? A. Add a long-acting β2-agonist B. Add theophylline C. Double the dose of inhaled glucocorticoid D. Obtain a bronchial challenge test

A. Add a long-acting β2-agonist Educational Objective: Manage asthma during pregnancy. The most appropriate next step in management is to add a long-acting β2-agonist and continue the use of inhaled glucocorticoids. Approximately one third of patients with asthma experience worsening of symptoms during pregnancy. Although this patient previously had good control of symptoms while taking medium-dose inhaled glucocorticoids, the same regimen is not providing adequate control at this point. The recommendation for step-up therapy in pregnant patients is similar to that for nonpregnant patients. The understanding is that it is safer for pregnant women to be exposed to asthma medications with limited human safety data than it is for them to experience ongoing symptoms and exacerbations of asthma. Long-acting β2-agonist are classified as pregnancy category C drugs, meaning that some safety data are lacking but the potential benefit of the drug may justify the potential risk. Therefore, the addition of a long-acting β2-agonist is recommended when symptoms are not controlled with medium-dose inhaled glucocorticoids. The addition of this medication results in better asthma control compared with doubling the dose of inhaled glucocorticoids. This recommendation is based on studies of nonpregnant patients. Large-scale studies of pregnant patients have not been performed. Theophylline is also classified as a pregnancy category C drug. Theophylline provides an alternative to step-up therapy with a long-acting β2-agonist. However, because the metabolism of the drug is altered during pregnancy, more frequent monitoring of serum levels and for evidence of toxicity is required. Therefore, a long-acting β2-agonist is preferred over theophylline. A bronchial challenge test (methacholine or mannitol) can be performed to evaluate patients whose symptoms are not clearly consistent with asthma. However, methacholine challenge is contraindicated during pregnancy. Mannitol is listed as a pregnancy category C drug; adequate information is not available on its safety during pregnancy. Furthermore, bronchial challenge is not likely to provide clinically useful information. KEY POINT: In pregnant patients with asthma, the addition of a long-acting β2-agonist is recommended when symptoms are not controlled with medium-dose inhaled glucocorticoids.

A 55-year-old woman is evaluated because of a recent increase in symptoms of asthma characterized by daily cough and dyspnea. She reports waking up two to three nights per week with her typical asthma symptoms. She has no postnasal drip, nasal discharge, fever, or heartburn. Her medications are medium-dose inhaled corticosteroids and albuterol as needed. She demonstrates proper use of her metered-dose inhalers. On physical examination, she appears comfortable and is in no respiratory distress. Pulse rate is 76/min, and respiration rate is 18/min. Pulmonary examination shows bilateral wheezing. The remainder of the findings on examination are normal. Which of the following is the most appropriate treatment? A. Add a long-acting β2-agonist inhaler B. Add an ipratropium metered-dose inhaler C. Double the dose of inhaled corticosteroids D. a 10-day course of a macrolide antibiotic ​

A. Add a long-acting β2-agonist inhaler Educational Objective: Treat inadequately controlled asthma. The most appropriate treatment is to add a long-acting β2-agonist inhaler. This patient was doing well until the recent exacerbation of asthma. Her asthma is now classified as moderate persistent, based on the National Asthma Education and Prevention Program guidelines (daily symptoms of asthma and nocturnal awakenings more than once per week). Symptoms are not well controlled on a moderate dose of inhaled corticosteroids. The guidelines recommend the addition of a long-acting β2-agonist in this case. This treatment has been shown to lead to greater improvement in asthma control compared with doubling the dose of inhaled corticosteroids. The systemic side effects of inhaled corticosteroids are relatively uncommon but occur in patients who are receiving high-dose therapy (particularly long-term, high-dose therapy). These effects include adrenal suppression, glaucoma, cataracts, osteopenia, and thinning of the skin. Therefore, the lowest dose that provides disease control should always be used. Finally, treatment of asthma with a metered-dose inhaler with various agents is essential. Patients should be shown the proper technique for using inhalers, and those with poorly controlled disease should be evaluated for proper inhaler technique. Ipratropium enhances the bronchodilator effect of β2-agonist when given for acute exacerbations of asthma. However, its use for long-term control of asthma generally is not recommended. Recent studies showed that the addition of a long-acting anticholinergic drug (tiotropium) was equivalent to the addition of a long-acting β2-agonist in patients with asthma whose symptoms were inadequately controlled with inhaled corticosteroids alone. Although macrolide antibiotics are beneficial in treating atypical respiratory tract infections (which can be associated with exacerbations of asthma and asthma-like symptoms), their routine use in the treatment of asthma is not recommended and has not been shown to improve asthma control. KEY POINT: Guidelines recommend the addition of a long-acting β2-agonist to medium dose corticosteroids in patients with moderate persistent asthma. This treatment has proved to lead to greater improvement in asthma control compared with doubling the dose of inhaled corticosteroids.

A 53-year-old man comes to the emergency department at 6:00 AM due to severe pain in his right great toe for the past 3 hours. His pain began suddenly as a dull ache, and rapidly worsened to severe throbbing that is not relieved by acetaminophen. Medical history is notable for hypertension, type 2 diabetes mellitus, and hypercholesterolemia. The patient has smoked 2 packs of cigarettes daily for 30 years. He drinks 2-3 beers and 3-4 cups of coffee daily, consumes fast food often, and does not exercise regularly. Current medications include metformin, losartan, amlodipine, sitagliptin, and atorvastatin. The patient's temperature is 36.8 C (98.2 F), blood pressure is 160/90 mm Hg, pulse Is 88/min, and respirations are 16/min. On examination, the right great toe appears markedly swollen, red, and warm to the touch. Which of the following interventions would be most appropriate to prevent development of further similar episodes in this patient? A. Alcohol cessation B. Discontinue atorvastatin C. Discontinue losartan D. Low-dose prednisone E. Moderation of coffee intake F. Smoking cessation

A. Alcohol cessation This patient has sudden-onset severe pain in the first metatarsophalangeal joint (podagra) consistent with acute gout. Initial treatment can include nonsteroidal anti-inflammatory drugs (eg, indomethacin), glucocorticoids, or colchicine. Urate-lowering medications (eg, allopurinolr febuxostat) are indicated for patients with recurrent attacks or complicated disease (eg, tophi, uric acid kidney stones). This patient is having his first attack and does not require urate-lowering medication; however, he has multiple risk factors for recurrent gout (ie, obesity, hypertension, insulin resistance) and should be counseled on lifestyle modification to reduce his risk. Weight loss and caloric restriction can reduce the risk of recurrent gout. In addition, specific dietary changes are also recommended. The risk of attacks is increased with intake of red meat and seafood but is reduced in diets emphasizing protein from vegetarian and low-fat dairy sources. Risk is also increased by heavy intake of beverages and foods containing fructose and other refined sugars. Heavy alcohol intake, especially from beer and distilled spirits, is also associated with an increased risk of gout attack. Ethanol increases uric acid production and may also decrease renal elimination of uric acid. Educational objective: Lifestyle modifications, including alcohol cessation and weight loss, are recommended to prevent recurrent gout attacks. Medications for lowering serum urate are indicated for patients with repeated attacks of gouty arthritis or complicated disease (eg, tophi, uric acid kidney stones).

Three years after a laparoscopic Roux-en-Y gastric bypass (LRYGB), a 45-year-old male presents with symptoms and signs of a small bowel obstruction (SBO). He reports a 150-lb weight loss. Which of the following is the most likely etiology? A. An internal hernia B. Adhesions C. Roux compression due to mesocolon scarring D. kinking of the jejunojejunostomy E. Incarcerated abdominal wall hernia ​

A. An internal hernia The most common etiology of small bowel obstruction in the United States is adhesions from previous abdominal surgery. However, this does not hold true for patients that have previously had a LRYGB. In this procedure, a potential hernia site (Petersen's space hernia) is created increasing the risk for the development of an internal hernia, which is the most common cause of SBO in this patient population with an incidence of 1% to 5%. This potential space results from herniation of intestinal loops through a defect in the mesentery and between small bowel limbs, transverse mesocolon, and the retroperitoneum. Additionally, when compared to its open counterpart, the laparoscopic approach further facilitates a Petersen's hernia because of the decreased frequency of postoperative adhesions, which seemingly have a physiologic role of preventing bowel mobility and thus, internal herniation. Risk of SBO is significantly higher with a retrocolic versus antecolic approach. Roux compression due to mesocolon scarring is the second most common etiology for SBO in patients with LRYGB followed by adhesions (B, C). Kinking of the jejunojejunostomy and incarcerated abdominal wall hernia occur less frequently (D, E).

A 48-year-old woman presents to her primary care physician because of swelling of her hands for the past 3 months. She says her skin feels firm and tight. For the past 5 years, she notes her hands turn very pale when she goes outside in cold weather, then they turn blue, and then return to a red color. She has a history of gastroesophageal reflux disease and is taking omeprazole. Recently she has begun having pain while swallowing solid foods. On physical examination, she has bilaterally swollen fingers and hands. The overlying skin appears very smooth, but on palpation, the hands are firm and indurated. On the finger pads there are several subcutaneous hard nodules. Which of the following will most likely be elevated in this patient? A. Anti-centromere autoantibody B. Anti-Scl-70 autoantibody C. Anti-Smith autoantibody D. Anti-SSA autoantibody E. Antineutrophil cytoplasmic autoantibody ​

A. Anti-centromere autoantibody This patient has symptoms and signs suggestive of systemic sclerosis, or scleroderma. The swollen, indurated fingers and hands are a classic finding in cutaneous systemic sclerosis. The fact that the skin findings are limited to the hands and do not extend proximally indicates that this woman has limited cutaneous systemic sclerosis (LCSS), as opposed to diffuse disease. LCSS is typically associated with the CREST syndrome: Calcinosis, Raynaud phenomenon, Esophageal dysmotility, Sclerodactyly, and Telangiectasias. This patient demonstrates many of the components of CREST, including nodules in the finger pads, which represent calcific deposits in the subcutaneous tissue. She also has Raynaud phenomenon, or episodic vasoconstriction of the small arteries in the fingers, causing pallor and cyanosis when patients are exposed to cold temperatures or emotional stress, and rubor on rewarming. She also has a history of reflux and dysphagia, which are signs of esophageal dysmotility. LCSS is typically associated with elevated levels of anti-centromere and/or anti-nucleolar autoantibodies. Bottom Line: Limited cutaneous systemic sclerosis is associated with the CREST syndrome (Calcinosis, Raynaud phenomenon, Esophageal dysmotility, Sclerodactyly, and Telangiectasias) and elevated anticentromere antibodies.

A 25-year-old woman is evaluated during a follow-up visit for systemic lupus erythematosus. She was feeling well until 2 weeks ago when she developed increased fatigue and diffuse arthralgia. Medications are hydroxychloroquine and ibuprofen as needed. On physical examination, temperature is 37.2 °C (99.0 °F), blood pressure is 140/80 mm Hg, pulse rate is 80/min, and respiration rate is 16/min. There is diffuse alopecia of the scalp. Malar erythema is noted. Heart sounds are normal, and the chest is clear. Examination of the abdomen is normal. Tenderness with minimal swelling of the proximal interphalangeal joints is present bilaterally. Small effusions on both knees with pain on range of motion are noted. Laboratory studies: Leukocyte count 3000/µL (3.0 x 109/L), with 900 lymphocytes Creatinine Normal Electrolytes Normal Urinalysis 2+ protein; trace blood Which of the following tests should be obtained next? A. Anti-double-stranded DNA antibodies B. Antinuclear antibodies Anti-Ro/SSA and anti-La/SSB antibodies C. Anti-Smith antibodies D. Anti-U1-ribonucleoprotein antibodies

A. Anti-double-stranded DNA antibodies Educational Objective: Confirm a flare of systemic lupus erythematosus with an anti-double-stranded DNA antibody measurement. Anti-double-stranded DNA antibodies correlate with systemic lupus erythematosus disease activity, particularly active kidney disease or glomerulonephritis. Measurement of anti-double-stranded DNA antibodies is appropriate for this patient who is having a flare of systemic lupus erythematosus (SLE). She has symptoms of fatigue, joint pain, rash, leukopenia, and lymphopenia. Urinalysis shows proteinuria and hematuria, indicating that she may have glomerulonephritis as well. Levels of anti-double-stranded DNA antibodies correlate with SLE disease activity; in particular, they correlate with active kidney disease or glomerulonephritis and might prompt further evaluation such as kidney biopsy. Thus, measuring anti-double-stranded DNA antibody titers may be useful in assessing this patient's recent symptoms. Following anti-double-stranded DNA antibody titers over time can be useful because it is a marker for risk of developing lupus nephritis. Antinuclear antibody (ANA) testing is a useful screening tool for SLE because more than 95% of patients with SLE are positive for ANA; however, ANA does not correlate with disease activity. Anti-Ro/SSA and anti-La/SSB antibodies can be present in patients with Sjogren syndrome as well as SLE. These antibodies correlate with SLE rashes and photosensitivity and are a risk factor for the development of neonatal lupus erythematosus; however, they do not correlate with disease activity. Anti-Smith antibodies are highly specific for the diagnosis of SLE; however, these antibodies also do not correlate with disease activity. Anti-U1-ribonucleoprotein antibodies are found in patients with SLE and with mixed connective tissue disease but do not correlate with disease activity.

A 49-year-old woman comes to the office due to difficulty swallowing solid food. The patient's symptoms are worse with breads and crackers, but she has no difficulty with liquids, and she uses sips of water to help with swallowing. She thinks that she may have lost weight. The patient's medical history is significant for hypertension and hypothyroidism treated with amlodipine and levothyroxine, respectively. She also uses over-the-counter saline eye drops for eye dryness. The patient has never used tobacco or alcohol. Family history is negative for cancer. She has been monogamous with her husband for 15 years. Blood pressure is 140/90 mm Hg and pulse is 60/min. BMI is 19 kg/m2. Physical examination shows oral thrush, prominent dental caries, and bilateral, firm submandibular nodules. There is no lymphadenopathy. Cardiopulmonary examination shows no abnormalities. Abdomen Is soft and nontender with no organomegaly. There is no peripheral edema. Which of the following is the best next step to confirm this patient's diagnosis? A. Antibodies to Ro/SSA and La/SSB CT scan of the neck B. Eating disorder aassessment C. Esophageal endoscopy with biopsy D. Submandibular gland biopsy E. Videofiuoroscopic swallowing study

A. Antibodies to Ro/SSA and La/SSB CT scan of the neck Sjogren syndrome (SS) is an autoimmune disorder characterized by inflammation of exocrine glands, with a greatest incidence in middle-aged women. Patients can have severe dry mouth (xerostomia) with dysphagia (difficulty swallowing), thrush, and dental caries. Involvement of lacrimal glands causes keratoconjunctivitis sicca (e.g., dry eyes). Extra-glandular features include arthritis, Raynaud phenomenon, cutaneous vasculitis, interstitial lung disease, and increased risk for non-Hodgkin lymphoma. The diagnosis of SS is made in patients with ocular or oral dryness when there is: « Evidence of dry mouth and eyes. In the Schirmer test, a sterile strip of filter paper is placed under the lower eyelid, and the area moistened by the absorbed fluid is measured after 5 minutes. « Either histologic evidence of lymphocytic infiltration of the salivary glands or presence of serum autoantibodies directed against SSA (Ro) and/or SSB (La). Antinuclear antibodies and rheumatoid factor are less specific but helpful when positive. Educational objective: The diagnosis of Sjogren syndrome requires evidence of dry mouth and eyes (e.g., positive Schirmer test result for decreased lacrimation) with either histologic evidence of lymphocytic infiltration of the salivary glands or serum autoantibodies against SSA (Ro) and/or SSB (La).

A 46-year-old woman comes to the physician after 2 months of pain and numbness of her right index and middle fingers. Her fingers turn white when subjected to cold; and they hurt until she thoroughly warms them. The patient has no problems with her left fingers or toes. Past medical history is significant for chronic cough; lower back pain, and acid reflux disease. She has a 28-pack-year smoking history. On examination, there are 2 small ulcers on the tip of her right index finger. Which of the following is the best next step in management of this patient? A. Antinuclear antibodies B. Arteriogram C. Losartan therapy D. Thyroid function studies E. Transthoracic echocardiogram

A. Antinuclear antibodies This patient's clinical features suggest Raynaud phenomenon (RP), which is caused by abnormal vasoconstriction of digital arteries in response to cold or emotional distress. Primary RP is not associated with a systemic disorder, but secondary RP (e.g., systemic lupus erythematosus, scleroderma, thromboangiitis obliterans) is. Patients typically experience sudden onset of cold at the extremities (fingers or toes) and sharply demarcated color changes (white or blue) of the digits. Primary RP typically affects young patients (ages 15-30, often female), who have symmetric episodic attacks without tissue injury. Patients with secondary RP are generally older (age >40, often male) and have asymmetric attacks with clinical features of tissue ischemia (e.g., numbness, ulcers) and systemic disease (autoimmune or vascular). Based on history and physical examination, workup for patients with suspected secondary RP may include: « Complete blood count and metabolic panel « Urinalysis « Antinuclear antibody (AMA) and rheumatoid factor « Erythrocyte sedimentation rate and complement levels (C3 and C4) If the ANA screen is positive, specific antibodies (e.g. antitopoisomerase-1 for systemic sclerosis) may be obtained. Educational objective: Raynaud phenomenon (RP) is abnormal vasoconstriction of digital arteries in response to cold or emotional stress. It may be primary, or may occur in association with systemic disease, especially autoimmune disorders. Patients with suspected secondary RP should be tested for autoantibodies and inflammatory markers.

A 32-year-old man presents with pain and swelling of one ankle and one knee. The history reveals that he had just returned from a trip to South America and developed diarrhea that was now subsiding. He denied any eye inflammation or skin lesions. Physical examination confirmed tenderness and swelling of an ankle and a knee. The next appropriate diagnostic test would be which of the following? A. Aspiration of the knee swelling and culture of the synovial fluid B. Initiate a course of broad-spectrum antibiotics. C. X-rays of the lumbar and cervical spine D. Check a blood test for the presence of antinuclear antibodies. E. Initiate an empirical trial of methotrexate. ​

A. Aspiration of the knee swelling and culture of the synovial fluid A patient with new-onset swelling in the joints, particularly when one or two joints are affected, should always raise the possibility of septic arthritis. When there has been a recent infection elsewhere in the body, it is possible the inciting infection could spread by a hematogenous route to seed the joint. Although the history is suggestive of reactive arthritis (ReA), it is important to rule out septic arthritis with certainty before entertaining that diagnosis. Culture of synovial fluid is the definitive test to rule out septic arthritis. Empirical use of broad-spectrum antibiotics rarely has a place in the management of new-onset arthritis. The first step is to make a diagnosis. X-rays of the spine are important in the evaluation of inflammatory back pain, but in this case there is only peripheral joint involvement. If the course of the arthritis becomes chronic or back pain develops over time, x-rays at that point would be appropriate. Antinuclear antibodies in the serum are the hallmark of many autoimmune rheumatic diseases—in particular, lupus. But with the clinical presentation in this case, there are no features to implicate lupus or related conditions, and it would not be appropriate to search for antinuclear antibodies. The immediate priority is to rule out septic arthritis. It would be inappropriate to institute methotrexate before that is resolved.

A 38-year-old woman has had nausea for 6 months. She reports no vomiting or diarrhea. On physical examination, there are no remarkable findings. Upper gastrointestinal endoscopy shows diffuse gastric mucosal erythema with focal mucosal erosions, but no ulcerations. The esophageal and duodenal mucosal surfaces appear normal. Gastric biopsies are obtained and microscopic examination shows focal mucosal hemorrhage, loss of the surface epithelium, and increased numbers of neutrophils, lymphocytes, and plasma cells in an edematous mucosa. No Helicobacter pylori organisms are seen. Laboratory studies show a normal serum gastrin level. Which of the following pharmacologic agents is most likely to produce these findings? A. Aspirin B. Chlorpromazine C. Cimetidine D. Clindamycin E. Omeprazole ​

A. Aspirin These findings are consistent with an acute gastritis. If significant inflammation is not present, then the term gastropathy is used. Heavy consumption of ethanol is probably the most common cause, but aspirin and nonsteroidal anti-inflammatory drugs (NSAIDs), smoking, and chemotherapy agents can produce the same findings. NSAIDs can be cofactors in peptic ulcer disease. Chlorpromazine (used to treat nausea) does not have the same association. Cimetidine and omeprazole are used to treat peptic ulcer disease by reducing gastric acid production, increasing the serum gastrin. Cimetidine is an H2 receptor blocker, and omeprazole is a proton pump inhibitor. Clindamycin is a broad-spectrum antibiotic that may alter flora in the lower gastrointestinal tract.

A 24-year-old woman is evaluated because of chronic cough that began after a respiratory tract infection 7 months ago. She notices that exposure to cold air and exercise, such as running, are likely to induce cough and wheezing. On rare occasions she is awakened at night with cough. She does not smoke, has no travel history works as an accountant, and is not exposed to animals. She has no personal or family history of allergy or asthma. She otherwise feels well and takes no medications. On physical examination, vital signs are normal. Upper airway examination is normal, and lungs are clear to auscultation. The remainder of the examination is normal. Chest radiography is normal. On office spirometry, forced expiratory volume in 1 second (FEV1) is 85% of predicted and FVC is 85% of predicted. The FEV1/forced vital capacity (FVC) ratio is 76%. The flow volume loop is normal. Pulmonary function testing shows total lung capacity of 105% of predicted. After the patient inhaled methacholine, there was a 20% drop in FEV1 and FVC. Which of the following is the most likely diagnosis? A. Asthma B. Cystic fibrosis C. Idiopathic pulmonary fibrosis D. cord dysfunction

A. Asthma Educational Objective: Diagnose cough variant asthma. The most likely diagnosis is asthma. The patient's history is suspicious for reactive airway disease with onset after a respiratory tract infection, symptoms of cough and wheezing when exposed to cold air or after exercise, and occasional nighttime symptoms. Findings on initial spirometry were normal. In patients who have symptoms suggestive of asthma but normal findings on spirometry bronchoprovocation with methacholine can help to establish the presence of airway hyperresponsiveness. Methacholine is administered by inhalation in increasing concentrations through a nebulizer. Cigarette smoking, chronic obstructive pulmonary disease, allergic rhinitis, and recent viral respiratory tract infection can lead to increased airway hyperresponsiveness and a positive result on methacholine challenge test. Therefore, a positive test should be correlated with other features of asthma before a clinical diagnosis can be established. The most appropriate use of this test is to exclude asthma in patients with normal findings on spirometry and methacholine challenge and symptoms consistent with, but not typical of, asthma. Nearly 10% of patients who are diagnosed with cystic fibrosis are older than 18 years of age. In cystic fibrosis lung disease, chest radiograph typically shows hyperinflation and accentuated broncho-vascular markings, appearing first in the upper lobes, followed by bronchiectasis and cyst formation. Pulmonary function testing shows an obstructive pattern. This patient has no findings consistent with cystic fibrosis. The hallmark of idiopathic pulmonary fibrosis is the finding of peripheral- and basal-predominant disease on computed tomography in a patient with progressive pulmonary symptoms associated with restrictive lung physiology. This picture is not compatible with the patient's presentation. When patients with asthma present with prominent wheezing that is more notable during inspiration (stridor), vocal cord dysfunction (VCD) should be suspected. Abrupt onset and termination of the episode is characteristic of VCD and is atypical for asthma. Flow-volume loops show inspiratory flow cutoff and preserved expiratory flow, in contrast to what is seen in asthma. In the absence of concomitant asthma, patients with VCD have negative findings on methacholine challenge test. KEY POINT: In patients who have symptoms suggestive of asthma but normal findings on spirometry, bronchoprovocation with methacholine can help to establish the presence of airway hyperresponsiveness.

A 42-year-old woman comes to the office due to a 1-year history of intermittent skin lesions. She has had ulcerating lesions on the lower extremities associated with inguinal adenopathy and low-grade fevers. Her medical history is notable for obesity, type 2 diabetes mellitus, and hyperlipidemia. Current medications Include metformin and rosuvastatin. The patient also frequently takes ibuprofen for joint pain. She does not use tobacco, alcohol, or illicit drugs. Her temperature is 37.2 C (99 F), blood pressure is 145/90 mm Hg, pulse is 90/min, and BMI is 32 kg/m2. Cardiopulmonary examination is normal. The abdomen is soft and nontender with normal bowel sounds and a palpable spleen tip on inspiration, Swelling and tenderness of the hand joints are present bilaterally. Skin examination shows several healed lesions on the lower legs. Laboratory results are as follows: Hemoglobin 13.2 g/dL Platelets 93,000/mm3 White blood cells 1800/mm3 Neutrophils 20% Creatinine 0.8 mg/dL Antinuclear antibody negative A. Autoimmune disorder B. Drug toxicity C. Lymphoproliferative disorder D. Myelodysplasia E. Viral infection F. Vitamin deficiency ​

A. Autoimmune disorder This patient has inflammatory arthritis, splenomegaly, and neutropenia. This triad of clinical findings is characteristic of Felty syndrome (FS). FS is most common in patients with long-established rheumatoid arthritis (RA) but occasionally precedes the diagnosis. Patients with FS typically have severe, seropositive RA with increased risk for extra-articular manifestations (eg, vasculitis, skin ulcers). The neutropenia of FS may be detected incidentally but increases the risk for recurring bacterial infections. FS is more common in women, with a peak incidence in the fourth and fifth decades. The pathophysiology is unknown, but FS is associated with HLA-DR4 and has an increased incidence In patients with a family history of RA, suggesting a genetic component. The diagnosis is based primarily on clinical features, but hematologic studies (eg, peripheral blood smear, bone marrow biopsy) are frequently performed to exclude other causes of neutropenia. FS usually improves with treatment of the underlying RA (eg, methotrexate). Educational objective: Felty syndrome is characterized by the triad of inflammatory arthritis, splenomegaly, ana neutropenia. It is most common in patients with established rheumatoid arthritis (RA) but can sometimes precede this diagnosis. Patients typically have severe, seropositive RA with increased risk for extra-articular manifestations.

A 67-year-old woman is evaluated in the emergency department for a 2 day history of fever, dyspnea, and increased cough with production of green sputum. She has severe COPD, which was diagnosed 2 years ago. She used albuterol several times yesterday with no relief of dyspnea, and she was unable to sleep last night. Her last spirometry, performed 6 months ago, showed an FEV1 of 48% of predicted. She is a current smoker with a 24-pack-year history. Medications are tiotropium and as- needed albuterol. On physical examination, temperature is 38.9 °C (102.0 °F), blood pressure is 124/80 mm Hg, pulse rate is 118/min, and respiration rate is 30/min. Oxygen saturation is 82% breathing ambient air. Pulmonary examination reveals bilateral diffuse expiratory wheezing. After an albuterol nebulizer treatment and breathing 2 L of oxygen by nasal cannula, oxygen saturation is 91%. She remains tachypneic with bilateral expiratory wheezing. Chest radiograph shows no infiltrate. In addition to continuing this patient's supplemental oxygen and short-acting bronchodilator, which of the following is the most appropriate treatment? A. Azithromycin and prednisone B. Clarithromycin and fluticasone C. Doxycycline and salmeterol/fluticasone D. Erythromycin and roflumilast

A. Azithromycin and prednisone Educational Objective: Treat an acute exacerbation of COPD with antibiotics and systemic glucocorticoid therapy. Key Point: Antibiotics and systemic glucocorticoids are primary treatments for acute COPD exacerbations in addition to short-acting bronchodilators and as needed oxygen supplementation. The most appropriate treatment is to start short courses of antibiotics and prednisone. This patient has an acute exacerbation of COPD. In addition to short-acting bronchodilators and supplemental oxygen as needed, antibiotics and systemic glucocorticoids (such as prednisone) are indicated for treatment of acute COPD exacerbations. Acute exacerbations of COPD are mostly caused by bacterial or viral infections. The combination of cough, dyspnea, and sputum purulence that is increased from baseline generally indicates the need for antibiotics in the treatment of acute COPD exacerbations, especially in patients with severe and very severe COPD. The most common bacterial pathogens that cause exacerbations are Hemophilus influenzae, Hemophilus parainfluenza, Streptococcus pneumoniae, and Moraxella catarrhalis. Empiric antibiotic therapy should be chosen on the basis of local bacterial resistance patterns. Commonly used regimens include an advanced macrolide, a cephalosporin, or doxycycline. Systemic glucocorticoids decrease recovery time, improve lung function, improve arterial hypoxemia, and reduce the risk of early relapse. A prednisone dose of 40 mg/d for 5 days is recommended. Earlier studies suggested a 2-week course of prednisone, but a recent study showed that a 5-day course is sufficient. A prompt follow-up visit is necessary after discharge, and pulmonary rehabilitation should be recommended. The benefits of glucocorticoids in acute COPD exacerbation have been documented with systemic treatment and not with inhaled agents such as fluticasone. Although adding a combination of a long-acting β2- agonist (LABA) and inhaled glucocorticoid at the time of discharge may be appropriate, treating an acute exacerbation with antibiotics and an inhaled glucocorticoid would not be appropriate. Short-acting β2-agonists (albuterol) with or without short-acting anticholinergic agents are preferred for treatment of acute COPD exacerbations. Guidelines suggest adding a LABA or a combination of a LABA and an inhaled glucocorticoid at the time of discharge, in addition to the long-acting anticholinergic agent (tiotropium) that this patient was already taking. However, a combination of a LABA and an inhaled glucocorticoid (such as fluticasone/salmeterol) is not appropriate therapy during an acute COPD exacerbation. Roflumilast is an oral phosphodiesterase-4 inhibitor that reduces airway inflammation. Roflumilast is indicated for patients with severe or very severe COPD with recurrent exacerbations; it is not indicated for use during acute exacerbations.

A 35-year-old woman comes to the office due to oral ulcers. She has multiple painful sores on her oral mucosa that began a few days ago. The patient had similar lesions 3 months ago that healed without scarring. In addition, she recently was evaluated by an ophthalmologist for blurred vision and was diagnosed with anterior uveitis. She has also had recurrent genital lesions requiring frequent gynecology appointments over the last year. On examination, there are visible oral ulcerations as well as scattered hyperpigmented skin lesions and tender, indurated areas on her legs. Which of the following is the most likely diagnosis? A. Behcet syndrome B. Herpes simplex infection C. Reactive arthritis D. Sarcoidosis E. Systemic lupus erythematosus

A. Behcet syndrome This patient has characteristic features of Behcet syndrome, including recurrent painful oral and genital aphthous ulcers, uveitis, and erythema nodosum (tender red nodules usually in the pretibial area). These patients are also at high risk of vascular disease (vasculitis) with venous and arterial thrombosis. Patients may also demonstrate pathergy, an exaggerated ulcerating skin response following minor injuries (e.gr] needlestick). Behcet syndrome is more common in patients of Turkish, Middle Eastern, and Asian descent. The diagnosis is made based on clinical features, and laboratory evaluation is primarily directed to ruling out other disorders. Biopsy of affected tissues is not usually needed but may show a nonspecific vasculitis, often involving small or medium-sized vessels. Educational objective: Behcet syndrome is a multisystem inflammatory condition characterized by recurrent oral and genital ulcers. Skin and ocular involvement is common. Thrombosis is a major cause of morbidity. It is most common in patients of Turkish, Middle Eastern, and Asian descent. The diagnosis is made based on clinical features.

A 66-year-old woman is evaluated for a 1-month history of abdominal discomfort. She describes a deeply painful sensation in the mid-epigastric region and also notes early satiety. Because of decreased appetite, she reports a 1.4-kg (3-lb) weight loss. The patient has taken an over-the-counter proton pump inhibitor for the past week that has helped relieve her symptoms. Medical history is otherwise unremarkable, and she takes no other medications. On physical examination, vital signs are normal. The abdomen is soft with normal bowel sounds and no masses. Deep palpation in the mid-epigastrium produces moderate tenderness. Upper endoscopy reveals a 9-mm ulcer in the gastric antrum proximal to the pylorus. Which of the following is the most appropriate management for this patient's ulcer? A. Biopsy of the ulcer B. Omeprazole, amoxicillin, and clarithromycin C. Rapid urease test D. Urea breath test ​

A. Biopsy of the ulcer Educational Objective: Manage a newly diagnosed gastric ulcer. The most appropriate management for this patient's gastric ulcer is biopsy.. Biopsies of all gastric ulcers should be performed because even small, benign-appearing gastric ulcers may harbor malignancy. In benign ulcers, biopsies can also provide evidence of the presence of Helicobacter pylori infection and guide appropriate therapy. KEY POINT: Biopsies of all gastric ulcers should be performed because even small, benign-appearing gastric ulcers may harbor malignancy. ​

A 30-year-old man is evaluated for chronic cough that has lasted nearly 1 year. He recalls that the cough began after a "bad cold. " He was treated with a course of a macrolide antibiotic without significant improvement. The cough is nonproductive, is most noticeable at night and on cold days, and sometimes occurs after exercise. He has no postnasal drip, nasal congestion, or heartburn. He does not smoke. He has no history of occupational or other exposures. Medical history is otherwise unremarkable except for gastroesophageal reflux for which he takes a daily proton pump inhibitor. On physical examination, vital signs are normal. The oropharynx is normal, without postnasal drip. Findings on pulmonary examination are normal. Spirometry shows forced expiratory volume in 1 second (FEV1) of 90% of predicted and an FEV1/forced vital capacity ratio of 80%. Chest radiograph is normal. Which of the following is the most appropriate diagnostic test to perform next? A. Bronchial challenge test B. Bronchoscopy C. Chest computed tomography D. Pertussis culture

A. Bronchial challenge test Educational Objective: Diagnose cough-variant asthma. The most appropriate diagnostic test is a bronchial challenge test. This patient has chronic cough. Asthma, postnasal drip, and gastroesophageal reflux are the most common diagnoses for patients with chronic cough and normal findings on chest radiograph. This patient has episodic cough triggered by cold air and hyperventilation. The symptoms are suggestive of cough-variant asthma. Negative findings on bronchial challenge testing in such a patient can exclude asthma, and positive findings suggest cough-variant asthma. Such testing is indicated when asthma is suspected even though routine spirometry shows no evidence of bronchospasm. Patients with underlying asthma typically have bronchial hyperresponsiveness and have an exaggerated response to agents that promote bronchoconstriction. Diagnostic criteria are based on the degree of airway obstruction triggered by inhalation of these medications relative to baseline. The diagnosis of cough-variant asthma should be confirmed with clinical response to usual asthma therapy. Although it is important to consider pertussis in patients with acute or subacute cough, this patient has had cough for nearly 1 year and has no recent symptoms to suggest acute infection. Furthermore, he has already received a course of antibiotics with a drug that should have been effective against Bordetella pertussis. Bronchoscopy should be reserved for patients with abnormal findings on chest radiograph and those who are at increased risk for lung cancer (e.g., long-standing history of smoking, older age). Chest computed tomography in a young nonsmoker with a clear chest radiograph would be unlikely to provide any useful information. Therefore, it is not indicated. K E Y P O I N T: Bronchial challenge testing in patients with suspected cough-variant asthma can exclude asthma if results are negative and suggest cough-variant asthma if results are positive.

A 67-year-old woman presents to the clinic complaining of increasing shortness of breath on exertion. She has no prior cardiac or pulmonary history, and reports no symptoms of chest discomfort, cough, sputum production, orthopnea or peripheral edema. Her physical examination including vital signs, cardiac and pulmonary examinations are completely normal. Her CXR, ECG, and CBC are also normal. She then undergoes pulmonary function tests to evaluate her symptoms of dyspnea. The most prominent finding is a reduction of the ratio of FEV1/FVC with no reversibility when given inhaled salbutamol. Which of the following is the most likely diagnosis? A. COPD B. ankylosing spondylitis C. pickwickian syndrome D. scleroderma of the chest wall E. lobar pneumonia ​

A. COPD In COPD the FEV1 and VC are reduced, but the FEV1 is reduced more because of high airway resistance resulting in a decreased FEV1/FVC ratio. The hallmark of COPD is airway obstruction (FEV1/FVC <70%) that is not reversible with salbutamol. In predominant emphysema, diffusing capacity is more profoundly decreased than in predominant bronchitis.

A 34-year-old man is evaluated for a 1-year history of decreased libido and severe erectile dysfunction, including absence of morning erections. He and his female partner would like to conceive. His medical history is otherwise unremarkable, and he takes no medications. On physical examination, vital signs are normal. BMI is 27. Atrophic testes are noted bilaterally. Laboratory studies: Follicle-stimulating hormone 1.2 mU/mL (1.2 U/L) Luteinizing hormone 0.8 mU/mL (0.8 U/L) Prolactin 110 ng/mL (110 µg/L) Morning testosterone (total) 120 ng/dL (4.2 nmol/L) Thyroid-stimulating hormone Normal MRI of the pituitary reveals a 0.8-cm anterior pituitary mass consistent with an adenoma. Which of the following is the most appropriate treatment? A. Cabergoline B. Clomiphene citrate C. Sildenafil D. Testosterone replacement therapy

A. Cabergoline Educational Objective: Treat hypogonadism secondary to hyperprolactinemia in a male patient. Key Point: Secondary hypogonadism in a male patient caused by hyperprolactinemia as a result of a prolactinoma is a possible cause of erectile dysfunction and should be treated with a dopamine agonist such as cabergoline. The most appropriate treatment is a dopamine agonist such as cabergoline. Hyperprolactinemia as a result of a prolactinoma is a possible cause of erectile dysfunction and decreased libido and may be successfully treated with a dopamine agonist. In addition to the sexual dysfunction associated with hyperprolactinemia, semen parameters are often abnormal; patients commonly demonstrate oligoasthenospermia (reduced sperm motility) and at times complete azoospermia. Three months of treatment are typically needed prior to return of improved semen parameters. Prolactin is secreted by the pituitary lactotroph cells under tonic inhibition by dopamine. Dopamine agonist therapy can normalize prolactin levels, reverse hypogonadism, and shrink tumors by at least 50% in almost 90% of patients. Evaluation of the infertile male with abnormal findings on semen analysis should always include investigation of the hypothalamic-pituitary-testicular (HPT) axis. Disturbances in this axis may result in failure of gonadotropin release from the anterior pituitary and insufficient testosterone production as well as absent or diminished spermatogenesis. Clomiphene citrate is effective only when the HPT axis is intact, which does not apply for this patient. Although the indication for clomiphene citrate in the infertile male population remains controversial, some clinicians use it to increase endogenous follicle-stimulating hormone and luteinizing hormone output from the anterior pituitary to support testosterone production by Leydig cells. Sildenafil may improve erectile dysfunction in this patient, but it will not increase endogenous testosterone levels and will not improve his hyperprolactinemic state. Testosterone replacement therapy would be helpful to alleviate the sexual side effects of this patient's hyperprolactinemic state; however, no restoration of spermatogenesis would occur, and therefore infertility would persist. Neither testosterone replacement nor sildenafil therapy will reduce the size of the patient's prolactinoma.

A 34-year-old woman is evaluated for amenorrhea, headache, and fatigue. She reports that from the time of menarche until 2.5 years ago, her menses were regular and predictable. Two and a half years ago, her menses became irregular and then stopped completely 6 months ago. She has had a few hot flushes and fatigue. She has noted galactorrhea. She began having headaches 2 years ago. In addition, she notes blurry peripheral vision. The rest of her medical history is unremarkable. She takes no medications. On physical examination, blood pressure is 112/72 mm Hg and pulse rate is 68/min. BMI is 21. White milky substance is expressed from her breasts bilaterally. Ocular movements and cranial nerves are intact. There are no stigmata of Cushing disease or acromegaly. Laboratory studies: Cortisol 8 AM after 1 mg of dexamethasone the night before 16 µg/dL (441.6 nmol/L) Estradiol <32 pg/mL (117.4 pmol/L) Follicle-stimulating hormone 1.1 mU/mL (1.1 U/L) Luteinizing hormone 0.8 mU/mL (0.8 U/L) Prolactin 472 ng/mL (472 µg/L) Thyroid-stimulating hormone 1.1 µU/mL (1.1 mU/L) MRI shows a 2.4-cm pituitary tumor that elevates the optic chiasm and surrounds the left carotid artery. Which of the following is the most appropriate treatment? A. Cabergoline B. Octreotide C. Radiation D. Surgery

A. Cabergoline Educational Objective: Treat a macroprolactinoma. Key Point: Dopamine agonists, such as cabergoline, are first-line therapy for symptomatic patients with prolactinomas. The patient has a macroprolactinoma, which is best treated with a dopamine agonist such as cabergoline. The most common cause of hyperprolactinemia is a prolactinoma, which is a benign adenoma. Microprolactinomas are less than 10 mm in diameter, and macroprolactinomas are 10 mm or greater in diameter. Prolactinomas are the most common type of secretory pituitary adenoma. The patient's tumor is causing significant mass effect, including compression of the optic chiasm, invasion into the cavernous sinus, secondary hypothyroidism, and likely growth hormone deficiency; however, even with these complications, the macroprolactinoma is best treated with medication instead of surgery. A dopamine agonist can cause a rapid decrease in the serum prolactin level and shrinkage of the prolactinoma. More specifically, in as many as 90% of patients, it can normalize prolactin levels, reverse hypogonadism, and shrink tumors by at least 50%. Because of these rapid decreases in tumor size, dopamine agonists can be used as first-line therapy, even in patients with mild visual field defects, as long as visual acuity is not threatened by rapid progression of the tumor or recent tumor hemorrhage. Octreotide is used to treat acromegaly but has no role in the first-line treatment of prolactinoma. Medical therapy is preferred over surgery and radiation for the treatment of prolactinomas. Stereotactic surgery (a specific kind of radiation therapy) would be considered in a patient with a tumor that is refractory to medical therapy and incompletely resectable. Because this patient has a prolactinoma, the treatment of choice is a dopamine agonist, not surgery. There is an excellent chance that the prolactinoma will respond to cabergoline, and the patient can avoid surgery.

A 77 year old woman with a history of generalised osteoarthritis (OA) is admitted to hospital with a delirious episode associated with dehydration and a urinary tract infection. During the admission, she develops pain, swelling and redness of the left wrist gradually worsening over a period of 4-6 hours. Blood tests reveal a neutrophilia (white cell count 12.5 x 109/L), a raised erythrocyte sedimentation rate (ESR; 65 mm/hr) and a raised C-reactive protein (CRP; 154 mg/L). What would be the most likely diagnosis? A. Calcium pyrophosphate deposition disease B. Gout C. Reactive arthritis D. Septic arthritis E. Vasculitis ​

A. Calcium pyrophosphate deposition disease Calcium pyrophosphate deposition disease is strongly associated with OA and typically affects those aged > 65 years, affecting women more commonly than men. The onset is sudden with join pain and swelling developing over a period of 4-6 hours. Dehydration is a common precipitating factor, and investigations typically reveal a neutrophilia and a raised ESR and CRP. Gout and reactive arthritis can present in a similar manner but would be less likely in a woman of this age. Septic arthritis is possible but usually has a subacute onset and tends to develop more slowly, over a period of 24-48 hours.

A 52-year-old woman complains of difficulty gripping her cup of coffee in the morning. She likes to solve crossword puzzles but is unable to hold a pen properly due to stiffness in her right hand. Her symptoms gradually improve during the day and she Is "fully functional" by Sate afternoon. Examination shows nontender nodules near the elbows over the back of her forearm. The erythrocyte sedimentation rate is 45 mm/hr. Which of the following would most likely be involved during the course of this patient's disease? A. Cervical spine B. Lumbar spine C. Sacral spine D. Sacroiliac joints E. Thoracic spine ​

A. Cervical spine This patient's presentation (morning stiffness, metacarpophalangeal joint involvement, nontender nodules on the elbow, high erythrocyte sedimentation rate) suggests rheumatoid arthritis (RA). RA is a chronic systemic inflammatory disorder that involves synovial inflammation leading to joint damage. Women are affected 2-3 times more frequently than men, with a peak Incidence at age 50-75. RA primarily involves peripheral joints and most commonly affects the metacarpophalangeal (MCP) and proximal interphalangeal (PIP) joints of the fingers, metatarsophalangeal (MTP) joints of the toes, and the wrist joints in the early phases. However, the distal interphalangeal (DIP) joints are not involved. The disease eventually can involve larger joints such as the knees, elbows, ankles, and shoulders. Symptoms include pain, swelling, and morning stiffness that improve with increased movement and during the course of the day. Reduced grip strength can occur occasionally and is a sensitive sign of early disease. Rheumatoid nodules can occur in up to 30%-40% of patients and present as firm and nontender subcutaneous nodules usually close to pressure points (eg, elbows). RA most commonly affects the cervical spine joints In the axial skeleton and can lead to cervical spine subluxation, which can also cause spinal cord compression. Patients with cervical spine involvement often complain of neck pain, stiffness, and radicular pain in the upper extremity. Subluxation with spinal cord compression can present with hyperreflexia or upgoing toes on Babinski testing. Educational objective: Rheumatoid arthritis (RA) presents as progressive and symmetrical involvement of the peripheral joints. The common sites of early involvement Include metacarpophalangeal and proximal interphalangeal joints of the fingers, metatarsophalangeal joints of the toes, and the wrist joints. RA most commonly affects the cervical spine joints in the axial skeleton and can cause spinal subluxation and spinal cord compression.

A 42-year-old Turkish restaurant owner presents to the Emergency Department with a 12 h history of severe abdominal pain, fever and myalgia. On examination, he is pyrexial, temperature 39 °C and has generalised abdominal tenderness and guarding. Bowel sounds are quiet. He has a healed laparotomy scar from a previous hospital admission with similar symptoms. The laparotomy was inconclusive. On this occasion, plain abdominal X-ray is unremarkable. He is reviewed by the surgical team and commenced on broad-spectrum intravenous antibiotics. Eighteen hours after admission, his symptoms have resolved. He comments that his brother has also had previous hospital admissions with similar symptoms. Which medication would you prescribe to affect long-term outcomes in this condition? A. Colchicine B. Tramadol C. Prednisolone D. Diclofenac E Indometacin ​

A. Colchicine This patient has familial Mediterranean fever (FMF), an inherited, autosomal recessive condition characterized by short, recurrent episodes of fever and peritonitis. Other associated symptoms include pleuritis, arthritis and skin rashes. It most commonly occurs in people of Mediterranean origin, but not exclusively. Attacks usually last 24-72 h. The abdominal symptoms often mimic an acute surgical abdomen and consequently many patients have undergone laparotomy. Despite the severity of the symptoms during an attack, patients quickly recover and are well between episodes. Colchicine is extremely effective in treating and preventing attacks of familial Mediterranean fever.

An 83-year-old woman is evaluated in the emergency department because of hematemesis associated with hypotension and tachycardia. Two 16-gauge intravenous catheters are placed, and intravenous fluid and erythrocyte resuscitation is initiated with bolus administration of 2 L of 0. 9% saline and two units of packed erythrocytes. Intravenous omeprazole is also begun, and she is transferred to the intensive care unit (ICU). The patient has no history of gastrointestinal bleeding or liver disease. She has osteoarthritis for which she takes daily aspirin. On physical examination in the ICU 2 hours later, blood pressure is 87 /58 mm Hg, pulse rate is 112/min, and respiration rate is 12/min. The abdomen is nontender, and there is no organomegaly. There are no stigmata of chronic liver disease. Laboratory studies show a hemoglobin level of 7 g/dL (70 g/L). Which of the following is the most appropriate next step in management? A. Continue intravenous fluid and erythrocyte resuscitation B. Initiate octreotide C. Perform immediate upper endoscopy D. Place a diagnostic nasogastric tube

A. Continue intravenous fluid and erythrocyte resuscitation Educational Objective: Manage upper gastrointestinal bleeding and hemodynamic instability. This patient should continue to receive intravenous fluid and erythrocyte resuscitation. She has experienced a large gastrointestinal hemorrhage with resultant hemodynamic instability. Her history of aspirin use suggests an upper gastrointestinal source of bleeding. Appropriate intravenous access has been obtained, and 0.9% saline fluid boluses and intravenous proton pump inhibitor infusions have been started. Despite this, she remains unstable and has a hemoglobin level of7 g/dL (70 g/L). The most important and urgent treatment for this patient is erythrocyte transfusion and intravenous crystalloid administration to achieve hemodynamic stability, as an intensive resuscitation strategy has been shown to improve mortality in patients with severe gastrointestinal bleeding. Volume loss is estimated by pulse rate, blood pressure, and the presence of orthostatic hypotension because changes in hemoglobin and hematocrit levels may not become evident immediately. An initial hemoglobin level of less than 8 g/dL (80 g/L) is concerning because re-equilibration in the 24 to 48 hours after the initial bleeding episode may reveal an even lower hemoglobin level. Octreotide is a somatostatin analogue that reduces portal venous pressure and has vasoconstrictive properties; it is therefore used to treat bleeding due to variceal hemorrhage, but would not be appropriately used empirically in this patient without known or suspected liver disease. After further resuscitation with packed red blood cells and intravenous crystalloid, this patient will require upper endoscopy within 24 hours of presentation. However, upper endoscopy is not appropriate at this time in an incompletely resuscitated patient with ongoing instability. Additionally medications administered for moderate sedation during standard upper endoscopy may result in hypotension and could cause further hemodynamic insult. Nasogastric tube placement can be helpful in selected patients in whom the location of bleeding is in question. However, there is a high enough false-negative rate (approximately 15%) and false-positive rate (due to nasogastric tube mucosa! irritation) that this is not uniformly recommended and would not be the first priority in this patient. Furthermore, this patient has hematemesis, indicating significant upper gastrointestinal hemorrhage. KEY POINT: Patients with upper gastrointestinal bleeding should be stabilized with intravenous fluid and erythrocyte resuscitation before diagnostic upper endoscopy is performed.

A 66-year-old man is evaluated because of slowly progressive shortness of breath and productive cough. He has a 5-year history of chronic obstructive pulmonary disease. He has a 55-pack-year smoking history, but stopped tobacco use 4 years ago. Medications are inhaled salbutamol and inhaled ipratropium. He uses inhaled albuterol as rescue medication. The patient continues to experience limiting dyspnea with activities of daily living. He demonstrates excellent inhaler technique. He reports no change in the color or volume of his sputum and does not have fever or chills. On physical examination, temperature is 37.5°C (99.5°F), blood pressure is 128/76 mm Hg, pulse rate is 94/min, respiration rate is 20/min, and oxygen saturation is 82% on ambient air. Body mass index is 20. Heart sounds are distant, and breath sounds are diminished bilaterally and associated with a prolonged expiratory phase. Spirometry shows Forced expiratory volume in 1 second (FEV1) of 40% of predicted and an FEV1/forced vital capacity ratio of 45%. Chest radiograph shows hyperinflation. Which of the following interventions is most likely to prolong the patient's survival? A. Continuous oxygen therapy B. Inhaled glucocorticoid therapy C. Long-term antibiotic therapy D. Pulmonary rehabilitation

A. Continuous oxygen therapy Educational Objective: Treat hypoxic respiratory failure with continuous oxygen therapy. Oxygen therapy is a major component of treatment for patients with very severe chronic obstructive pulmonary disease (COPD). This treatment is indicated for patients who have resting hypoxemia, defined as arterial PO2 of 55 mm Hg or less or arterial oxygen saturation of 88% or less. In patients who qualify for continuous therapy because of resting hypoxemia, oxygen treatment should be administered for at least 15 h/day. The use of long-term oxygen therapy in patients with chronic respiratory failure improves survival and has a beneficial effect on hemodynamics, hematologic characteristics, exercise capacity, and mental status. Antibiotic therapy is most beneficial in treating infectious exacerbations of COPD that are characterized by increases in dyspnea and sputum volume and purulence. Antibiotics in this setting improve airflow and reduce the mortality rate and the rate of treatment failure, especially in patients who have more severe exacerbations. Although there is some evidence of their effectiveness, more definitive data are awaited on the long-term use of prophylactic antibiotics to prevent exacerbations. There is no evidence that long-term antibiotic therapy prolongs survival. An inhaled glucocorticoid is widely prescribed for COPD and is frequently added to treatment with long-acting bronchodilators when patients remain symptomatic or have repeated exacerbations. In some patients with severe COPD and inadequately controlled symptoms, the combination of a long-acting p2-agonist, a long-acting anticholinergic agent, and an inhaled glucocorticoid is used. There is some evidence that triple therapy may be beneficial, but the efficacy and safety of this intervention have not been definitively proved. There is no evidence that inhaled glucocorticoid therapy alone or in combination prolongs survival. Pulmonary rehabilitation can be considered for all symptomatic patients with forced expiratory volume in 1 second of less than 50% of predicted. When added to other forms of therapy, pulmonary rehabilitation reduces the perceived intensity of breathlessness, decreases dyspnea and fatigue, increases participation in daily activities, improves quality of life (including a reduction of anxiety and depression associated with COPD), and reduces the number of hospitalizations and the length of stay. Pulmonary rehabilitation, although indicated in this patient, does not improve survival. KEY POINT: The use of long-term oxygen therapy in patients with chronic respiratory failure improves survival as well as hemodynamics, hematologic characteristics, exercise capacity, and mental status.

A 25-year-old man comes to his physician because of decreased libido and feeling tired. He has not been able to complete his workouts at the gym lately, and he recently was involved in a motor vehicle collision, which he attributes to "not paying attention" to a car approaching from the side. His temperature is 36.7°C (98°F), pulse is 110/min. blood pressure is 100/75 mm Hg, and respirations are 12/min. Physical examination of the heart reveals an S1 and S2 heart sound. There is a grade l/IV systolic ejection murmur heard best at the apex. Pulmonary auscultation reveals good breath sounds bilaterally. Laboratory studies shows decreased thyroid-stimulating hormone (TSH), growth hormone (GH), and ACTH. Additional laboratory tests would most likely show which of the following? A. Elevated corticotropin-releasing hormone (CRH) B. Elevated cortisol C. Elevated insulin-like growth factor-l (IGF-1) D. Reduced prolactin E. Reduced thyroid-releasing hormone (TRH)

A. Elevated corticotropin-releasing hormone (CRH) This patient most likely has a pituitary adenoma that is compressing the anterior pituitary, and has most likely grown large enough to compress the optic chiasm, causing bitemporal hemianopsia (which could explain his "inattention" to a car approaching from the side). Most patients with pituitary adenomas present with signs and symptoms of hormone hypersecretion (hypercortisolism, excess growth hormone, hyperprolactinemia) Most pituitary adenomas are secretory, with prolactinoma being the most common in this category. However, ~30% of patients have nonfunctioning adenomas, as seen in our patient. These patients can present with neurologic symptoms resulting from mass effects (most commonly compression of the optic chiasm), with signs and symptoms of pituitary hypofunction due to compression of pituitary tissue by the tumor. In addition, "stalk effect" (compression of the infundibulum) occurs, resulting in elevated prolactin levels due to interruption of normal inhibition of prolactin by dopamine. Some cases can also be found as an incidental finding on imaging. One of the most common neurologic finding resulting from a mass effect is a visual field loss due to compression of the optic chiasm, potentially leading to a bitemporal hemianopsia. Patients presenting with neurologic symptoms may also have symptoms such as fatigue, lethargy, decreased libido, erectile dysfunction, and menstrual cycle disorders. The most common pituitary hormone deficiencies are growth hormone (GH), LH/FSH (hypogonadotropic hypogonadism), ACTH (secondary adrenal insufficiency), and TSH (secondary hypothyroidism). The reduction in ACTH would subsequently reduce cortisol secretion. Cortisol normally feeds back to inhibit further secretion of CRH and ACTH. Without an intact feedback system (see figure). CRH levels would be elevated (choice A).

A 55-year-old male with a history of interstitial pulmonary fibrosis undergoes pulmonary function testing. What parameter would most likely be decreased in this restrictive lung disease patient? A. FVC (forced vital capacity) B. Peak expiratory flow rate C. FEV1 (forced expiratory volume in 1 second) D. FEV1/FVC E. Fraction of expired O2 ​

A. FVC (forced vital capacity) Restrictive lung disease is associated with lung stiffening, which limits lung expansion. This manifests as a decrease in forced vital capacity (FVC) in pulmonary function tests. In practice, such patients can voluntarily inhale and exhale less air volume than a healthy person of comparable age, sex, and height. Peak expiratory flow rate and forced expiratory volume in 1 second (FEV1) may or may not be normal. The FEV1/FVC ratio is increased because FVC is typically reduced significantly. The fraction of expired O2 would not be changed by restrictive lung disease.

A 58-year-old woman is evaluated during a follow-up examination. The patient was evaluated in the emergency department 8 weeks ago because of progressively worsening epigastric pain. Upper endoscopy showed a 4-mm duodenal ulcer; biopsy specimens revealed mild gastritis with no evidence of malignancy Histologic evaluation of the tissue showed infection with Helicobacter pylori. She was treated with a 14-day course of omeprazole, clarithromycin, and amoxicillin followed by an additional 2 weeks of omeprazole alone. Her symptoms resolved at the completion of therapy. Medical history is otherwise unremarkable, and she is currently taking no medications. On physical examination, vital signs are normal. The abdomen is soft without mid-epigastric tenderness. The remainder of the examination is unremarkable. Which of the following is the most appropriate next step in management? A. Fecal H. pylori antigen testing B. Proton pump inhibitor therapy for at least 6 months C. Repeat upper endoscopy D. No additional testing or treatment ​

A. Fecal H. pylori antigen testing Educational Objective: Manage Helicobacter pylori infection in a patient with peptic ulcer disease. This patient should undergo fecal Helicobacter pylori antigen testing to verify eradication of the infection. The two most common causes of peptic ulcer disease are infection with H. pylori and the use of nonsteroidal anti-inflammatory drags (NSA!Ds). Once a diagnosis of H. pylori infection has been established, treatment followed by verification of eradication is essential. The initial diagnosis of H. pylori infection may be done by endoscopic studies (either histologic examination of biopsy tissue or rapid urease testing) or non-endoscopic studies (serum antibody tests, the urea breath test, or stool examination for H. pylori antigens). Because results of serum antibody studies remain positive after therapy, the urea breath test and fecal antigen testing are practical and noninvasive methods for detecting any residual H. pylori infection. The sensitivity of both tests is reduced when patients have recently taken a proton pump inhibitor (PPI) or antibiotic, and a period of 4 weeks without such drugs is recommended to optimize test accuracy. KEY POINT: Follow-up testing using the urea breath test or fecal Helicobater pylori antigen testing is indicated after treatment of H. pylori infection in a patient with peptic ulcer disease; testing should be performed at least 4 weeks following completion of proton pump inhibitor and antibiotic therapy.

A 49-year-old woman has a history of peptic ulcer disease for which she has been treated with proton pump inhibitors. She has had nausea with vomiting for the past 2 months. Upper GI endoscopy reveals three circumscribed, round, smooth lesions in the gastric body from 1 to 2 cm in diameter. Biopsies are taken and microscopically show the lesions to consist of irregular glands that are cystically dilated and lined by flattened parietal and chief cells. No inflammation, Helicobacter pylori, metaplasia, or dysplasia is present. What is the most likely diagnosis? A. Fundic gland polyps B. Gastric adenomas C. Hyperplastic polyps D. Hypertrophic gastropathy ​

A. Fundic gland polyps There is an association of fundic gland polyps with use of proton pump inhibitors and also with familial adenomatous polyposis (FAP); increased gastrin may drive glandular hyperplasia. Gastric adenomas are most common in the antrum, have intestinal metaplasia with dysplasia, and are precursors to adenocarcinoma; they may occur with FAP. Hyperplastic polyps are associated with chronic gastritis, often from H. pylori infection. One form of hypertrophic gastropathy is Menetrier disease, which results from excessive secretion of transforming growth factor alpha (TGF-α) with diffuse enlargement of gastric rugae and protein-losing enteropathy.

A 67-year-old man is evaluated in the emergency department for an explosive headache and blurred vision that began 4 hours ago. He reports a 3-month history of fatigue, weight gain (total, 4.5 kg [10 lb.]), and erectile dysfunction. The patient has a 2-year history of atrial fibrillation treated with warfarin and metoprolol. On physical examination, he is pale and appears uncomfortable. Blood pressure is 88/56 mm Hg, pulse rate is 88/min, and respiration rate is 18/min. Visual field examination reveals bitemporal hemianopia. The remainder of the examination is unremarkable. Results of laboratory studies are significant for a serum sodium level of 128 mEq/L (128 mmol/L) . A non-contrast computed tomographic scan of the head shows a heterogeneous sellar mass with suprasellar extension and bowing of the optic chiasm. In addition to neurosurgical consultation, which of the following is the most appropriate initial management? A. Glucocorticoid administration B. Levothyroxine administration C. Lumbar puncture D. Serum prolactin measurement

A. Glucocorticoid administration Educational Objective: Manage pituitary tumor apoplexy. This patient has acute bleeding into the pituitary gland (pituitary tumor apoplexy) and should receive glucocorticoids in addition to undergoing surgical removal of the tumor. His history of fatigue, weight gain, and erectile dysfunction and the laboratory finding of hyponatremia suggest panhypopituitarism, and his acute headache is consistent with hemorrhage. Pituitary tumor apoplexy usually occurs in the setting of a preexisting pituitary adenoma, and thus a neuroimaging scan was appropriately obtained to confirm the diagnosis and show the pituitary anatomy. He also has evidence of bitemporal hemianopia caused by optic chiasmal compression by the mass. The anticoagulant taken by this patient may have predisposed him to hemorrhage. Pituitary tumor apoplexy is generally a neurosurgical emergency. On occasion, hemorrhagic infarction of a pituitary adenoma may be less urgent, especially in the absence of associated mass effect, and can be managed with conservative follow-up monitoring. In the setting of local mass effect and severe headache, however, neurosurgical decompression of the pituitary gland is necessary. Urgent glucocorticoid administration is often required because of acute adrenocorticotropic hormone deficiency. The leading cause of death in patients with pituitary tumor apoplexy is adrenal insufficiency. Levothyroxine is not indicated at this time. Although pituitary tumor apoplexy may result in the loss of thyroid-stimulating hormone with subsequent thyroid hormone deficiency requiring treatment, replacement of adrenal hormone with glucocorticoids is indicated acutely. A lumbar puncture is useful in patients with suspected meningitis or in whom subarachnoid hemorrhage is suspected despite a negative imaging study. However, the imaging study already provides an explanation for this patient's stiff neck and headache, and a lumbar puncture is not only inappropriate but contraindicated in a patient with pituitary tumor apoplexy. Measurement of the serum prolactin level typically is obtained after a diagnosis of pituitary adenoma is made to exclude prolactinoma. Because this patient's disorder is a neurosurgical emergency, this test is inappropriate before the apoplexy is addressed. K E Y P O I N T: In addition to neurosurgical decompression of the pituitary gland, urgent glucocorticoid administration is often necessary in patients with pituitary tumor apoplexy because of acute adrenocorticotropic hormone deficiency.

A previously healthy 45-year old man is woken up at 0400 hrs with acute pain, swelling and redness of the right ankle with no obvious trigger factor. Height is 168 cm, weight 104 kg. He has a history hypertension treated with bendroflumethazide 2.5 mg daily. He drinks 2-3 pints of beer each night and consumes 26 units of alcohol per week. What is the most likely diagnosis? A. Gout B. Osteoarthritis C. Psoriatic arthritis D. Rheumatoid arthritis E. Septic arthritis ​

A. Gout The clinical presentation is typical of gout, given the acute onset, involvement of a single joint and risk factors of obesity, thiazide therapy and excessive alcohol intake. The pattern of involvement and acute onset is not consistent with rheumatoid arthritis, psoriatic arthritis or OA. Septic arthritis is possible but unlikely in the absence of a previous history of joint disease or site of infection. ​

The above patient undergoes upper GI endoscopy, which shows a 2-cm chronic ulcer on the lesser curve of the stomach with no stigmata of recent hemorrhage. The rest of the upper GI tract is normal. Which statement regarding his management is the most appropriate? A. He must have biopsies of the gastric ulcer to rule out malignancy and must have a repeat endoscopy 6-8 weeks later to confirm full healing after treatment B. He should avoid eating citrus fruits as these may delay healing of peptic ulcers C. He should be referred to a surgeon for consideration of highly selective vagotomy D. He should only have antral biopsies to check for presence of H. pylori infection, as gastric ulcers are usually benign E. There is no strong indication to stop smoking, as this has no impact on healing rates ​

A. He must have biopsies of the gastric ulcer to rule out malignancy and must have a repeat endoscopy 6-8 weeks later to confirm full healing after treatment Gastric ulcers may occasionally be malignant and therefore must always be biopsied and followed up to ensure healing. He should, of course, have antral biopsies to check for H. pylori and this should be eradicated if positive, but he should also have the ulcer edge biopsied. Surgery is no longer an option in the management of peptic ulcer disease unless there are severe complications such as gastric outlet obstruction, uncontrollable bleeding or perforation. Once a peptic ulcer forms, it is more likely to cause complications and less likely to heal if the patient continues to smoke. There is no specific dietary advice and citrus fruits have no relevance in this situation.

In patients who have chronic obstructive pulmonary disease and are being treated for an exacerbation that has caused hypercarbia despite antibiotics, corticosteroids, and inhaled bronchodilator therapy, the intervention that has been demonstrated to improve outcomes is: A. Noninvasive positive pressure ventilation (NIPPV). B. Administration of a continuous infusion of aminophylline. C. Administration of a respiratory stimulant such as progesterone. D. Pulmonary rehabilitation. E. Increasing the corticosteroid dose

A. Noninvasive positive pressure ventilation (NIPPV). NIPPV can shorten the hospital stay, reduce the need for endotracheal intubation and mechanical ventilation, and reduce mortality in individuals being treated for an exacerbation of chronic obstructive pulmonary disease complicated by hypercarbic respiratory failure.

A 75-year-old man is seen for routine follow-up for very severe COPD. He has constant dyspnea and air hunger and spends most of the day in a chair. He has had no change in baseline cough and sputum production. He has had multiple COPD exacerbations that required ICU admission and intubation. He has not benefited from pulmonary rehabilitation in the past. He quit smoking 3 years ago. His medical history is also notable for hypertension, type 2 diabetes mellitus, and a myocardial infarction 3 years ago. His medications are lisinopril, insulin glargine, budesonide/formoterol, tiotropium, roflumilast, as-needed albuterol, and 2 L of oxygen by nasal cannula. Spirometry performed 1 year ago showed an FEV1 of 21% of predicted and a DLCO of 35% of predicted. Residual volume/total lung capacity is 105% of predicted. On physical examination, the patient is very thin and demonstrates a significantly increased work of breathing. He is afebrile, blood pressure is 125/80 mm Hg, pulse rate is normal, and respiration rate is 32/min; BMI is 17. Oxygen saturation is 90% breathing 2 L of oxygen. Pulmonary examination reveals significantly decreased breath sounds, with no crackles or wheezing, and the remainder of the examination is unremarkable. Laboratory studies reveal a serum albumin level of 2.3 g/dL (23 g/L). Arterial blood gas studies reveal a PCO2 of 55 mmHg (7.3 kPa). Chest radiograph shows no acute changes. Echocardiogram shows normal left ventricular function; the estimated pulmonary artery pressure is elevated, suggesting cor pulmonale. CT scan shows diffuse emphysema. Which of the following is the most appropriate management? A. Hospice referral B. Lung transplantation C. Lung volume reduction surgery D. Repeat pulmonary rehabilitation

A. Hospice referral Educational Objective: Manage advanced COPD with referral to hospice care. Key Point: In patients with COPD, parameters that portend a poor prognosis and trigger more extensive discussions regarding end-of-life care include an FEV1 of less than 30% of predicted, oxygen dependence, multiple hospital admissions for COPD exacerbations, significant comorbidities, weight loss and cachexia, decreased functional status, and increasing dependence on others. The most appropriate management is to refer this patient for hospice care. Hospice is considered appropriate for patients in whom attempted curative therapy is not likely to be beneficial, and specifically in those who are predicted to have less than 6 months to live. In hospice care, treatment goals are refocused from cure and life-prolonging therapy toward maintaining the highest possible quality of life. In patients with COPD, parameters that portend a poor prognosis and trigger more extensive discussions regarding end-of-life care include an FEV1 of less than 30% of predicted, oxygen dependence, multiple hospital admissions for COPD exacerbations, significant comorbidities, weight loss and cachexia, decreased functional status, and increasing dependence on others. This patient has very severe COPD, cor pulmonale, decreased functional capacity, constant dyspnea and air hunger, poor nutritional status, and a history of multiple COPD exacerbations on maximal medical therapy and home oxygen. Given these factors and an overall poor prognosis, discussion of a hospice approach to care would be an appropriate next step in management. In any patient with advancing COPD, it is important to have ongoing discussions regarding the goals of care as the disease progresses so that appropriate management decisions may be made based on anticipated outcomes and patient values and preferences. Involvement of clinicians trained in palliative care may be helpful in these discussions. Palliative care focuses on the many implications of any significant illness with an emphasis on establishing patient-centered goals of care and symptom management. Palliative care does not preclude active treatment of disease and is appropriate regardless of estimated survival time. Although this patient might meet several of the criteria for possible lung transplantation, most transplant centers use 65 years of age as an arbitrary cutoff, and the presence of other comorbidities such as diabetes mellitus, coronary artery disease, and osteoporosis should also be considered. This patient's age and comorbid conditions likely would preclude lung transplantation. Lung volume reduction surgery (LVRS) is indicated for patients with severe COPD with predominant upper lobe emphysema who are symptomatic despite optimal medications. Because this patient has diffuse emphysema, he is not a candidate for LVRS. This patient has completed pulmonary rehabilitation in the past without improvement, and chair-bound patients may not benefit from it; therefore, pulmonary rehabilitation is not an appropriate option for this patient.

A 60-year-old woman is brought to the hospital due to lethargy and confusion that have progressed over the last 2 days. This morning she was hard to arouse. The patient's husband says that she has lost 4.5 kg (10 lb) over the preceding 2-month period and has had a dry cough. She has no other medical problems and takes no medication. The patient has a 40-pack-year smoking history. She does not drink alcohol. Temperature is 37 C (98.6 F), blood pressure is 110/70 mm Hg; pulse is 75/min, and respirations are 16/min. She is not orthostatic. The patient is markedly confused. Her mucous membranes are moist. There is no jugular venous distension. Breath and heart sounds are normal. Neurologic examination shows no focal abnormalities. There is no ankle edema or ascites. Laboratory results are as follows: Sodium 117 mEq/L Potassium 3.9 mEq/L Bicarbonate 22 mEq/L Blood glucose 100 mg/dL Blood urea nitrogen 10 mg/dL Serum osmolality Is 250 mOsm/kg H2O; and urine osmolality Is 500 mOsm/kg H2O. Chest x-ray reveals a mass in the right hilar region. CT scan of the head is unremarkable. Which of the following is the best next step in management of this patient? A. Hypertonic saline B. Intravenous 0.45% solution C. Intravenous desmopressin D. Intravenous hydrocortisone E. Mannitol therapy F. Normal saline G. Sodium bicarbonate therapy

A. Hypertonic saline This patient's chest x-ray findings of a hilar mass in the setting of weight loss, cough, and a significant smoking history are highly suspicious for small cell lung cancer, which is a common cause of the syndrome of inappropriate antidiuretic hormone secretion (SIADH). The characteristic findings of SIADH are hyponatremia, serum osmolality <275 mOsm/kg H2O and urine osmolality >100 mOsm/kg H2O (inappropriately elevated) in a euvolemic patient. The symptoms of SIADH depend on the degree of hyponatremia and the rate at which it occurred. Rapid decreases (over <48 hours) in serum sodium concentration lead to profound fluid shifts and severe symptoms. Patients with serum sodium 130-135 mEq/L are usually asymptomatic; those with chronic hyponatremia with serum sodium 120-130 mEq/L may be asymptomatic or display mild symptoms (lethargy, forgetfulness). These patients usually respond to fluid restriction (<800 mL/day), sometimes with the addition of salt tablets. Patients with serum sodium <120 mEq/L may have severe symptoms (e.g., profound confusion, seizures, coma), which signal an increased risk of cerebral edema and brainstem herniation. These patients require an urgent increase in their serum sodium concentration that is best accomplished with administration of hypertonic (3%) saline. Extreme care must be taken with the rate of correction (<8 mEq/L over the first 24 hours) as rapid correction can lead to osmotic demyelination syndrome, a devastating and potentially fatal complication (e.g., dysarthria, quadriparesis, coma). Educational objective: Patients with the syndrome of inappropriate antidiuretic hormone secretion who are asymptomatic or have mild symptoms usually respond to fluid restriction. Patients with severe symptoms require treatment with hypertonic (3%) saline.

A 45-year-old woman comes to the physician with a 3-month history of exertional dyspnea and nonproductive cough. She has also been having difficulty swallowing. Her other medical problems include Raynaud phenomenon, heartburn, and poorly controlled hypertension.. The skin over her hands, arms, and trunk is thick and firm. Which of the following is the most likely cause of her pulmonary complaints? A. Interstitial lung disease B. Parenchymal granulomatous nodules C. Pleural effusion D. Pulmonary arterial hypertension E. Restricted chest wall movement

A. Interstitial lung disease This patient with dysphagia, Raynaud phenomenon, and extensive skin thickening, most likely has diffuse systemic sclerosis (SSc). SSc displays extensive heterogeneity in manifestations and severity, though some degree of skin thickening is seen in almost all patients. Patients with sclerosis limited to the skin of the hands, wrists, and face (limited SSc) may have esophageal dysmotility and vascular symptoms (e.g., Raynaud phenomenon), but extradermal manifestations are otherwise limited in most cases. In contrast, diffuse cutaneous SSc, with dermal thickening involving the proximal extremities or trunk, is much more likely to cause extradermal complications and carries a more severe prognosis. A wide range of pulmonary complications can occur in patients with SSc. They affect up to 30% of patients, and are the leading cause of death in SSc. The most common pulmonary complication in patients with diffuse SSc is interstitial fibrosis, which develops in about 40% of patients. Educational objective: Patients with diffuse systemic sclerosis (SSc) frequently have extradermal organ involvement, especially of the lungs. The most common pulmonary complication in patients with SSc is interstitial fibrosis.

The most common metabolic disorder after gastric resection is a deficiency of: A. Iron B. Vitamin B12 C. Folate D. Calcium E. Vitamin D ​

A. Iron Gastric resection leads to numerous disturbances in metabolism. These include deficiencies of iron, vitamin B12 (B), folate (C), fat-soluble vitamins (E), and calcium (D). Of these, iron deficiency is the most common. Iron is absorbed in the duodenum and is facilitated by an acidic environment. After gastric resection, overall iron intake is decreased, and the reduced acidity impairs absorption. Reduction in the parietal cell mass from gastric resection leads to a decrease in intrinsic factor, which is necessary for the enteric absorption of vitamin B12, occurring in the terminal ileum. This leads to a megaloblastic anemia. Furthermore, an acidic environment facilitates the bioavailability of vitamin B12. Vitamin B12 deficiency usually only develops when at least one-half of the stomach is resected. Fat malabsorption can occur after gastrectomy (particularly with a Billroth II reconstruction) because of inadequate mixing of food with bile and digestive enzymes. This leads to a decreased absorption of fat-soluble vitamins. Calcium is absorbed in the duodenum and small bowel and is also facilitated by an acid environment. Long-term deficiencies manifest as osteoporosis. Folate deficiency is rare.

A 52-year-old woman is evaluated for a 6-year history of Sjogren syndrome. During the past 3 months, she has had low-grade fevers up to 37.5 °C (99.5 °F), weight loss of 6.8 kg (15 lb), and increased fatigue and sicca symptoms. She recently noted a rash on her legs. She reports no current joint pain. Medications are hydroxychloroquine and acetaminophen as needed. On physical examination, temperature is 37.2 °C (99.0 °F), blood pressure is 135/85 mm Hg, and pulse rate is 82/min. BMI is 28. The oral mucosa is dry. Bilateral parotid fullness is present. There is bilateral cervical adenopathy. The tip of the spleen is palpable. There are a few scattered palpable purpura on the lower legs. The remainder of the physical examination is unremarkable. Laboratory studies show a normal complete blood count except for a hemoglobin level of 11 g/dL (110 g/L); serum C3 and C4 levels are low, and serum and urine protein electrophoresis reveals M-component. Chest radiograph and echocardiogram are normal. CT scan of the abdomen shows numerous enlarged retroperitoneal lymph nodes and splenomegaly. Which of the following is the most appropriate next step in the management of this patient? A. Obtain a lymph node biopsy B. Obtain a skin bbiopsy C. Order heterophile antibody testing D. Start prednisone E. Start prednisone and cyclophosphamide

A. Obtain a lymph node bbbiopsy Educational Objective: Recognize lymphoma in a patient with Sjogren syndrome. Patients with Sjogren syndrome have up to a 44-fold higher risk of developing lymphoma, the most common being diffuse B-cell and mucosa-associated lymphoid tissue (MALT) lymphomas. The most appropriate next step in the management of this patient is to obtain a lymph node biopsy. This patient has Sjogren syndrome, an immune-mediated disease manifesting primarily as inflammation of exocrine glands, including the major and minor salivary glands, lacrimal glands, and, less commonly, other exocrine glands such as the pancreas. Patients with Sjogren syndrome are at significant risk of developing lymphoma, the most common being diffuse B-cell and mucosa- associated lymphoid tissue (MALT) lymphomas; this risk is up to 44-fold higher than in the general population. It is thought that chronic B-cell activation may lead to the development of a clone of malignant B cells. Hypocomplementemia, splenomegaly, lymphadenopathy, gammopathy, skin vasculitis, and cryoglobulinemia predict the development of, and accompany, lymphoma. With this patient's clinical history and findings, lymphoma must be in the differential and should be evaluated with a lymph node biopsy.

A 43-year-old woman presents to the emergency department complaining of confusion. Her other complaints include Increased thirst and "using the bathroom more frequently." She denies any fever, chills, headache, chest pain, shortness of breath, or cough. Her past medical history is significant for bipolar disorder that is well-controlled by medication. She does not drink alcohol, smoke cigarettes, or use illicit drugs. Her vital signs are stable, and physical exam is unremarkable. Laboratory studies reveal the following: Sodium 154 mEq/L Potassium 4.1 mEq/L Chloride 116 mEq/L Bicarbonate 28 mEq/L Glucose 95 mg/dL Urine osmolality 250 mQsm/L Plasma osmolality 326 mOsm/L What is the most likely cause of this patient's symptoms and laboratory findings? A. Lithium B. Divalproic acid C. Craniopharyngioma D. Head trauma E. Psychogenic polydipsia

A. Lithium Explanation: Diabetes insipidus (DI) is a leading cause of euvolemic hypernatremia. It typically presents with severe polyuria and mild hypernatremia, it can be divided into two types based on urine osmolality, as well as etiology. Based on urine osmolality, DI may be complete or partial Complete DI - the urine osmolality is less than 300 mOsm/kg (often less than 100 mOsm/kg) Partial DI - urine osmolality ranges from 300-600 mOsm/kg. The serum osmolality is elevated in both types. Based on etiology DI may be central or nephrogenic. Central DI is due to decreased production of antidiuretic hormone (ADH). Common causes include trauma, hemorrhage, infection, and tumors. Nephrogenic DI results from renal ADH resistance. Common causes include hypercalcemia, severe hypokalemia, tubulointerstitial renal disease, and medications. The most commonly implicated mediations are lithium, demeclocycline, foscarnet, cidofovir, and amphotericin. In this vignette, the patient's clinical history, presentation and laboratory findings are suggestive of nephrogenic diabetes insipidus most likely caused by lithium, which is one of the first-line drugs for bipolar disorder. Educational objective: Lithium is a common cause of nephrogenic diabetes insipidus. Lithium-induced nephrogenic DI Is treated with salt restriction and discontinuation of lithium.

A 24-year-old man is evaluated for shortness of breath with occasional wheeze that occurs several times per week. He develops significant shortness of breath when playing tennis or soccer outdoors, particularly during cool weather. He notes that it takes several minutes to recover his breath when this occurs. He also notes coughing during exercise as well as episodes of cough in the evenings and nighttime, even when not exerting himself. Medical history is significant only for seasonal allergies as a child, although he now rarely experiences allergy symptoms. He takes no medications. On physical examination, vital signs are normal. Mild nasal congestion is noted. Lung examination reveals normal air movement and no wheezes. Heart examination is unremarkable, and the remainder of the examination is normal. A chest radiograph is normal. Spirometry findings are within normal parameters. Which of the following is the most appropriate next step in management? A. Methacholine challenge test B. Nasal glucocorticoid C. Short-acting β2-agonist inhaler as needed D. Clinical observation

A. Methacholine challenge test Educational Objective: Diagnose exercise-induced bronchospasm and possible asthma with bronchial challenge testing. Key Point: In patients with symptoms of exercise-induced bronchospasm and/or asthma but normal spirometry findings, bronchial challenge testing is warranted for further evaluation. The most appropriate next step in management is a methacholine challenge test. Testing will confirm the degree to which this patient's symptoms are caused by hyperreactivity of the lungs. The frequency and degree of this patient's symptoms are concerning, and he has several risk factors and clinical factors (such as a history of allergies, cough without exercise, and symptoms occurring several times a week) that suggest the possibility of asthma as the underlying diagnosis. Though some patients with asthma wheeze with exercise, it is not a necessary symptom to consider the diagnosis; exercise intolerance, breathlessness, and cough can be the primary symptoms as well. Because of the significant morbidity and mortality associated with undiagnosed and untreated asthma, further evaluation of this possibility is indicated in this patient. Because spirometry results are normal, which can occur in patients with asthma, further confirmatory testing with a bronchial challenge test (such as with methacholine) is warranted to further evaluate for possible asthma. Managing only this patient's allergic rhinitis with a nasal glucocorticoid would not treat possible underlying asthma adequately to protect him from nighttime cough and shortness of breath during exercise. However, counseling avoidance of exercise in locations with allergens would be appropriate. Although a therapeutic trial of an as-needed p2-agonist inhaler may seem reasonable, a more extensive assessment and proper diagnosis of this patient's symptoms concerning for asthma will ensure that adequate medications are prescribed for chronic management. In patients with asthma, spirometry results between episodes are often normal. Additionally, this patient is experiencing frequent and disruptive symptoms. Therefore, clinical observation without further evaluation would not be appropriate.

A 52-year-old man is hospitalized for an acute COPD exacerbation and agrees to treatment with 3 days of IV methylprednisolone followed by a taper of oral prednisone over 10 days. Which of the following is this patient most likely to experience as a potential adverse effect of these medications? A. Muscle pain and weakness B. Serious infections C. Skin atrophy and purpura D. Psychiatric disturbances E. Hypertension F. Hyperglycemia G. Osteoporosis H. Chronic adrenal suppression ​

A. Muscle pain and weakness Systemic corticosteroids have a long list of adverse effects when used chronically; however, there is debate as to the frequency of adverse effects when used over a short period of time. All of the answer choices are potential consequences of chronic therapy, but the one answer choice that most commonly occurs with a short course of steroids is hyperglycemia. Other notable side effects of chronic steroids (i.e., iatrogenic Cushing syndrome) are cataracts, dyslipidemia, pancreatitis, hypokalemia, and VZV reactivation (herpes zoster).

A 65-year-old male comes to the emergency department with increasing respiratory distress over the past 2 days. He is a smoker and has known COPD, for which he has been hospitalized twice in the past year for exacerbations. The patient appears cyanotic, in moderate distress utilizing accessory musculature and perched forward sitting on the stretcher. He is alert and reports feeling "better" since getting to the emergency department. Lung sounds are quite diminished in all lung fields, and an ABG on room air shows pH 7.30, PCO2 63 mmHg, and a PO2 of 44 mmHg. The respiratory therapist is awaiting your orders. Your orders for initial oxygen therapy should be which of the following: A. Nasal oxygen, 1-2 L/min with follow-up ABG in 20 minutes. Increase oxygen flow based on the ABG to a PO2 of 55-60 mmHg. B. Intubate the patient and place on mechanical ventilation with an initial FiO2 of 100%. C. O2 100% by non-rebreather face mask with continuous pulse oximetry. D. Nasal oxygen initially, 3-4 L/min with pulse oximetry to achieve a saturation of 92% or greater. E. Withhold oxygen therapy at this time since patient is stable and you do not want to induce further respiratory acidosis. F. Institute BiPAP therapy because patient is already showing signs of inspiratory fatigue and respiratory acidosis.

A. Nasal oxygen, 1-2 L/min with follow-up ABG in 20 minutes. Increase oxygen flow based on the ABG to a PO2 of 55-60 mmHg The treatment imperative is to provide this patient with supplemental oxygen to achieve a hemoglobin saturation of 88-90%. This is to prevent further hypoxemia, which can cause cardiac arrhythmias and acute cor pulmonale. Oxygen supplementation may result in worsening respiratory acidosis. (HINT: not through reduction in respiratory drive—COPD patients are just like the rest of us in that our chemiotaxic centers for the control of ventilation are CO2 sensitive, not O2 driven!) Worsening respiratory acidosis can be detected only by ABGs, not pulse oximetry. If providing supplemental FiO2 to achieve an oxygen saturation of 88-90% results in severe respiratory acidosis, then positive pressure ventilation via non-invasive methods or conventional ventilation must be considered. Board Testing Point: Recognize how to treat a severe COPD exacerbation. ​

A 17-year-old girl presents to the emergency department with a 1-month history of fevers, joint pain, and generalized weakness. Physical examination shows mild symmetric arthritic changes in the fingers, generalized mild muscle weakness in the legs and arms, and an erythematous rash over the cheeks and nose. Serum testing demonstrates anti-dsDNA antibodies. Which of the following are all associated with the disease process affecting this patient? A. Oral aphthous ulcers, photosensitivity, pericarditis, and seizures B. Oral ulcers, uveitis, toxic megacolon, and arthritis C. Proximal muscle weakness, violaceous periorbital rash, occult malignancy D. Telangiectasias, esophageal dysmotility, and subcutaneous calcium nodules E. Uveitis, hilar lymphadenopathy, and subcutaneous nodules ​

A. Oral aphthous ulcers, photosensitivity, pericarditis, and seizures This patient suffers from systemic lupus erythematosus (SLE). SLE is an autoimmune disorder with a variable effect on multiple organs and organ systems, including skin, joints, kidneys, brain, and heart. The most common early presentation signs include fevers, arthritis, arthralgias, malar rash, and pleuritis. A useful mnemonic to remember SLE signs and symptoms is DOPAMINE RASH: Discoid rash, Oral ulcers, Photosensitivity, Arthritis, Malar rash, Immunologic criteria, Neurologic symptoms (including seizures), Elevated erythrocyte sedimentation rate. Renal disease, Antinuclear antibody positive, Serositis (including pericarditis), and Hematologic abnormalities. Bottom Line: Remember the signs and symptoms of SLE using DOPAMINE RASH: Discoid rash, Oral ulcers, Photosensitivity, Arthritis, Malar rash, Immunologic criteria, Neurologic symptoms, Elevated erythrocyte sedimentation rate, Renal disease, Antinuclear antibody positivity, Serositis, and Hematologic abnormalities. ​

A 65-year-old woman presents with complaints of increasing pain in her fingers and hands. She states that she has morning stiffness in those regions that tends to resolve within 10 to 15 minutes after start of activity. There are no other rheumatic or systemic symptoms, and physical examination of the hands is completely normal. Which of the following is the most likely clinical diagnosis? A. Osteoarthritis B. Rheumatoid arthritis C. Fibromyalgia D. Scleroderma/systemic sclerosis E. Systemic lupus erythematosus (SLE) ​

A. Osteoarthritis Morning stiffness that resolves in 10 to 15 minutes is most characteristic of osteoarthritis. The morning stiffness of rheumatoid arthritis and other inflammatory rheumatic diseases typically lasts longer: at least 1 hour and even for much of the day. Hand involvement with scleroderma/systemic sclerosis is characteristically associated with initial swelling of the digits, then progressing to skin tightness, which was not observed in this patient. Fibromyalgia involves widespread body pain of both sides, including upper and lower extremities, back, and neck. This would be an unusual age for presentation with SLE, which typically begins in women in the age range of 15 to 45 years ​

A 20-year-old woman is admitted to the hospital for an elective adrenalectomy. Her history is significant for recent onset of hypertension, weight gain, generalized weakness, and easy bruising. Laboratory evaluation demonstrated an elevated urine free cortisol and lack of suppression of serum cortisol following a low-dose overnight dexamethasone suppression test. Subsequent testing revealed a suppressed adrenocorticotropic hormone level. A contrast-enhanced adrenal CT scan revealed a well-circumscribed 3.7- cm right adrenal mass with a contrast washout of greater than 50% at 10 minutes. Which of the following is the most appropriate perioperative management? A. Postoperative hydrocortisone B. Postoperative mitotane C. Postoperative norepinephrine D. Preoperative phenoxybenzamine

A. Postoperative hydrocortisone Educational Objective: Manage a patient with Cushing syndrome following adrenalectomy. Key Point: Following adrenalectomy, patients with adrenal Cushing syndrome should be treated with stress-dose glucocorticoids during the postoperative period to avoid the risk of acute hypocortisolism. The most appropriate management of this patient undergoing right adrenalectomy for Cushing syndrome (CS) is postoperative hydrocortisone. The patient has adrenocorticotropic hormone (ACTH)- independent CS, and a contrast-enhanced adrenal CT scan demonstrated a right adrenal mass with imaging characteristics consistent with a benign adenoma. Adrenocortical adenomas typically have low attenuation on unenhanced CT scan (density less than 10 Hounsfield units) and exhibit rapid washout of intravenous iodine contrast media (>50% at 10 minutes). Following adrenalectomy, patients with adrenal CS may develop acute adrenal failure because of hypothalamic-pituitary-adrenal (HPA) axis suppression and contralateral adrenal atrophy. All patients should be treated with stress-dose glucocorticoids and tapered to physiologic replacement until HPA axis recovery is confirmed. Most patients have adrenal insufficiency lasting as long as 12 months. Postoperative administration of mitotane, an adrenolytic drug, is recommended as adjuvant therapy for patients with locally persistent or metastatic adrenocortical carcinoma (ACC). In these patients, mitotane is associated with objective remissions in approximately 25% of patients. This patient's CT scan findings are not consistent with ACC. Adrenal cancers are typically large (>4-6 cm) with irregular margins and areas of necrosis or calcification. Unenhanced CT will demonstrate high attenuation (density >10 Hounsfield units), and washout of intravenous iodine contrast media is less than 50% at 10 minutes. Postoperative norepinephrine is not indicated. If the patient were to experience hypotension or shock postoperatively, treatment with vasopressors in addition to glucocorticoid replacement would be considered; however, this would not replace the administration of glucocorticoids in this population at risk for acute hypocortisolism. Preoperative phenoxybenzamine is indicated in the management of patients with pheochromocytoma, not CS. The purpose of preoperative α-blockade is to provide blood pressure control and decrease the risk of cardiovascular complications related to excessive catecholamine release during intraoperative manipulation of the tumor.

A 38-year-old male postgraduate student from Hong Kong presents to his general practitioner (GP) complaining of sudden-onset left ear pain, and with an inability to sleep on the affected side. He reports that he is feeling very low because he has had a number of health problems since coming to this country 6 months ago. On further questioning, he admits to having general aches and pains, including pains in various joints over this time. He reports that both his eyes are red all the time but that antiallergy drops bought from the chemist have had no effect. He has also had a couple of nose bleeds, which is a first for him and which he blames on the increased pollution in the air, which he also feels is responsible for the hoarseness in his voice. He denies any known medical problems and, apart from analgesics, he is not on any regular medication. Positive findings on clinical examination include left auricular chondritis, bilateral eyelid edema and bilateral mild episcleritis, tenderness to palpation over the anterior trachea and pain on palpation over the costochondral joints. The GP orders some blood tests and X-rays, including rheumatoid factor, which is negative. What is the most likely diagnosis? A. Relapsing polychondritis B. Reiter's syndrome C. Wegener's granulomatosis/granulomatosis with polyangitis (GPA) D. Cogan's syndrome E. Polyarteritis nodosa

A. Relapsing polychondritis This young man has inflammation of cartilaginous structures - the pinna (red, painful ear), nasal septum (epistaxis) and larynx (hoarse voice), as well as costochondritis and the C-rings of the trachea, consistent with relapsing polychondritis (RP). There is no gender difference in the distribution of the disease, and although it can occur at any age it is most common in the fifth decade. The patient's reported symptoms depend on the area of the body affected. General symptoms include intermittent fever and weight loss, but other more specific symptoms include sudden onset of ear pain with an inability to sleep on the affected side, diminished hearing, mono- or polyarthritis, back pain, myalgias, mild epistaxis, saddle-shaped nose, redness of the eyes indicative of conjunctivitis, episcleritis and/or scleritis, hoarseness of the voice and recurrent respiratory infections. There are no specific diagnostic laboratory findings in patients with RP, but the non-specific indicators of inflammation (ESR, CRP) are often elevated. No specific therapy exists for RP, but the goal of treatment is to control the patient's symptoms and to preserve the integrity of the affected cartilaginous structures. The mainstay of treatment is systemic corticosteroids. It is important in these patients to investigate for the presence of other concurrent autoimmune diseases.

A patient with a history of pancreatitis develops severe bleeding from gastric varices. You do not see any esophageal varices on EGD. U/S shows splenic vein thrombosis. The portal vein is patent. The best Tx for this patient's most likely condition is: A. Splenectomy B. TIPS procedure C. Spleno-renal shunt D. Porto-caval shunt ​

A. Splenectomy Gastric varices without esophageal varices are most likely from splenic vein thrombosis related to pancreatitis and not cirrhosis. U/S or CT scan are used to demonstrate splenic vein thrombosis. Tx for symptomatic isolated gastric varices related to splenic vein thrombosis is splenectomy Asymptomatic splenic vein thrombosis should be left alone. ​

A 65-year-old former smoker with COPD of Global Initiative for Chronic Obstructive Lung Disease grade 3, group D, is referred to the intensive care unit because of an acute exacerbation of his disease, presenting with increased dyspnea and purulent sputum. Despite inhalation of salbutamol, intravenous antibiotics and corticosteroids, his condition worsens gradually over 30 min. He has not eaten or drunk for the last 5 h. On arterial blood gas analysis, pH is 7.25, PaO2 is 6.6 kPa (49.5 mmHg) and PaCO2 is 8.0 kPa (60 mmHg). He is agitated but cooperates with inhalation and opens his eyes on request. His respiratory rate is 26 breaths per min. Which therapeutic option is most appropriate in this situation? A. Start NIV and oxygen B. Start CPAP and oxygen C. Intubate and start mechanical ventilation D. Add salbutamol i.v. and low-dose (2.5 mg) morphine i.v. E. Add inhaled short-acting anticholinergic and low-dose (2.5 mg) morphine i.v. ​

A. Start NIV and oxygen NIV should be used in exacerbations of COPD when pH is <7.35 and PaCO2 is >6.0 kPa (>45 mmHg). Although the case presented here shows severe acidotic decompensation, a NIV trial is warranted under intensive care unit conditions, as even a failed trial of NIV leading to a delayed endotracheal intubation does not lead to higher mortality. The efficacy of NIV in treating acute exacerbations of COPD was studied in a European, randomised, multicentre study conducted in 85 COPD patients assigned to receive conventional treatment (oxygen therapy plus drugs) or NIV. The group of patients treated with NIV had fewer intubations (26% versus 74%, p<0.001), fewer complications (14% versus 45%, p<0.01), shorter length of hospital stay (23±17 versus 35±33 days, p<0.02) and lower mortality (9% versus 29%; p<0.02). CPAP has not been extensively studied in COPD exacerbations. CPAP would not actively support ventilation so hypercapnia would not be expected to improve; therefore, bilevel NIV is preferred.

A 38-year-old woman presents to the hospital with fever, cough, and shortness of breath. On imaging, a lobar pneumonia is confirmed, but a lung mass is also noted. She is treated with antibiotics and at a later time the mass is biopsied via bronchoscopy. Eventually the patient is discharged to follow up as an outpatient. The biopsy report suggests a benign lesion, and the patient agrees to have the lesion followed with imaging. Several months later, the patient presents with difficulty breathing for a few weeks. Her vitals are normal, but inspiratory and expiratory stridor is heard along with rhonchi on lung auscultation. There are no wheezes or rales. Examination of the oropharynx is unremarkable. Spirometry with flow-volume loops shows a plateau during inspiration and expiration, with decreased peak inspiratory and expiratory flow. What is the most likely diagnosis? A. Subglottic stenosis B. Carcinoid tumor C. Viral bronchiolitis D. Postobstructive pneumonia ​

A. Subglottic stenosis. Auscultation of lung sounds helps to define the site of pathology within the airway anatomy. Rhonchi are low pitched, sonorous sounds that typically indicate secretions in the upper airway. High-pitched sounds as a result of upper airway obstruction are termed stridor. Wheezes are high-pitched musical sounds caused by narrowing of bronchioles. Rales, also known as crackles, come in two types (wet and dry), and the pathology in both types involve the distal airway. Wet rales are caused by fluid accumulation within alveoli, which overwhelms the mechanism of surfactant to decrease surface tension and causes the alveoli to collapse and open back up during the end of inspiration; dry rales (velcro sound) typically involve the smaller airways leading into the alveoli and are usually due to interstitial processes such as pulmonary fibrosis. This patient has stridor, which suggests that the problem involves the upper airway. In addition, the reduction in peak inspiratory and expiratory flow with plateaus seen on the flow-volume loops are indicative of a fixed obstruction. The only answer choice that fits with a fixed obstruction of the upper airway is subglottic stenosis, which can be congenital or acquired from trauma during procedures such as bronchoscopy. The rhonchi are likely a result of excess secretion build up below the obstruction. It is likely that the tumor is indeed benign and is not responsible for this patient's symptoms. (B, C) A carcinoid tumor and viral bronchiolitis would both produce wheezing, which is not heard in this patient. (D) Postobstructive pneumonia could occur in this patient given that she has a lung mass near the airway, however this would produce rales and not stridor.

Which statement regarding gastrointestinal stromal cell tumors (GISTs) is correct? A. They are differentiated from other mesenchymal tumors by expression of the c-kit proto-oncogene B. They are invariably benign and do not require any specific management C. They are particularly aggressive and require resection and treatment with imatinib (a tyrosine kinase inhibitor) D. They arise from the interstitial cells of Lieberkuhn E. They only bleed if patients also take NSAIDs ​

A. They are differentiated from other mesenchymal tumors by expression of the c-kit proto-oncogene GISTs arise from the interstitial cells of Cajal. They are differentiated from other mesenchymal tumors by expression of the c-kit proto-oncogene. They are usually benign tumors, particularly the smaller lesions < 2 cm, and asymptomatic, but the larger ones may have malignant potential. Very large lesions should be treated pre-operatively with imatinib to reduce their size and make surgery easier. Imatinib can also be used for prolonged control of metastatic GISTs. They can bleed independently of NSAIDs.

A 54-year-old man presents with a 2-month history of increasing lethargy. He also reports that he has been bruising more easily of late. On direct questioning he admits to passing dark urine, but denies any other urinary symptoms. A year ago, he had finished a 6-month period of anticoagulant therapy for a spontaneous deep vein thrombosis. His full blood count demonstrates hemoglobin 6.5 g/dl, white blood-cell count 2.8 x 109/1, and platelets 27 x 109/1. What is the most appropriate management for his anemia? A. Androgen 'hormonal' therapy B. Erythropoietin injections C. Folic acid therapy D. Transfusion with blood substitutes E. Transfusion with red cells ​

A. Transfusion with red cells Transfusion with red cells is the best way to manage this man's anemia. Essentially, this man is pancytopenic. He has a background history of a deep vein thrombosis, and this combination should suggest a diagnosis of paroxysmal nocturnal hemoglobinuria (PNH). This can also be associated with aplastic anemia, and a bone marrow examination should certainly be performed to diagnose this. There is historical literature to suggest that washed red cells was a superior transfusion product for PNH patients. However, there is no evidence that survival of washed red cells is better than that of standard red cells. ​

A 48-year-old man presents with complaints of dyspnea that tends to occur with exercise. He has a 40-pack-year history of smoking and has been diagnosed with exercise-induced asthma in the past, but denies any other medical problems. On spirometry, his expiratory loop is normal, but he has a flattened inspiratory loop. What is the most likely diagnosis? A. Vocal cord dysfunction B. COPD C. Asthma D. Restrictive lung disease

A. Vocal cord dysfunction Vocal cord dysfunction occurs when the vocal cords move toward midline during inspiration or expiration, leading to varying degrees of obstruction. It is often misdiagnosed as exercise-induced asthma. Precipitating factors, including exercise, psychological conditions, irritants, sinusitis, and gastroesophageal reflux disease. Spirometry generally will show a normal expiratory loop with a flattened inspiratory loop. In asthma and COPD, the FEV1/FVC ratio is decreased, resulting in a concave shape in the expiratory portion of the flow-volume curve. The inspiratory loops are generally normal. Patients with restrictive lung disease have a normal FEV1/FVC ratio with a reduced FVC.

A 35-year-old man is evaluated in the clinic for symptoms of shortness of breath. He reports no other lung or heart disease. He smokes half pack a day for the past 10 years. On examination, his JVP is at 2 cm, heart sounds normal, and lungs are clear. A CXR shows hyperinflation and increased lucency of the lung fields. A chest CT reveals bullae and emphysematous changes in the lower lobes, while pulmonary function tests show an FEV1/FVC ratio of <70%. Evaluation of his family reveals other affected individuals. Which of the following is the most likely diagnosis? A. alpha1-antitrypsin deficiency B. beta-glycosidase deficiency C. glucose-6-phosphatase deficiency D. glucocerebrosides deficiency E. growth hormone deficiency ​

A. alpha1-antitrypsin deficiency Most people have two MM genes and a resultant alpha1-antitrypsin level in excess of 2.5 g/L. Homozygotes with ZZ or SS genotypes have severe alpha1-antitrypsin deficiency and develop severe panacinar emphysema in the third or fourth decade of life. Smoking is an important cofactor in the development of disease. Heterozygotes (MZ or MS) have intermediate levels of alpha1-antitrypsin (ie, genetic expression is that of an autosomal codominant allele). This heterozygous state is common (5-14% of general population), but it is unclear whether it is associated with lung function abnormalities.

A 39-year-old woman is evaluated for anemia. Her laboratory studies reveal a hemoglobin of 7.4 g/dL, hematocrit of 23.9%, mean corpuscular volume of 72 fL, mean cell hemoglobin of 25 pg, and mean cell hemoglobin concentration of 28%. The peripheral smear is shown in Figure. Which of the following tests is most likely to be abnormal in this patient? A.Ferritin B.Haptoglobin C.Hemoglobinelectrophoresis D. Glucose-6-phosphate dehydrogenase E. Vitamin B12 ​

A.Ferritin This patient with anemia demonstrates a low mean cell volume, low mean cell hemoglobin, and low mean cell hemoglobin concentration. The peripheral smear demonstrates microcytic and hypochromic cells, which would be expected given these laboratory findings. In addition, there is marked variation in size (anisocytosis) and shape (poikilocytosis). These findings are consistent with severe iron-deficiency anemia, and serum ferritin would be expected to be <10 to 15 g/L. A low haptoglobin level would be seen in cases of hemolysis, which can be intravascular or extravascular in origin. In intravascular hemolysis, the peripheral smear would be expected to show poikilocytosis with the presence of schistocytes (fragmented red blood cells [RBCs]). In extravascular hemolysis, the peripheral smear would typically shows spherocytes. Hemoglobin electrophoresis is used to determine the presence of abnormal hemoglobin variants. Sickle cell anemia is the most common form and demonstrates sickled RBCs. Thalassemias are also common inherited hemoglobinopathies. The peripheral smear in thalassemia often shows target cells. Glucose-6-phosphate dehydrogenase deficiency leads to oxidant-induced hemolysis with presence of bite cells or blister cells. Vitamin B12 deficiency leads to macrocytosis, which is not consistent with this case.

Which of the following best describes the expected histopathology of the lungs in the patient described in Question 01? A. Destruction of the walls of airspaces without fibrosis B. Interstitial fibrosis of the lung parenchyma C. Lymphocytes restricted to the interstitium D. Prominent bronchial smooth muscle cell hyperplasia E. Thickening of the epithelial basement membrane

A: Destruction of the walls of airspaces without fibrosis. Emphysema is a chronic lung disease characterized by enlargement of the airspaces distal to the terminal bronchioles, with destruction of their walls, but without fibrosis or inflammation.

A 43-year-old woman is evaluated for progressive weight gain over the past 2 years. Her previous weight was 72.6 kg (160 lb) but has steadily risen to her current weight of 106.6 kg (235 lb). She notes a slight increase in her appetite but minimal change in her lifestyle or activity level. She has tried to lose weight with increased exercise and nutritional counseling but without significant results. More recently she reports having trouble sleeping and decreased exercise tolerance with activities such as walking up steps. Medical history is significant for impaired fasting glucose, hypertension, and hyperlipidemia. Medications are hydrochlorothiazide and atorvastatin. She has not been prescribed glucocorticoids or had glucocorticoid joint injections. On physical examination, temperature is 37.2 °C (99.0 °F), blood pressure is 136/86 mm Hg, pulse rate is 88/min, and respiration rate is 12/min. BMI is 38. She has rounded facies, thin hair, mild hirsutism, and prominent fat deposition in the dorsocervical and supraclavicular areas. Her skin is thin, and she bruises easily, although striae are not present. Her examination is otherwise unremarkable. Laboratory studies are significant for a fasting plasma glucose level of 120 mg/dL (6.7 mmol/L) and normal thyroid-stimulating hormone level. Which of the following is the most appropriate next step in evaluation? A. Adrenocorticotropic hormone mmeasurement B. 1-mg dexamethasone suppression test C. 8-mg dexamethasone suppression test D.Pituitary MMRI E. Serum cortisol measurement

B. 1-mg dexamethasone suppression test Educational Objective: Diagnose Cushing syndrome. Key Point: To screen for Cushing syndrome, biochemical evidence of hypercortisolism must be confirmed by a 1-mg dexamethasone suppression test, 24-hour urine free cortisol testing, and/or measurement of evening salivary cortisol levels. The most appropriate next diagnostic test for this patient is the 1-mg dexamethasone suppression test. She has the typical clinical features and findings of cortisol excess, or Cushing syndrome. The most common cause of Cushing syndrome is exogeneous glucocorticoid use; however, she has not received glucocorticoids. To evaluate for Cushing syndrome, biochemical evidence of hypercortisolism must be confirmed by use of several screening tests. Three screening tests are used for Cushing syndrome: the 1-mg dexamethasone suppression test (given late at night with assessment of cortisol suppression the next morning), 24-hour urine free cortisol excretion (to quantify total daily cortisol secretion), and measurement of evening salivary cortisol (which normally reaches a nadir at that time but remains elevated in patients with Cushing syndrome). At least two abnormal first-line screening tests are required for diagnosis. Only after establishing biochemical hypercortisolism should the source of excess cortisol production be sought. Measurement of adrenocorticotropic hormone (ACTH) is not a screening test for Cushing syndrome. After documentation of excess cortisol production, ACTH levels may be useful in determining if hypercortisolism is ACTH-dependent or -independent; however, it is not an appropriate initial screening test. An 8-mg dexamethasone suppression test is helpful in differentiating between Cushing disease (pituitary tumor-secreting ACTH) and ectopic ACTH production. However, it is not a screening test for Cushing syndrome and would be appropriate only in specific situations after Cushing syndrome is diagnosed. A pituitary MRI should be ordered only after hypercortisolism and Cushing syndrome are diagnosed and a pituitary adenoma is suspected as a cause. Measurement of serum cortisol levels lacks sensitivity and specificity for diagnosing Cushing syndrome, primarily due to the pulsatile nature of cortisol secretion, and is not used as a screening test.

A 66-year-old woman is evaluated in the emergency department for a 1-day history of nausea, vomiting, weakness, and confusion. Today, she has difficulty walking and has fallen several times. Medical history is significant for a recent diagnosis of depression for which fluoxetine was started 3 weeks ago. She takes no other medications. On physical examination, the patient appears chronically ill. She is unable to stand without assistance because of generalized weakness. Temperature is normal, blood pressure is 130/78 mm Hg and pulse rate is 68/min without orthostatic changes; respiration rate is 18/min. Cardiopulmonary examination is normal. There is no peripheral edema. The neurologic examination is nonfocal. Laboratory studies: Serum creatinine 0.9 mg/dL (79.6 µmol/L) Serum sodium 115 mEq/L (115 mmol/L) Glucose, random plasma 105 mg/dL (5.8 mmol/L) Serum osmolality 245 mOsm/kg H20 Urine osmolality 408 mOsm/kg H20 Urine sodium 90 mEq/L (90 mmol/L) Which of the following is the most appropriate treatment? A. 0.9% Saline infusion B. 3% Saline iinfusion C. Furosemide D. Tolvaptan

B. 3% Saline iinfusion Educational Objective: Treat a patient who has symptomatic hyponatremia. This patient has symptomatic hypo-omolar hyponatremia, and a rapid increase in the serum sodium level using 3% saline infusion is indicated. The patient's presentation is consistent with the syndrome of inappropriate antidiuretic hormone secretion (SIADH) as the cause of her hyponatremia, likely from her recently started antidepressant medication. This diagnosis is based on her low serum osmolality associated with a urine sodium level exceeding 40 mEq/L (40 mmol/L) and urine osmolality that is inappropriately concentrated relative to her serum osmolality (above 100 mOsm/ kg H20 and usually greater than 300 mOsm/kg H20) without evidence of hypovolemia. Although free water restriction is the recommended therapy for asymptomatic patients with SIADH, hypertonic saline (such as 3% saline) is used to treat patients who are symptomatic. Because of its hypertonicity, 3% saline rapidly increases the serum sodium level; a typical goal for treating symptomatic hyponatremia is to increase the serum sodium level to approximately 120 mEq/L (120 mmol/L) . However, hypertonic saline must be used with extreme caution to avoid overcorrection and the risk of central nervous system damage resulting from changes in serum osmolality. Recent evidence suggests than an increase in the serum sodium level by approximately 4 to 6 mEq/L (4-6 mmol/L) over the first 24 hours is sufficient in symptomatic patients. If the extracellular fluid osmolality rapidly normalizes in a patient with chronic hyponatremia, cell shrinkage may occur and can precipitate osmotic demyelination syndrome. Use of 0. 9% (normal) saline in patients with SIADH can result in excretion of most of the infused sodium and retention of a significant portion of the infused water, leading to positive water balance and worsening hyponatremia. Furosemide can be used as adjunctive management of hyponatremia caused by SIADH because this agent interferes with urine concentration and causes excretion of a more dilute urine, thus increasing total water excretion. However, the rapidity of correction expected with furosemide alone is insufficient to increase the serum sodium concentration to an appropriate level in a symptomatic patient. Tolvaptan, an oral V2 receptor vasopressin antagonist, is approved to treat patients with asymptomatic euvolemic and hypervolemic hyponatremia. However, the safety of this agent in the management of patients with symptomatic hyponatremia has yet to be established, and too rapid correction and overcorrection of the serum sodium level have been reported. K EY P O I N T: Patients with symptomatic hyponatremia due to the syndrome of inappropriate antidiuretic hormone secretion (SIADH) require a rapid increase in the serum sodium level using 3% (hypertonic) saline infusion.

A 30-year-old woman is evaluated during a follow-up visit for systemic lupus erythematosus. She was diagnosed 3 months ago after presenting with pericarditis and arthritis. She was initially treated with prednisone, 40 mg/d, with improvement of her presenting symptoms. The prednisone has been tapered over 3 months to her current dose of 10 mg/d with no recurrence. She also takes vitamin D and a calcium supplement. On physical examination, vital signs are normal. BMI is 25. Cardiac examination is normal. There is no evidence of arthritis. The remainder of the examination is normal. Which of the following is the most appropriate next step in treating this patient? A. Add azathioprine B. Add hydroxychloroquine C. Add mycophenolate mofetil D. Add a scheduled NSAID ​

B. Add hydroxychloroquine Educational Objective: Treat mild systemic lupus erythematosus. Antimalarial therapy such as hydroxychloroquine in systemic lupus erythematosus (SLE) has documented benefit for reducing disease activity, improving survival, and reducing the risk of SLE-related thrombosis and myocardial infarction. Hydroxychloroquine is an appropriate agent to address milder systemic manifestations of systemic lupus erythematosus (SLE) such as arthritis and pericarditis, and it can act as a glucocorticoid-sparing agent. All patients with SLE who can tolerate it should be taking hydroxychloroquine. Antimalarial therapy such as hydroxychloroquine in SLE has documented benefit for reducing disease activity, improving survival, and reducing the risk of SLE-related thrombosis and myocardial infarction. Azathioprine is generally reserved for more severe manifestations of SLE not responsive to low-dose prednisone and hydroxychloroquine but can be associated with serious toxicity. Azathioprine has generally been supplanted by the use of mycophenolate mofetil in SLE. Mycophenolate mofetil may be appropriate for this patient if she had more serious disease activity such as nephritis or if her arthritis or pericarditis recurred while taking hydroxychloroquine. NSAIDs, often with colchicine, are first-line therapy for most patients with pericarditis, although glucocorticoids may be indicated in patients with pericarditis associated with a systemic inflammatory disease such as in this patient. However, there is no indication to start an NSAID now given resolution of her symptoms, and doing so would increase her risk of gastrointestinal complications if used along with her daily glucocorticoid. ​

A 68-year-old woman is hospitalized for an acute exacerbation of chronic obstructive pulmonary disease (COPD). She responds well to treatment and is discharged. At her follow-up appointment, she states that she has been compliant with treatment but has had 2 acute exacerbations in the last 9 months. In addition to COPD, her medical history is significant for hypertension, hyperlipidemia, and atrial fibrillation. She takes hydrochlorothiazide, simvastatin, diltiazem, and salmeterol. She also takes inhaled albuterol and ipratropium as needed. She does not smoke, is up to date with the appropriate vaccinations, and is undergoing pulmonary rehabilitation. Her vitals are taken: blood pressure 132/86 mmHg, heart rate 87 beats per minute, respiratory rate 16 breaths per minute, and oxygen saturation 94% on room air. Which of the following is the most appropriate next step in management? A. Add theophylline B. Add inhaled fluticasone C. Add oral prednisone D. Start home oxygen therapy E. No change

B. Add inhaled fluticasone. This patient's COPD is progressively worsening and she suffered another acute exacerbation, which makes it necessary to step up her treatment to the next level. Treatment of COPD progresses in a stepwise fashion based on symptoms, number of exacerbations, and GOLD staging criteria, which relies on forced expiratory volume in 1 second (FEV1) for staging. For the shelf examination, the most important task is to recognize when a patient requires a step-up in therapy and then to know what the next step in therapy is. The first step in all of these patients is to decrease risk factors: smoking cessation and vaccinations to prevent lung infections (pneumococcal vaccine and annual influenza vaccine). Pulmonary rehabilitation is another supplemental therapy that is useful in patients with at least moderate symptoms. The treatment steps in pharmacologic therapy are summarized in Box in a stepwise fashion; when a patient is experiencing worsening of symptoms or acute exacerbations, the next step should be added to the current regimen. Other complicated treatments for advanced disease, such as lung volume reduction surgery, are not typically tested on the shelf examination. This patient's regimen includes short-acting bronchodilators and a LABA, and therefore the next step in management is an inhaled corticosteroid. (A) Theophylline blocks phosphodiesterase and increases intracellular cAMP, leading to increased catecholamine release. It is not a first-line therapy, but it may be used in severe COPD for refractory disease. As a sympathomimetic, it can exacerbate tachyarrhythmias and therefore is not a good option in this patient with atrial fibrillation. (C) Oral corticosteroids are used for acute exacerbations but not in chronic disease due to the negative side-effect profile as well as an increase in mortality. (D) This patient does not meet criteria for home oxygen therapy at this time (criteria explained later). (E) The patient has worsening symptoms and the next step in therapy is warranted.

A 32-year-old woman presents with 3 weeks of symmetrical swelling and stiffness in her PIPs and MCPs. The most helpful laboratory test result in establishing a diagnosis of RA in this patient would be: A. ESR of 65 mm/hour. B. RF of 120 IU (normal <40). C. Anti-CCP of 87 (normal <20). D. Positive ANA. ​

C. Anti-CCP of 87 (normal <20). The presence of anti-CCP antibodies in a patient with an inflammatory arthritis is 99% specific for the diagnosis of RA even when it is only of short duration as in this patient with only 3 weeks of disease. This is critically important because it allows very early institution of DMARD therapy. The presence of a positive ANA, an elevated RF, or elevated ESR would not be particularly helpful—all are nonspecific. ​

A 48 year old asthmatic is referred to clinic because of increased frequency of asthma exacerbations. He has been waking at night with cough and breathlessness that require extra doses of inhaled salbutamol. Spirometry reveals an obstructive defect and blood tests reveal an eosinophilia of 0.67 × 109/L. CXR is clear. The patient's current therapy includes Flixotide 500 µg/ salmeterol 25 µg 1 puff twice a day; salbutamol 2 puffs, as required. The patient has started prednisolone 40 mg for 5 days as prescribed by his family physician today. In line with British Thoracic Society guidelines, what should be suggested in order to step-up therapy? A. Add amoxicillin B. Add montelukast C. Double prednisolone dose D. Provide home nebuliser E. Start omalizumab

B. Add montelukast The scenario does not provide enough information to advocate initiation of omalizumab (we do not know patient's body mass index (BMI), total IgE, sensitisation to allergens). Montelukast and oral theophylline preparations are recommended as additional therapy at this stage. Doubling the prednisolone would increase side-effects without increasing efficacy. There is no indication for an antibiotic here. Home nebulisers are not recommended in asthma because of the risk of late presentation with significant exacerbation.

A 23-year-old man is evaluated in the emergency department for a worsening asthma exacerbation that began 2 days ago following an upper respiratory infection. He has been using an albuterol inhaler at home without improvement. He has a history of poorly controlled asthma and has been hospitalized once a year for the past 4 years; he required intubation 2 years ago. Medical history is otherwise unremarkable. Medications are a long-acting glucocorticoid and an as-needed albuterol inhaler. He is a never-smoker. On physical examination, he is in moderate discomfort. Temperature is 37.3 °C (99.1 °F), blood pressure is 138/85 mm Hg, pulse rate is 124/min, and respiration rate is 20/min. Audible wheezing is heard, and pulmonary examination reveals diffuse expiratory wheezes. Except for tachycardia, the remainder of the examination is normal. Chest radiograph shows hyperinflated lungs but is otherwise normal. Arterial blood gas studies breathing ambient air show a pH of 7.48, a PCO2 of 30 mm Hg (4.0 kPa), and PO2 of 85 mm Hg (11.3 kPa). Systemic glucocorticoids and frequent β2-agonist nebulizer treatments are administered. After 1 hour, he notes that he feels better. He appears tired, is not speaking in full sentences, and is using accessory muscles of breathing. Pulse rate is now 119/min, respiration rate is 19/min, and repeat arterial blood gas studies breathing ambient air show a pH of 7.38, a PCO2 of 43 mm Hg (5.7 kPa), and a PO2 of 80 mm Hg (10.6 kPa). Which of the following is the most appropriate management? A. Admit to the general medical floor B. Admit to the ICU C. Continue current treatment and reassess in 2 hours D Discharge home with next-day follow-up

B. Admit to the ICU Educational Objective: Identify the appropriate venue of care for a high-risk asthma exacerbation. Key Point: Patients with high-risk asthma exacerbations who are not responding to initial treatment and have persistently high work of breathing are best managed in an ICU. The most appropriate management is to admit the patient to the ICU because of his high-risk asthma and arterial blood gas studies suggesting the possibility of impending respiratory failure. Patients with asthma who are at risk of poorer outcomes include those with limited access to health care or difficulty with medication adherence, poor perception of low lung function (and inability to gauge worsening of airways obstruction), an acute increase in use of short-acting β2-agonist therapy, and lack of improvement with outpatient treatment. This patient has several of these factors, and he also had severe symptoms before seeking care. His initial blood gas studies showed an acute respiratory alkalosis, which is a typical finding in patients with an acute asthma exacerbation. However, his second blood gas determination after 1 hour of treatment showed a developing respiratory acidosis despite a continued high work of breathing, as reflected by his respiration rate. These changes suggest progressively ineffective ventilation and the possibility of impending respiratory collapse. Therefore, the most appropriate venue for treatment is an ICU setting, in which ventilatory failure could be managed if it occurs. Admission to a general medical floor would not be appropriate because this patient's clinical indicators suggest the need for more intense monitoring that could not be provided in that setting. Although ventilatory failure could be managed in an emergency department setting, continued therapy with reassessment in 2 hours would not be appropriate given this patient's changing clinical course. Discharge with close follow-up would not be appropriate given this patient's current respiratory status and previous issues with poor asthma control.

A 72-year-old woman is evaluated for follow-up because of exacerbation of COPD. She has severe COPD without resting hypoxemia. The patient presented 1 week ago with fever, productive cough, and mild dyspnea over her baseline. Use of an albuterol inhaler was increased to six times daily, and a β-lactam/β-lactamase inhibitor and glucocorticoid taper were started. On follow-up today, she is fatigued and dyspneic relative to baseline. The medical history is otherwise unremarkable. Medications are tiotropium, fluticasone-salmeterol, and albuterol. On physical examination, temperature is 37.8°C (100.0°F), blood pressure is 130/85 mm Hg, pulse rate is 95/min and regular, and respiration rate is 28/min. She is dyspneic at rest. Pulmonary examination shows bilateral expiratory wheezing. Oxygen saturation is 86% on ambient air and 92% on 2L oxygen via nasal cannula. Chest radiograph shows no infiltrate and no cardiomegaly. Which of the following is the most appropriate next step in management? A. Add home oxygen treatment B. Admit to the hospital C. Expand the antibiotic spectrum D. Prolong the glucocorticoid taper

B. Admit to the hospital Educational Objective: Manage an acute exacerbation of chronic obstructive pulmonary disease (COPD). The most appropriate next step in management is hospital admission. Many exacerbations of COPD can be managed with in-home treatment. However, hospital admission should be considered in patients with severe disease, advanced age, significant comorbidities, a marked increase in intensity of symptoms, newly occurring arrhythmias, diagnostic uncertainty, insufficient home support, onset of new physical signs, or poor response to initial medical management, as in this patient. This patient has not responded to appropriate outpatient treatment and now has a mild oxygen requirement. She should be admitted to the hospital for more aggressive treatment with inhaled bronchodilators, continuous oxygen therapy, pulmonary toilet, antibiotics, glucocorticoids, and monitoring for potential complications. Home oxygen treatment is frequently used in patients with COPD. However, this patient's new resting hypoxia suggests lack of improvement or worsening of the current exacerbation. Therefore, providing home oxygen treatment alone would not be appropriate as a next therapeutic step. This patient has purulent sputum, an increase in sputum volume, and worsening dyspnea. Therefore, antibiotic treatment is appropriate. The initial antibiotic choice was appropriate, but empiric expansion of antibiotic coverage is not indicated in the absence of additional risk factors. Glucocorticoids have been shown to be effective in treating acute exacerbations of COPD. Although the ideal dose and duration of therapy are not well defined, prolonging the treatment course alone in a patient who is not responding to treatment would not be appropriate. KEY POINT: Patients who have exacerbations of COPD should be admitted to the hospital if they have underlying severe COPD and advanced age, significant comorbidities, a marked increase in the intensity of symptoms, newly occurring arrhythmias, diagnostic uncertainty, insufficient home support, or onset of new physical signs, or if they do not respond to initial medical management.

A 52-year-old woman with RA treated with methotrexate and adalimumab presents to the ER with fevers, cough, and increasing shortness of breath. A chest x-ray film shows diffuse patchy infiltrates. You would: A. Admit, stop her adalimumab, and give her broad-spectrum antibiotics. B. Admit, stop both her adalimumab and methotrexate, and aggressively pursue work-up for infectious and inflammatory etiologies. C. Treat her for community-acquired pneumonia. D. Admit, stop adalimumab, and aggressively pursue work-up for infectious etiology. ​

B. Admit, stop both her adalimumab and methotrexate, and aggressively pursue work-up for infectious and inflammatory etiologies. All TNF inhibitors have a black box warning for fatal opportunistic infections. Tuberculosis, histoplasmosis, coccidioidomycosis, pneumocystis, legionellosis, and others have all been reported. The key to patient survival is aggressive diagnosis so appropriate treatment can be started rapidly. Methotrexate can be associated with unusual infections rarely, but the biggest concern here is that the patient may have methotrexate pneumonitis. This can be fatal if the methotrexate is not stopped. It usually presents with low-grade fevers, nonproductive cough, increasing dyspnea, and patchy infiltrates on chest x-ray films. Treatment of methotrexate pneumonitis after stopping methotrexate is usually supportive, but occasionally high-dose steroids may be indicated. ​

A 27-year-old patient with asthma presents to your office complaining of shortness of breath with wheezing. Which of the following medications is indicated in the initial treatment of this patient? A. Salmeterol B. Albuterol C. Cromolyn sodium D. Ipratropium bromide E. Theophylline ​

B. Albuterol Asthma is a reversible obstructive lung disorder that is characterized by reactive airways. The condition is thought to be inherited; however, some individuals may be affected without a family history. Many factors may precipitate an attack, including infections, smoke, cold weather, exercise, toxic fumes, and stress. Symptoms include wheezing, shortness of breath, tachypnea, cough (particularly in children), and tightness or pressure in the chest. The mainstay of acute treatment (reliever therapy) is an inhalant form of a β2-adrenergic agonist, such as albuterol. Inhaled corticosteroids and salmeterol (a long-acting β2-adrenergic agonist) are used in chronic therapy. For patients who have more severe asthmatic attacks, short courses of oral corticosteroids may be necessary, particularly with upper respiratory infections. Theophylline, once readily prescribed, is used less frequently, and its benefits are controversial. Cromolyn sodium, a mast-cell stabilizer; ipratropium bromide, an anticholinergic medication; and the leukotriene modifiers can be used for chronic asthmatic conditions. Pulmonary function tests in patients affected with asthma usually show a normal or decreased vital capacity, decreased forced expiratory volume in 1 second, increased residual volume, increased total lung capacity, and a positive response to inhaled bronchodilators. In children, rescue β2-adrenergic agonists are the treatment of choice for mild intermittent asthma.

A 63 year old woman with a 10-year history of rheumatoid arthritis presents with gradually worsening pain and swelling of the left knee joint over a period of 2-3 days. Her arthritis has generally been under good control with methotrexate 20 mg weekly and the tumor necrosis factor alpha (TNF-α) inhibitor etanercept 50 mg weekly. On examination the knee is warm and swollen, with signs of an effusion. What would be the most appropriate course of action? A. Aspirate the knee and inject with 80 mg methylprednisolone? B. Aspirate the knee and send the synovial fluid for culture and microscopy C. Commence treatment with a broad-spectrum antibiotic D. Commence treatment with diclofenac 75 mg twice daily E. Increase the dose of methotrexate to 25 mg weekly ​

B. Aspirate the knee and send the synovial fluid for culture and microscopy The history is suggesting of septic arthritis given the history of rheumatoid arthritis (RA) and immunosuppressive therapy with methotrexate and a TNF-α inhibitor. Options A, D and E would not be appropriate until infection had been excluded, nor would option C. ​

A 62-year-old man is evaluated for 2-year history of increasing shortness of breath. His symptoms are worse when he walks up steps or lifts heavy objects. He also has an occasional nonproductive cough. His medical history is significant for hypertension, and his only medication is chlorthalidone. He is an active smoker with a 56-pack-year smoking history. He is a former construction worker and worked in a steel mill when he was in high school. On physical examination, the patient is afebrile, blood pressure is 125/78 mm Hg, pulse rate is 90/min, and respiration rate is 18/min; BMI is 31. Oxygen saturation at rest breathing ambient air is 94%. Pulmonary examination reveals a mildly prolonged expiratory phase but is otherwise normal; no wheezes or crackles are noted. The remainder of the examination is unremarkable. Chest radiograph shows mildly increased lung markings but no focal findings. Spirometry shows an FEV1/FVC ratio of 65%, an FEV1 of 52% of predicted, and an FVC of 80% of predicted. Lung volumes show a total lung capacity of 120% of predicted, a residual volume of 125% of predicted, and a DLCO of 65% of predicted. Based on this patient's clinical findings, which of the following is the most likely diagnosis? A. Bronchiectasis B. COPD C. Idiopathic pulmonary fibrosis D. Obesity hypoventilation syndrome

B. COPD Educational Objective: Diagnose COPD using clinical history and pulmonary function testing. Key Point: The FEV1/FVC ratio is used to assess for airway obstruction; a value less than 70% (the lower limit of normal) is consistent with airflow obstruction. The most likely diagnosis is COPD. The spirometric findings, in conjunction with this patient's clinical history, are most consistent with this diagnosis. His FEV1/FVC ratio is low, consistent with an obstructive lung defect. The FEV1/FVC ratio is used to assess for airway obstruction; a value less than 70% (the lower limit of normal) is consistent with airflow obstruction. With evidence of obstruction, the degree of reduction in FEV1 is then used to characterize the degree of obstruction. An FEV1 of 50% to 80% of predicted is classified as moderately reduced, 34% to 50% of predicted is severely reduced, and less than 34% of predicted is very severely reduced. This patient's reduction in FEV1 qualifies as moderately severe. Increased lung volumes with higher than predicted total lung capacity suggests hyperinflation, and high residual volume suggests air trapping from increased lung compliance. The lower than predicted DLCO suggests an effect on the lung parenchyma, which in this patient is most consistent with emphysema. This patient's clinical history demonstrates a progressive course of dyspnea, in contrast to an episodic course with resolution, which would be more compatible with asthma. Bronchiectasis is a condition that shares features with COPD, including mild to moderate airflow obstruction on pulmonary function testing and an abnormal lung examination with wheezing and crackles. However, patients with bronchiectasis usually have significant coughing with daily sputum (often thick sputum) and airways that are easily inflamed and collapsible. Chest radiographs and CT imaging often show distorted airway architecture. This patient, in contrast, has features of indolent dyspnea, a significant smoking history, and relatively unremarkable chest radiograph, which is more compatible with COPD. Idiopathic pulmonary fibrosis is a form of diffuse parenchymal lung disease. This condition more typically causes a restrictive pattern on spirometry with parallel decreases in both the FEV1 and FVC, unlike in this patient who has a decreased FEV1 but preserved FVC. Additionally, pulmonary fibrosis is more likely to result in decreased lung volumes in contrast to the increased lung volumes seen in this patient. Obesity hypoventilation syndrome is characterized by fatigue and daytime somnolence in patients who are obese (BMI >30), and the diagnosis is confirmed by arterial blood gas testing showing daytime hypercapnia with an arterial PCO2 greater than 45 mm Hg (6.0 kPa). However, pulmonary function testing typically shows a restrictive pattern without obstruction, with a decreased FEV1 and FVC but preserved FEV1/FVC ratio. Although this patient is obese, his clinical symptoms and pulmonary function studies are not consistent with a diagnosis of obesity hypoventilation syndrome.

Antibodies to centromeres occur most commonly in which of the following diseases? A. Progressive systemic sclerosis B. CREST syndrome C. Polymyositis D. Dermatomyositis E. Sjögren syndrome

B. CREST syndrome Antibodies to centromere proteins occur most commonly in CREST syndrome or limited scleroderma. This syndrome is characterized by calcinosis, Raynaud phenomenon, esophageal dysmotility, and telangiectasias. In contrast to the CREST syndrome, the diffuse form of progressive systemic sclerosis demonstrates antibodies to a protein called SCL-70 or DNA topoisomerase. The basis for these patterns of expression is unclear. Both antigens are ubiquitous and intracellular. It is difficult to understand why these antibodies would be associated with distinct clinical patterns even if there is some overlap in the clinical features. ​

A 68-year-old man comes to the office with a 2-day history of right foot pain and swelling. The pain started acutely when he awoke in the morning. Initially, the patient had pain only on dorsiflexion of the foot, but he progressively developed pain with any movement at the ankle or weight bearing. The patient has not had similar symptoms before. He is normally very active, and had spent a week bicycling in Utah 3 weeks prior to this presentation. Medical history is notable for type 2 diabetes mellitus, hypertension, and hyperlipidemia. Temperature is 37 C (98.6 F), blood pressure is 140/90 mm Hg, and pulse is 90/min. Examination of the right foot shows warmth, swelling, and tenderness over the dorsal surface of the foot and around the right ankle. Both dorsiflexion and plantar flexion are restricted due to pain. The skin is intact without ulcers or breaks. Pulses are full and equal in both feet, and sensory sensations are normal. X-rays reveal soft tissue swelling, small tibiotalar joint effusion, and chronic calcification of the articular cartilage. There are no fractures or dislocation. Which of the following is the most likely diagnosis? A. Bacterial infection of the ankle joint B. Calcium pyrophosphate arthritis C. Monosodium urate arthritis D. Neuropathic joint E. Plantar fasciitis F. Stress fracture

B. Calcium pyrophosphate arthritis This patient has characteristic findings of acute calcium pyrophosphate dihydrate (CPPD) crystal arthritis (pseudogout), including onset at age >65, monoarticular arthritis, and chondrocalcinosis (calcification of articular cartilage). Synovial CPPD deposition can also cause a chronic inflammatory arthritis resembling rheumatoid arthritis and a noninflammatory degenerative process resembling osteoarthritis. Pseudogout commonly affects the knees and ankles, where it may be difficult to distinguish from urate gout, but it also frequently affects the large joints of the upper extremity and may involve multiple joints concurrently. Attacks of pseudogout often occur in the setting of trauma/overuse. surgery, or medical illness. The finding of chondrocalcinosis in a joint with a typical acute inflammatory arthritis establishes a diagnosis of probable CPPD crystal arthritis. Synovial fluid analysis will show an inflammatory effusion (15,000-30,000 cells/mm3) along with rhomboid-shaped, weakly positively birefringent CPPD crystals. Arthrocentesis for synovial fluid analysis is advised to exclude urate gout (up to 50,000 cells/mm3, needle-shaped negatively birefringent urate crystals) and septic arthritis (>50,000 cells/mm3, visible organisms on Gram stain). Educational objective: The finding of chondrocalcinosis in a joint with a typical acute inflammatory arthritis establishes a diagnosis of probable calcium pyrophosphate dihydrate (CPPD) crystal arthritis (pseudogout). Synovial fluid analysis will show an inflammatory effusion and rhomboid-shaped, weakly positively birefringent CPPD crystals.

A 52-year-old man notes nausea with abdominal dis-comfort after meals. On physical examination, there are no abnormal findings. Upper endoscopy is performed, and there are three ovoid nodules in the fundus and antrum ranging from 0.3 to 1.2 cm in size. They have rounded, smooth surfaces. Biopsies are taken and on microscopic examination there are irregular, cystically dilated and elongated foveolar glands. Which of the following treatment strategies is most appropriate for his gastric lesions? A. Antibiotics B. Chemotherapy C. Corticosteroids D. Multivitamins E. Total gastrectomy F. Vagotomy ​

B. Chemotherapy Gastric inflammatory/hyperplastic polyps may arise in the setting of Helicobacter pylori infection. They are the most common type of gastric polyp. They may be precursors to gastric adenocarcinomas, particular lesions larger than 1.5 cm and with high-grade dysplasia. The other listed options are not appropriate for an infectious etiology.

A 56-year-old male with a 65-pack-per-year smoking history presents to his family physician for a checkup. He has not seen a doctor in 15 years. His vital signs are temperature of 98.7°F, respiratory rate of 16 breaths per minute, heart rate of 85 bpm, and blood pressure of 159/85 mm Hg. During the history, he relates that he has been coughing up thick yellow green material for about the last three years, on and off, but more frequently for at least half of the year. Physical examination reveals some ronchi, but no crackles. His fingernail beds are slightly blue, and a pulse oximeter reveals an oxygen saturation of 88%. He does not report having difficulty breathing. Of the following, what is the most likely diagnosis? A. Emphysema B. Chronic bronchitis C. Lobar pneumonia D. Granulomatosis with polyangiitis E. Chronic eosinophilic pneumonia ​

B. Chronic bronchitis Of the diseases listed, emphysema and chronic bronchitis are the most common (A, B). With chronic bronchitis, patients have a productive cough for several months in two consecutive years (B). Physical examination can reveal ronchi and signs of hypoxemia, with dyspnea occurring later in the course of the disease, whereas in emphysema, dyspnea occurs earlier ("pink puffers"). Without a fever, lobar pneumonia is not likely, and the time course is too long (C). (D, E) are possible, but rarer. ​

A 48-year-old man is evaluated because of a 1-year history of cough. He has no shortness of breath, heartburn, or change in appetite or weight. He has a 35-pack-year history of smoking. He has no seasonal allergies. The medical history is significant for hypertension. His only medication is losartan. On physical examination, vital signs are normal. Pulmonary examination shows normal breath sounds that are equal bilaterally without wheezing. No nasal polyps are noted. Findings on abdominal examination are unremarkable. There is no cyanosis, clubbing, or edema. Pulmonary function tests show forced expiratory volume in 1 second (FEV1) of 75% of predicted and an FEV1/forced vital capacity (FVC) ratio of 63%. After administration of a bronchodilator, there is no significant change in the FEV/FVC ratio and FEV1 is 83% of predicted. Chest radiograph shows no masses, and normal lung markings are seen. Which of the following is the most likely cause of this patient's cough? A. Asthma B. Chronic obstructive pulmonary disease C. Gastroesophageal reflux disease D. Losartan use

B. Chronic obstructive pulmonary disease Educational Objective: Diagnose chronic obstructive pulmonary disease (COPD). The most likely cause of this patient's cough is COPD. The post-bronchodilator forced expiratory volume in 1 second (FEV1)/forced vital capacity (FVC) ratio of less than 70% confirms airflow limitation and a diagnosis of COPD. The relatively preserved FEV1 suggests that the COPD is mild. COPD should be considered in any patient older than 40 years of age who has dyspnea, chronic cough or sputum production, or a history of risk factors (e.g., exposure to tobacco smoke, dust, chemicals, outdoor air pollution, or biomass smoke). Spirometry is the gold standard for diagnosing COPD and monitoring its progress. It should be performed to confirm the diagnosis and exclude other diseases. The American College of Physicians and Global Initiative for Chronic Obstructive Lung Disease (GOLD) guidelines require an FEV1/FVC ratio of less than 70% of predicted to establish the diagnosis of COPD. The GOLD guidelines use the degree of airflow obstruction as measured by FEV1 to further describe the level of disease. Level 1 (mild) COPD is characterized by FEV1 of80% or greater of predicted. Level 2 (moderate) COPD is characterized by FEV1 of 50% to 79% of predicted. Level 3 (severe) COPD is characterized by FEV1 of 30% to 49% of predicted. Level 4 (very severe) COPD is characterized by FEV1 of less than 30% of predicted. Although asthma may present with cough, asthma is unlikely in this patient because of the lack of atopy and history of respiratory symptoms as a child. There are no other clinical findings consistent with bronchospasm. Gastroesophageal reflux disease may cause cough. However, COPD is the more likely cause of cough in this patient who does not have heartburn, has a history of smoking, and shows airflow obstruction on pulmonary function testing. Although angiotensin-converting enzyme inhibitors may be associated with cough, angiotensin receptor blockers (e.g., losartan) are less likely to cause cough and would not be an expected cause of this patient's cough, given the other clinical findings. KEY POINT: Spirometry is essential for diagnosing COPD and assessing its severity. A postbronchodilator FEV1/FVC ratio of less than 70% confirms airflow limitation.

A 65 year old man presents to his family physician complaining of pain in the left knee, worse on ascending and descending stairs. He smokes 15 cigarettes a day, and drinks about 4 units of alcohol daily (28 units per week). He is a former amateur soccer player who suffered a cruciate ligament tear in his 30s. He has lived alone since his wife died 5 years previously. He has been avoiding dairy products since he thinks they cause gastrointestinal upset. Which of the following risk factors predispose to the development of osteoarthritis? A. Alcohol intake > 21 units per week B. Cigarette smoking C. Immobilisation D. Low dietary calcium intake E. Previous anterior cruciate ligament tear ​

B. Cigarette smoking Osteoarthritis is characterised by joint space narrowing due to cartilage erosion and subchondral sclerosis. Erosions at all MCPJs and periarticular osteoporosis are more typical of RA (though inflammatory OA typically causes inflammation at index and middle finger MCPJs) whereas punched-out erosions of the first MTP joint of the feet suggest gout. Irregularity and fusion of the sacroiliac joint (SIJ) suggests axial spondyloarthritis (axSpA) or psoriatic arthritis (PsA). ​

A 22-year-old woman is evaluated for a 2-year history of recurrent abdominal pain often accompanied by fever; episodes occur every 3 to 4 months and last 1 to 3 days and resolve completely. She went to the emergency department during an episode 6 weeks ago. She was noted to be mildly febrile, and laboratory studies showed an erythrocyte sedimentation rate of 84 mm/h and a leukocyte count of 16,000/µL (16 x 109/L) with neutrophilia. She was diagnosed with viral gastroenteritis and recovered completely with supportive treatment. She has been treated on several occasions for cellulitis that occurs on her foot or lower extremity, but the reason for repeated infection or a responsible organism has not been identified. On two occasions, she had pain and swelling in the knee that lasted several weeks and was not associated with the abdominal pain. She has tried naproxen without relief. She currently feels well and has no complaints. On physical examination today, temperature is 36.6 °C (97.9 °F), blood pressure is 120/74 mm Hg, pulse rate is 74/min, and respiration rate is 14/min. BMI is 23. The cardiopulmonary and abdominal examinations are normal. There is no joint swelling. Current laboratory studies, including complete blood count, chemistry panel, and erythrocyte sedimentation rate, are normal. Which of the following is the most appropriate treatment? A. Anakinra B. Colchicine C. Indomethacin D. Prednisone

B. Colchicine Educational Objective: Treat a patient who has familial Mediterranean fever with colchicine. Familial Mediterranean fever is characterized by episodes of fever, polyserositis, arthritis, erysipeloid rash around the ankles, and elevated acute phase reactants; a response to colchicine is useful in the clinical diagnosis. Colchicine is appropriate for this patient who has familial Mediterranean fever (FMF), an autosomal recessive disease characterized by episodes of fever, polyserositis, arthritis, erysipeloid rash around the ankles, and elevated acute phase reactants. Attacks last 1 to 3 days and are self-limited but can be dramatic. FMF is associated with mutation of the MEFV1 gene; testing for MEFV mutations is available and should be considered as an aid to diagnosis, although not all mutations have been identified. AA amyloidosis is a potential long-term consequence of FMF due to the production and accumulation of serum amyloid A. Colchicine affects the function of various inflammatory cells that are thought to play a role in the cytokine overproduction seen in FMF. A response to colchicine is useful in the clinical diagnosis of FMF because it can prevent attacks as well as AA amyloidosis. Treatment with colchicine is therefore indicated for this patient who most likely has FMF, as manifested by her history of febrile attacks of abdominal serositis, erysipelas-like skin lesions that mimic cellulitis, and inflammatory arthritis occurring independently of the febrile episodes. FMF genetic mutations affect the function of pyrin that results in overactivation of the inflammasome. The inflammasome is an important constituent of the innate immune system that is responsible for production of interleukin (IL)-1; dysregulation of the inflammasome, as in FMF, results in overproduction of IL-1. The IL-1 inhibitor anakinra can be used in patients with FMF who are unresponsive to colchicine; however, this agent is not first-line therapy. NSAIDs and prednisone have not been shown to have a significant impact on the disease progression or symptoms associated with FMF.

A 30-year-old woman is evaluated in follow-up after recently discovering she is 6 weeks pregnant with her first child. Her medical history is notable for mild persistent asthma related to seasonal ragweed allergy. Her symptoms are well controlled on a low-dose inhaled glucocorticoid and an as-needed β2-agonist. However, symptoms usually flare in fall and winter and with moderate exertion, and she treats these symptoms with an add-on oral leukotriene inhibitor. She currently has no asthma symptoms. Medications are budesonide and albuterol inhalers and montelukast when needed. On physical examination, vital signs are normal. The lungs show good air movement and no wheezes. The remainder of the physical examination is unremarkable. The patient's spirometry at the time of diagnosis showed an FEV1 of 85% of predicted without a significant bronchodilator response. Her last routine spirometry 6 months ago demonstrated normal values with an FEV1 of 93% of predicted. Which of the following is the most appropriate treatment? A. Continue montelukast and albuterol but stop budesonide B. Continue the current drug regimen C. Increase budesonide to high dose D. Switch to as-needed albuterol therapy only

B. Continue the current drug regimen Educational Objective: Treat asthma during pregnancy. Key Point: Inhaled glucocorticoids, most β2-agonists, and the leukotriene antagonist montelukast are considered safe in pregnancy. The most appropriate management is to continue the current drug regimen. As in all patients with asthma, treatment of asthma during pregnancy should aim for optimal control, and a regimen that adequately controls asthma should be continued. Controlled asthma ensures safety for the fetus as well as the mother and reduces the risk for preeclampsia and preterm birth. Inhaled glucocorticoids are considered safe in pregnancy, and abundant long-term safety evidence exists for budesonide. Short-acting β2-agonists are considered safe in pregnancy, and existing data for long-acting β2-agonists are reassuring that these agents are safe as well. The leukotriene antagonist montelukast is also considered safe in pregnancy. In this patient, who has a history of reduced lung function, it is appropriate to maintain the current regimen that is controlling her asthma because the medications she is taking have evidence of safety in pregnancy. This patient also should have frequent monitoring with regular visits, should be offered a peak flow monitor, and should be counseled regarding the safety of and need for continued asthma medications. The most appropriate management is to continue the current regimen and follow up frequently. Empirically stepping up asthma therapy in anticipation of worsening asthma symptoms during pregnancy is not warranted. During pregnancy, one third of women have no significant change in their asthma, one third experience worsening of symptoms, and one third experience improvement of symptoms. Stopping inhaled glucocorticoids and/or montelukast would not be appropriate owing to this patient's likely impairment without controller medications, particularly because these agents are considered safe in pregnancy.

A 66-year-old woman is brought to the emergency department due to agitation, restlessness, and poor sleep. Over the past 3 months, she has bad headaches and gained 6.3 kg (13.9 lb). The patient's medical history is unremarkable, and she takes no medications. She has smoked a pack of cigarettes daily for the last 50 years but does not use illicit drugs. Blood pressure is 160/110 mm Hg and pulse is 90/min. Skin examination shows facial plethora and scattered bruises on her extremities. Laboratory results are as follows: Sodium 147 mEq/L Potassium 3.2mEq/L Chloride 98 mEq/L Bicarbonate 26 mEq/L Blood urea nitrogen 12 mg/dL Creatinine 0.9 mg/dL Glucose 205 mg/dL Calcium 9.4 mg/dL Which of the following is the most likely cause of this patient's hypertension? A. Adrenal medullary disease B. Cushing syndrome C. Parathyroid gland disease D. Renal artery stenosis E. Renal parenchymal disease

B. Cushing syndrome This patient's hypertension, neuropsychiatric disturbances, hyperglycemia, and weight gain suggest hypercortisolism (Cushing syndrome). Other manifestations can include proximal muscle weakness, central adiposity with enlargement of the supraclavicular and dorsal neck fat pads, abdominal stria, facial rounding, and ecchymosis. Most cases of Cushing syndrome are due to exogenous glucocorticoid Intake, ectopic ACTH production (e.g., small cell lung cancer), or ACTH-producing pituitary adenomas (Cushing disease). Primary adrenal hypercortisolism (e.g., adrenal adenoma) is less common. Associated hypokalemia suggests severe hypercortisolism and is most often seen with ectopic ACTH-producing tumors. Initial testing for suspected hypercortisolism can include a 24-hour assay for urine free cortisol, a late-night salivary cortisol measurement, or a low-dose dexamethasone suppression test. Once hypercortisolism is established, an ACTH level can determine whether it is ACTH-dependent (Cushing disease or ectopic ACTH production) or ACTH-independent (adrenal disease or exogenous glucocorticoid intake). Patients with an elevated ACTH should have a high-dose dexamethasone suppression test to determine whether ACTH production is pituitary (dexamethasone suppresses cortisol production) or ectopic (dexamethasone does not suppress cortisol). Educational objective: Cushing syndrome is characterized by hypertension, hyperglycemia, and weight gain. Common causes include excess glucocorticoid intake, ACTH-producing pituitary adenoma, and ectopic ACTH production.

A 64-year-old man comes to the office due to increasing pain in his right groin for the past several months. The pain increases with activity and is relieved with rest, it sometimes radiates to the upper thigh. The patient has no history of trauma or fails. There Is no associated fever, weight loss, or change in appetite. He has a history of lumbar disk herniation but has no back pain currently. His other medical problems include type 2 diabetes mellitus and hypertension. The patient is 168 cm (66 in) tall and weighs 95 kg (210 lb.), with a calculated BMI of 34 kg/m2. His vital signs are otherwise normal. Examination shows pain on passive internal rotation of the right hip joint. Direct pressure over the groin does not increase the pain. Reflexes are 2+ in the lower extremities symmetrically, and there are no sensory deficits. Muscle bulk, tone, and power are normal. Dorsalis pedis and posterior tibial pulses are 2+ in both legs. Which of the following is the most likely cause of this patient's hip pain? A. Aortoiliac vascular occlusion B. Cutaneous nerve compression C. Degenerative joint disease D. Disruption of bone vasculature inflammation of the trochanteric bursa E. Referred pain from the lumbosacral spine

C. Degenerative joint disease This patient with slowly progressive pain relieved by rest has degenerative joint disease (osteoarthritis) of the hip. Osteoarthritis is the most common joint disorder and is characterized by progressive destruction of the articular cartilage. Predisposing factors include advanced age; obesity; and diabetes mellitus. Osteoarthritis typically affects the interphalangeal joints of the hands and the large weight-bearing joints of the lower extremity. The pain of hip osteoarthritis is typically felt in the groin, buttock, or pelvis and can radiate to the lower thigh or knee. Patients may have mild pain and stiffness with prolonged rest, but the worst pain usually occurs with activity and weight bearing. Symptoms are insidious, with onset at age >40. Examination findings include decreased internal and external rotation, but synovitis (redness, warmth, tenderness) is absent. Common x-ray findings include loss of joint space, osteophyte formation, and subchondral sclerosis. Educational objective: Osteoarthritis presents in adults age >40 with indolent, progressive pain that is worsened by activity and relieved by rest. The joint is not tender and systemic symptoms are absent.

A 60-year-old woman is evaluated during a follow-up visit for Sjogren syndrome. She reports persistent eye discomfort described as a sandy or gritty sensation. She has had recurrent corneal abrasions and erosions. She has been using artificial tears with minimal improvement. She saw her ophthalmologist who inserted punctal plugs, but they caused excessive tearing and were removed. She otherwise feels well and reports no fever, chills, weight loss, rash, joint pain, chest pain, or dyspnea. On physical examination, temperature is 36.7 °C (98.0 °F), blood pressure is 135/80 mm Hg, pulse rate is 75/min, and respiration rate is 18/min. The oral mucosa is dry. Moderately injected sclerae are noted. There is no cervical or supraclavicular adenopathy and no parotid gland swelling. Lung, heart, musculoskeletal, and skin examinations are normal. Which of the following is the most appropriate treatment? A. Certolizumab pegol B. Cyclosporine drops C. Hydroxychloroquine D. Olopatadine drops E. Prednisone ​

B. Cyclosporine drops Educational Objective: Treat dry eyes in a patient with Sjogren syndrome. Topical cyclosporine improves the symptoms of dry eyes in patients with primary Sjogren syndrome. Cyclosporine drops are appropriate for this patient. She has primary Sjogren syndrome with dry eyes (keratoconjunctivitis sicca), which has not responded to topical lubrication or punctal plugs. Sjogren syndrome is an immune-mediated disease of unknown cause manifesting primarily as inflammation of exocrine glands, including the major and minor salivary glands, lacrimal glands, and, less commonly, other exocrine glands such as the pancreas. The most prominent clinical feature is dryness (sicca), particularly of the eyes and mouth. Dry eyes can lead to corneal damage and visual impairment. Sicca symptoms are primarily treated with hydration and lubrication, although other local measures and medications may be helpful. Topical cyclosporine has been demonstrated in trials to improve the symptoms of dry eyes in patients with primary Sjogren syndrome. An alternative therapy is punctal occlusion (placement of plugs in the tear drainage duct openings of the lower eyelids to increase eye moisture). There is controversy as to the timing and type of plug to use when performing this procedure.

A 59-year-old man has had nausea and vomiting for 5 months. He has experienced no hematemesis. On physical examination, there is no abdominal tenderness, and bowel sounds are present. Upper gastrointestinal endoscopy shows erythematous areas of mucosa with thickening of the rugal folds in the gastric antrum. The microscopic appearance of a gastric biopsy specimen with a Steiner silver stain is shown in the figure. Which of the following factors is most likely responsible for this gastric mucosal pathology? A. Cysteine proteinase B. Cytotoxin-associated gene A C. Heat-stable enterotoxin D. Shiga toxin E. Verocytotoxin ​

B. Cytotoxin-associated gene A Helicobacter pylori organisms shown in the figure reside in the mucus layer above the gastric mucus and are associated with various gastric disorders, ranging from chronic gastritis with erythema and thickened rugal folds, as in this case, to peptic ulcers and to adenocarcinoma. H. pylori organisms elaborate several toxic substances that injure the epithelium. The H. pylori gene from a pathogenicity island encodes cytotoxin-associated antigen (CagA) and is present in many patients with chronic gastritis and peptic ulcers; it increases the risk for gastric cancer. Cysteine proteinases produced by Entamoeba histolytica aid in tissue invasion. Heat-stable enterotoxin is produced by strains of Escherichia coli that cause traveler's diarrhea. Shiga toxin is elaborated by Shigella flexneri organisms, which cause a form of bacillary dysentery. Verocytotoxin produced by some E. coli strains is associated with hemolytic uremic syndrome mediated by endothelial injury.

A patient with a BMI of 34, DM, and GERD is best treated with: A. Diet modification B. Diet modification and medical weight management program C. Gastric banding D. Gastric bypass E. Nothing ​

B. Diet modification and medical weight management program. This patient doesn't quite meet criteria for surgical intervention. ​

A 27-year-old woman is evaluated for a 7-month history of abdominal discomfort after eating. Initially, her discomfort was intermittent, but during the last 4 months it has been present with every meal. The discomfort is described as a sense of fullness over the mid-epigastric region associated with bloating, and lasts for approximately 1 hour before resolving. Eating smaller meals decreases the intensity and duration of the discomfort. Calcium carbonate antacids and simethicone do not relieve her symptoms. She does not have heartburn, regurgitation, vomiting, belching, dysphagia, odynophagia, diarrhea, or constipation. The patient does not smoke cigarettes or drink alcoholic beverages. She has gained approximately 9 kg (20 lb.) over the past year. Medical history is otherwise unremarkable, and she takes no medications. On physical examination, vital signs are normal. Body mass index is 32. The remainder of the examination, including abdominal examination, is normal. A complete blood count is normal. A urine pregnancy test is negative. Which of the following is the most likely diagnosis? A. Biliary colic B. Dyspepsia C. Gastroesophageal reflux disease D. Peptic ulcer disease

B. Dyspepsia Educational Objective: Diagnose dyspepsia. The most likely diagnosis is dyspepsia. The Rome III criteria for dyspepsia are one or more of the following predominant symptoms: (1) bothersome postprandial fullness, (2) early satiety, (3) epigastric pain, or (4) epigastric burning. The symptoms should be consistent for at least 3 months with an initial onset at least 6 months before diagnosis. This patient does not have alarm features such as unintentional weight loss, anemia, vomiting, or dysphagia that might suggest an alternative diagnosis and the need for immediate upper endoscopy. KEY POINT: The Rome Ill criteria for the diagnosis of dyspepsia include one or more of the following predominant symptoms for at least 3 months: (1) bothersome post prandial fullness, (2) early satiety, (3) epigastric pain, or (4) epigastric burning. ​

A 32-year-old woman with a history of moderate persistent asthma complains of "white stuff " on her tongue and a feeling of "cotton mouth." Her asthma is well controlled without any recent exacerbations. Examination shows white plaques adherent to the tongue and buccal mucosa that are removed with scraping. Which of the following is most likely responsible? A. Albuterol B. Fluticasone C. Prednisone D. Salmeterol E. Montelukast F. Epstein-Barr virus ​

B. Fluticasone. Inhaled corticosteroids cause local immunosuppression and increase the risk of oral candidiasis. Local treatment is preferable to systemic treatment, and a good option is nystatin swish and swallow. (A, C, D, E) Prednisone is the only other medication on this list that would also increase the risk of oral candidiasis; however, this patient has no reason for taking oral steroids (they are given during exacerbations and for severe persistent asthma). Albuterol and salmeterol are short- and long-acting β2 agonists, respectively, and would not cause oral candidiasis. Montelukast is a leukotriene inhibitor with adverse effects of drowsiness, GI complaints, and hypersensitivity reactions. (F) Epstein-Barr virus causes oral hairy leukoplakia, which manifests with white plaques that cannot be removed with scraping (differentiates it from candidiasis). Unlike oral leukoplakia, which is a precancerous lesion, oral hairy leukoplakia is not precancerous and is fairly specific for HIV infection.

A 31-year-old woman is evaluated following her recent discovery that she is pregnant at approximately 10 weeks' gestation. Medical history is significant for a prolactinoma diagnosed 2 years ago during an evaluation for amenorrhea. At the time of diagnosis, her serum prolactin level was 184 ng/mL (184 µg/L), and a 1.4-cm pituitary adenoma extending above the sella was detected on MRI without evidence of mass effect. She was treated with bromocriptine with return of regular menses. She discontinued the bromocriptine when she found that she was pregnant. She is currently without symptoms. She does not have new or severe headache. Medical history is otherwise unremarkable, and her only current medication is a prenatal multivitamin. On physical examination, vital signs are normal. Visual fields are full to confrontation, and the remainder of her examination is normal. Which of the following is the most appropriate next step in management? A. Check serum prolactin level B. Formal visual field testing C. Repeat pituitary MMMRI D. Restart bromocriptine

B. Formal visual field testing Educational Objective: Treat prolactinoma during pregnancy. Key Point: Because prolactinomas may increase in size in pregnant women and lead to loss of vision, close clinical monitoring and formal visual field testing should be performed during each trimester. The patient should undergo formal visual field testing. During pregnancy, there is concern that prolactinomas can grow due to estrogenic stimulation. The risk of significant growth depends on the size of the prolactinoma prior to pregnancy. With microadenomas (<10 mm), the risk is considered to be low, whereas the risk is higher with macroprolactinomas (≥10 mm). Significant expansion may cause vision loss by compressing the optic chiasm. It is therefore appropriate to obtain visual field testing during pregnancy in women with macroadenomas even without symptoms to diagnose vision field loss (such as bitemporal hemianopsia) that may be due to an enlarging prolactinoma. Testing in these women is recommended every trimester of pregnancy. Because this patient is in her first trimester and has a history of a macroadenoma, visual field testing now is indicated. It is not helpful to check the serum prolactin level. Prolactin is normally elevated in pregnancy and can be greater than 200 ng/mL (200 µg/L). Additionally, this patient has a known prolactinoma, so her prolactin will likely be elevated for both reasons. Elevated prolactin itself is not necessarily harmful. Prolactinomas cause concern when they cause hypogonadism or mass effect. In this patient, an elevated prolactin level will not change the treatment plan because it is expected to be elevated and does not help clarify whether the prolactinoma is causing harm.

A 66-year-old man is admitted to the hospital for left Sower quadrant abdominal pain, fever, and vomiting. CT scan of the abdomen reveals diverticulitis. The patient is started on supportive care and antibiotics. His symptoms improve over 2 days; however, on the third day of hospitalization he reports acute pain in the right ankle, which becomes "unbearable" within a few hours. The patient had a similar pain 1 year ago that resolved with over-the-counter medication. Medical history is notable for type 2 diabetes, hyperlipidemia, and hypertension. His temperature is 37.1 C (98.8 F), blood pressure is 140/90 mm Hg, and pulse is 98/min. BMI is 36 kg/m2. Examination shows a swollen, erythematous right ankle with moderate restriction of movement due to pain; the left ankle is normal. Both ankles were normal at the time of admission to the hospital. Which of the following is the most likely diagnosis? A. Enteropathic arthritis B. Gout C. Neuropathic joint D. Osteoarthritis E. Psoriatic arthritis F. Rheumatoid arthritis G. Septic arthritis ​

B. Gout This patient with acute, rapidly progressing ankle pain and a prior history of similar pain has acute gout. Although it is most common in the first metatarsophalangeal joint gout can also involve the knee and ankie. Gout is characterized by recurrent attacks that typically develop overnight or early in the morning, reaching maximum intensity within 12-24 hours. Definitive diagnosis is obtained via synovial fluid analysis showing inflammatory cells and needle-shaped, negatively birefringent urate crystals. A gout attack can be precipitated by conditions that cause increased production or decreased elimination of uric acid, or an acute change in uric acid levels. Common triggers include heavy alcohol consumption, intake of urate-rich foods, trauma/surgery, dehydration, and medications that raise (eg, thiazide diuretics, cyclosporine) or lower (eg, allopurinol) uric acid levels. Educational objective: Gout typically presents as an acute monoarticular arthritis that quickly progresses to maximum intensity within 12-24 hours. Triggers include alcohol use, surgery/trauma, dehydration, and certain medications (eg, diuretics).

A 55-year-old carpenter is newly diagnosed with RA. His medical history includes type II diabetes for 15 years. CBC shows low-grade anemia with normal red cell indices, Hgb A1c is 7, creatinine is 2.1, and urinalysis shows 3+ proteinuria. Laboratory studies are otherwise unremarkable. Appropriate initial DMARD therapy would be: A. Cyclosporine. B. Hydroxychloroquine. C. Methotrexate. D. Sulfasalazine. ​

B. Hydroxychloroquine. Although methotrexate at the doses used to treat RA rarely affects the kidney, well-functioning kidneys are important for the clearance of methotrexate. Methotrexate, if used, should be used with extreme caution and close monitoring in this situation. Cyclosporine, although effective in some cases of RA, also has renal toxicity and is not a good choice. Hydroxychloroquine is the best choice here—in addition to being effective against RA, it has been shown to lower blood sugar, particularly in type II diabetes. Additionally, patients with RA who are on hydroxychloroquine have a 60% reduction in their risk for incident diabetes. Finally, sulfasalazine would be a possible choice but not as good as hydroxychloroquine. ​

A 59-year-old woman is evaluated in follow-up in November after being diagnosed with moderate COPD 3 months ago. She has never received the influenza or pneumococcal vaccine. She has no allergies. Her medications are a long-acting inhaled anticholinergic agent and an as- needed short-acting β2-agonist inhaler. On physical examination, vital signs and the remainder of the physical examination are normal. Which of the following vaccinations are recommended for this patient? A. Inactivated influenza vaccine and 13-valent pneumococcal conjugate vaccine (PCV13) B. Inactivated influenza vaccine and 23-valent polysaccharide pneumococcal vaccine (PPSV23) C. Inactivated influenza vaccine, PPSV23, and PCV13 D. Live attenuated influenza vaccine and PPSV23 D. Live attenuated influenza vaccine, PPSV23, and PCV13

B. Inactivated influenza vaccine and 23-valent polysaccharide pneumococcal vaccine (PPSV23) Educational Objective: Recommend appropriate influenza and pneumococcal vaccinations in a patient with COPD. Key Point: Patients with COPD should receive inactivated influenza vaccination annually, and the 23-valent polysaccharide pneumococcal vaccine (PPSV23) should be given to all patients aged 19 to 64 years with COPD, with revaccination at age 65 years or older if 5 years have elapsed since the previous pneumococcal immunization; the 13-valent pneumococcal conjugate vaccine should also be administered at age 65 years or older if 1 year has elapsed since the last PPSV23 immunization. This patient should receive inactivated influenza vaccine and 23-valent polysaccharide pneumococcal vaccine (PPSV23). Influenza vaccination has been shown to reduce serious illness (such as lower respiratory tract infections that require hospitalization) and death in patients with COPD. These vaccines should be administered annually in all patients with COPD. Pneumococcal vaccination with PPSV23 should be given to all patients aged 19 to 64 years with COPD, with revaccination at age 65 years if 5 years have elapsed since the previous pneumococcal immunization. All patients (with or without COPD) should also receive the 13-valent pneumococcal conjugate vaccine (PCV13) at age 65 years, although the polysaccharide and conjugate vaccines should be given sequentially rather than together for optimal effect. Ideally, PCV13 should be administered at least 1 year following PPSV23. PPSV23 and PCV13 are also indicated in patients with functional or anatomic asplenia, cochlear implants, persistent cerebrospinal fluid leak, and significant immunocompromising conditions. Preferably, these patients should receive PCV13 first followed by PPSV23 at least 8 weeks later. This patient will require another dose of PPSV23 at the age of 65 years. Patients with COPD who received their first PPSV23 dose at age 65 years or older do not require revaccination. Because this patient is younger than 65 years, she does not require the PCV13 vaccination at this time. The live attenuated influenza vaccination is recommended for healthy persons between the ages of 2 and 49 years. Because this patient has COPD and because she is older than 49 years, live attenuated influenza vaccination should not be administered.

A 30-year-old man comes to the office due to low back pain and stiffness for 2 months. His pain is worse in the morning and improves with activity. The patient has also had intermittent diarrhea and lower abdominal pain for the past 3 months. He tried ibuprofen, which helped his back pain but made his diarrhea worse. He has not traveled outside the United States. The patient has been sexually active with the same female partner for 2 years. He has no other medical problems and does not use tobacco, alcohol, or recreational drugs. Vital signs are normal. Physical examination shows limited spine flexion and tenderness in the lower back. Laboratory studies show anemia and thrombocytosis. Stool cultures are negative. Plain radiographs reveal sacroiliac joint inflammation. Which of the following is the most likely cause of this patient's symptoms? A. Celiac disease B. Inflammatory bowel disease C. Paraneoplastic syndrome D. Reactive arthritis from diarrhea E. Rheumatoid arthritis F. Spinal osteomyelitis

B. Inflammatory bowel disease This patient likely has inflammatory bowel disease (IBD) complicated by spondylarthritis. IBD, which includes Crohn disease and ulcerative colitis, frequently presents in patients age 15-40 and again at age 50-80. Both Crohn disease and ulcerative colitis have multiple extraintestinal manifestations, including arthritis, eye (eg, uveitis, episcleritis) and skin (eg, pyoderma gangrenosum) involvement, and hepatobiliary disease (eg, primary sclerosing cholangitis). Arthritis occurs in up to 45% of patients with IBD and can involve axial (eg, spine) or peripheral (eg, knee) joints. Patients with spondylarthritis or sacroiliitis commonly report prolonged stiffness and low back or buttock pain that improves with activity. There are no specific tests for the diagnosis of ISO-associated arthritis. Nonsteroidal anti-inflammatory drugs (NSAIDs) can relieve arthritis symptoms but exacerbate the underlying bowel disease. Consequently, although NSAIDs are frequently tried, many of the medications (eg, sulfasalazine) used to treat the bowel disease itself are also used to treat joint disease. Educational objective: Inflammatory bowel disease is frequently complicated by arthritis, which occurs in up to 45% of patients and can involve axial or peripheral joints. Treatment with nonsteroidal anti-inflammatory drugs may exacerbate the underlying disease.

A 63-year-old man is admitted to the ICU for an exacerbation of COPD. He has a 10-year history of COPD with chronic hypercapnia. He recently developed a viral upper respiratory tract infection that worsened his baseline dyspnea, and his family reports increased use of his rescue inhaler. He was brought to the hospital because he became confused. His medical history is otherwise unremarkable. His medications on admission are tiotropium and fluticasone/salmeterol metered-dose inhalers and an as-needed albuterol metered-dose inhaler. On physical examination, he is responsive but confused and disoriented. Temperature is 36.9 °C (98.4 °F), blood pressure is 117/83 mm Hg, pulse rate is 99/min, and respiration rate is 32/min; BMI is 20. Oxygen saturation is 86% breathing 60% oxygen by Venturi mask. Use of accessory muscles of breathing is present. On oral examination, pooling of secretions in the posterior pharynx and diminished gag reflex are noted. There is no jugular venous distention. Pulmonary examination reveals transmitted upper airway noise and decreased breath sounds with polyphonic end-expiratory wheezing heard throughout both lung fields. There is no clubbing or peripheral edema. A chest radiograph demonstrates hyperinflation but no infiltrates. Laboratory studies: Leukocyte count 11,000/µL (11 x 109/L) Hematocrit 32.8% Arterial blood gases (breathing 60% oxygen): pH 7.25 PCO2 72 mm Hg (9.6 kPa) PO2 48 mm Hg (6.4 kPa) Glucocorticoids, antibiotics, and inhaled albuterol by nebulizer are started. Which of the following is the most appropriate next step in management? A. Increase oxygen to 100% by nonrebreather mask B. Intubate and start mechanical ventilation C. Start noninvasive ventilation with continuous positive airway pressure D. Start noninvasive ventilation with inspiratory pressure support and positive end-expiratory pressure

B. Intubate and start mechanical ventilation Educational Objective: Manage hypoxic and hypercapnic respiratory failure in a patient with a contraindication to noninvasive ventilation. Key Point: Contraindications to noninvasive ventilation (NIV) include conditions that make airway protection difficult (for example, encephalopathy or bulbar dysfunction), medical instability (such as hemodynamic instability, severe acidosis, arrhythmias, or active upper gastrointestinal bleeding), and mechanical issues associated with NIV (including upper airway obstruction, recent facial trauma or surgery, or esophageal or transsphenoidal surgery). The most appropriate next step in management is intubation and mechanical ventilation. This patient is experiencing an exacerbation of his COPD with hypercapnic respiratory failure with hypoxia and mental status changes. Although the majority of patients with acute exacerbation of COPD should undergo a trial of noninvasive ventilation (NIV) and reevaluation prior to intubation and mechanical ventilation, encephalopathy that impairs the ability to cooperate with NIV or manage secretions is a contraindication to NIV. Other contraindications to NIV include conditions that make airway protection difficult (for example, bulbar dysfunction or ineffective gag or cough reflexes), medical instability (such as hemodynamic instability, severe acidosis, arrhythmias, or active upper gastrointestinal bleeding), and mechanical issues associated with NIV (including upper airway obstruction, recent facial trauma or surgery, or esophageal or transsphenoidal surgery). Because this patient would likely not be able to adhere with NIV because of confusion and may not be able to protect his airway, intubation and mechanical ventilation are indicated. Although increasing the level of inspired oxygen to maintain an oxygen saturation of greater than 90% in acute COPD exacerbations has been shown to be beneficial, doing so would not address this patient's hypercapnia and associated mental status changes, which need to be treated with ventilatory support. The use of NIV in eligible patients is effective and can avoid intubation in several disease states, particularly with COPD exacerbations or decompensated heart failure. In COPD, NIV has been shown to decrease the need for intubation, decrease mortality, and reduce length of hospital stay and complication rates. Compared with invasive mechanical ventilation, NIV uses pressure instead of volume to assist ventilation, and multiple different modes are available for use with limited data regarding the effectiveness of one mode of ventilation relative to another. However, none of the NIV modes would be appropriate in this patient with a contraindication to this therapy.

A 50-year-old woman presents with symptoms of early satiety, nausea, vomiting, and epigastric pain. Upper endoscopy reveals a large mass of undigested food particles in the stomach that is partially obstructing the pylorus. Which of the following is true regarding this condition? A. Most patients require surgery. B. It can be treated with oral administration of cellulase. C. Psychiatric treatment is critical in long-term management. D. The patient likely has patchy areas of alopecia. E. Peptic ulcer disease is a risk factor. ​

B. It can be treated with oral administration of cellulase. Bezoars are accumulations of indigestible material in the stomach. Bezoars often produce nonspecific symptoms and are usually found incidentally in patients undergoing upper gastrointestinal endoscopy or imaging. There are two types. Phytobezoars are composed of undigested vegetable matter (as in this patient). Risk factors for phytobezoars include previous gastric surgery and gastroparesis such as from diabetes. Peptic ulcer disease is not a risk factor (E). Bezoars produce obstructive symptoms but can also cause ulceration and bleeding. Diagnosis is suggested by an upper GI series and confirmed by endoscopy. Treatment generally consists of a combination of enzymatic degradation, endoscopic disruption, irrigation, and removal. Enzyme therapy can be performed with papain (present in meat tenderizers) or with cellulase. However, the use of papain has been associated with hypernatremia, gastric ulceration, and esophageal perforation so that cellulase is preferred. More recently, nasogastric Coca-Cola lavage has been successfully used. The mechanism responsible is believed to be a combination of the mucolytic effect of sodium bicarbonate (NaHCO3) and digestion of the bezoar by CO2 bubbles, all of which is exaggerated by the cola's acidity. Trichobezoars are composed of hair. It occurs most commonly in girls and young women who swallow their hair (trichophagia). Interestingly, most have long hair with patchy areas of alopecia (D), and many have an underlying psychiatric disorder, so psychiatric care is important in prevention (unlike phytobezoars) (C). The hair creates a cast of the stomach and strands of hair can extend into the small bowel (the so-called Rapunzel syndrome). Large trichobezoars are likely to require surgical removal because they are less likely to respond to enzymatic degradation (A).

A 75 year old woman is referred to the rheumatology clinic complaining of pain and swelling affecting the proximal interphalangeal joints (PIPJs) and distal interphalangeal joints (DIPJs) of both hands. Investigations show a haemoglobin of 118 g/L, white cell count 6.3 x 109/L, platelets 355 x 109/L and ESR 20 mm/hr. An X-ray of the hands and wrists is performed. Which radiological features are typical of osteoarthritis? A. Irregularity and fusion of the sacroiliac joints B. Joint space narrowing and subchondral sclerosis of the PIPJ and DIPJ of the hands C. Marginal erosions affecting the MCPJ of the hands D. Periarticular osteoporosis affecting the PIPJs and DIPJs in the hands E. Punched-out erosions of the first MTP joint of the feet ​

B. Joint space narrowing and subchondral sclerosis of the PIPJ and DIPJ of the hands Osteoarthritis is characterised by joint space narrowing due to cartilage erosion and subchondral sclerosis. Erosions at all MCPJs and periarticular osteoporosis are more typical of RA (though inflammatory OA typically causes inflammation at index and middle finger MCPJs) whereas punched-out erosions of the first MTP joint of the feet suggest gout. Irregularity and fusion of the sacroiliac joint (SIJ) suggests axial spondyloarthritis (axSpA) or psoriatic arthritis (PsA). ​

A 25-year-old man comes to the clinic due to a 3-month history of right shoulder pain. He also has swelling and pain at his heel that is worse when walking, jogging, or climbing stairs. The patient has tried using a topical "arthritis cream" without improvement, although over-the-counter analgesics have provided some relief. Medical history includes influenza pneumonia at age 12, but he has otherwise been healthy and takes no regular medications. The patient does not use tobacco, alcohol, or illicit drugs. His temperature is 37.3 C (99.2 F), blood pressure is 115/75 mm Hg, pulse is 56/min and regular, and respirations are 12/min. On examination, there is pain with resisted abduction at the shoulder and tenderness at the acromioclavicular junction. Palpation over the heels, iliac crests, and tibial tuberosities also elicits tenderness. Which of the following additional findings is most likely in this patient? A. Hand joint deformities B. Limited spine mobility C. Positive rheumatoid factor D. Proteinuria E. Subcutaneous nodules ​

B. Limited spine mobility Enthesitis is characterized by inflammation and pain at sites where tendons and ligaments attach to bone. It is commonly due to tendon or ligament stress and can manifest as acute swelling and pain. Examination findings include tenderness at tendon insertion sites. Enthesitis at the insertion of the Achilles tendon at the heel is often the most prominent presentation. However, enthesitis can also be seen at the costosternal junction, shoulders, elbows, hips, iliac crests, tibial tuberosities, and other locations. Chronic complications of enthesitis include fibrosis and calcification. Enthesitis may occur as an isolated disorder (e.g., plantar fasciitis) but is a characteristic finding in the spondyloarthropathies such as ankylosing spondylitis (AS), psoriatic arthritis, and reactive arthritis. It is particularly prominent in AS and classically manifests with heel pain at the insertion of the Achilles tendon; it can worsen with activity. Enthesitis of multiple regions is also frequently seen, as in this case. Low back pain (that classically improves with activity), loss of spinal mobility, and peripheral arthritis are other common musculoskeletal manifestations of AS. Educational objective: Enthesitis is characterized by inflammation and pain at sites of tendon and ligament attachment to bone. It is a common finding in ankylosing spondylitis and other spondyloarthropathies.

A 60-year-old woman is evaluated for a 6-month history of worsening dyspnea, especially with exertion. She was diagnosed with COPD 4 years ago, and pulmonary function tests performed 3 months ago showed an FEV1of 32% of predicted and an FEV1/FVC ratio of 55%. She has had three exacerbations in the last year. She completed pulmonary rehabilitation twice within the last 2 years and quit smoking 1 year ago. Medications are combination long-acting β2-agonist and inhaled glucocorticoid, roflumilast, and albuterol as needed. On physical examination, she is afebrile. Blood pressure is 120/74 mm Hg, pulse rate is 94/min, and respiration rate is 20/min. BMI is 28. Oxygen saturation breathing ambient air is 89%. There is no jugular venous distention. Pulmonary examination reveals decreased breath sounds. Cardiovascular examination reveals a loud pulmonic component of S2. Bilateral lower extremity edema is noted to a level above the ankles. Chest radiograph shows no infiltrate or mass. Arterial PO2 is 57 mm Hg (7.6 kPa) breathing ambient air. Echocardiogram reveals an ejection fraction of 60% and no valvular or wall motion abnormalities. The estimated mean pulmonary artery pressure is 52 mm Hg. Which of the following is the most appropriate treatment? A. Daily prednisone B. Long-term oxygen therapy C. Overnight pulse oximetry D. Repeat pulmonary rehabilitation

B. Long-term oxygen therapy Educational Objective: Treat COPD with long-term oxygen therapy. Key Point: Long-term oxygen therapy is indicated if patients with COPD meet the following criteria: (1) chronic respiratory failure and/or severe resting hypoxemia, defined as an arterial PO2 less than or equal to 55 mm Hg (7.3 kPa) or oxygen saturation less than or equal to 88% breathing ambient air, with or without hypercapnia; and/or (2) if there is evidence of pulmonary hypertension, peripheral edema suggesting right-sided heart failure, or polycythemia, in combination with an arterial PO2 less than 60 mm Hg (8.0 kPa) or oxygen saturation less than 88% breathing ambient air. The most appropriate treatment is to start long-term oxygen therapy (LTOT) for greater than 15 hours per day. The need for oxygen therapy should be evaluated in all stable patients with an FEV1 less than 35% of predicted or in patients with clinical symptoms or signs suggestive of respiratory failure or right-sided heart failure. LTOT is indicated if patients meet the following criteria: (1) chronic respiratory failure and/or severe resting hypoxemia, defined as an arterial PO2 less than or equal to 55 mm Hg (7.3 kPa) or oxygen saturation less than or equal to 88% breathing ambient air, with or without hypercapnia; and/or (2) if there is evidence of pulmonary hypertension, peripheral edema suggesting right-sided heart failure, or polycythemia, in combination with an arterial PO2 less than 60 mm Hg (8.0 kPa) or oxygen saturation less than 88% breathing ambient air. This patient's examination findings (loud pulmonic component of S2, peripheral edema) and estimated mean pulmonary artery pressure of 52 mm Hg on echocardiography indicate the presence of pulmonary hypertension. Therefore, this patient is an appropriate candidate for LTOT. The resting arterial PO2 or oxygen saturation values should be repeated and confirmed twice over a 3-week period before a decision about LTOT is made in a stable patient. A short course of systemic glucocorticoids is recommended in the treatment of acute exacerbations of COPD. However, long-term therapy with systemic glucocorticoids is associated with significant side effects and is not recommended in the management of COPD. Nocturnal oximetry may be useful in diagnosing hypoxia occurring overnight in patients with COPD that might benefit from oxygen therapy while sleeping. However, this patient's indication for continuous LTOT may be established with available data; therefore, an overnight pulse oximetry study would not be needed to guide therapy. Although pulmonary rehabilitation is helpful in improving quality of life in patients with advanced COPD, this patient has completed therapy on two occasions, and repeating pulmonary rehabilitation would not address her hypoxemia-associated pulmonary hypertension. However, after LTOT is started, entering a pulmonary rehabilitation program might be appropriate.

A 31-year-old Caucasian male complains of joint pains. He describes right knee pain, right heel pain and low-back pain. He was recently treated for urethral discharge at an outside clinic. He has no history of trauma or illicit drug use. He is afebrile, and his vital signs are stable. His right knee is swollen, tender and warm to touch; tenderness is also present over the Achilles tendon. Oral examination shows mouth ulcers. Synovial fluid analysis from the right knee shows a white blood cell count of 10,000/mm3 with many polymorphonuclear leukocytes but a negative Gram stain. Which of the following is the most appropriate initial pharmacotherapy for this patient? A. Antihistamines B. NSAIDs C. Antibiotics D. Colchicine E. Allopurinol F. Chondroitin sulfate ​

B. NSAIDs Reactive arthritis is a type of seronegative spondyloarthropathy. Classic reactive arthritis consists of a triad of nongonococcal urethritis, asymmetric oligoarthritis and conjunctivitis. The arthritis often involves the knee and sacroiliac spine, which explains this patient's back pain. In addition to the classic triad, mucocutaneous lesions and enthesitis (Achilles tendon pain) are common findings in reactive arthritis. Because not all symptoms are always present, there should be a high suspicion for reactive arthritis in the case of any asymmetric oligoarthritis associated with urethritis, conjunctivitis or mouth ulcers. Synovial fluid analysis is usually sterile. Nonsteroidal anti-inflammatory agents (NSAIDs) are the first line therapy during the acute phase of this condition. Educational objective: Reactive arthritis is a seronegative spondyloarthropathy resulting from enteric or genitourinary infection. Findings in reactive arthritis may include urethritis, conjunctivitis, mucocutaneous lesions, enthesitis and asymmetric oligoarthritis. Not all extra-articular manifestations need be present to suspect the diagnosis. NSAIDs are the first line therapy.

A 45-year-old male is evaluated in the clinic for asthma. His symptoms began 2 years ago and are characterized by an episodic cough and wheezing that responded initially to inhaled bronchodilators and inhaled corticosteroids but now require nearly constant prednisone tapers. He notes that the symptoms are worst on weekdays but cannot pinpoint specific triggers. His medications are an albuterol MDI, a fluticasone MDI, and prednisone 10 mg po daily. The patient has no habits and works as a textile worker. Physical examination is notable for mild diffuse polyphonic expiratory wheezing but no other abnormality. Which of the following is the most appropriate next step? A. Exercise physiology testing B. Measurement of FEV1 before and after work C. Methacholine challenge testing D. Skin testing for allergies E. Sputum culture for Aspergillus fumigatus ​

B. Measurement of FEV1 before and after work The patient presents with typical asthma symptoms; however, the symptoms are escalating and now require nearly constant use of oral steroids. It is of note that the symptoms are worse during weekdays and better on weekends. This finding suggests that there is an exposure during the week that may be triggering the patient's asthma. Often textile workers have asthma resulting from the inhalation of particles. The first step in diagnosing a work-related asthma trigger is to check FEV1 before and after the first shift of the workweek. A decrease in FEV1 would suggest an occupational exposure. Skin testing for allergies would not be likely to pinpoint the work-related exposure. Although A. fumigatus can be associated with worsening asthma from allergic bronchopulmonary aspergillosis, this would not result in a fluctuation in symptoms throughout the week. The patient does not require further testing to diagnose that he has asthma; therefore, a methacholine challenge is not indicated. Finally, the exercise physiology test is generally used to differentiate between cardiac and pulmonary causes or deconditioning as etiologies for shortness of breath.

A 65-year-old man comes to the office with a 1-year history of erectile dysfunction and decreased libido. He has hypertension that was diagnosed 5 years ago and is controlled with diet. The patient has no history of cardiac problems, unexpected weight loss, visual changes, or symptoms of neuropathy. Family history is unremarkable. On examination, there Is minimal bilateral gynecomastia without galactorrhea. Genitourinary examination shows decreased testicular size. The remainder of the physical examination, including visual fields on confrontation, is unremarkable. Laboratory results are as follows: Serum chemistry Creatinine 1.1 mg/dl Alanine aminotransferase 21 U/L Endocrine Hemoglobin A1c 5.8% FSH, serum 5 mU/mL LH, serum 3 mU/mL Testosterone, serum 174 ng/dL (normal, 300-800 ng/dL) TSH 1.3 µU/ml Repeat early-morning testosterone level is 162 ng/dL. Which of the following is the best next step in management of this patient? A. Angiography of the deep arteries of the penis B. Measurement of serum prolactin level C. Measurement of transferrin saturation D. Sildenafil ttreatment E. Testosterone therapy

B. Measurement of serum prolactin level This patient has gynecomastia and testicular atrophy. His laboratory results show low testosterone levels and inappropriately normal gonadotropin (i.e., LH, FSH) levels, indicating secondary (central) hypogonadism. Low levels of testosterone normally increase LH and FSH levels due to loss of feedback inhibition. Therefore, low or even normal LH and FSH levels suggest an inappropriate hypothalamic-pituitary response. Secondary hypogonadism can be due to mass lesions in the hypothalamus or pituitary, hyperprolactinemia (due to suppression of gonadotropin-releasing hormone), long-term use of glucocorticoids or opiates, or severe systemic illness. Secondary hypogonadism Is also more common In individuals with obesity or type 2 diabetes mellitus. Patients with secondary hypogonadism should have measurement of serum prolactin. In men with prolactinoma, gynecomastia is present only in approximately one third of patients, and galactorrhea is uncommon. Visual field defects are present only when the tumor impinges on the optic chiasm. Patients should also have screening for other pituitary hormone deficiencies (e.g., TSH). Pituitary imaging with MRI is indicated for patients with elevated prolactin, mass-effect symptoms (e.g., visual field defects), very low (<150 ng/dL) testosterone levels, or disruptions in other pituitary hormones. Educational objective: Secondary hypogonadism can be due to mass lesions in the hypothalamus or pituitary, hyperprolactinemia, long-term use of glucocorticoids or opiates, or severe systemic illness. Patients should have measurement of serum prolactin and screening for other pituitary hormone deficiencies. Pituitary imaging is indicated for patients with elevated prolactin, mass-effect symptoms, very low testosterone levels, or disruptions in other pituitary hormones.

A 64 year old woman with a 10-year history of rheumatoid arthritis affecting the hands and wrists, which is controlled with sulfasalazine 3 g daily and hydroxychloroquine 200 mg twice daily, presents with a disturbance of sensation and tingling affecting the thumb and anterior aspects of the index and second fingers of the right hand. On examination there is no evidence of active synovitis but there is altered perception of fine touch on examination of the affected digits in the right hand. Investigations are as follows: haemoglobin 120 g/L, white cell count 6.5 x 109/L, platelets 456 x 109/L, ESR 20 mm/ hr., CRP 6 mg/L. What is the most likely cause of the symptoms? A. Bone erosions secondary to the long-standing RA B. Median nerve compression C. Mononeuritis associated with rheumatoid vasculitis D. Osteoarthritis of the first CMC joint E. Ulnar nerve compression ​

B. Median nerve compression The symptoms are typical of median nerve compression syndrome, which is a recognised complication of RA. Mononeuritis can occur secondary to vasculitis in RA but this is unlikely since the disease is under good control. Osteoarthritis of the CMC joint presents with local pain rather than neurological symptoms. Bone erosions are a cause of pain in RA but not neurological symptoms. ​

A 14-year-old boy is brought to the emergency department after accidental ingestion of a chicken bone that lodged in his esophagus. Upper endoscopy is performed and the bone is successfully removed. However, the patient is incidentally found to have mild hypercalcemia on laboratory testing. On follow-up with his primary care provider 2 weeks later, he has no symptoms and clinical examination is unremarkable. Further questioning reveals that several of his family members also have mild hypercalcemia. Subsequent laboratory studies show a borderline high parathyroid hormone concentration, very low urinary calcium level, and normal 25-hydroxyvitamin D level. A mutation in which of the following receptors is most likely responsible for this patient's laboratory abnormalities? A. Intracellular receptor with a DNA-binding domain B. Membrane-bound receptor coupled with a G protein C. Transmembrane ligand-gated ion channel D. Transmembrane receptor associated with intrinsic tyrosine kinase activity Q E. Transmembrane receptor causing activation of Janus kinase/STAT pathway

B. Membrane-bound receptor coupled with a G protein Calcium-sensing receptors (CaSRs) are transmembrane Gq protein-coupled (metabotropic) receptors that regulate the secretion of parathyroid hormone (PTH) in response to changes in circulating calcium levels. Binding of calcium to CaSRs leads to the inhibition of PTH release, whereas low calcium levels allow increased PTH release. Familial hypocalciuric hypercalcemia (FHH) is a benign autosomal dominant disorder caused by defective CaSRs in the parathyroid gland and kidneys. In FHH, higher serum calcium levels are required to suppress the secretion of PTH. This raises the set point of calcium-induced regulation of PTH secretion. Patients with FHH have mild asymptomatic hypercalcemia, reduced urinary excretion of calcium, and high normal or mildly elevated PTH. Educational objective: Calcium-sensing receptors are G protein-coupled receptors that regulate the secretion of parathyroid hormone in response to changes in circulating calcium levels. Familial hypocalciuric hypercalcemia is a benign autosomal dominant disorder caused by defective calcium-sensing receptors in the parathyroid gland and kidneys.

A 19-year-old boy complains of difficulty breathing during exercise. He reports being in good physical shape, but occasionally experiences coughing and have to stop and catch his breath. This seems to occur more often in cold weather. The patient has no significant medical history other than seasonal allergies, and he takes no medication. He has some patchy dry skin over the elbows with some erythema and excoriations; otherwise the physical examination is normal. He is referred for spirometry, which is normal. Which of the following is the most appropriate next step in management? A. Albuterol challenge B. Methacholine challenge C. Chest x-ray D. Allergen skin testing E. Trial of albuterol ​

B. Methacholine challenge Methacholine challenge. Although the diagnosis of asthma is tested more often on the Pediatrics shelf examination, it may show up on the Medicine shelf examination and therefore a brief overview may be useful. Asthma is an intermittent obstructive lung disease that is classically described as airway hyper-responsiveness with variable airflow obstruction. It is a common condition, especially in those with other features of the atopic triad (seasonal allergies, eczema, and asthma). Symptoms are usually caused by common triggers such as infections, environmental exposures (smoke, allergens, etc.), medications (β-blockers, aspirin, NSAIDs), cold air, and exercise. Because symptoms of asthma may be intermittent, the physical examination is often normal. If spirometry is consistent with an obstructive pattern, then asthma can be differentiated from COPD by assessing the improvement in FEV1 after bronchodilator administration (e.g., albuterol); if FEV1 improves by at least 12%, then the diagnosis is more consistent with asthma. If spirometry is normal, as in this patient, a provocative test such as the methacholine challenge can be performed. Methacholine is a muscarinic agonist that will cause bronchoconstriction; asthmatics will be much more sensitive to lower doses of methacholine than the regular population and will develop an obstructive pattern on spirometry with a prolonged FEV1. (A) This patient has normal findings on spirometry and therefore the administration of albuterol will not change the findings. (C) A chest x-ray would likely be normal in this patient. During an acute exacerbation, there may be hyperinflation of the lungs. (D) Allergen skin testing is not a bad idea, especially since the patient has a history of seasonal allergies and eczema, but it is not the best next step in management. (E) If the initial workup is negative, then the patient might be encouraged to measure peak expiratory flow or try a bronchodilator while symptomatic. Since he is undergoing spirometry already, a methacholine challenge should be attempted first to make the

A 58-year-old woman comes to the clinic due to a 6-month history of left knee pain. The pain is worse with activity. She initially had pain only with ambulation, but then progressively developed rest pain at night. In addition, the patient has had intermittent morning stiffness of 10-15 minutes duration. There is no associated fever or weight loss. Her medical history is significant for hypertension, type 2 diabetes mellitus, gastroesophageal reflux disease, and obstructive sleep apnea. Current medications include hydrochlorothiazide, lisinopril, omeprazole, atorvastatin, insulin glargine, metformin, and aspirin. The patient's temperature is 37.2 C (98.9 F) and blood pressure is 146/86 mm Hg. Examination shows a mild effusion, tenderness, and decreased range of motion of the left knee. A fluctuant swelling along the posterior aspect of the knee is palpable. The patient undergoes arthrocentesis with synovial fluid analysis as follows: White blood ceils 1100/mm3 Gram stain no organisms Crystals absent Plain film x-ray of her knee joint would most likely reveal which of the following? A. Calcification of cartilaginous structures B. Narrowing of joint space and osteophytes C. Normal joint space with soft tissue swelling D. Periarticular osteopenia and joint margin erosions E. Punched-out erosions with a rim of cortical bone

B. Narrowing of joint space and osteophytes This patient with chronic knee pain and a bland synovial effusion (no organisms, <2,000 white blood cells/mm5) has typical features of osteoarthritis (OA). OA is a progressive arthritis that commonly affects the hands and weight-bearing joints. Major risk factors include obesity, advancing age. diabetes, and prior joint injury. Most patients will have exertional pain with few associated symptoms, although a subset can develop swelling, rest pain, and brief morning stiffness. Unlike autoimmune inflammatory arthritis (eg, rheumatoid arthritis [RA]), systemic symptoms are generally absent. The diagnosis of OA is based on clinical and x-ray findings. Crepitus on movement, periarticular bony enlargement, and painful or decreased range of motion are characteristic. This patient also has an effusion and a popliteal (Baker) cyst (posterior extrusion of synovial fluid into the gastrocnemius or semimembranosus bursa). Although the synovium can have signs of chronic inflammation on arthroscopic inspection or biopsy, joint fluid cell counts will be significantly lower than in RA or crystal-induced arthritis. Diagnostic findings on x-ray include a narrowed joint space, osteophytes, and subchondral sclerosis/cysts. Educational objective: Osteoarthritis is a progressive disorder that commonly affects the hands and weight-bearing joints. Major risk factors include obesity, advancing age. diabetes, and prior joint injury. Most patients will have exertional pain with few associated symptoms, although a subset can develop swelling, rest pain, and brief morning stiffness.

A 44-year-old man is evaluated in the emergency department because of polyuria and polydipsia. Over the past week he has noted increased urination and almost constant thirst. Medical history is significant for bipolar disorder. His only medication is lithium, and he does not take any over-the-counter preparations. On physical examination, temperature is normal, blood pressure is 135/80 mm Hg, pulse rate is 78/min, and respiration rate is 12/min. The remainder of the examination is normal. Results of laboratory studies show a serum sodium level of 152 mEq/L (152 mmol/L) , random plasma glucose level of 125 mg/dL (6.9 mmol/L) , and urine osmolality of 117 mOsm/kg. Injection of arginine vasopressin results in no significant increase in urine osmolality within 1 to 2 hours. Which of the following is the most likely cause of this patient's hypernatremia? A. Central diabetes insipidus B. Nephrogenic diabetes insipidus C. Osmotic diuresis D. Primary polydipsia

B. Nephrogenic diabetes insipidus Educational Objective: Diagnose nephrogenic diabetes insipidus. The most likely cause of this patient's hypernatremia is nephrogenic diabetes insipidus (DI) . This patient has serum hyperosmolality, as estimated by multiplying the serum sodium level by 2 (304 mOsm/kg [304 µmol/kg] ; normal, 275-295 mOsm/kg [275-295 µmol/kg]). The appropriate kidney response to hyperosmolality is to retain free water, which results in concentration of the urine above serum osmolality, up to a potential maximally concentration of greater than 800 mOsm/kg [800 µmol/kg] in younger, healthy patients. This response is not seen in this patient. Therefore, he has either DI or an osmotic diuresis. Patients with hyperosmolality who have sub-maximally concentrated urine without evidence of an osmotic diuresis have DI by definition. The diagnosis may be confirmed by a water deprivation test, which should normally demonstrate an increase in urine osmolality with decreased water intake and the resulting increase in serum osmolality; patients with DI fail to show an appropriate increase in urine osmolality as the serum osmolality rises. Distinguishing between central and nephrogenic DI in a patient who already demonstrates hyperosmolality can be done by measuring antidiuretic hormone (ADH) levels (patients with central DI have an inappropriately low level, whereas patients with nephrogenic DI have a normal to elevated level) or by evaluating the response to administering the vasopressor analogue desmopressin (arginine vasopressin) . A significant increase in urine osmolality (greater than 50%) within 1 to 2 hours after administration indicates insufficient endogenous ADH secretion, and, therefore, central DI, whereas a lack of response indicates kidney resistance to the effects of arginine vasopressin and, therefore, nephrogenic ID. Nephrogenic DI is an insensitivity of the cortical collecting duct to circulating ADH and can be caused by medications (for example, lithium) , hypokalemia, hypercalcemia, sickle cell disease and trait, and amyloidosis. Treatment requires adequate water intake, salt restriction, and, in some patients, a thiazide diuretic. Thiazide diuretics effectively block sodium reabsorption in the distal renal tubule, thereby causing natriuresis. An osmotic diuresis is most often caused by hyperglycemia and may be occur with ingestion of osmotically active substances such as mannitol. This patient's plasma glucose level does not exceed the kidney threshold for glucose reabsorption (which in most persons is 200 to 225 mg/dL [11.1-12.5 mmol/L]), and he does not report intake of any other potentially causative agents. Furthermore, solute diuresis is usually characterized by isotonicity of the urine, whereas this patient has a markedly hypotonic urine. Consequently, an osmotic diuresis is an unlikely cause of his hypernatremia. Patients with primary polydipsia also manifest polyuria and polydipsia but do not develop hypernatremia and hyperosmolality. These patients may develop hyponatremia and typically have a clearly identifiable psychiatric illness. K E Y P O I N T: Patients with hyperosmolality without glucosuria who have sub-maximally concentrated urine have diabetes insipidus by definition

Which of the following is a common complication of seronegative (ACPA and rheumatoid factor negative) rheumatoid arthritis? A. Felty's syndrome B. Osteoporosis C. Rheumatoid nodules D. Uveitis E Vasculitis ​

B. Osteoporosis Osteoporosis is a common complication of RA whether seropositive or seronegative. Felty's syndrome, vasculitis and nodules can occur in RA but they are rare complications and occur in seropositive RA. Uveitis is a feature of axial spondyloarthritis (axSpA), not RA. ​

A 55-year-old man comes to the physician with a 3-month history of headache, periodic loss of vision, and easy bruising. Physical examination shows splenomegaly. His hemoglobin concentration is 13.8 g/dL, leukocyte count is 8000/mm3, and platelet count is 995,000/mm3. Bone marrow biopsy shows markedly increased megakaryocytes with hyper-lobulated nuclei. Genetic analysis shows upregulation of the JAK-STAT genes. The pathway encoded by these genes is also physiologically responsible for signal transmission of which of the following hormones? A. Oxytocin B. Prolactin C. Glucagon D. ACTH E. Glycerol

B. Prolactin Easy bruising, periodic loss of vision, and splenomegaly in this patient with marked thrombocytosis and megakaryocytic hyperplasia are suggestive of essential thrombocythemia. The vast majority of patients with essential thrombocythemia have mutations involving the JAK-STAT signaling pathway. The hormones produced by the acidophil cells of the pituitary gland also transmit their signals via this pathway. ​

A 32-year-old man is evaluated for a 4-month history of an "upset stomach" that usually occurs after meals. He describes a sense of abdominal fullness and bloating associated with eating. He also has heartburn symptoms after meals at least four times each week. He has not had pain, difficulty swallowing, vomiting, weight loss, altered stool habits, or blood in the stool. There is no family history of gastrointestinal malignancy. He occasionally takes an over-the-counter H2 receptor antagonist antacid that temporarily improves his symptoms. On physical examination, vital signs are normal. Abdominal examination discloses a nontender epigastrium and no masses or lymphadenopathy. A complete blood count is normal. Which of the following is the most appropriate management? A. H2 receptor antagonist therapy B. Proton pump inhibitor therapy C. Test for Helicobacter pylori and treat if positive D. Upper endoscopy ​

B. Proton pump inhibitor therapy Educational Objective: Manage gastroesophageal reflux disease with dyspeptic features. This patient is considered to have gastroesophageal reflux disease (GERD) with dyspeptic features, and the most appropriate initial management is acid suppression with a proton pump inhibitor (PP!). Studies have shown that the majority of patients with dyspepsia who undergo upper endoscopy have normal findings, and those who do have abnormalities are found to have esophagitis. Therefore, predominant heartburn or regurgitation symptoms should be categorized as GERD rather than dyspepsia. Empiric therapy with a proton pump inhibitor (PP!) is the most cost-effective management strategy. There are six PPIs available in the United States: omeprazole, esomeprazole, lansoprazole, dexlansaprazole, pantoprazole, and rabeprazole, and all have approximately similar efficacy. PPI therapy is usually given once a day before a meal, usually before breakfast. In patients who require twice-daily dosing for symptom control (for example, patients with noncardiac chest pain, extraesophageal manifestations, incomplete response to standard therapy, or Barrett esophagus), the second dose should be administered before dinner. Patients should be reassessed in 6 weeks; if symptoms respond, the drug dosage can be decreased to the lowest effective dose. Long-term PPI therapy is generally considered safe for patients who require ongoing acid suppression. Although this patient had temporary relief with low-dose H2 receptor antagonist therapy, PP!s are more potent and longer-acting than H2 receptor antagonists and are considered to be superior for treat¬ment of both dyspepsia and heartburn. A test-and-treat strategy for Helicobacter pylori infection is appropriate for patients with dyspeptic symptoms without heartburn or alarm symptoms and who are from a geographic area (such as developing countries) with a high prevalence of H. pylori infection (>20%). Because this patient also has heartburn, this approach is not indicated. There has recently been some controversy as to whether eradication of H. pylori in patients with dyspepsia may increase the risk of post-eradication GERD, but this has not yet been definitively established. KEY POINT: Patients with gastroesophageal reflux disease with dyspeptic features should be treated with proton pump inhibitor therapy.

A 60-year-old man presents with dyspnea on exertion beginning approximately 2 months ago. He denies chest pain, cough, and wheezing but does admit to occasional chest tightness, particularly at night. He has no history of cardiovascular disease, lung disease or diabetes. He has a 30 pack-year history of cigarette smoking but quit five years ago. He takes medication daily for hypertension and gastroesophageal reflux disease. He does not exercise regularly. His BMI is 36 kg/m2. On physical examination his blood pressure is 140/88, and his other vital signs and examination are normal. Pulmonary function tests reveal a reduced FEV1 and preserved FVC, with an FEV1/FVC ratio of 65%, consistent with airflow limitation. Chest radiograph, complete blood count, electrocardiogram, and brain natriuretic peptide levels are normal. What is the next appropriate step in this patient's evaluation? A. Echocardiogram B. Pulse oximetry with ambulation (6-minute walk test) C. Exercise treadmill test D. High-resolution chest computed tomography 24-hour E. Holter monitoring

B. Pulse oximetry with ambulation (6-minute walk test) This patient likely has chronic obstructive pulmonary disease (COPD) manifesting as dyspnea on exertion. His main risk factor is cigarette smoking. The presence of hypertension, obesity, and lack of exercise can suggest cardiovascular disease as a second and possibly contributing cause. A normal brain natriuretic peptide level and electrocardiogram make primary cardiac disease less likely but do not exclude it. Anemia is excluded by a normal complete blood count. COPD is the likely diagnosis, and oxygen testing to exclude hypoxemia is the best first test, and this test may also explain his hypertension. Additional testing likely will be required if the oxygen titration test is unrevealing. ​

A 24-year-old nonsmoking woman was diagnosed with asthma 9 months ago, and has been on 500 μg beclomethasone and 9 μg formoterol, both twice daily, plus salbutamol as needed, since then. She has been asymptomatic for the past 3 months. Her FEV1 is 3.8 L (97% predicted). Which one of the following should you advise her to do? A. Continue the same treatment B. Reduce beclomethasone dosage C. Discontinue formoterol D. Discontinue salbutamol E. Discontinue medication ​

B. Reduce beclomethasone dosage The patient is on a high-dose inhaled corticosteroid (ICS) (1000 μg beclomethasone per day) and has been asymptomatic for 3 months. According to the Global Initiative for Asthma, asthma patients should have their dose adjusted to the lowest possible that assures control. High-dose ICS carries a risk of long-term side-effects, so her ICS should be reduced to a medium dose. Stepping down ICS doses by 25-50% at 3-month intervals is feasible and safe for most patients. If the patient remains asymptomatic for the next 3 months, formoterol may be discontinued. Salbutamol is prescribed as needed and should not be discontinued. Lastly, stepping down should be performed in intervals and, therefore, discontinuation of medication in a patient on high-dose treatment is contraindicated.

A 7-year-old boy is being evaluated for growth retardation. Brain MRI shows a 4 cm multiloculated, cystic, suprasellar lesion, which is bulging into the floor of the third ventricle and base of the brain. Calcifications are present. From which of the following structures is this mass most likely derived? A. Prolactin secreting cells of the anterior pituitary B. Remnants of the Rathke pouch C. Astrocytes D. Arachnoid cap cells E. Posterior pituitary cells

B. Remnants of the Rathke pouch Craniopharyngiomas are tumors arising from remnants of Rathke's pouch. The anterior pituitary is formed from an out-pouching of the pharyngeal roof and is called Rathke's pouch. The posterior pituitary gland arises from an extension of the hypothalamic neurons. Together, the anterior and posterior pituitary glands lie in the sella turcica at the skull base. During the time of pituitary development, remnants of Rathke's pouch cells can remain in the diencephalon (the posterior region of the forebrain). Neoplastic transformation of these "pouch cells" is called a craniopharyngioma. Typically, craniopharyngiomas have three components: solid, comprised of the actual tumor cells: cystic, filled with "machinery oil" liquid; and a calcified component. Any suprasellar mass with three components is highly suggestive of craniopharyngioma. Craniopharyngioma symptoms include headaches, visual field defects, and hypopituitarism, evidenced by the growth retardation of this child. Ultimately, compression of the pituitary stalk by craniopharyngioma leads to hyperprolactinemia by loss of dopaminergic inhibition. Craniopharyngiomas are usually tumors of childhood, being most frequently discovered between the ages of 5 and 10 years of age. Educational Objective: The anterior pituitary is formed from an out-pouching of the pharyngeal roof and is called Rathke's pouch. The posterior pituitary gland arises from an extension of the hypothalamic neurons. Craniopharyngiomas are tumors arising from Rathke's pouch remnants in the anterior pituitary. They characteristically have three components: solid, cystic, and calcified. They present during childhood, usually, with mass effect and visual deficits.

A 28-year-old woman seeks preconception counseling. She has a 4-year history of systemic lupus erythematosus (SLE) with manifestations of photosensitive rash, arthritis, and pericarditis; she has been treated with hydroxychloroquine and low-dose prednisone with good control of her symptoms for 18 months. She has never been pregnant. She also takes vitamin D and calcium. The physical examination and vital signs are normal. Laboratory studies indicate that the patient's SLE is quiescent. A recent urinalysis is normal, and a previously checked antiphospholipid panel and lupus anticoagulant were negative. SLE antinuclear antibody profile: Antinuclear antibodies: Positive (titer: 1:320), speckled pattern Anti-Ro/SSA antibodies: Positive Anti-double-stranded DNA antibodies: Negative Anti-U1 -ribonucleoprotein antibodies: Negative Anti-Smith antibodies: Negative The increased risk of preeclampsia and preterm delivery in SLE as well as avoidance of NSAIDs prior to conception and in the later stages of pregnancy is discussed. Which of the following also needs to be discussed with this patient based on her antibody profile? A. Need to discontinue hydroxychloroquine B. Risk of congenital heart block in her child C. Risk of developing lupus nephritis D. Risk of developing subacute cutaneous lupus

B. Risk of congenital heart block in her child Educational Objective: Provide preconception counseling to a patient with systemic lupus erythematosus who has positive anti-Ro/SSA antibodies. Neonatal congenital heart block affects approximately 2% of pregnancies in which the mother is positive for anti-Ro/SSA or anti-La/SSB antibodies. Preconception counseling regarding congenital heart block in her child is appropriate for this patient with systemic lupus erythematosus (SLE) who is positive for anti-Ro/SSA antibodies. Patients with SLE experience miscarriage, stillbirth, preeclampsia and premature delivery two to five times more often than patients without the disease. This patient has mild SLE, and her disease has been quiescent for 18 months; therefore, this is an appropriate time to attempt pregnancy. Expert opinion recommends conception when SLE has been quiescent for at least 6 months. A major risk to her child would be congenital heart block, which affects approximately 2% of pregnancies in which the mother is positive for anti-Ro/SSA or anti-La/SSB antibodies. Some of these newborns require pacing from birth if there is complete heart block at delivery. Pregnancies in mothers who are positive for anti-Ro/SSA or anti-La/SSB antibodies should be monitored closely and should include input from high-risk obstetrics and neonatology because these antibodies can pass the placenta and affect the developing cardiac conduction system. If the mother has had a previously affected child, subsequent pregnancies carry a 12% risk of congenital heart block. Positivity for anti-Ro/SSA or anti-La/SSB antibodies also confers a risk for neonatal lupus erythematosus, which is characterized by rash as well as hematologic and hepatic abnormalities that generally resolve when the antibody dissipates. The use of phototherapy for neonatal hyperbilirubinemia may cause the rash to develop because the antibody is associated with photosensitivity.

Considering the acute management of the patient in previous Question, what is the most important action? A. She should be immediately referred to the on-call surgical team B. She should be resuscitated with colloids/ crystalloids followed by blood products C. She should be started on octreotide to stop the bleeding D. She should be started on tranexamic acid to stop the bleeding E. She should undergo immediate endoscopy to treat any bleeding lesions ​

B. She should be resuscitated with colloids/ crystalloids followed by blood products This patient is clearly hemodynamically compromised and in shock. The most immediate management should focus on resuscitation. Upper GI endoscopy should not be carried out before resuscitation. ​

The best test to confirm eradication of H. pylori after treatment is: A. H. pylori serology B. Urea breath test C. Histologic biopsy D. Rapid urease test E. Antral mucosal biopsy with culture ​

B. Urea breath test A urea breath test is the best way to confirm eradication of H. pylori. The test relies on the fact that the bacteria hydrolyze urea. The patient is given radiolabeled urea to ingest orally. If H. pylori is present, the urea will be converted to ammonia and radiolabeled bicarbonate, which is then exhaled as carbon dioxide. The amount of exhaled carbon dioxide is quantified. Positive H. pylori serology (A) provides evidence of current infection if the patient has never been treated for it but will remain positive even after successful treatment; thus, it is not useful in this setting. Antral mucosa biopsy (E) with histologic examination (C) for the organism is the gold standard test. It is useful in the initial evaluation of patients with upper GI symptoms because it permits evaluation of the stomach via endoscopy at the time of biopsy. However, given its invasive nature and increased cost, it is not routinely recommended to confirm eradication. Cultures of the gastric mucosa are not routinely available at every laboratory, and a repeat endoscopy is required. The rapid urease test, also known as the campylobacter-like organism (CLO) test (D), is ideally used if another endoscopy and biopsy are being performed. The study requires placing a sample of gastric mucosa in a urea solution and then using a pH indicator to demonstrate the production of ammonia.

A medical student participating in an ultramarathon decides to avoid drinking during the event, believing that it will enhance his performance. When he collapses and is unable to complete the race, he is noted to be dehydrated with an associated increase in his plasma osmolality and a reduction in urine output. The later homeostatic response is most directly attributable to which of the following hormones: A. Insulin B. Vasopressin C. Aldosterone D. Cortisol E. Parathyroid hormone

B. Vasopressin Insulin and cortisol have no direct effects on urine output, ruling out options A and D. Parathyroid hormone regulates levels of calcium and phosphorus, but these minerals are not major determinants of urine flow, ruling out option E. Aldosterone and vasopressin act on the kidney, but aldosterone synthesis is triggered by a decrease in plasma sodium, and acts to increase sodium reabsorption in the kidney. While the latter action would decrease urine flow, plasma sodium would be increased rather than decreased in this dehydrated student, ruling out option C. On the other hand, vasopressin is released following stimulation of receptors in the hypothalamus by a rise in plasma osmolality, and increases the reabsorption of water in the kidney tubule, decreasing urine production. ​

A 66-year-old man has progressive shortness of breath due to COPD. He is currently able to do his activities of daily living, but has trouble walking more than 1 block. His physical examination reveals hyperinflation, increased resonance to percussion, and bilateral expiratory wheezes. He is on appropriate medical therapy for his stage of COPD. Which of the following is also indicated in the management of this condition? A. meningococcal vaccination B. yearly influenza vaccination C. weight reduction if obese D. Haemophilus influenzae B vaccination E. pneumococcal vaccination ​

B. Yearly influenza vaccination Yearly influenza vaccination is indicated for patients with COPD. Evidence for pneumococcal vaccination is not definitive but some advocate giving it as well. There is no role for vaccination with H influenzae B or meningo-coccus in patients with COPD. Dietary support to prevent malnutrition and improve muscle strength can be helpful. Exercise programs seem to provide subjective improvement as well. Obviously, stopping smoking is crucial.

A 32-year-old man presents to the clinic with symptoms of wheezing, cough, and shortness of breath. The symptoms started gradually at first but are worse now and interfering with his daily activities. He cannot identify any triggers and has no family or childhood history of asthma or eczema. On examination, he appears well and the pertinent findings are bilateral expiratory wheezes on auscultation, and the rest of the physical is normal. Further evaluation with pulmonary function tests reveals a reduced FEV1/FVC ratio that corrects with the administration of inhaled salbutamol. Which of the following statements about a diagnosis of idiosyncratic asthma is correct? A. known antigenic stimulus B. adult onset C.history of atopy D. positive skin tests E. high immunoglobulin E (IgE) levels

B. adult onset Idiosyncratic asthma is more likely to have its onset in adult life. A significant portion of asthmatics have no known personal or family history of atopy and have normal IgE levels. Upper respiratory infections can serve as triggers for idiosyncratic asthma.

A 23-year-old man is experiencing a flare of his asthma. He is using his salbutamol inhaler more frequently than usual and despite increasing his inhaled steroids he is still short of breath. Previously, his asthma was considered mild with no severe exacerbations requiring oral steroids or hospitalization. With the current flare, he is experiencing recurrent episodes of bronchial obstruction, fever, malaise, and expectoration of brownish mucous plugs. On examination, there is bilateral wheezing. The heart, abdomen, neurologic, and skin exams are normal. A CXR reveals upper lobe pulmonary infiltrates; the eosinophil count is 3000/mL, and serum precipitating antibodies to Aspergillus are positive. Which of the following is the most likely diagnosis? A. ascaris infestation B. allergic bronchopulmonary aspergillosis C. Churg-Strauss allergic granulomatosis D. Löeffler syndrome E. hypereosinophilic syndrome ​

B. allergic bronchopulmonary aspergillosis Allergic bronchopulmonary aspergillosis (in asthmatics) is the most likely diagnosis in this patient. Other causes of pulmonary infiltrates and eosinophilia include parasitic reactions, drugs, idiopathic causes which includes Löeffler syndrome (benign, acute eosinophilic pneumonia), chronic eosinophilic pneumonia, hypereosinophilic syndrome, and Churg-Strauss allergic granulomatosis.

A 75-year-old man presents to the clinic for follow-up assessment of his COPD. He has a 45 pack/year history of smoking and quit 1 year ago. At his last visit he complained of worsening dyspnea symptoms despite being on a long acting bronchodilator and tiotropium. Pulmonary function tests (PFTs), a blood gas on room air and CXR were ordered to evaluate his worsening symptoms. The PFTs confirm severe air flow obstruction that is nonreversible and the CXR shows hyperinflation but no other pulmonary pathology. His ABG reveals a PO2 of 52 mm Hg, and a PCO2 of 45 mm Hg. Which of the following statements concerning hypoxemia in COPD is correct? A. erythrocytosis is an appropriate compensation for hypoxemia, and phlebotomy will worsen symptoms B. continuous oxygen therapy is effective in producing symptomatic and hemo-dynamic improvement in severe hypoxia C. a PO2 of <60 mm Hg is an indication for continuous oxygen therapy D. a PO2 of 65 mm Hg or below is an indication for supplemental oxygen during air travel E. continuous supplemental oxygen improves functional ability but does not alter the natural history of obstructive airways disease with severe hypoxemia

B. continuous oxygen therapy is effective in producing symptomatic and hemo-dynamic improvement in severe hypoxia Continuous oxygen supplementation improves symptoms, prolongs life and decreasing hospitalizations in COPD patients with hypoxemia. In prolonged air travel, even those with PO2 in the mid 70s should be considered for oxygen therapy. A PO2 below 55 mm Hg is an indication for oxygen therapy, but between 55 and 60 mm Hg, associated evidence of right heart dysfunction should also be present before therapy is commenced.

Which of the following patients is appropriately diagnosed with asthma? A. A 24-year-old woman treated with inhaled corticosteroids for cough and wheezing that has persisted for 6 weeks following a viral upper respiratory infection. B. A 26-year-old man who coughs and occasionally wheezes following exercise in cold weather. C. A 34-year-old woman evaluated for chronic cough with an FEV1/FVC ratio of 68% with an FEV1 that increases from 1.68 L (52% predicted) to 1.98 L (61% predicted) after albuterol (18% change in FEV1). D. A 44-year-old man who works as a technician caring for the mice in a medical research laboratory complains of wheezing, shortness of breath, and cough that are most severe at the end of the week. E. A 60-year-old man who has smoked two packs of cigarettes per day for 40 years who has dyspnea and cough, and airway hyperreactivity in response to methacholine. ​

C. A 34-year-old woman evaluated for chronic cough with an FEV1/FVC ratio of 68% with an FEV1 that increases from 1.68 L (52% predicted) to 1.98 L (61% predicted) after albuterol (18% change in FEV1). The preferred method for diagnosing asthma is demonstration of airflow obstruction on spirometry that is at least partially reversible. This is demonstrated in option C with a decreased FEV1/FVC ratio, decreased FEV1, and a significant increase in FEV 1 following administration of albuterol. For an individual to be considered responsive to a bronchodilator, the individual should experience an increase in either FEV 1 or FVC of at least 200 mL and 12%. Option A describes someone with postviral cough syndrome, which can persist for several weeks following a viral upper respiratory infection. Option B describes someone with exercise-induced bronchoconstriction (EIB), which, in the absence of other symptoms to suggest asthma, should not be diagnosed as asthma. Isolated EIB lacks the characteristic airway inflammation of asthma and does not progress to asthma. While it is estimated that 80-90% of individuals with asthma experience EIB, many individuals who have EIB do not also have asthma.

A 45 year old smoker is sent for pulmonary function testing because of breathlessness, wheeze and productive cough. There are no other significant respiratory exposures. The family physician has trialed a salbutamol inhaler, which has been somewhat helpful. Spirometry reveals: FEV1 2.6 L FVC 4.4 L FEV1/FVC 0.59 Following nebulization of 2.5 mg salbutamol: FEV1 3.2 L FVC 4.4 L FEV1/FVC 0.73 What does the respiratory function testing reveal? A. A mixture of obstruction and restriction B. A restrictive defect C. A reversible obstructive defect D. An irreversible obstructive defect E. Inhaled salbutamol has confounded testing ​

C. A reversible obstructive defect The patient has an obstructive defect that entirely resolves following nebulized bronchodilator; this is in keeping with asthma despite the patient's smoking history. Inhaled therapy taken on the morning of reversibility testing can confound the test but in this instance there is a clear result with strong evidence of reversibility.

A 24-year-old woman is evaluated because of worsening symptoms of asthma. She uses an as-needed albuterol inhaler two to three times per week and has been waking at night at least once a week with asthma symptoms that require use of the inhaler. She is still able to perform most of her daily activities, including regular exercise, if she uses albuterol for prevention. She is allergic to house dust mites, ragweed, grass, trees, and cats. On physical examination, vital signs are normal. Pulmonary examination is normal with no wheezing. Spirometry shows forced expiratory volume in 1 second (FEV1) of 85% of predicted and an FEV1/forced vital capacity ratio of 80% of predicted. Which of the following is the most appropriate treatment? A. Add a long-acting β2-agonist B. Add a long-acting β2-agonist and a low-dose inhaled glucocorticoid C. Add a low-dose inhaled glucocorticoid D. Advise scheduled use of albuterol E. Refer for allergen immunotherapy ​

C. Add a low-dose inhaled glucocorticoid Educational Objective: Treat mild persistent asthma. The most appropriate treatment is to add a low-dose inhaled glucocorticoid. This patient has mild persistent asthma. She has symptoms more than 2 days per week but not daily, and she wakes up once a week but not nightly. The preferred therapy for this patient is a low-dose inhaled glucocorticoid added to an as-needed short-acting β2-agonist. Alternatives to inhaled glucocorticoids include a leukotriene receptor antagonist and theophylline. Adding a long-acting β2-agonist is not recommended for patients with asthma who are not already receiving inhaled glucocorticoid therapy. Providing a combination of long-acting β2-agonist and inhaled corticosteroid therapy is not indicated. Based on the National Asthma Education and Prevention Program guidelines and a Food and Drag Administration black box warning, treatment with inhaled glucocorticoids should be started first. Long-acting β2-agonist should be added only if medium-dose inhaled glucocorticoid therapy does not control symptoms. Scheduled use of albuterol is not recommended because it can mask ongoing airway inflammation and the need to provide anti-inflammatory therapy with inhaled glucocorticoids. Allergen immunotherapy is an option for some patients. However, its benefits are mostly for those with allergic rhinitis. It would not be recommended for patients with mild persistent asthma. KEY POINT: The preferred therapy for mild persistent asthma is a low-dose inhaled glucocorticoid added to an as-needed short-acting β2-agonist.

A 58-year-old woman is evaluated because of daily wheezing and breathlessness during allergy season. She has a long history of seasonal allergies, and her usual symptoms include itchy eyes and runny nose. However, over the past several years, these symptoms have been periodically accompanied by episodes of wheezing and shortness of breath. These episodes have increased to the point that they limit her activities several times a week. Medical history is otherwise unremarkable. Her medications are as-needed fexofenadine and antihistamine eye drops. She is a never-smoker. On physical examination, the patient is afebrile, blood pressure is 130/76 mm Hg, pulse rate is 79/min, and respiration rate is 14/min; BMI is 28. Watery eyes and conjunctival irritation are noted. Pulmonary examination reveals expiratory wheezing. Cardiac examination is normal. A chest radiograph is unremarkable. Spirometry shows an FEV1 of 74% of predicted, which improves by 18% with a bronchodilator. In addition to a short-acting β2-agonist, which of the following is the most appropriate treatment? A. Add a leukotriene antagonist B. Add a low-dose inhaled glucocorticoid C. Add a low-dose inhaled glucocorticoid and long-acting β2-agonist D. Recommend daily oral antihistamine use

C. Add a low-dose inhaled glucocorticoid and long-acting β2-agonist Educational Objective: Treat moderate persistent asthma with combination inhaled glucocorticoid and long-acting β2-agonist therapy. Key Point: The most appropriate therapy for moderate persistent asthma is combination low-dose inhaled glucocorticoid and long-acting β2-agonist therapy. The most appropriate treatment is the combination of a low-dose inhaled glucocorticoid and long-acting β2-agonist (LABA) in addition to an as-needed short-acting β2-agonist. Appropriate initial classification of asthma is important in guiding the strength and type of therapy that is likely to be most effective. Underestimating asthma severity may delay resolution of symptoms and may inadequately manage airway inflammation. This patient has newly diagnosed moderate persistent asthma based on her daily symptoms and spirometry showing an FEV1 of greater than or equal to 60% but less than 80% of predicted that responds to bronchodilator therapy. The preferred regimen for treating moderate persistent asthma is the combination of a low-dose inhaled glucocorticoid and LABA. Adding a LABA to inhaled glucocorticoid therapy has increased synergistic anti-inflammatory properties compared with either agent alone. Additionally, recent data have demonstrated that long-acting anticholinergic therapy may be appropriate as step-up therapy if moderate persistent asthma does not resolve with combination inhaled glucocorticoid and LABA therapy. A leukotriene agonist or low-dose inhaled glucocorticoid are recommended treatment options for patients with mild persistent asthma, defined as asthma symptoms occurring more than 2 days per week but not daily with an FEV1 of greater than or equal to 80% of predicted. The addition of either therapy alone may not adequately treat the degree of asthma seen in this patient. This patient requires anti-inflammatory controller therapy. Recommending daily use of her oral antihistamine would not address her lower airway inflammation and would not be an appropriate intervention to control her asthma symptoms.

A 28-year-old man with a history of asthma is hospitalized for an acute exacerbation. After discharge, he follows up in clinic. A further history is obtained, and the patient reports daily symptoms with night-time awakenings occurring more than 1 night each week. The symptoms are especially prominent with exercise. He uses inhaled albuterol as his reliever medication and inhaled fluticasone as his controller medication. Spirometry is performed and shows a mildly reduced FEV1/FVC ratio from his baseline. Which of the following changes to the patient's therapeutic regimen would have been appropriate and might have prevented the acute exacerbation? A. Adding ipratropium B. Adding oral steroids C. Adding salmeterol D. Adding tiotropium E. Adding theophylline F. Adding montelukast G. No change to current reg ​

C. Adding salmeterol. Based on his symptoms, this patient's asthma severity can be graded as "moderate." Appropriate management of asthma is somewhat similar to COPD in that there is a stepwise approach to achieve adequate control of the disease. The goal is to have daily symptoms 2 or less times a week with no nightly symptoms. All patients should be on a reliever medication (rapid acting β2 agonist such as albuterol), but their controller medications can be adjusted based on the severity. Because this patient requires a step up in therapy, the most appropriate next step would be to add a long acting β2 agonist (e.g., salmeterol). These medications are especially good for asthma with a strong exercise component. (A, D) Short and long-acting anticholinergics are more useful in COPD when compared to asthma. (B) Oral steroids are used for acute exacerbations and for chronic severe asthma, using the lowest dose possible; they are not used for chronic therapy in COPD. The patient might be on a steroid taper after his recent acute exacerbation; however, the question is asking about changes to his chronic regimen. (E, F) Theophylline is not a first-line treatment but may be used in patients with difficulty to control asthma. Montelukast and other leukotriene antagonists may be useful as adjunctive therapy. (G) A change in therapy is warranted to achieve symptomatic control.

A 45-year-old woman comes to the physician with knee pain. She has a long history of rheumatoid arthritis affecting the joints of her hands, wrists, and knees. The patient has mild residual pain and joint stiffness on low-dose methotrexate but has otherwise done well. Over the last 2 days, the pain in her right knee has become much more severe and was associated with an episode of chills. Her temperature is 38.7 C (101.6 F), blood pressure is 120/70 mm Hg, pulse is 100/min, and respirations are 18/min. Physical examination shows mild swelling in the joints of her hands and wrists. The right knee is red and more swollen than on previous examinations; active and passive range of motion at the right knee are limited due to pain. After appropriate diagnostic testing, which of the following is most likely to be indicated to treat this patient's current condition? A. Allopurinol B. Anti-cytokine agent C. Antibiotics D. Colchicine E. Glucocorticoids F. High-dose methotrexate ​

C. Antibiotics This patient's clinical presentation (acute monoarthritis and fever with underlying rheumatoid arthritis [RAJ) is suspicious for acute septic arthritis. Septic arthritis is more common in patients with underlying joint abnormalities, such as osteoarthritis, RA, or prosthetic joints. Patients with RA are at especially high risk due to impaired phagocytosis and systemic immunosuppression. Occasionally, septic arthritis in patients with underlying RA may present without overt signs of infection (eg, fever, chills, peripheral leukocytosis); this is especially common in elderly patients and may lead to a delayed diagnosis. Diagnosis of septic arthritis is confirmed with synovial fluid analysis showing elevated leukocytes (50,000-150,000/mm'), positive Gram stain, and positive fluid culture. Blood cultures are positive in 50% of patients and should be obtained prior to antibiotic therapy. Treatment of septic arthritis requires prompt administration of intravenous antibiotics and adequate joint drainage to lessen the likelihood of joint destruction. Educational objective: Septic arthritis is characterized by acute monoarthritis, often with fever and restricted range of motion. It is more common in patients with underlying joint disease. Synovial fluid analysis should be performed promptly to confirm the diagnosis.

What is the antigenic target of rheumatoid factors? A. Lambda light chain B. Kappa light chain C. Antibody Fc D. Antibody Fab E. Antibody F(ab′)2.

C. Antibody Fc RFs are IgM antibodies directed to the Fc portion of the IgG molecule. These antibodies likely react with a conformational determinant that becomes available when an IgG molecule binds to its antigen. Denatured IgG preparations may also expose this determinant. Although the predominant antigenic target of RFs is the Fc portion, some antibodies to IgG may react to other parts of the IgG molecule.

A 23-year-old cross country runner with a history of asthma complains of worsening dyspnea and cough during exercise. He currently takes inhaled fluticasone and a long-acting β2 agonist, with inhaled albuterol used as a rescue medication. He is allergic to aspirin and penicillin. He asks about adding montelukast to his regimen. What is the mechanism of action of this medication? A. Blocks muscarinic acetylcholine receptors B. Irreversible inhibition of cyclooxygenase-1 and 2 C. Blocks the receptor for LTC4, LTD4, and LTE4 in the leukotriene pathway D. Inhibition of phospholipase A2 and disruption of neutrophil endothelial adherence

C. Blocks the receptor for LTC4, LTD4, and LTE4 in the leukotriene pathway Blocks the receptor for LTC4, LTD4, and LTE4 in the leukotriene pathway. Montelukast acts as a leukotriene modifier and is a great therapy for asthma in a subset of patients, especially those with an aspirin allergy or with primarily exercise-induced asthma. (A) This is the mechanism of ipratropium, which can be used in acute asthma exacerbations but is used more commonly in COPD (both for chronic treatment and acute exacerbations). (B) Aspirin irreversibly inhibits COX-1 and COX-2. Both the cyclooxygenase pathway and the leukotriene pathway are part of the eicosanoid pathway, and they diverge from the precursor arachidonic acid. (D) Arachidonic acid is produced from membrane phospholipids via the enzyme phospholipase A2, which is inhibited by glucocorticoids. Glucocorticoids also cause lymphocyte destruction and a disruption in the adherence of neutrophils to endothelium, which is why patients that acutely start systemic steroids will have an increased white blood cell count that are predominantly neutrophils.

A 72-year-old man is evaluated for follow-up after his first exacerbation of moderate chronic obstructive pulmonary disease. The dose of prednisone was tapered, and treatment was stopped 1 week ago. He is afebrile and his cough has decreased. He still has dyspnea despite adherence to the treatment regimen. Medical history is otherwise unremarkable. He has a 45-pack-year smoking history but stopped 6 months ago. Medications are tiotropium, a salmeterol dry powder inhaler, and an albuterol metered-dose inhaler as needed. He uses albuterol up to six times daily. On physical examination, vital signs are normal. Pulmonary examination shows occasional expiratory wheezes. Oxygen saturation is 94% on ambient air. Which of the following is the most appropriate next step in management? A. Add a fluticasone inhaler B. Add oxygen therapy C. Check the patient's inhaler technique D. Resume treatment with prednisone ​

C. Check the patient's inhaler technique Educational Objective: Manage lack of response to appropriate therapy for chronic obstructive pulmonary disease (COPD) by checking the patient's inhaler technique. The most appropriate next step in management is to check the patient's inhaler technique. This patient has moderate (stage 2) COPD and is on an appropriate medication regimen that includes a long- acting anticholinergic agent and a long-acting P2-agonist. The patient is adherent to this regimen. The most appropriate next step is to check his inhaler technique. Inhaler therapy is very important in the treatment of COPD. Several drag- and patient-dependent factors, including age, eyesight, finger dexterity, degree of lung function, cognitive function, breathing pattern, inhaler technique, drug formulation, and device-related variables, affect the distribution of the drug and clinical outcomes. Patient-dependent factors also change over time. It is very important to ensure that the inhaler technique is correct. Adding an inhaled glucocorticoid is not an appropriate option before the inhaler technique is assessed. Oxygen therapy is a major component of treatment for patients with very severe COPD. It is usually prescribed for patients who have arterial PO2 of 55 mmHg (7.3 kPa) or less or oxygen saturation of 88% or less with or without hypercapnia. It is also prescribed for those with arterial PO2 of 56 to 59 mmHg (7.4-7.8 kPa) or oxygen saturation of less than 89% with one or more of the following: pulmonary hypertension, evidence of cor pulmonale or edema as a result of right sided heart failure, or hematocrit greater than 56%. This patient has none of these indications for oxygen therapy. Resuming treatment with prednisone is not appropriate at this time because this patient was treated adequately with a short course of a systemic glucocorticoid. Long-term use of glucocorticoids is not recommended because there is no evidence of benefit and these drugs have many side effects. KEY POINT: If a patient is not responding to appropriate therapy for COPD, the inhaler technique should be assessed before therapy is adjusted.

A 70-year-old man presents with an 8-hour history of acute abdominal pain. On examination, the patient is febrile to 101°F, with a blood pressure of 105/70 mm Hg and a heart rate of 130 beats per minute and has diffuse abdominal tenderness with rebound and guarding; the rectal examination is guaiac positive. Laboratory values are significant for a white blood cell count of 16,000 cells/µL and a hematocrit of 26%. CT demonstrates extravasation of oral contrast in the proximal duodenum. After resuscitation, management consists of: A. Closure of the perforation with omental patch plus an HSV B. Closure of the perforation and omental patch via the open approach C. Closure of the perforation with omental patch and duodenotomy with oversewing of posterior ulcer D. Vagotomy and antrectomy with oversewing of the posterior ulcer and omental patch E. Closure of the perforation and omental patch via laparoscopic approach ​

C. Closure of the perforation with omental patch and duodenotomy with oversewing of posterior ulcer. The presentation of oral contrast extravasation in the proximal duodenum (or free air under the diaphragm) combined with anemia and guaiac-positive stool is highly suggestive of a "kissing" duodenal ulcer. This represents a rare combination of an anterior duodenal ulcer that perforates into the peritoneum and a posterior ulcer that erodes into the gastroduodenal artery and bleeds. The majority of perforated ulcers can be managed by simple ulcer closure with an omental (Graham) patch. This can be achieved via an open or laparoscopic approach. In this patient, one must rule out a bleeding posterior ulcer. This would best be achieved via an anterior duodenotomy across the pylorus. If a posterior ulcer is identified, it should be oversewn.

A 55-year-old man is being evaluated for constipation. There is no history of prior gastrectomy or of upper GI symptoms. Hemoglobin is 10 g/dL, mean corpuscular volume (MCV) is 72 fL, serum iron is 4 μg/dL (normal 50-150 μg/dL), iron-binding capacity is 450 μg/dL (normal 250-370 μg/dL), saturation is 1% (normal 20%-45%), and ferritin is 10 μg/L (normal 15-400 μg/L). Which of the following is the best next step in the evaluation of this patient's anemia? A. Red blood cell folate B. Serum lead level C. Colonoscopy D. Bone marrow examination E. Hemoglobin electrophoresis with A2 and F levels

C. Colonoscopy The patient has a microcytic anemia. A low serum iron, low ferritin, and high iron-binding capacity are diagnostic of iron-deficiency anemia. Most iron-deficiency anemia is explained by blood loss. The patient's symptoms of constipation point to blood loss from the lower GI tract. Colonoscopy would be the highest-yield procedure. Barium enema misses 50% of polyps and a significant minority of colon cancers. Even patients without GI symptoms who have no obvious explanation (such as menstrual blood loss or multiple prior pregnancies in women) for their iron deficiency should be worked up for GI blood loss. Folate deficiency presents as a megaloblastic anemia with macrocytosis (large, oval-shaped red cells) and hypersegmentation of the polymorphonuclear leukocytes. Lead poisoning can cause a microcytic hypochromic anemia, but this would not be associated with the abnormal iron studies and low ferritin seen in this patient. Basophilic stippling or target cells seen on the peripheral blood smear would be important clues to the presence of lead poisoning. Although a bone marrow examination will prove the diagnosis by the absence of stainable iron in the marrow, the diagnosis of iron deficiency is clear from the serum studies. Thalassemia (diagnosed by hemoglobin electrophoresis) is not associated with abnormal iron studies. The most important issue is now to find the source of the iron loss.

A 48 year old woman with seropositive rheumatoid arthritis (RA) has persistent pain and swelling of the wrists, elbows, shoulders and MCP joints of both hands despite 12 months' therapy with methotrexate 25 mg weekly, folic acid 5 mg weekly, sulfasalazine 3 g daily and hydroxychloroquine 400 mg daily. Examination reveals tenderness and swelling of 10 affected joints, and a raised ESR (45 mm/hr.). She rates the activity of her arthritis as 80/100 on a visual analogue scale yielding a DAS28 of 6.45. What would be the most appropriate next course of action? A. Commence treatment with apremilast 30 mg twice daily B. Commence treatment with the interleukin (IL)-17 blocker secukinumab 150 mg monthly C. Commence treatment with the TNF-α inhibitor etanercept 50 mg weekly D. Increase the frequency of methotrexate to 3 times weekly E. Stop sulfasalazine and substitute leflunomide 10 mg daily ​

C. Commence treatment with the TNF-α inhibitor etanercept 50 mg weekly The patient has failed to respond adequately to triple therapy and progression to biologic treatment would be indicated. Neither secukinumab nor apremilast is effective in RA and would not be appropriate choices. Substitution of sulfasalazine with leflunomide is unlikely to be effective. Increasing the frequency of methotrexate would not be indicated since the patient is already on the maximum recommended dose. ​

A 45-year-old man comes to the emergency department with a 2-week history of progressively worsening shortness of breath. The patient has no history of cardiopulmonary disease, although his wife, who has accompanied him, states that he snores heavily at night. In addition, over the last several months his hands and feet have become enlarged; he is not able to take his wedding ring off and his shoes no longer fit. The patient has no significant medical history and takes no medications. He does not use tobacco or alcohol. On examination, he has coarse facial features with prominent frontal bones and jaw and crowding of the pharyngeal space. Extremities show enlarged, swollen hands and feet. There are fine crackles at the lung bases. Which of the following is most like to be found on echocardiogram? A. Apical left ventricular aneurysm B. Asymmetric thickening of the interventricular septum C. Concentric left ventricular hypertrophy D. Flail posterior mitral valve leaflet E. Moderate pericardial effusion

C. Concentric left ventricular hypertrophy his patient has several typical manifestations of acromegaly, including coarsening of facial features, pharyngeal crowding, and enlargement of hands and feet. Acromegaly is caused by excessive secretion of growth hormone, usually from a pituitary somatotroph adenoma. Growth hormone increases hepatic production of insulin-like growth factor-l (IGF-I), which causes excessive growth of bone and soft tissues, leading to the clinical manifestations of the disease. In the heart, acromegaly causes concentric myocardial hypertrophy leading to diastolic dysfunction, along with left ventricular dilation and global hypokinesis. This cardiomyopathy Is worsened by concurrent hypertension, obstructive sleep apnea (seen in this patient), and valvular heart disease, which are common in acromegaly. Complications include heart failure (eg, dyspnea, crackles at bases) and arrhythmias. Cardiovascular disease is the leading cause of death in patients with acromegaly, but normalization of growth hormone levels following successful treatment markedly reduces cardiovascular mortality. Educational objective: Acromegaly causes concentric myocardial hypertrophy, diastolic dysfunction, left ventricular dilation, and global hypokinesis. This cardiomyopathy is often worsened by concurrent hypertension, obstructive sleep apnea, and valvular heart disease. Complications include heart failure and arrhythmias.

A 65-year-old man comes to the office due to intermittent right knee pain for the last 3 or 4 months. His symptoms are worse at the end of the day; and he has had mild swelling at the knee following strenuous exertion. There is no associated fever, malaise, or skin rash. The patient takes metformin for type 2 diabetes mellitus and valsartan for hypertension. On examination, he is afebrile and has a BMI of 36 kg/m2. The right knee has bony enlargement and a small effusion but no warmth or erythema. Range of motion elicits pain. Arthrocentesis is performed, and synovial fluid analysis shows a leukocyte count of 1800/mm3 with no crystals. His serum creatinine is 0.8 mg/dL and serum uric acid level Is 5.6 mg/dL. In addition to a supervised exercise program, which of the following medications is the most appropriate treatment for relief of this patient's symptoms? A. Acetaminophen with oxycodone B. Colchicine C. Diclofenac D. Febuxostat E. Prednisone ​

C. Diclofenac This patient has features suggesting osteoarthritis (OA), including chronic joint pain, periarticular bony enlargement, and a bland synovial effusion (ie, low synovial leukocyte count, no crystals). Risk factors for OA include advancing age, obesity, diabetes, and prior joint injury. Patients typically have no or only mild inflammatory features (eg, erythema, morning stiffness, systemic symptoms). Examination can show crepitus and a small effusion. The diagnosis can be confirmed with x-ray, revealing narrowed joint space, osteophytes, and possible subchondral sclerosis. The initial management of OA should emphasize nonpharmacologic measures including exercise, weight loss, and activity modification. Nonsteroidal anti-inflammatory drugs (NSAIDs) (eg, diclofenac) are the drug of choice for relief of pain in OA but do not alter the progression of the disease. Educational objective: The initial management of osteoarthritis should emphasize nonpharmacologic measures including exercise, weight loss, and activity modification. Nonsteroidal anti-inflammatory drugs are the drug of choice for relief of pain in osteoarthritis but do not alter the progression of the disease.

A 24-year-old woman with a history of asthma complains of worsening chest tightness and cough. She currently has an albuterol inhaler that she uses as needed, but says that it does not help very much. She has no other medical problems and takes no other medications. The physical examination is unremarkable. Which of the following is the most appropriate next step in management? A. Add an inhaled corticosteroid B. Add an inhaled anticholinergic C. Education about proper use of inhalers D. Workup for a different diagnosis

C. Education about proper use of inhalers Education about proper use of inhalers. Education is truly a cornerstone of asthma therapy. Patients should be educated about how to properly use their inhaler, how to monitor their pulmonary function (with peak expiratory flow), and how to recognize and avoid triggers. Proper education about inhalers includes shaking the medication before use, correct positioning, taking a slow deep breath while administering the medication, and holding the medication within the lungs for at least 5 seconds before breathing out. In addition to the importance of technique, patients should also be educated about the timing of use. Patients that know that they will be experiencing a trigger should use the inhaler 10 minutes in advance (e.g., used before exercise). As a general rule, the correct answer on the shelf examination usually follows the principle that the least invasive maneuver should be attempted first, and therefore choices such as obtaining a further history and patient education are usually the right answers. (A) An inhaled corticosteroid is the next step in therapy for worsening symptoms, however an additional medication might be avoided in this patient if she begins to use her inhaler correctly. (B) An inhaled anticholinergic such as ipratropium can be used to improve delivery of the β2 agonist reliever medications and improve bronchodilation, but this is not the next step. (D) Medication compliance and proper technique should be addressed before assuming that the diagnosis of asthma is incorrect.

A 34-year-old woman comes to the clinic with a facial rash, joint pains and oral ulcers. She has no associated fever or other systemic symptoms and otherwise feels well. Medical history is unremarkable, and the patient's only concurrent medication is an oral contraceptive. She does not smoke and consumes alcohol only on social occasions. Antinuclear and anti-double-stranded ONA antibody titers are elevated. Creatinine and complement levels are normal. The patient is diagnosed with systemic lupus erythematosus, and therapy with hydroxychloroquine is initiated. Which of the following evaluations should be done periodically while the patient is taking this medication? A. Audiometry B. Echocardiogram C. Eye examination D. Liver function panel E. Pulmonary function tests F. Thyroid-stimulating hormone

C. Eye examination This patient, with a new diagnosis of systemic lupus erythematosus (SLE), has limited disease, with skin, joint, and mucosal manifestations. Disease activity in SLE should be monitored with periodic blood counts, inflammatory markers (e.g., erythrocyte sedimentation rate), complement levels, and anti-double-stranded DMA antibody titers. Patients should also be screened regularly for lupus nephritis with urinalysis, serum creatinine, and urine protein assay. The choice of drug therapy in SLE depends on the degree of organ involvement. Most patients with active SLE benefit from hydroxychloroquine, a safe and well-tolerated medication. However, it can cause retinal toxicity on rare occasions, with a potential for irreversible vision loss. This is most common after 5-7 years of therapy. For this reason, patients treated with hydroxychloroquine should have a baseline ophthalmologic evaluation, with annual reassessment beginning after 5 years. Educational objective: Hydroxychloroquine is used for patients with active systemic lupus erythematosus. However, it can cause retinal toxicity with prolonged use. Patients treated with hydroxychloroquine should have a baseline ophthalmologic evaluation and periodic reassessment.

A 69-year-old man is evaluated to determine the appropriateness of air travel. He has moderate-severity COPD (FEV1 is 47% of predicted) with stable exertional dyspnea in the setting of previous cigarette smoking. He and his wife are planning a 4-hour flight. He has not flown in 5 years. His medical history is also notable for coronary artery disease. His medications are a tiotropium inhaler, an albuterol/ipratropium inhaler, aspirin, rosuvastatin, and metoprolol. On physical examination, temperature is 36.6 °C (97.9 °F), blood pressure is 124/74 mm Hg, pulse rate is 80/min, and respiration rate is 16/min; BMI is 23. Oxygen saturation is 93% at rest breathing ambient air. Pulmonary examination reveals distant breath sounds, resonance to percussion, and no wheeze. A chest radiograph performed 6 months ago showed a flattened diaphragm. Which of the following is the most appropriate management? A. Arterial blood gas studies B. Echocardiographic assessment of pulmonary artery pressures C. Hypoxia altitude simulation test D. Recommend not flying

C. Hypoxia altitude simulation test Educational Objective: Manage air travel in a patient with lung disease. Key Point: In patients with lung disease and sea-level oxygen saturation between 92% and 95%, hypoxia altitude simulation testing can be used to determine the need for oxygen supplementation during air travel. The most appropriate next step is a hypoxia altitude simulation test (HAST). Initial evaluation of patients with lung disease who are not on long-term oxygen therapy and are planning air travel is usually with pulse oximetry at sea level. If the oxygen saturation is greater than 95%, it is unlikely that the patient will have significant desaturation at higher altitudes, and further testing or supplemental oxygen is not usually indicated. For patients with an oxygen saturation of less than 92%, in-flight supplemental oxygen is usually recommended. In patients with sea-level oxygen saturation between 92% and 95%, simulators that mimic altitude hypoxia, available at some centers, can be used to determine the need for oxygen supplementation during flight. Patients at high risk for in-flight hypoxia and its complications are those with COPD with hypercapnia, a recent exacerbation of chronic lung disease, pulmonary hypertension, and restrictive lung disease, in addition to those who have had previous inflight symptoms. This patient, who has a resting oxyhemoglobin saturation of 93% breathing ambient air, has risk factors for altitude-related hypoxia, which include an FEV1 less than 50% of predicted and cardiac disease. The HAST predicts the in-flight arterial partial pressure of oxygen and whether supplemental oxygen is needed, accounting for the pressurization of airline cabins that simulate an altitude of less than 2440 meters (8000 feet). In patients with COPD and a sea-level oxygen saturation between 92% and 95%, an alternative approach in locations where HAST testing is not available is to perform a 6-minute walk test. An oxygen saturation less than 84% suggests likely significant desaturation with altitude, and supplemental oxygen is usually prescribed without additional testing. Those with a walk test saturation greater than or equal to 84% should be referred for HAST testing. Arterial blood gas studies alone are unlikely to provide important information beyond what is already known about this patient's oxygenation status. It is possible that this patient has chronic hypercapnia due to underlying lung disease, a finding that would support further testing in the form of HAST. Similarly, echocardiographic evidence of pulmonary hypertension would represent a risk factor for in-flight hypoxia and would require further evaluation, but it does not have adequate predictive value alone in determining the need for in-flight oxygen. With a proper pre-travel evaluation, it is unlikely that this patient with stable chronic lung disease would have an in-flight problem. Unless the patient declines further testing, recommending against travel is premature.

A 17-year-old, previously well, Caucasian girl presented to her general practitioner with tiredness. The following blood counts were obtained on laboratory testing: haemoglobin 6.8 g/ dl, white blood cell count 10 x 109/1, platelets 365 x 109/1, and mean cell volume (MCV) of 67 fl. She had been on a paracetamol and codeine compound preparation for menorrhagia and dysmenorrhoea. She eats lots of vegetables, liver, eggs, steak and cereals. Which one of these causes is playing a significant role in the aetiology of her anaemia? A. Chronic gastrointestinal (GI) blood loss B.Having 12 cups of tea and other binders in her diet C. Heavy and irregular periods D. Infestation with hookworm E.Poor diet with foods not rich in iron

C. Heavy and irregular periods Heavy and irregular periods are the most likely significant aetiology of her anaemia. She has a microcytic anaemia with a normal white cell count and platelet count. We are told that she has menorrhagia. Iron balance in young women is often very borderline, and in the presence of heavy periods, can result easily in iron deficiency, despite a good diet. ​

A 35-year-old man has had epigastric pain for more than 1 year. The pain tends to occur 2 to 3 hours after a meal and is relieved if he takes antacids or eats more food. He has noticed a 4-kg weight gain in the past year. He does not smoke and drinks 1 glass of Johannisberg Riesling daily. The result of a urea breath test is positive, and a gastric biopsy specimen contains urease. He begins a 2-week course of antibiotics, but on day 4, he feels better and discontinues treatment. Three weeks later, the epigastric pain recurs. If he does not seek further treatment, which of the following complications is he most likely to develop? A. Carcinoid syndrome B. Fat malabsorption C. Hematemesis D. Migratory thrombophlebitis E. Vitamin B12 deficiency ​

C. Hematemesis The clinical symptoms in this case suggest peptic ulcer disease. In most cases, peptic ulcers are associated with Helicobacter pylori infection. These bacteria secrete urease, which can be detected by oral administration of urea 14C. After drinking the labeled urea solution, the patient blows into a tube. If H. pylori urease is present in the stomach, the urea is hydrolyzed, and labeled carbon dioxide is detected in the breath sample. In the biopsy urease test, antral biopsy specimens are placed in a gel containing urea and an indica-tor, and if H. pylori is present, the color changes within minutes. If not properly treated, peptic ulcers can produce many complications, including massive bleeding that can be fatal. Carcinoid tumors can occur in the stomach, but they are rare and are not related to peptic ulcer disease, which this patient has. He does not have fat malabsorption because fat absorption does not occur in the stomach. Peptic ulcers rarely progress to gastric carcinoma. The stomach has numerous arterial supplies and therefore is unlikely to be affected by focal thrombosis. Vitamin B12 deficiency can occur with autoimmune atrophic gastritis because intrinsic factor, which is required for vitamin B12 absorption, is produced in gastric parietal cells.

A 68-year-old man recently diagnosed with lung cancer presents to his oncologist complaining of increasing fatigue over the past 2 months. He states that although he is tired more often, he is relieved that he has been "putting some of my old weight back on." His shortness of breath and cough are unchanged from baseline but he notes that he has been thirstier than usual. His temperature is 37.1°C (98.8°F)f blood pressure is 145/88 mm Hg, and pulse is 74/min. Physical examination reveals an elderly man with a round face and a small mound of fat at the base of the neck. Body habitus is thin with a disproportionate amount of fat in the abdominal region. Testing reveals a plasma ACTH level of 73 pg/mL (normal: 9-52 pg/mL). Which of the following is necessary to make the correct diagnosis? A. Cosyntropin stimulation test B. Fasting blood glucose measurement C. High-dose dexamethasone suppression test D. Low-dose dexamethasone suppression test E. Urinary free cortisol measurement

C. High-dose dexamethasone suppression test This is a case of Cushing syndrome resulting from ectopic ACTH production from small cell lung cancer (SCLC). Cushing syndrome is characterized by an excess of cortisol, resulting in classic features that include fatigue, weight gain, polydipsia, round or "moon1' facies, dorsal fat pad ("buffalo hump"), and peripheral wasting with central obesity. This excess of cortisol may arise from a defect within the pituitary or the adrenal glands, from exogenous steroid administration, or from an ectopic or nonadrenal source of glucocorticoids. SCLC is a well-known cause of Cushing syndrome. In ectopic or adrenal tumors producing cortisol, cortisol levels are not suppressed by a high-dose dexamethasone suppression test. Of note, while Cushing syndrome can have many sources (as it merely refers to elevated cortisol levels), Cushing disease refers only to elevated cortisol levels secondary to an ACTH-producing pituitary tumor. Bottom Line: Cushing syndrome can result from ectopic ACTH production due to small cell lung cancer. In ectopic or adrenal tumors producing cortisol, cortisol levels are not suppressed by a high-dose dexamethasone suppression test.

An older man with a history of COPD presents for follow-up after being hospitalized with an acute exacerbation. He asks you what treatments are available to decrease his risk of death. Which of the following therapies has been shown to reduce long term mortality in this condition? A. Guaifenesin B. Oral prednisone C. Home oxygen therapy D. Ipratropium ​

C. Home oxygen therapy. The two most important interventions for reducing mortality in COPD are smoking cessation and home oxygen therapy, provided that the patient meets certain criteria. Patients with an SaO2 ≤88% or PaO2 ≤55 mmHg during rest, exercise, or sleep will benefit from chronic home oxygen therapy with a significant reduction in mortality. The cutoffs are slightly higher in a patient that has polycythemia or cor pulmonale. There are other interventions that show some mortality benefit, but the two interventions mentioned above are the most likely interventions to be tested on the shelf examination. (A) Mucolytics such as guaifenesin may partially reduce symptoms from excess airway secretions; however, there is debate over their usefulness and they do not reduce mortality. (B) Oral prednisone is used in acute exacerbations to reduce airway inflammation, but chronic treatment with systemic steroids has many side effects and can increase mortality. (D) Inhaled ipratropium is useful both in chronic treatment and in acute exacerbations. It decreases symptoms and improves lung function but does not decrease overall mortality.

A 38-year-old man with panhypopituitarism is evaluated for worsening fatigue and weight gain over the past 3 months. He is sleeping 9 hours each night, but he feels tired during the day and has decreased his usual exercise level. He has morning erections but reports low libido and occasional erectile dysfunction during intercourse. He wakes once during the night to urinate and drink water. He estimates that he urinates 5 to 8 times during the day, which is unchanged. Medical history is significant for transsphenoidal resection of a craniopharyngioma at age 18 years. He has required anterior pituitary hormone replacement and desmopressin since that time. Medications are desmopressin, hydrocortisone, levothyroxine, somatropin, and testosterone enanthate. On physical examination, temperature is 37.0 °C (98.6 °F), blood pressure is 118/64 mm Hg, pulse rate is 74/min, and respiration rate is 14/min. No orthostatic changes are noted. BMI is 24. There are no facial changes suggestive of acromegaly or Cushing syndrome. The thyroid is normal without goiter or nodules. Hair distribution and skin turgor are normal. There is no gynecomastia or striae. Normal penis and testicular volume are noted. Visual fields are intact on neurologic examination. Laboratory studies: Sodium 138 mEq/L (138 mmol/L) Insulin-like growth factor 1 Normal Prolactin 18 ng/mL (18 µg/L) Thyroid-stimulating hormone 0.8 µU/mL (0.8 mU/L) Thyroxine (T4), free 0.7 ng/dL (9.0 pmol/L) Testosterone (11 days after injection) 482 ng/dL (16.7 nmol/L) Follow-up MRIs show no residual tumor. Which of the following is the most appropriate management? A. Increase desmopressin B. Increase hhydrocortisone C. Increase levothyroxine D. Increase testosterone enanthate E. Stop somatropin

C. Increase levothyroxine Educational Objective: Manage hormone replacement therapy in a patient with panhypopituitarism. Key Point: Patients with secondary hypothyroidism from pituitary dysfunction have low or low-normal thyroid-stimulating hormone values, so levothyroxine dose should be adjusted based on free thyroxine (T4) level. The patient's hypothyroidism is inadequately treated, causing symptoms of fatigue and weight gain and a low free thyroxine (T4) level. Therefore, his levothyroxine dose should be increased. Patients with secondary hypothyroidism from pituitary dysfunction have low or low-normal thyroid-stimulating hormone (TSH) values which cannot be used to assess the adequacy of thyroid hormone replacement. Because of this, the levothyroxine dose is adjusted based on free T4 levels instead of TSH values. His free T4 is low, suggesting inadequate treatment as the likely cause of his symptoms. The patient's desmopressin dosing is adequate to treat his diabetes insipidus. His symptoms are well controlled without evidence of excessive urination, and his serum sodium level is normal. In addition, increasing the dose would risk potentially causing water retention and hyponatremia. He is on a physiologic dose of hydrocortisone. Hydrocortisone dose is not adjusted based on laboratory test results because his endogenous adrenocorticotropic hormone (ACTH) and cortisol levels will remain low on adequate therapy and are therefore not used to alter therapy. Instead, hydrocortisone is adjusted based on symptoms, such as orthostasis, weight loss, nausea, vomiting, and lightheadedness. He does not have these symptoms, so his cortisol deficiency is adequately treated. Increasing his hydrocortisone to higher than necessary doses increases the risk of iatrogenic Cushing syndrome and glucocorticoid-induced osteoporosis. His testosterone value is normal, and he has normal morning erections. These are two signs that his hypogonadism is adequately treated. Possibly, his erectile dysfunction is a result of fatigue from hypothyroidism or is functional instead of physiologic. There is no reason to stop somatropin. He has no evidence of residual tumor. Growth hormone (GH) replacement can improve lean mass distribution and quality of life in a patient with true GH deficiency, so it is reasonable to continue. Discontinuing GH will likely worsen his fatigue.

A 26-year-old man is evaluated because of 6-month history of chest tightness, cough, and shortness of breath that occur after running. He has no symptoms while running. His symptoms usually start a few minutes after he finishes running and continue for the next 30 minutes. Symptoms are worse when he runs in cold weather. He has no other medical problems and takes no medications. On physical examination, temperature is 36.5°C (97.7°F), blood pressure is 120/70 mm Hg, pulse rate is 65/min, respiration rate is 14/min, and oxygen saturation is 98% on ambient air. Body mass index is 21. Pulmonary examination shows good air movement without wheezing. Office spirometry is normal. After an exercise challenge test, forced expiratory volume in 1 second decreased by 15% from the baseline measurement obtained before exercise. Which of the following is the most appropriate treatment for this patient? A. Inhaled glucocorticoid before exercise B. Inhaled long-acting β2-agonist daily C. Inhaled short-acting β2-agonist before exercise D. Oral Ieukotriene receptor agonist daily

C. Inhaled short-acting β2-agonist before exercise Educational Objective: Treat exercise-induced bronchospasm. The most appropriate treatment for this patient is an inhaled short acting β2-agonist before exercise. Exercise-induced bronchoconstriction (EIB) is common in patients with asthma, and some patients only experience symptoms after exercise. EIB is diagnosed by measuring Forced expiratory volume in 1 second (FEV1) before and after exercise. A decrease in FEV1 of greater than 10% within 30 minutes of exercise is diagnostic of EIB. First-line therapy is administration of an inhaled short-acting β2-agonist 15 to 20 minutes before exercise. These agents work by stimulating airway β2 receptors, resulting in muscle relaxation and bronchodilation. When given before exercise, in approximately 80% of patients, these drugs are usually effective for 2 to 4 hours in protecting against bronchoconstriction. Nonpharmacologic approaches to prevent EIB include gradual warm-up before intense exercise, using a mask over the nose and mouth during cold weather, and avoiding high-intensity intermittent exercise. The use of an inhaled glucocorticoid immediately before exercise is not effective in preventing EIB. Leukotriene-modifying drugs can also be used to prevent EIB. However, they are not as effective as inhaled short-acting β2-agonists. Inhaled long-acting β2-agonist are not used as monotherapy for asthma. These drugs control asthma symptoms but provide no anti-inflammatory effects. Therefore, single-agent treatment of asthma with long-acting β2-agonist can mask worsening of airway inflammation and lead to increased risk of asthma-related complications, including death. K E Y P O I N T: First-line therapy for exercise-induced asthma is administration of an inhaled short-acting β2-agonist 15 to 20 minutes before exercise.

A 19-year-old man with asthma complains of shortness of breath and wheezing when playing sports. Otherwise he has no symptoms. The best preventive treatment is A. Antileukotrienes before exercise B. Regular inhaled steroids C. Inhaled β agonist before exercise D. Long-acting β agonist E. Anxiolytic medication

C. Inhaled β agonist before exercise Exercise-induced asthma occurs mainly in patients already diagnosed with asthma. Wheezing usually begins shortly after the initiation of exercise and can be debilitating and limit participation. Short acting β2 agonists (15 to 30 minutes before exercise) are recommended first-line agents for pharmacologic treatment, although leukotriene-receptor antagonists or inhaled corticosteroids with or without long-acting β2 agonists may be needed in refractory cases. If symptoms persist despite treatment, alternative diagnoses such as cardiac or other pulmonary etiologies, vocal cord dysfunction, or anxiety should be considered. The regular use of inhaled corticosteroids is best in the setting of asthmatic symptoms occurring in addition to exercise. Other preventive measures include a slow warm-up and the avoidance of very warm and very cold conditions.

A 49-year-old man comes to the office due to a 3-month history of fatigue. He also reports diffuse joint pain; finger swelling: and difficulty gripping objects with his right hand. The patient has poorly controlled hypertension despite being compliant with medications, a low-salt diet, and regular exercise. Temperature is 36.7 C (98 F), blood pressure is 146/98 mm Hg; pulse is 90/min, and respirations are 14/min. On examination, facial features appear coarse and differ significantly from his driver's license photograph taken 3 years ago. The palms are sweaty and have a doughy feel. The skin is oily. Multiple skin tags are noted, particularly on the neck area. Tapping the ventral aspect of the right wrist produces shooting pain on the lateral side of the right hand. Which of the following is the best next step in evaluating this patient? A. Fasting growth hormone level B. Growth hormone level following an oral glucose load C. Insulin-like growth factor 1 level D. MRI of the pituitary gland E. Thyrotropin-releasing hormone stimulation test

C. Insulin-like growth factor 1 level This patient's presentation (coarse facial features, arthralgias, uncontrolled hypertension, enlargement of the digits, carpal tunnel syndrome) is consistent with acromegaly. This condition is caused by excessive secretion of growth hormone (GH)r usually due to a pituitary somatotroph adenoma. Other common features include malocclusion of the jaw, hyperhidrosis, heart failure, macroglossia, and local mass-effect symptoms (eg, headache, visual field defects). GH stimulates hepatic insulin-like growth factor 1 (IGF-1) secretion, which is responsible for most of the clinical manifestations of acromegaly. IGF-1 levels in acromegaly are consistently elevated throughout the day. In contrast, GH levels can fluctuate widely and cannot be used alone to diagnose acromegaly (Choice A). As a result, IGF-1 is the preferred initial test. However, IGF-1 levels decrease with age and require interpretation according to age-adjusted norms. Educational objective: Acromegaly is caused by excessive secretion of growth hormone (GH), usually due to a pituitary somatotroph adenoma. Insulin-like growth factor 1 mediates most of the effects of GH and is the preferred initial test in suspected acromegaly. GH is not as sensitive due to wide fluctuations in circulating levels.

A 35 year old woman with previous gastric bypass presents to the ED with crampy abdominal pain. The most important issue to assess for is: A. Appendicitis B. Cholecystitis C. Internal hernia D. Diverticulitis E. Gallstone Ileus ​

C. Internal hernia Internal hernia can be devastating. Patients with previous gastric bypass are at high risk for this complication at any point following surgery. Clinical suspicion should remain high.

A 70-year-old woman with aortic stenosis complains of increasing shortness of breath associated with occasional chest pain. On examination she looks anaemic and has bilateral crepitations on auscultation. A late systolic murmur is heard at the cardiac base. Results of laboratory examination are: Hb 7.4 g/litre; MCV 70 ft; leukocytes 5.4 x 109/litre; platelets 580 x 109/litre. The blood smear shows hypochromic, microcytic erythrocytes. Which one of the following is the most likely diagnosis? A. Anaemia of chronic disease B. Autoimmune haemolytic anaemia C. Iron deficiency anaemia D. Folate deficiency anaemia E. Microangiopathic haemolytic anaemia

C. Iron deficiency anaemia This lady has a microcytic anaemia associated with a thrombocytosis. The anaemia is quite significant, and the microcytosis is quite pronounced, suggesting iron deficiency anaemia more than the anaemia of chronic disease.

A 42-year-old man presents to his physician with complaints of weight gain circumferentially around his abdomen and a "hump" on his neck. Physical examination shows a rounded face and purple stretch marks on the abdomen. A random blood glucose level is 340 mg/dL. On CT a solitary pituitary adenoma is noted. He is offered the option of medical management or surgical resection as possible treatment approaches for the adenoma. After understanding the options, he wishes to try medical therapy first. Which of the following treatment strategies is most appropriate for this patient? A. Glucocorticoids B. Insulin C. Ketoconazole D. Observation E. Pituitary irradiation

C. Ketoconazole This patient presents with the stigmata of hypercortisolism ("buffalo1' hump, truncal obesity with striae, and "moon facies"). His CT revealed a pituitary adenoma which is the most common cause of Cushing disease. Excess production of ACTH by the pituitary adenoma causes the adrenal cortex to produce excess cortisol. When surgical removal of the tumor is not an option, medical therapy is required to control the hypercortisolism. There are several drug options available to decrease cortisol production (metyrapone, mitotane, and ketoconazole) with ketoconazole being the most commonly used although not FDA approved, Ketoconazole inhibits 17α-hydroxylase (desmolase), which is the first step in cortisol biosynthesis (conversion of cholesterol to pregnenolone). Ketoconazole also decreases production of cortisol by inhibiting the conversion of 11-deoxycortisol to cortisol (mediated by 11β-hydroxylase). Metyrapone also decreases cortisol production by inhibiting 11β-hydroxylase. Bottom Line: Ketoconazole decreases adrenal steroidogenesis by inhibiting several key enzymes involved in the biosynthetic pathway of cortisol.

A 65 year old man with known COPD and mild airways obstruction on spirometry presents with increasing shortness of breath on exertion. He mainly struggles walking on inclines or climbing stairs. He does not experience exacerbations of COPD and he has had no courses of oral prednisolone or antibiotics in the last 2 years. His current inhaled therapy includes a long-acting muscarinic antagonist (LAMA) and an SABA. Examination reveals decreased air entry bilaterally, with loss of cardiac dullness on percussion. The patient does not desaturate on an incremental walk test. What would be a reasonable addition for this patient to escalate therapy? A. Ambulatory oxygen B. Inhaled corticosteroid (ICS)/LABA combination inhaler C. LAMA/LABA combination inhaler D. Oral prednisolone E. Salbutamol nebuliser ​

C. LAMA/LABA combination inhaler The patient's main complaint is of increased shortness of breath on exertion. ICS would be indicated if there was an increase in exacerbation frequency. Oral prednisolone would be useful in the context of a current exacerbation of COPD. Ambulatory oxygen is not indicated without desaturation on exertion. Whilst a nebulizer might well improve the patient's symptoms, an escalation of inhaled therapy would be a better approach. LABA/ LAMA combination inhalers offer enhanced bronchodilatation and improvements in exercise tolerance. Combination therapy with a β2-agonist and an anticholinergic provides a greater benefit in terms of symptoms and bronchodilation (FEV1) than does either agent alone. It is also convenient having both drugs in a single MDI. ​

52-year old woman is referred to the clinic with a 3-month history of polyuria. She additionally complains of increased thirst, drinking up to 5 L per day. Fasting plasma glucose is normal at 4.2 mmol/L (76 mg/dL). She attends for a water deprivation test. Which of the following confirms a diagnosis of diabetes insipidus? A. 24-hour urine volume of 3 L B. Plasma osmolality <280 mOsm/kg at the start of the test C. Plasma osmolality >300 mOsm/kg and urine osmolality <600 mOsm/kg D. Plasma sodium concentration 145 mmol/L E. Reduction in body weight of 1% over the test period

C. Plasma osmolality >300 mOsm/kg and urine osmolality <600 mOsm/kg Low plasma osmolality at the start of the test may suggest primary polydipsia as the cause of her symptoms. A plasma sodium concentration above the normal range and 24-hour urine volume greater than 3 L can be an indicator of diabetes insipidus but are not diagnostic. A water deprivation test should be discontinued if body weight falls by 3% or more and if plasma osmolality rises to >300 mOsm/kg with continued dilute urine (osmolality <300 mOsm/ kg). DDAVP (2 µg intramuscularly) should subsequently be administered to assess response and determine if the underlying diagnosis is cranial or nephrogenic diabetes insipidus. ​

A 28-year-old woman presents to her primary care physician complaining of a rash and hair loss. She says that while on a cruise last month she got a bad sunburn on her cheeks and nose, but the redness has not subsided. She also notes that she has been losing large clumps of hair in the shower for the past several weeks. On further review of systems, she notes feeling very fatigued for the past few months and has lost 4.5 kg (10 lb.). She has a temperature of 38.3°C (100.9°F), heart rate of 92/min, and blood pressure of 110/72 mm Hg. Her muscles are diffusely tender and her shoulders, knees, and ankles have decreased passive range of motion secondary to pain and stiffness. She has 1+ pitting edema in her legs up to the mid-calf bilaterally. Laboratory work-up reveals positive antinuclear antibody and positive anti-double-stranded DNA antibody. Which of the following would most likely be found in this patient? A. Anticentromere antibodies B. Antineutrophil cytoplasmic antibodies C. Leukopenia D. Macrocytic anemia E. Thrombocytosis ​

C. Leukopenia This woman is presenting with signs and symptoms suggestive of systemic lupus erythematosus (SLE). It can cause widespread systemic disease including renal failure, central nervous system disturbances, and pulmonary disease. Antinuclear antibody is positive in almost all patients with SLE, but it is not specific. Anti-double-stranded DNA and anti-Sm antibodies are highly specific for lupus, so their presence confirms the diagnosis of SLE. Patients with SLE frequently have hematologic abnormalities. Leukopenia is a very common finding. It is usually lymphopenia as opposed to neutropenia, so patients are not at significantly increased risk of infection. Bottom Line: SLE can cause widespread systemic disease including renal failure, central nervous system disturbances, pulmonary disease, as well as hematologic abnormalities including leukopenia. The presence of anti-double-stranded DNA and anti-Smith antibodies confirm the diagnosis. ​

A 55 year old man has smoked 30 cigarettes per day since he was 15 years old. He is a taxi driver. He finds he is increasingly breathless on exertion. Oxygen saturations are 98% on room air. Examination reveals tracheal tug, reduced cricosternal distance and a barrel chest. He has reduced cardiac dullness and symmetrically reduced air entry. CXR reveals hyperinflation and spirometry reveals moderate airways obstruction. The patient walks 300 m on an incremental walk test before becoming breathless; oxygen saturations are maintained. What pathological change best explains why he is breathless on exertion? A. Activation of central chemoreceptors B. Exercise-induced bronchospasm C. Loss of elastic recoil D. Paradoxical diaphragm movement E. Pulmonary hypertension ​

C. Loss of elastic recoil In COPD, loss of elastin fibres results in small airway collapse and air trapping during expiration. This dynamic hyperinflation is initially noticed on exertion because expiration time is shortened during exercise. Exercise-induced bronchospasm would be more likely in asthma. There are no examination findings that suggest sufficient pulmonary hypertension to cause breathlessness. There is no reason for the diaphragm to move paradoxically in this case. Central chemoreceptors are stimulated by a rise in CO2, which might be expected in more advanced disease.

A 60-year-old man is evaluated for a history of advanced COPD. He has fatigue and dyspnea with mild exertion, which significantly impairs his ability to perform routine activities. He is adherent to his medication regimen, and his inhaler technique is good. He has a history of multiple exacerbations, which have required hospitalization over the past year. He participated in pulmonary rehabilitation after his last hospitalization. Medical history is otherwise unremarkable. He quit smoking 3 years ago. Medications are roflumilast, tiotropium, mometasone/formoterol, as-needed albuterol, and 2 L of oxygen via nasal cannula. On physical examination, respiration rate is 22/min; other vital signs are normal. BMI is 20. Pulmonary examination reveals decreased breath sounds bilaterally with no wheezing or crackles. The remainder of the examination is unremarkable. Chest radiograph shows changes consistent with COPD, and chest CT shows bilateral homogeneous emphysema. Pulmonary function testing shows an FEV1 of 18% of predicted, an FEV1/FVC ratio of 33%, and a DLCO of 52% of predicted. Arterial blood gases on 2 L of oxygen reveal a pH of 7.37, a PCO2 of 64 mmHg (8.5 kPa), a PO2 of 62 mmHg (8.2 kPa), and an oxygen saturation of 91%. Which of the following is the most appropriate next step in management? A. Daily oral glucocorticoid therapy B. Hospice care C. Lung transplantation D. Lung volume reduction surgery

C. Lung transplantation Educational Objective: Treat advanced COPD with lung transplantation. Key Point: In patients with advanced COPD, criteria for referral for lung transplantation include a history of exacerbations associated with acute hypercapnia (arterial PCO2>50 mm Hg [6.7 kPa]); pulmonary hypertension, cor pulmonale, or both despite oxygen therapy; or FEV1 less than 20% of predicted with DLCO less than 20% of predicted or homogeneous distribution of emphysema. The most appropriate management is to evaluate for lung transplantation. Criteria for referral for lung transplantation include a history of COPD exacerbations associated with acute hypercapnia (arterial PCO2 >50 mm Hg [6.7 kPa]); pulmonary hypertension, cor pulmonale, or both despite oxygen therapy; or FEV1 less than 20% of predicted with DLCO less than 20% of predicted or homogeneous distribution of emphysema. This 60-year-old patient has a history of hospitalization for exacerbations, hypercapnia, an FEV1 of less than 20% of predicted, and bilateral homogeneous distribution of emphysema. Therefore, he is a candidate for lung transplantation. Absolute contraindications to lung transplantation include malignancy within the last 2 years, infection with hepatitis B or C virus with histologic evidence of significant liver damage, active or recent cigarette smoking, drug or alcohol abuse, severe psychiatric illness, documented nonadherence with medical care, and absence of social support. Age greater than 65 years is a relative contraindication, as well as the presence of multiple comorbid conditions, which are not present in this patient. Short-term use of systemic glucocorticoids is recommended for acute exacerbations of COPD. However, long-term use of oral glucocorticoids has not been shown to improve quality of life or reduce the rate of exacerbations. Because this patient has no evidence of an acute exacerbation, there is no indication for oral glucocorticoid therapy. Lung volume reduction surgery (LVRS) is indicated in patients with predominantly upper lobe emphysema. Because this patient has homogeneous emphysema, LVRS is not an option. Additionally, LVRS results in higher mortality in patients with an FEV1 less than 20% of predicted. Hospice is an approach to care in patients with life-limiting illness and focuses on quality of life rather than an attempt at cure of the underlying disease based on the patient's and family's goals and values. Although this patient is significantly limited in his daily activities, it is not clear that he has entered the last months to weeks of life, which is the usual time frame for hospice care (compared with palliative care, which is appropriate for all patients with severe or advanced disease). Additionally, because this patient meets criteria for possible lung transplantation, it is reasonable to offer this as a potential treatment option. Hospice care may be a consideration if he is not a transplant candidate or if he declines further aggressive treatment and his condition progresses.

A 26-year-old G1P1 woman presents to the clinic because of cloudy, milky nipple discharge for 4-5 weeks. She is obese and was diagnosed with schizophrenia when she was 19 years old. After initially having difficulty controlling her symptoms and requiring several medication changes, she is currently well managed by a local psychiatrist. She does not remember what medications she takes. However, she does recall her psychiatrist recently changing her medication because her old drug caused her to develop a blood disorder. Her last normal menstrua! period was 6 weeks ago. Review of systems is positive for occasional mild headaches and feelings of restlessness, but she denies changes in vision or sensation. What is the most likely cause of her symptoms? A. Central nervous system sarcoidosis B. Idiopathic prolactinemia C. Medication effect D. Pregnancy E. Prolactinoma

C. Medication effect Under normal conditions, dopamine is delivered from the hypothalamus to the pituitary in a tonic fashion, constitutively inhibiting the release of prolactin into the bloodstream. Typical antipsychotics, such as haloperidol, block dopamine receptors on the surface of prolactin-secreting cells in the anterior pituitary. This leads to inappropriate release of prolactin. She is also predisposed to the galactorrheic effect of prolactin by virtue of her parity. Another clue to this answer is her history of a "blood disorder." Given her history of difficulty controlling her schizophrenia symptoms, it is possible that she was on an atypical antipsychotic such as clozapine. This drug is only recommended in patients with treatment resistant schizophrenia because it can cause agranulocytosis. This may have prompted her psychiatrist to switch to a typical antipsychotic such as haloperidol, which generally does not produce that adverse effect. Bottom Line: Typical antipsychotics such as haloperidol block dopamine receptors on the surface of prolactin-secreting cells in the anterior pituitary, leading to inappropriate release of prolactin.

A 28-year-old accountant with RA is considering another pregnancy. Reasons for concerns for conception and the health of her fetus during the first trimester include which of the following? A. She is currently on 5 mg of prednisone. B. She is currently taking sulfasalazine. C. She has a history of treatment with leflunomide. D. She is taking Naprosyn 500 mg. ​

C. She has a history of treatment with leflunomide. Leflunomide is a significant teratogen and has a half-life that is extremely prolonged. Any potential mother who has ever taken it needs to have blood levels drawn before conception. Leflunomide can be rapidly eliminated from the body by treatment with cholestyramine. Prednisone is considered safe in pregnancy; significant data suggest that sulfasalazine is well tolerated by both mother and child and NSAIDs are safe until the last trimester. ​

A 72-year-old man is admitted to the hospital overnight following an elective inguinal hernia repair. There are no immediate postoperative complications, but he awakens the next morning with severe right knee pain. Medical history is notable for hypertension, chronic obstructive pulmonary disease, and mild aortic stenosis. The patient smokes half a pack of cigarettes daily and drinks alcohol on social occasions. His temperature Is 38.1 C (100.5 F)h blood pressure Is 160/110 mm Hg, pulse Is 80/min, and respirations are 16/min. Physical examination shows redness and swelling of the right knee with limited motion due to pain. Diagnostic arthrocentesis is performed, with results from synovial fluid analysis as follows: White blood cells 30,000/mm3 Neutrophils 90% Polarized microscopy Few rhomboid-shaped crystals Gram stain No organisms seen with this patient's current condition? A. Conjunctivitis B. Heberden nodes C. Meniscal calcification D. Rheumatoid factor E. Tophi F. Transient bacteremia ​

C. Meniscal calcification Pseudogout is a form of acute arthritis induced by the release of calcium pyrophosphate dihydrate (CPPD) crystals from sites of chondrocalcinosis (calcification of articular cartilage) into the joint space, it often occurs in the setting of recent surgery or medical illness. Patients present with acute pain, swelling, redness, and limited motion of the involved joint(s), with the knee most commonly affected. Fever and a mild leukocytosis may occur in pseudogout, although these findings are less common than in urate gout. Identifying chondrocalcinosis on x-ray of a joint with acute arthritis establishes a diagnosis of probable CPPD crystal arthritis. Synovial fluid analysis will show an inflammatory effusion (15,000-30,000 cells/mm5). The rhomboid-shaped, positively birefringent crystals of CPPD can be difficult to detect on synovial fluid analysis but are diagnostic when present. Educational objective: Pseudogout is an acute inflammatory arthritis caused by calcium pyrophosphate crystals. It often occurs in the setting of surgery or medical illness. Pseudogout is diagnosed with synovial fluid showing rhomboid-shaped, positively birefringent crystals and radiographic evidence of chondrocalcinosis (calcified articular cartilage).

A 36 year old woman presents with 3-month history of joint pain and swelling affecting the wrists, MCPJs and proximal interphalangeal joints (PIPJs) of the hands, both shoulders, both knees and the MCPJs of both feet. Laboratory investigations reveal an ACPA level of 145, an ESR of 68 mm/hr and a CRP of 84 mg/L. On examination she has 22 tender and 16 swollen joints and rates the activity of her arthritis as 65/100, giving a Disease Activity Score 28 (DAS28) of 7.54. What would be the most appropriate initial treatment? A. Adalimumab 40 mg every 2 weeks and prednisolone 5 mg daily B. Hydroxychloroquine 200 mg twice daily and ibuprofen 400 mg 3 times daily C. Methotrexate 15 mg weekly, folic acid 5 mg weekly and prednisolone 30 mg daily D. Prednisolone 30 mg daily, ibuprofen 400 mg 3 times daily and omeprazole 30 mg daily E. Rituximab 1000 mg on two occasions a fortnight apart combined with prednisolone 5 mg daily ​

C. Methotrexate 15 mg weekly, folic acid 5 mg weekly and prednisolone 30 mg daily Methotrexate is the core disease-modifying antirheumatic drug (DMARD) in the management of RA and it is often combined with prednisolone therapy at first presentation to gain disease control. Option B would be inappropriate in view of the very active disease given that hydroxychloroquine has relatively weak immunosuppressive effects. Options A and E would not be indicated as first-line treatments. Option D would not give adequate disease control. ​

A 62 year old woman attends the family practice surgery with increasing shortness of breath on exertion and is sent for full pulmonary function testing. She is an ex-smoker of 25 pack years and works caring for horses at a riding stable. Respiratory function testing reveals: FEV1 3.2 L FVC 3.8 L FEV1/FVC 0.84 Total lung capacity (TLC) 4.1 L Residual volume (RV) 0.6 L Transfer factor for carbon monoxide (TLCO) 1.5 (44%) Transfer coefficient (KCO) 1 (56%) How should the respiratory function testing be interpreted? A. Gas transfer reduction indicates emphysema B. Gas transfer reduction indicates hypersensitivity pneumonitis C. No evidence of airways obstruction D. There is a restrictive picture E. There is an obstructive picture

C. No evidence of airways obstruction The gas transfer is reduced and partially corrects for lung volume. There is no evidence of airways obstruction. The respiratory function tests are non-diagnostic and further clinical details and, potentially, imaging studies are likely to be required. FEV1, FVC and FEV1/FVC are within normal limits.

Which of the following is the gold standard for the diagnosis of gastroparesis? A. Upper endoscopy B. Plain abdominal x-rays C. Nuclear medicine scan D. CT E. It is considered a clinical diagnosis. ​

C. Nuclear medicine scan Gastroparesis is defined as delayed gastric emptying without a mechanical cause for obstruction. Although diabetes is the most common known cause of gastroparesis (29%), idiopathic gastroparesis occurs more frequently (36%). The most common symptoms are nausea, early satiety, and abdominal bloating. Most patients do not have abdominal pain. Although symptoms alone can be suggestive of this condition, it needs to be confirmed with imaging (E). Gastric emptying scintigraphy (delayed gastric emptying study) is the gold standard in diagnosing gastroparesis. This involves asking the patient to eat a small meal along with a radioactive tracer. The rate of emptying is measured 1, 2, 3, and 4 hours after the meal is ingested, and if more than 10% of the meal remains in the stomach after 4 hours, the study is considered consistent with gastroparesis (A, B, D).

A 27 year old woman with severe atopic asthma is 26 weeks pregnant. She has been awake at night coughing and short of breath for 48 hours. She has been trying to avoid using her salbutamol inhaler but nonetheless has had to take it 6 to 8 times in the last 24 hours. She has no nasal symptoms and no reflux. Observations: oxygen saturations 94% on room air, respiratory rate 22 breaths/min, BP 110/65 mmHg, pulse 96 beats/min, apyrexial; peak expiratory flow rate (PEFR) 240 L/min (best 450 L/min). Examination of the chest reveals bilateral wheeze, mild bilateral pedal oedema, heart sounds dual, no murmur. The patient's current medications include: regular high-dose ICS/long-acting β2-agonist (LABA) and oral montelukast; inhaled short-acting β2-agonist (SABA) as required. She insists she has been adherent with medication although she is very worried about taking any medications whilst pregnant. What should be the next step in her management? A. Low-molecular-weight heparin (LMWH) B. Oral amoxicillin C. Oral prednisolone D. Reduce high-dose ICS/LABA E. Stop montelukast

C. Oral prednisolone This patient is on maximal therapy for atopic asthma but has features of poor control suggesting a significant exacerbation. She needs to have oral corticosteroid therapy, nebulised bronchodilator and be admitted for observation. Her therapy should not be reduced because the greater risk to patient and fetus is uncontrolled asthma. Antibiotics would only be considered where there was strong objective evidence of infection (fever, sputum culture positive, CXR infiltrate). The presentation is not suggestive of pulmonary thromboembolism.

A 28-year-old man presents with a 2-year history of low back pain and intermittent arthritis in the lower extremity. Physical examination demonstrates bilateral Achilles tendonitis and tenderness to direct pressure over the sacroiliac joints. Which of the following would not support the diagnosis of spondyloarthritis (SpA)? A. Presence of HLA-B27 B. History of anterior uveitis C. Positive rheumatoid factor D. Elevated CRP E. History of inflammatory bowel disease ​

C. Positive rheumatoid factor The history and the other optional features would be entirely consistent with a diagnosis of SpA. HLA-B27 is strongly associated with SpA in general, and its strongest association is with ankylosing spondylitis (AS). Anterior uveitis occurs in approximately 30% of AS cases, and is itself associated with HLA-B27 even when AS is not present. Positive rheumatoid factor is characteristic of rheumatoid arthritis but not of SpA, so it would not be expected to be positive in such patients. Elevated C-reactive protein (CRP) is a nonspecific marker of inflammation and would be commonly seen in patients with active SpA. Inflammatory bowel disease occurs in approximately 10% of AS cases, and its presence strongly suggests that the underlying arthritis is part of the SpA spectrum. ​

A 64-year-old woman is seen for follow-up evaluation. Two weeks ago, she was in a car accident, and an incidental pituitary adenoma was found on a cervical spine CT scan. She has no residual injuries from the car accident. She is otherwise healthy and takes no medications. She went through menopause at age 51. She has night sweats two to three times per month and occasional hot flushes. These have improved over the past decade and are not bothersome. She is not sexually active. She has never taken hormone replacement therapy. She has had no change in vision, headaches, or galactorrhea. On physical examination, temperature is 37.5 °C (99.5 °F), blood pressure is 110/63 mm Hg, pulse rate is 82/min, and respiration rate is 14/min. BMI is 26. There is axillary and pubic hair loss. Visual fields are intact. There are no findings suggestive of Cushing syndrome or acromegaly. Laboratory studies: Estradiol <20 pg/mL (73.4 pmol/L) Follicle-stimulating hormone 6.4 mU/mL (6.4 U/L) Luteinizing hormone 3.2 mU/mL (3.2 U/L) Prolactin 53 ng/mL (53 µg/L) Thyroid-stimulating hormone 3.2 µU/mL (3.2 mU/L) Thyroxine (T4), free 1.1 ng/dL (14.2 pmol/L) Pituitary MRI shows a 7-mm adenoma in the anterior sella. The tumor is not invasive. It does not approximate the optic chiasm. The pituitary stalk is mid-line. Which of the following is the most appropriate management? A. Begin dopamine agonist B. Gamma knife stereotactic radiosurgery C. Repeat testing in 12 months D. Transphenoidal resection

C. Repeat testing in 12 months Educational Objective: Treat a microprolactinoma in a postmenopausal woman. Key Point: Microprolactinomas in asymptomatic patients do not require treatment; however, surveillance is recommended. No therapy is necessary at this time, and the patient should be retested in 12 months. The patient has a microprolactinoma, but she is postmenopausal. Luteinizing hormone and follicle-stimulating hormone levels are normally high in postmenopausal women because of ovarian failure; however, her levels are lower than expected, likely because the elevated prolactin is providing negative feedback. This causes hypogonadism but is not clinically relevant because she is already hypogonadal from normal menopause. She has minimal symptoms from menopause and is tolerating it well. The prolactinoma was found incidentally. On MRI, it has no concerning features, and her other pituitary hormone levels are normal. Although no treatment is necessary for this asymptomatic patient, it is advisable to retest in 6 to 12 months to make sure that the tumor does not grow.

A 65-year-old woman is evaluated 1 week after undergoing upper endoscopy for persistent abdominal pain. The study showed a 1-cm, clean-based ulcer in the duodenum. Biopsy specimens from the stomach showed no evidence of Helicobacter pylori infection, and a serum antibody test for H. pylori was also negative. Proton pump inhibitor therapy was started, and the patient's symptoms were alleviated. She has a history of mild osteoarthritis and osteoporosis. Medications are a nonprescription analgesic for arthritis and a calcium supplement, vitamin D, and alendronate. On physical examination, vital signs are normal. The abdominal examination reveals no tenderness, hepatomegaly, or palpable masses. A complete blood count is normal. Which of the following is the most appropriate next step in this patient's management? A. Measure the serum gastrin level B. Repeat upper endoscopy with biopsy of the ulcer C. Review the nonprescription arthritis analgesic D. Stop alendronate therapy ​

C. Review the nonprescription arthritis analgesic Educational Objective: Evaluate peptic ulcer disease. The type of nonprescription analgesic that this patient is taking should be identified. The two most common causes of peptic ulcer disease are nonsteroidal anti-inflammatory drugs (NSAIDs) and Helicobacter pylori infection, which account for more than 90% of cases. This patient has a history of arthritis for which she takes an over-the- counter analgesic. Therefore, inadvertent use of NSAIDs, which are widely available without a prescription and are often used as analgesics for arthritis, needs to be considered. Many patients who take such non-prescription medications are unaware that they are taking NSAIDs. KEY POINT: In patients with peptic ulcers, nonprescription medications should be reviewed to determine whether NSAIDs are being inadvertently used. ​

A 70-year-old man is evaluated in follow-up for COPD, which was diagnosed 1 year ago. He has had two exacerbations in the last year, with the second exacerbation 1 month ago. He has also had baseline chronic cough with mucoid sputum consistently for the last 3 years. Sputum culture 6 months ago grew Hemophilus influenzae and Mycobacterium avium-intracellulare. He has to stop to catch his breath after walking 100 meters (328 feet). His inhaler technique is good. He quit smoking 2 years ago and completed a pulmonary rehabilitation program 4 months ago. His medications are tiotropium, fluticasone/salmeterol, and as-needed albuterol. Spirometry performed 3 months ago showed an FEV1 of 32% of predicted and an FEV1/FVC ratio of 50%. Chest radiograph performed last month showed no infiltrate, mass, or increased vascular congestion. On physical examination, vital signs are normal. Oxygen saturation is 92% breathing ambient air. Pulmonary examination reveals diminished breath sounds. Which of the following is the most appropriate long-term COPD treatment? A. Daily azithromycin B. Daily prednisone C. Roflumilast D. Simvastatin

C. Roflumilast Educational Objective: Treat severe COPD with recurrent exacerbations. Key Point: Roflumilast is a phosphodiesterase-4 inhibitor that is used as add-on therapy to reduce exacerbations in patients with severe COPD associated with chronic bronchitis and a history of recurrent exacerbations despite other therapies. The most appropriate treatment is roflumilast. Roflumilast is a phosphodiesterase-4 (PDE-4) inhibitor that is used as add-on therapy to reduce exacerbations in patients with severe COPD associated with chronic bronchitis and a history of recurrent exacerbations despite other therapies. Inhibition of PDE-4 decreases inflammation, which may be helpful in a limited number of patients with COPD in whom inflammation is a significant factor. Roflumilast has minimal bronchodilator activity; however, small improvements in FEV1 are seen in patients also treated with a long-acting anticholinergic agent or a long-acting [β2-agonist. Roflumilast should always be used with at least one long-acting bronchodilator. This patent has severe symptomatic COPD (Global Initiative for Chronic Obstructive Lung Disease category D) with recurrent exacerbations, so roflumilast is indicated, along with other agents. Some studies of macrolide antibiotics suggest that their use reduces exacerbations in a select group of patients with COPD. Macrolides have been shown to reduce exacerbations and improve lung function in patients with cystic fibrosis. However, there is currently insufficient evidence to routinely recommend daily macrolide therapy for the long-term treatment of COPD. There are concerns that long-term azithromycin therapy may be associated with increasing bacterial resistance, especially in patients who previously had Mycobacterium avium-intracellulare infection, and an increased risk of QT prolongation. Oral glucocorticoids should be reserved for limited periodic use to treat acute exacerbations in patients with COPD. Long-term oral glucocorticoid therapy has been shown to have limited, if any, benefits in COPD. In addition, long-term glucocorticoid therapy has a high risk for significant side effects, including diabetes mellitus, muscle weakness, osteoporosis, and decrease in functional status. The anti-inflammatory effects of simvastatin were previously believed to reduce exacerbations in COPD. However, a large randomized prospective study showed that, in patients at risk for exacerbations, simvastatin along with usual treatment did not reduce exacerbations or the time to first exacerbation. In addition, simvastatin was found to have no effect on lung function, quality of life, rate of severe adverse events, or mortality.

A 58-year-old man is evaluated for chronic cough, occasional wheezing, and shortness of breath associated with frequent stops to catch his breath when walking one to two blocks on level ground. His medical history is notable for an episode of bronchitis, for which he underwent outpatient treatment 6 months ago. He is a current smoker with a 30-pack-year smoking history. On physical examination, vital signs are normal. Examination of the lungs shows mildly decreased breath sounds throughout both lung fields and occasional scattered expiratory wheezes. The remainder of the physical examination is normal. Spirometry shows an FEV1 of 70% of predicted and a postbronchodilator FEV1/FVC ratio of 62%. His modified Medical Research Council (mMRC) symptom score is 2. In addition to smoking cessation, which of the following is the most appropriate treatment? A. Combination inhaled glucocorticoid and a long-acting bronchodilator B. Phosphodiesterase-4 inhibitor and combination inhaled glucocorticoid and long-acting bronchodilator C. Short-acting bronchodilator as needed, a long-acting bronchodilator, and pulmonary rehabilitation D. Short-acting bronchodilator as needed and an inhaled glucocorticoid

C. Short-acting bronchodilator as needed, a long-acting bronchodilator, and pulmonary rehabilitation Educational Objective: Treat COPD in a patient with risk category B disease. Key Point: The recommended therapy for stable patients with symptomatic (Global Initiative for Chronic Obstructive Lung Disease risk category B) COPD consists of a short-acting bronchodilator as needed and regular use of a long-acting bronchodilator, as well as with pulmonary rehabilitation. The most appropriate treatment for this patient with COPD (Global Initiative for Chronic Obstructive Lung Disease [GOLD] risk category B) is a short-acting bronchodilator as needed, regular use of a long-acting bronchodilator, and pulmonary rehabilitation. The new GOLD risk stratification can be used to classify risk and help make management decisions. Risk category A COPD is characterized by an FEV1/FVC ratio less than 70%, an FEV1 greater than or equal to 50% of predicted, and zero to one exacerbation(s) in the past year. These patients have mild or infrequent symptoms. Symptoms can be objectively quantified using either the Modified Medical Research Council (mMRC) Questionnaire or the COPD Assessment Test (CAT) score. Patients in risk category A have a mMRC score of 0 to 1 or CAT score less than 10. Therapy for category A COPD is an inhaled bronchodilator as needed; a short-acting bronchodilator is preferred. Patients in risk category B have all the same spirometry measurements and exacerbation frequency as in category A but are more symptomatic, often having to walk slowly or stop owing to breathlessness (mMRC score of >2 or CAT score >10). These patients are also treated with bronchodilators, typically a short-acting bronchodilator as needed and regular use of a long-acting bronchodilator, as well as with pulmonary rehabilitation. Combinations of a long-acting [β2-agonist (LABA) and long- acting anticholinergic (also known as long-acting muscarinic agent [LAMA]) bronchodilators can be used as alternative therapy. Risk category C COPD is characterized by an FEV1/FVC ratio less than 70%, an FEV1 less than 50% of predicted, or two or more exacerbations per year or one or more hospitalizations for an exacerbation. Otherwise, patients have mild or infrequent symptoms (mMRC score of 0-1 or CAT score <10). Regular treatment with a combination inhaled glucocorticoid plus a LABA or monotherapy with a LAMA is preferred. Alternatives may include dual combination therapy with a LAMA plus an inhaled glucocorticoid or a LABA, or a phosphodiesterase-4 (PDE-4) inhibitor and a long-acting bronchodilator. Patients in risk category D have all the same spirometry measurements and exacerbation frequency as category C, but they are more symptomatic (mMRC score of >2 or CAT score >10). Preferred therapy includes a short-acting bronchodilator as needed and regular use of combination therapy with an inhaled glucocorticoid and a LABA and/or a LAMA, and pulmonary rehabilitation. Alternative therapy includes triple combinations of two long-acting bronchodilators and an inhaled glucocorticoid; an inhaled glucocorticoid plus a LABA and PDE-4 inhibitor; or double combinations of two long- acting bronchodilators, or a LAMA and PDE-4 inhibitor.

A 50-year-old man is evaluated for a 2-year history of intermittent, nonproductive, chronic cough, as well as mild dyspnea with exertion. He has not had fever, chest pain, heartburn, loss of appetite, or weight loss. He has a 20-pack-year history of smoking and is a current smoker. His medical history is otherwise unremarkable and he takes no medications. On physical examination, blood pressure is 125/76 mm Hg, pulse rate is 78/min, and respiration rate is 15/min; oxygen saturation is 98% breathing ambient air. BMI is 25. He appears comfortable. There is no jugular venous distention. Heart sounds are normal, and there is no murmur. The lungs are clear. No edema is noted. The remainder of the examination is normal. A chest radiograph and electrocardiogram are normal. Which of the following is the most appropriate next step in management? A. CT of the chest B. Polysomnography C. Spirometry D. Trial of a proton pump inhibitor

C. Spirometry Educational Objective: Diagnose COPD with spirometry. Key Point: Spirometry should be performed to evaluate for COPD in patients with suggestive symptoms (dyspnea, cough, sputum production) and risk factors such as a history of smoking. The most appropriate next step in management is spirometry. This patient, who is a smoker with intermittent cough and dyspnea, likely has COPD. Although dyspnea, cough, and sputum production are characteristic symptoms of COPD, these symptoms can be variable and the cough can be nonproductive. This patient requires spirometry, which measures the FEV1 and FVC, and the FEV1/FVC ratio to assess for and quantify any degree of airflow obstruction present. Patients with COPD have a FEV1/FVC ratio of less than or equal to 70%. Spirometry is reproducible and provides an objective measurement of airflow obstruction. Screening for COPD with spirometry in individuals without respiratory symptoms is not recommended, but it is indicated in this patient with symptoms and a suggestive clinical history for COPD. CT scanning is used to evaluate parenchymal lung lesions (such as nodules) and other potential chest pathology (such as pleural or mediastinal disease). However, it is not effective in diagnosing COPD, particularly in this patient with a normal plain chest radiograph. In addition, guidelines recommend low-dose CT scanning for current or former smokers aged 55 to 80 years with a smoking history of at least 30-pack-years with no history of lung cancer; this patient does not fit these criteria. Polysomnography is used for evaluation of suspected sleep-disordered breathing (such as sleep apnea). However, this patient does not have symptoms suggestive of sleep apnea or other sleep or nocturnal breathing disorders. Therefore, polysomnography is not indicated. Although gastroesophageal reflux disease (GERD) is a common cause of chronic cough, and a trial of an empiric proton pump inhibitor is reasonable in patients with unexplained cough and otherwise negative evaluation, GERD would not account for this patient's dyspnea symptoms. Because his clinical presentation is more consistent with mild COPD, empiric treatment for GERD would not be the preferred next management step.

A 58-year-old man is evaluated in the emergency department for painless bright red blood per rectum that began 3 hours ago. The bleeding was accompanied by a presyncopal episode. Medical history is unremarkable. He does not smoke, drink alcoholic beverages, or use illicit drugs. He is on no prescription medications but has been taking over-the-counter ibuprofen recently for a knee sprain. On physical examination, temperature is 37.2°C (99.0°F), blood pressure is 88/58 mm Hg, pulse rate is 132/min, and respiration rate is 24/min. Abdominal examination is normal. Rectal examination discloses bright red blood in the rectal vault. Laboratory studies reveal a hemoglobin level of 7.3 g/dL (73 g/L). Nasogastric tube aspirate shows no evidence of blood or coffee ground material. Intravenous fluid resuscitation is begun. Which of the following is the most appropriate diagnostic test to perform next? A. Colonoscopy B. Tagged red blood cell scan C. Upper endoscopy D. Video capsule endoscopy ​

C. Upper endoscopy Educational Objective: Evaluate the source of suspected upper gastrointestinal bleeding. The most appropriate diagnostic test to perform next is upper endoscopy. In a patient with suspected upper gastrointestinal bleeding, the presenting symptoms can suggest the degree and volume of blood loss. The presence of melena (black, tarry stools) suggests an upper gastrointestinal tract source but can be associated with loss of as little as 150 to 200 mL of blood. Hematemesis of bright red blood is associated with ongoing upper gastrointestinal bleeding, whereas hematochezia secondary to an upper gastrointestinal source is suggestive of brisk ongoing bleeding of at least 1000 mL of blood. Presyncope and syncope may occur with hypovolemia secondary to bleeding. This patient has hematochezia, significant anemia, and hemodynamic instability. His use of ibuprofen for his knee injury increases the chance of an upper gastrointestinal source of bleeding. The absence of blood or coffee-ground material in the nasogastric tube aspirate does not rule out an upper gastrointestinal bleeding source; nasogastric tube placement can miss up to 1S% of actively bleeding lesions, especially if no bile is noted on the aspirate. Therefore, placement of a nasogastric tube when there is a high suspicion of upper gastrointestinal bleeding is not very helpful and should not guide a decision as to whether or not to perform upper endoscopy. A brisk upper gastrointestinal source of bleeding can cause hematochezia and can be life threatening if not acted upon early. If an upper gastrointestinal source is suspected, urgent upper endoscopy should be performed. If the upper endoscopy is unrevealing, a rapid lavage and colonoscopy is the next study, followed by a nuclear medicine tagged red blood cell scan if no bleeding source is found. Video capsule endoscopy is reserved for persistent occult (or overt) gastrointestinal bleeding without an identified upper or lower gastrointestinal source. KEY POINT: If an upper gastrointestinal source of bleeding is suspected in a patient with hematochezia, upper endoscopy is the most appropriate diagnostic procedure.

A 43-year-old man is evaluated in the emergency department for the "worst headache of my life." It occurred suddenly without warning. He has had mild headaches that come and go over the past 3 years, but nothing this severe. Soon after the headache began, he lost vision in his left eye, and the vision in his right eye became blurry. He vomited twice in the emergency department. His medical history is significant for progressive erectile dysfunction and loss of libido over the past 3 years. On physical examination, temperature is 37.4 °C (99.3 °F), blood pressure is 156/92 mm Hg, pulse rate is 104/min, and respiration rate is 16/min. BMI is 28. He has loss of vision in his left eye and in the upper quadrants of his right eye. He also has left eye ptosis. Other cranial nerves are intact. Strength and sensation in all extremities are normal as are his speech and gait. CT of the head shows acute pituitary hemorrhage. Pituitary MRI shows a 3.1 × 2.5 × 2.2-cm pituitary mass with central hemorrhage. The mass compresses the optic chiasm and the left cavernous sinus. After administering high-dose glucocorticoids, which of the following is the most appropriate immediate management? A. Assess pituitary function B. Repeat imaging in 2 weeks C. Urgent transsphenoidal pituitary decompression D. Whole brain external beam radiation

C. Urgent transsphenoidal pituitary decompression Educational Objective: Treat pituitary apoplexy. Key Point: Patients with pituitary apoplexy and vision loss should receive immediate stress-dose glucocorticoids in addition to undergoing urgent transsphenoidal pituitary decompression. The patient has acute apoplexy caused by pituitary hemorrhage and requires urgent transsphenoidal decompression of the hemorrhage to preserve vision. Given his history of previous headache, loss of libido, and erectile dysfunction, he likely had a preexisting prolactinoma that acutely bled. Prolactinomas are almost always treated with dopamine agonists, but this patient requires urgent surgery to decrease pressure on the optic chiasm to save his vision. Transsphenoidal resection is the preferred method of pituitary surgery. He also requires urgent stresss dose glucocorticoids because of risk of secondary cortisol deficiency, which would be life-threatening. The patient has appropriately received glucocorticoids for possible adrenocorticotropic hormone deficiency. There is no indication to assess other pituitary function. In the next 2 to 4 weeks, thyroid function will need to be assessed, but thyroxine (T4) has a long half-life and does not need to be emergently measured or replaced. Treatment of hypogonadism or growth hormone deficiency is not urgent; surgical decompression is. Waiting 2 weeks to repeat imaging could lead to permanent vision loss. He needs immediate intervention to preserve his vision. Similarly, whole brain external beam radiation is not appropriate for this patient who needs rapid surgical decompression

A 56-year-old man presents to the clinic for assessment of symptoms of chronic cough. It is present most of the time and is progressively getting worse over the past 3 years. With the cough he usually has white to yellow sputum that he has to expectorate. There is no history of wheezing, asthma, heart failure (HF), or acid reflux disease. He currently smokes 1 pack a day for the past 35 years. On examination, his chest is clear. CXR is normal and his forced expiratory volume in 1 second (FEV1) and forced vital capacity (FVC) on spirometry are normal. Which of the following is the most likely diagnosis? A. chronic obstructive pulmonary disease (COPD) B. early cor pulmonale C. chronic bronchitis D. asthma E. emphysema ​

C. chronic bronchitis Chronic bronchitis is a clinical diagnosis defined by the presence of chronic productive cough for 3 months in each of 2 successive years in a patient in whom other causes of chronic cough have been excluded. Emphysema is a pathologic term describing the abnormal permanent enlargement of airspaces distal to the terminal bronchioles, accompanied by destruction of their walls without obvious fibrosis. Emphysema may be noted in patients with COPD.

A 57-year-old man presents to the clinic for assessment of shortness of breath on exertion. The symptoms started many months ago after a "cold" and seem to be getting worse. There is no associated cough or sputum production, and he reports a 40-pack-per-year history of smoking. He otherwise feels well and his only past health history is hypertension that is well controlled on amlodipine. On physical examination, there are bilateral wheezes on expiration and increased resonance to percussion of the chest. Pulmonary function tests confirm the diagnosis of chronic obstructive lung disease (COPD). Which of the following is the best definition of this condition? A. it is caused by bronchial asthma B. it is preceded by chronic bronchitis C. it is airflow limitation that is not fully reversible D. it is due to destruction and dilatation of lung alveoli E. is due to small airways disease only

C. it is airflow limitation that is not fully reversible COPD is defined as a disease state that is characterized by airflow limitation that is not fully reversible. Emphysema and chronic bronchitis are closely related, and the term COPD is often used to encompass both. Chronic bronchitis is a clinical syndrome defined as excessive tracheobronchial mucous production severe enough to cause productive cough for at least 3 months of the year for at least 2 consecutive years. Emphysema is defined as the distention of air spaces distal to the terminal bronchiole, with destruction of alveolar septa. It is primarily a histologic diagnosis. Smoking is the usual antecedent for COPD.

A 22-year-old woman is evaluated for a 6-month history of decreased exercise tolerance, particularly with activities such as running. She is otherwise healthy and eats a normal diet. Medical history is unremarkable. She notes no menstrual abnormalities and takes no medications. On physical examination, temperature is 36.7°C (98.0°F), blood pressure is 110/72 mm Hg, pulse rate is 88lmin, and respiration rate is 161min. Body mass index is 22. The patient has pale conjunctivae. Examination of the heart and lungs is normal. There is no splenomegaly. The neurologic examination is normal. Laboratory studies: Hemoglobin 7.9 g/dL (79 g/L) Leukocyte count 5600/µL (5.6 x 109/L) Mean corpuscular volume 62 fl. Platelet count 625,000/µL (625 x 109/L) Red blood cell distribution width 22% (normal range: 14.6-16.5%) A peripheral blood smear is notable for microcytic, hypochromic erythrocytes with marked anisopoikilocytosis. Which of the following is the most appropriate treatment for this patient? A.Erythropoietin B. Erythrocyte transfusion C.Oral ferrous sulfate D.Parenteral (intramuscular or intravenous) iron

C.Oral ferrous sulfate Educational Objective: Treat iron deficiency in a menstruating woman. The most appropriate treatment is oral ferrous sulfate. Iron deficiency can result from blood loss or malabsorption in addition to increased iron use. Women of reproductive age may lose enough iron through normal menstrual blood loss to become iron deficient in the absence of uterine or gastrointestinal disease. Patients with mild iron deficiency may note fatigue, malaise, irritability, decreased exercise tolerance, and headaches before symptoms of overt anemia occur. This patient has signs and symptoms of iron deficiency, likely secondary to menstrual blood loss. The variation in the size of erythrocytes is quantified in the red blood cell distribution width (ROW) measurement. An increased ROW is most often associated with a nutrient deficiency such as iron, folate, or vitamin B12• Patients with iron deficiency anemia caused by blood loss can have mild thrombocytosis, which resolves with treatment of iron deficiency. For simple iron deficiency, oral ferrous sulfate, which is relatively inexpensive and available without a prescription, would be effective and is therefore the most appropriate treatment option. Initiation of erythropoiesis-stimulating agents should only be considered after patients have been evaluated for other causes of anemia and iron stores are adequate (serum ferritin level >100 ng/mL [100 µg/L] and transferrin saturation >20%). Erythrocyte transfusions are reserved for patients with severe symptomatic anemia in whom rapid correction is necessary to prevent cardiovascular complications, including heart failure and infarction. Parenteral iron, either intramuscular iron dextran or intravenous iron sucrose, is reserved for patients receiving dialysis or for patients who cannot absorb or tolerate oral iron replacement. KEY POINT: For patients with simple iron deficiency, oral ferrous sulfate is the least expensive and simplest treatment option.

A 55-year-old man is admitted to the hospital with increasing shortness of breath and dry cough for the past few years. He smokes 1.5 packs of cigarettes and drinks about four bottles of beer a day. He is constantly "gasping for air" and now walks with difficulty because he becomes breathless after only a few steps. Prolonged expiration with wheezing is noted. Physical examination shows a barrel chest, hyperresonance on percussion, and clubbing of the digits. The patient's face is puffy and red, and he has pitting edema of the legs. A chest X-ray discloses hyperinflation, flattening of the diaphragm, and increased retrosternal air space. Which of the following is the appropriate diagnosis? A. Asthma B. Chronic bronchitis C. Emphysema D. Hypersensitivity pneumonitis E. Usual interstitial pneumonia ​

C: Emphysema. Chronic obstructive pulmonary disease is a nonspecific term that describes patients with chronic bronchitis or emphysema who evidence a decrease in forced expiratory volume. Emphysema is characterized principally by hyperinflated lungs. Chronic bronchitis (choice B) occurs after recurrent infections and, like asthma (choice A), is not generally associated with hyperinflated lungs. The major cause of emphysema is cigarette smoking, and moderate-to-severe emphysema is rare in nonsmokers. ​

Given the previous clinical scenario, a histologic section of his lung would most likely reveal which of the following? A. Noncaseating granulomas and transmural neutrophilic vasculitis B. Hypertrophy of smooth muscle in the airways C. Alveolar macrophage infiltrate throughout all alveolar airspaces D. Hypertrophy of mucous glands in the airways E. Patchy fibrosis of the alveolar septa ​

D. Hypertrophy of mucous glands in the airways This patient has signs and symptoms consistent with chronic bronchitis. With chronic bronchitis, patients have a productive cough for several months in two consecutive years. Physical examination can reveal ronchi and signs of hypoxemia, with dyspnea occurring later in the course of the disease; whereas, in emphysema, dyspnea occurs earlier ("pink puffers"). The histologic finding of chronic bronchitis is hypertrophy of the mucous glands in the airways (D). The other histologic features are not typical of chronic bronchitis (A-C, E).

A 33-year-old man presents to the hospital complaining of dyspnea. Over the past year he has had increasing shortness of breath while running that limits the distance he can run. In addition, he has a persistent cough that is bothering him, and during the past few days he has had increasing sputum production that was previously white and is now yellow. He has not visited a doctor yet because he does not have medical insurance. The patient denies any significant medical history but has a positive family history of emphysema. He smokes half a pack of cigarettes daily and drinks alcohol moderately. On examination, he has a temperature of 37.9C, blood pressure of 122/76 mmHg, heart rate of 93 beats per minute, respiratory rate of 24 breaths per minute, and oxygen saturation of 92% on room air. He has scattered wheezes throughout both lung fields with a normal cardiac examination. A chest x-ray shows loss of interstitial lung markings primarily at the lung bases. Which of the following is the mechanism by which this disease produces liver disease? A. Failure to inhibit the destruction of connective tissue by elastase B. Pulmonary artery hypertension leading to increased pressure in the hepatic vein C. Direct hepatic injury resulting in Mallory bodies D. Accumulation of abnormal proteins within hepatocytes

D. Accumulation of abnormal proteins within hepatocytes Accumulation of abnormal proteins within hepatocytes. For a young patient presenting with symptoms and signs of COPD, always consider α1-Antitrypsin deficiency as the cause. Although this autosomal recessive disease usually presents in patients after the age of 40, it can present earlier if the patient is a smoker. α1-Antitrypsin is a protease inhibitor that is produced in the liver and inhibits elastase in the lungs. During acute inflammation, neutrophils release elastase in the lungs that can degrade elastin, an important structural protein that helps tissues maintain their shape. Normally this is inhibited by α1-antitrypsin; however, patients with α1-Antitrypsin deficiency will develop panacinar emphysema due to unopposed destruction of elastin in the lung parenchyma. If a patient is exposed to greater than average amounts of toxins (e.g., smoking) that produce lung injury and inflammation, they will develop manifestations of emphysema at a younger age. The clues to the diagnosis in this case are the fact that the patient is very young, has a family history of emphysema, and has emphysema that is predominantly affecting the lung bases (seen on chest x-ray). Another clue is that the question stem states that liver disease is associated with the condition. Not all mutations cause liver disease, but some genotypes lead to the production of an abnormal protein within hepatocytes that polymerizes and causes cell apoptosis. (A) Therefore, the mechanism of liver disease is different from the mechanism of lung disease, with connective tissue destruction not being a prominent feature. When the liver is biopsied, the cytoplasmic inclusions are periodic acid-Schiff positive. Skin findings (e.g., necrotizing panniculitis) are another extrapulmonary manifestation that may be associated with this disease. Diagnosis of α1-Antitrypsin deficiency is made by finding low serum levels of α1-Antitrypsin as well as genotyping the patient. Treatments include smoking cessation, IV supplementation of α1-Antitrypsin, standard treatments for COPD based on severity of disease, and lung or liver transplant if end-stage disease is present. (B) Emphysema causes pulmonary vascular destruction, which can lead to increased pulmonary artery pressure and eventually cor pulmonale. Congestive hepatopathy as a result of elevated venous pressure would exacerbate the liver disease, but it is not the primary cause of the liver disease in α1-Antitrypsin deficiency. (C) Mallory bodies are cytoplasmic inclusions seen within hepatocytes in alcoholic liver disease. They are not 100% specific to alcoholic liver disease and may rarely be seen on histology in α1-Antitrypsin deficiency. However, this is not the best explanation for the mechanism of liver disease in these patients

A 21-year-old woman with type 1 diabetes mellitus suddenly develops tremors, cold sweats, and confusion while on a backpacking trip with friends. She is only oriented to person and is unable to follow commands. Her fingerstick blood glucose concentration is 28 mg/dL. Her friend administers an intramuscular injection with a substance that reverses her symptoms. Which of the following is the most likely mechanism of action of this drug? A. Activation of Glucokinase B. Inhibition of glucose-6-phosphatase C. Inhibition of α-glucosidase D. Activation of Adenylyl cyclase E. Inhibition of glycogen phosphorylase

D. Activation of Adenylyl cyclase Reversal of this patient's symptoms of severe hypoglycemia was achieved by injecting glucagon.

A 29-year-old woman with a history of asthma presents with progressive worsening of respiratory function. She reports that her symptoms have developed gradually over the past few months, with occasional fevers and episodes of mild hemoptysis. She denies any weight loss, skin changes, or diarrhea. Her medications include inhaled albuterol as needed and OCPs. She has no family history of cancer, no recent travel, and no recent sick contacts. She works as an accountant and does not smoke or use any illicit drugs. On examination, she has a low-grade fever with scattered wheezes and rhonchi over bilateral lung fields. After coughing vigorously, a brown mucus plug is expectorated. Her laboratory values show eosinophilia with an elevated total serum IgE. A chest x-ray shows interstitial infiltrates in the upper lobes with some areas of atelectasis bilaterally, and a CT scan shows enlarged airways primarily in the upper lobes with bronchial wall thickening. Which of the following is the most likely diagnosis? A. Tuberculosis B. Pseudomonas aeruginosa C. Strongyloides stercoralis D. Allergic bronchopulmonary aspergillosis E. Bronchial carcinoid tumor F. Worsening asthma

D. Allergic bronchopulmonary aspergillosis Allergic bronchopulmonary aspergillosis. The key to answering this question is knowing what conditions are associated with an underlying diagnosis of asthma. Allergic bronchopulmonary aspergillosis (ABPA) is a hypersensitivity reaction to Aspergillus colonization in the bronchi. These patients may develop worsening respiratory function with hemoptysis, and can eventually develop bronchiectasis, which was seen on this patient's CT scan. Be suspicious about this diagnosis in patients with asthma, peripheral eosinophilia, and elevated IgE, and imaging findings of lung infiltrates and bronchiectasis. Diagnosis is further suggested by a skin prick test to assess for reactivity to Aspergillus antigens. Treatment is with systemic glucocorticoids and itraconazole. Other important associations with asthma include aspirin allergy (asthma with sensitivity to aspirin and nasal polyps), Churg-Strauss (asthma with vasculitis), and the atopic triad (asthma with eczema and allergic rhinitis). (A) The patient has no risk factors for tuberculosis, and this would not present with eosinophilia and enlarged airways (chronic untreated tuberculosis could cause airway destruction and bronchiectasis, but an acute infection would not). (B) Pseudomonas is a common cause of pulmonary infection in cystic fibrosis patients, but this patient lacks other findings that suggest cystic fibrosis (pancreatic insufficiency, family history, etc.). (C) Strongyloides stercoralis is a parasitic nematode that can penetrate the skin and migrate to the lungs, resulting in wheezing and coughing. Eosinophilia and elevated IgE may be seen in parasitic infections; however, she has no apparent risk factors for S. stercoralis and the CT scan is consistent with ABPA, not S. stercoralis infection. (E) Carcinoid syndrome is caused by a neuroendocrine tumor that is usually in the GI tract or bronchus. There is a lower rate of carcinoid syndrome in bronchial tumors than in GI tumors due to less production of serotonin and other vasoactive amines. Other symptoms of carcinoid syndrome include cutaneous flushing and diarrhea. (F) Infiltrates and bronchiectasis on imaging cannot be explained solely by worsening asthma.

Bleeding from a Dieulafoy gastric lesion is due to: A. Antral vascular ectasia B. Abnormal gastric rugal folds C. Ingested foreign material D. An abnormal submucosal vessel E. A premalignant lesion ​

D. An abnormal submucosal vessel A Dieulafoy lesion is a congenital malformation in the stomach (typically on the lesser curvature) characterized by a submucosal artery that is abnormally large and tortuous. As a result of its relatively superficial location, it may erode through the mucosa and become exposed to gastric secretions, leading to massive upper GI hemorrhage. On endoscopy, the mucosa of the stomach appears normal, and the only finding is a pinpoint area of mucosal defect with brisk arterial bleeding. The lesion may easily be missed if the bleeding is not active. Dieulafoy lesion is not premalignant (E) and is not associated with the ingestion of foreign material (C). Treatment is endoscopic, via electrocautery, heater probe, or injection with a sclerosing agent. Surgery, which consists of a wedge resection, is reserved for the rare patient who is not controlled endoscopically. Antral vascular ectasia (A) is seen in a condition known as watermelon stomach and can lead to significant acute or chronic GI blood loss. Dilated mucosal blood vessels containing thrombus, mucosal fibromuscular dysplasia, and hyalinization are prominent features. It derives its name from the mucosal vessels that create parallel lines in the mucosal folds (B). The stomach is typically not enlarged. It is seen predominantly in elderly women with autoimmune disease or elderly males with cirrhosis.

Which of the following antibodies is a classification criterion for systemic lupus erythematosus? A. Anti-La B. Anti-Ro C. Anti-RNP D. Anti-Sm E. Anti-ribosomal P ​

D. Anti-Sm Although the other antibodies are often found in patients with lupus, only anti-Sm is a classification criterion. Anti-Sm and anti-RNP commonly occur together, and in lupus, anti-RNP may be more frequent. AntiRNP, however, has been associated with an entity known as mixed connective tissue disease, which has overlapping features with other rheumatic diseases. As a result, anti-RNP has not been a classification criterion. ​

A 52-year-old woman is experiencing abdominal discomfort after meals as well as early in the morning. There is no weight loss or constitutional symptoms, and she has tried antacids but experiences minimal relief. Upper endoscopy reveals a duodenal ulcer and the biopsy is negative for malignancy. Which of the following is the most appropriate next step in management? A. 6-8 weeks of omeprazole or ranitidine B. long-term acid suppression with omeprazole C. antibiotic therapy D. antibiotic therapy and omeprazole E. bismuth citrate therapy ​

D. Antibiotic therapy and omeprazole Antibiotic therapy for eradication of H pylori in combination with acid suppression with a proton pump inhibitor like omeprazole is the most effective treatment for duodenal ulcer disease. The most popular regimes include antibiotics and acid suppression medications.

The most significant etiologic factor in peptic ulcer disease is: A. Duodenogastric reflux B. Acid hypersecretion C. Nonsteroidal antiinflammatory drug (NSAID) ingestion D. Bacterial colonization E. Smoking ​

D. Bacterial colonization Helicobacter pylori is considered the most significant etiologic factor in peptic ulcer disease. It is reportedly found in 80% to 90% of duodenal ulcers and more than 70% of gastric ulcers. H. pylori is a helix-shaped, microaerophilic gram-negative rod and is more prevalent in low and middle income countries, where as many as 70% of the population are colonized with H pylori. H. pylori also causes acute gastritis. The exact mechanism by which it causes ulceration is unclear, but it may be by the production of local toxic products, induction of a local immune response, or an increase in gastrin levels leading to an increase in acid (B). In fact, the organism seems to be the cause of hypergastrinemia in duodenal ulcers because the infection reduces the number of antral D cells, thus reducing somatostatin levels, which then disinhibits G (gastrin) cells. H. pylori is a potent producer of urease, which splits urea into ammonia and bicarbonate. This allows the bacteria to survive in a microenvironment of relative alkalinity within the stomach. It only lives in gastric epithelium (including heterotopic gastric mucosa) because it expresses specific adherence receptors recognized by the bacteria. After H. pylori, ingestion of NSAIDs (C) and smoking (E) are the next most important risk factors. Smoking increases gastric acid secretion and duodenogastric reflux (A) and decreases pancreaticoduodenal bicarbonate production.

A 45-year-old woman is undergoing an exploratory laparotomy for Zollinger-Ellison syndrome (ZES). Preoperative localization studies failed to demonstrate the location of the tumor. At surgery, no obvious tumor is seen despite an extensive Kocher maneuver and careful inspection. An intraoperative ultrasound scan is negative. The next step in the management would be: A. Closing the abdomen B. Distal pancreatectomy and splenectomy C. Proximal pancreaticoduodenectomy D. Blind proximal duodenotomy E. Blind distal duodenotomy ​

D. Blind proximal duodenotomy More than 80% of gastrinomas are localized preoperatively. For those that cannot be localized, surgical exploration is still indicated because excision of the primary tumor leads to a decreased rate of liver metastasis. When exploring, it is important to be aware that 80% of gastrinomas are found within the gastrinoma (Passaro) triangle, an area defined by the junction of the cystic duct and common bile duct, the second and third portions of the duodenum, and the neck and body of the pancreas. As many as 60% of gastrinomas are within the wall of the duodenum, primarily in the first and second portions and can be very small. Thus, the next maneuver would be to perform a blind proximal duodenotomy to manually palpate the duodenal wall for tumors. Closing the abdomen (A) would be inappropriate. Blind distal pancreatectomy and splenectomy (these share blood supply) (B) or distal duodenotomy (E) would have very low yields. A pancreaticoduodenectomy (Whipple procedure) (C) would not be indicated in this setting. It is potentially indicated for multiple duodenal or proximal pancreatic head tumors that could not be enucleated.

Which of the following is the best management for primary gastric (non-MALT [mucosa- associated lymphoid tissue]) lymphoma? A. Surgical resection B. Chemotherapy followed by surgical resection C. Radiation therapy D. Chemotherapy E. Surgical resection followed by radiation therapy ​

D. Chemotherapy Gastric lymphoma is a subtype of non-Hodgkin lymphoma (NHL) and considered the most common form of extranodal NHL. The vast majority are either MALT gastric lymphoma or diffuse large B-cell lymphoma (DLBCL). The current foundation for the treatment of gastric DLBCL is chemotherapy (A, B). Depending on the response, and initial tumor size, locoregional radiation therapy may be added (C, E).

A 68-year-old woman with a history of hypothyroidism and COPD presents for follow-up with her primary care physician. She has no complaints and denies any recent fatigue, weight loss, increase in cough or sputum production, or chest pain. Her medications include inhaled ipratropium and levothyroxine. She is normotensive with an SaO2 of 92% on room air. On examination, her lungs are hyper-resonant to percussion with some scattered wheezes and rhonchi. She has clubbing of the digits, which was not present 1 year ago. What is the most appropriate next course of action? A. Reassurance and regular follow-up B. Home oxygen therapy C. Pulmonary function tests D. Chest imaging ​

D. Chest imaging Clubbing of the digits is not associated with COPD, and therefore any new onset of clubbing in a COPD patient is a red flag for another etiology such as bronchogenic carcinoma. Chest imaging is an important diagnostic step to look for cancer. Other etiologies of clubbing include interstitial lung disease, cystic fibrosis, congenital heart disease, and malabsorption. (A) Because the clubbing is unrelated to the patient's COPD reassurance is inappropriate. (B) This patient does not meet criteria for home oxygen therapy (SaO2 ≤ 88%). (C) The patient is not having an exacerbation and has no reason to undergo pulmonary function testing at this time.

Which of the following is associated with hypergastrinemia? A. Diabetes B. Hypothyroidism C. Hyperparathyroidism D. Chronic gastritis E. D-cell hyperplasia ​

D. Chronic gastritis When considering gastrinoma, it is important to be aware of the differential diagnosis of an elevated gastrin level. Causes of hypergastrinemia with increased acid production include gastrinoma, G-cell hyperplasia (not D-cell) (E), retained antrum after distal gastrectomy, renal failure, and gastric outlet obstruction. Hypergastrinemia with normal or low acid production includes pernicious anemia, postvagotomy states, use of acid-suppressive medication, and chronic gastritis. Hypothyroidism is associated with a low gastrin level, whereas hyperthyroidism increases gastrin levels (B). Diabetes (A) and hyperparathyroidism (C) do not affect gastrin levels.

A 40-year-old woman is evaluated for amenorrhea of 4 months' duration. She has had weight gain, facial hair, alopecia, and debilitating fatigue. Her medical history is significant for psoriasis. She seems to be gaining weight in her face, abdomen, and neck. She also bruises easily. Her only medication is clobetasol for psoriasis. On physical examination, temperature is 37.6 °C (99.7 °F), blood pressure is 148/90 mm Hg, pulse rate is 88/min, and respiration rate is 12/min. BMI is 38. She is obese with a round face. She has terminal hairs on her chin, upper lip, chest, and back. Mild facial acne, central obesity, and a few wide purple striae on the back of her arms are also noted. She has supraclavicular fat. Her skin has psoriatic plaques. Muscle strength in the upper and lower extremities is 4/5. Which of the following is the most likely diagnosis? A. Adrenocortical carcinoma B. Cushing disease C. Ectopic adrenocorticotropic hormone production IIatrogenic D. Cushing syndrome

D. Cushing syndrome Educational Objective: Diagnose Cushing syndrome from exogenous glucocorticoids. Key Point: The most common cause of Cushing syndrome is an elevated level of cortisol resulting from both endogenous and exogenous exposure to glucocorticoids. The patient has iatrogenic Cushing syndrome caused by use of topical glucocorticoids in the treatment of psoriasis. Cushing syndrome presents similarly whether it is due to a pituitary adenoma (Cushing disease), adrenal tumor cortisol production, ectopic adrenocorticotropic hormone production, or excessive use of glucocorticoids. Her presentation would be consistent with any of these diagnoses; however, she is on high potency topical glucocorticoids, so this alone explains her symptoms and presentation. Exogenous glucocorticoid use as a cause of Cushing syndrome is common, whereas the other causes are rare.

In a laboratory research project, a mouse is treated with a drug that inhibits the activity of the major enzyme responsible for the catabolism of circulating thyroid hormones. Identify the pattern of changes in TSH, free T4 and protein-bound T4 that are most likely to be detected in this animal immediately following the drug treatment: A. Increased, increased, increased B. Increased, decreased, decreased C. Increased, decreased, increased D. Decreased, increased, increased E. Decreased, increased, decreased ​

D. Decreased, increased, increased A reduction in thyroid hormone catabolism will acutely increase the quantity of free T4 in the circulation (ruling out options B and C), which will also equilibrate with plasma binding proteins to increase the levels of bound T4 (ruling out options B and E). An increase in circulating free T4 will feedback on the pituitary to reduce the synthesis and secretion of TSH (ruling out options A, B, and C). Thus, by a process of elimination, the only possible option is D. Of course, if treatment is continued over a longer time period, a new steady state will likely be reached whereby thyroid secretion is suppressed, secondary to the reduction in TSH levels, to reestablish homeostasis. ​

A 40-year-old woman with moderate persistent asthma has been under good control for 3 months and is currently using her albuterol MDI for symptomatic control once weekly. She awakens at night twice monthly with asthma symptoms, but continues to exercise regularly without difficulties. Her other medications include fluticasone inhaled 88 µg/puff twice daily and salmeterol 50 µg twice daily. Her FEV1 is currently at 83% of her personal best. Which action is most appropriate at the present time? A. Add montelukast 10 mg once daily, as the current albuterol usage suggests poor asthma control. B. Decrease the fluticasone to 44 µg/puff twice daily. C. Discontinue the fluticasone. D. Discontinue the salmeterol. E. Do nothing, as the current albuterol usage suggests poor asthma control. ​

D. Discontinue the salmeterol. A step-down in asthma therapy can be considered when an individual has been clinically stable for 3-6 months. Factors demonstrating appropriate asthma control include daytime symptoms 2 or fewer times weekly, nighttime symptoms 2 or fewer times monthly, use of rescue inhaler 2 or fewer times weekly, FEV1 or peak expiratory flow rate at least 80% of personal best, or appropriate control by validated asthma control questionnaires such as the Asthma Control Test or Asthma Therapy Assessment Questionnaire. When stepping down therapy, it is important to review the medications and their dosages. This patient is currently being managed with low-dose inhaled corticosteroids plus a long-acting beta-agonist. At this point, the best course of therapy is to stop the long-acting beta-agonist salmeterol. Since the dose of fluticasone the patient is receiving is already at a low dose, it would not be recommended to decrease it further, and it is never appropriate to treat with a long-acting beta-agonist alone without inhaled corticosteroids. In a large clinical trial, asthma mortality increased in individuals treated with long-acting beta-agonists in the absence of therapy with inhaled corticosteroids. Adding another medication is not indicated as the patient is demonstrating good asthma control.

A 30-year-old woman with a history of asthma presents to her primary care physician complaining of increased urination and thirst over the past 2 months. She has also recently gained 3.2 kg (7 lb). She has a history of multiple hospitalizations for asthma exacerbations, the last of which was 1 month ago and for which she received intravenous methylprednisolone. Her regimen was changed from inhaled fluticasone to an oral agent 8 months ago secondary to an increase in frequency of the asthma exacerbations. Current medications consist of albuterol as needed and oral methylprednisolone every other day. Laboratory tests show her plasma ACTH level at 8 a.m. is 5.2 pg/mL (normal: 20-80 pg/mL) and her fasting blood glucose level is 328 mg/dL. Which of the following is the most likely explanation for these findings? A. ACTH-secreting pituitary adenoma B. Congenital adrenal hyperplasia C. Ectopic production of ACTH D. Exogenous corticosteroids E. Polycystic ovaries leading to increased circulating androgens

D. Exogenous corticosteroids This patient exhibits signs of increased blood glucose and weight gain. In the setting of chronic steroid use, this should raise the suspicion for Cushing syndrome. Cushing syndrome, or hypercortisolism, can cause weight gain and symptoms of diabetes, such as increased urination, thirst, and elevated blood glucose. Plasma ACTH levels are low when hypercortisolism is due to an exogenous cause. ACTH levels vary throughout the day with the highest serum levels in the early morning and the lowest serum levels in late evening. Thus when there is suspicion of adrenal insufficiency or (as in this case) exogenous steroid use, one should measure ACTH levels in the morning. Bottom Line: In the setting of chronic steroid use, signs of increased circulating androgens and weight gain should raise the suspicion for Cushing syndrome. ​

A patient with a BMI of 45, HTN, sleep apnea, and significant knee pain would be best treated with: A. Diet modification B. Diet modification and medical weight management program C. Gastric banding D. Gastric bypass E. Nothing ​

D. Gastric bypass For patients with this high of a BMI, medical therapy and a weight loss program are very unlikely to work, although patients should be started on these programs for education about future dietary habits. The best long term solution for this patient is laparoscopic gastric bypass. ​

A 28-year-old woman comes to the office with acute pain in her left knee joint She had mild discomfort and pain in her right wrist 4 days ago and left ankle pain 2 days ago The patient has had no recent respiratory illness, diarrhea, urinary symptoms, or vaginal discharge. Her medical history is unremarkable, and she takes no medications except oral contraceptives. The patient drinks a glass of wine daily, but does not smoke or use intravenous drugs. She is single and sexually active. Her last menstrual period was a week ago. Temperature is 38.5 C (101.3 F), blood pressure is 120/80 mm Hg, pulse is 98/min, and respirations are 15/min. Inspection of the knee shows a moderate effusion and faint erythema. There is warmth and tenderness on palpation Range of motion is limited by significant pain. No skin lesions are present Synovial fluid analysis shows a white blood cell count of 50,000/mm3 with a predominance of neutrophils Which of the following is the most likely cause of this patient's symptoms? A. Acute HIV infection B. Acute rheumatic fever C. Crystal-induced arthritis D. Gonococcal septic arthritis E. Infective endocarditis F. Non-gonococcal septic arthritis G. Reactive arthritis ​

D. Gonococcal septic arthritis This patient has features suggesting septic arthritis, including fever and a neutrophilic inflammatory joint effusion. In a young, sexually active individual, septic arthritis is most often caused by disseminated gonococcal infection (DGI). DGI may present with asymmetric polyarthralgias (often but not always associated with tenosynovitis and skin rash) or an isolated septic monoarthritis or oligoarthritis As in this patient, migratory polyarthralgias may precede purulent monoarthritis, and overlap syndromes can occur. Although the infection originates in genitourinary, pharyngeal, or rectal mucosa, the primary infection is usually silent. Diagnosis of DGI requires joint aspiration for synovial fluid analysis . The synovial white blood cell count is elevated but highly variable and typically lower than in septic arthritis due to other causes. Diagnosis is confirmed by Gram stain of the synovial fluid, blood cultures, and genital/pharyngeal nucleic acid amplification tests for Neisseria gonorrhoeae. Educational objective: Disseminated gonococcal infection may present with asymmetric polyarthralgias (often with tenosynovitis and rash) or isolated septic arthritis. Neisseria gonorrhoeae is the most common cause of septic arthritis in young, sexually active patients. Diagnosis is confirmed by Gram stain of the synovial fluid, blood cultures, and genital/pharryngeal nucleic acid amplification tests.

A 68-year-old woman presents to the Emergency Department with shortness of breath and right upper quadrant pain. She has a long history of hypertension and COPD, for which she takes captopril and a variety of inhaled medications including ipratropium, albuterol, salmeterol, and fluticasone. Her vitals show a temperature of 37 C, blood pressure of 102/64 mmHg, heart rate of 108 beats per minute, respiratory rate of 30 breaths per minute, and oxygen saturation of 83% on room air. When speaking, she is gasping for air in between words and using accessory muscles of respiration. Her jugular veins are distended, and she has a right ventricular heave with an S4 and a holosystolic murmur at the left lower sternal border. Breath sounds are decreased bilaterally, with scattered wheezes and rhonchi heard over the lung fields without significant rales. In addition, there is hepatomegaly and pitting edema around the ankles. An ECG shows right axis deviation with a right bundle branch block. Which of the following could have delayed the onset of this complication? A. Digoxin B. Diuresis with furosemide C. Vaccination against pneumococcus and influenza D. Home oxygen therapy

D. Home oxygen therapy Cor pulmonale is defined as right ventricular dysfunction as a result of lung disease. COPD is the most common cause of cor pulmonale and causes pulmonary hypertension from emphysematous destruction of the lung parenchyma and vasculature. Chronic pulmonary hypertension will eventually cause right heart failure, which will manifest with jugular venous distention, hepatomegaly, and edema. Important signs of right heart strain from pulmonary hypertension include tricuspid regurgitation, right ventricular heave, S4, and ECG findings of right ventricular hypertrophy. Home oxygen therapy delays the onset of this deadly complication of COPD by decreasing hypoxemic pulmonary vasoconstriction and therefore decreasing the pressure in the pulmonary artery. This is an important treatment for advanced COPD and has been shown to improve mortality. (A) Digoxin is a cardiac glycoside that increases contractility of the ventricles. It is used for symptomatic relief in heart failure; however, it is not very useful in isolated right heart failure and would not delay the development of cor pulmonale in this patient. (B) Diuresis is an option for treating heart failure; however, it would not delay the development of cor pulmonale. Diuresis is complicated in patients with cor pulmonale. Some reduction in excessive preload will improve right ventricular cardiac output, but they are also very preload dependent and their cardiac output will decrease quickly if they are diuresed too much. (C) Vaccination is an important preventive measure in patients with COPD to reduce the risk of serious lung infections that cause life-threatening acute exacerbations. This patient is afebrile and having shortness of breath as a result of right heart failure, not a respiratory infection.

A 42-year-old woman comes to the office due to several months of episodic shortness of breath and cough. The patient has a history of migraines and occasionally takes acetaminophen. She is a school teacher and does not use tobacco; alcohol, or illicit drugs. Temperature is 36.1 C (97 F), blood pressure is 110/80 mm Hg. and pulse is 84/min. Pulse oximetry shows 96% on room air. Lung examination and chest x-ray are normal. Spirometry shows that FEV1 is 71% of the predicted value. Sputum analysis shows predominant eosinophils. Exposure to which of the following is most closely associated with this patient's symptoms? A. Acetaminophen B. Gliadin-containing grains C. Histoplasma microconidia D. House dust mites E. Shellfish and other seafood F. Thermophilic actinomycetes ​

D. House dust mites This patient's intermittent respiratory symptoms (e.g., dyspnea, cough) in the setting of a normal chest x-ray, sputum eosinophils, and reduced FEV1 (<80% predicted) are highly suggestive of asthma. Asthma is a reversible obstructive pulmonary disease characterized by bronchial hyperresponsiveness to various triggers, such as exercise, cold air, respiratory infection, and exposure to inhaled allergens. Common aeroallergens include dust mites, cockroaches, pet dander, mold, and pollen. In sensitive individuals, allergens promote bronchospasm and airway inflammation by interacting with IgE bound to pulmonary mast cells: subsequent mast cell activation leads to the release of inflammatory mediators (e.g., histamine, prostaglandins, leukotrienes) and recruitment of inflammatory cells, particularly eosinophils. Therefore, allergen avoidance is an important preventive measure for patients with asthma. Educational objective: Intermittent respiratory symptoms in a patient with a normal chest x-ray, sputum eosinophils, and reduced FEV1 suggest asthma. Common asthma triggers include exercise, cold air, respiratory infection, and exposure to inhaled allergens (e.g., dust mites, cockroaches, pet dander, mold, pollen).

A 61-year-old man with increasing fatigue, early satiety, and nausea for 5 months vomited dark granular material yesterday. Endoscopy reveals a large ulcerated mass in the gastric fundus. Biopsies are taken and microscopically the mass is composed of spindle cells that are positive for c-Kit with immunohistochemical staining. Mitoses are frequent. Gastrectomy is performed, and the 10-cm circumscribed mass arises from the gastric wall. Which of the following therapies is most likely to be a useful adjunct in treatment of his disease? A. Amoxicillin B. Azathioprine C. Cyclophosphamide D. Imatinib E. Prednisone F. Radiation ​

D. Imatinib This gastrointestinal stromal tumor (GIST) is derived from the interstitial cell of Cajal, and hence of mesenchymal origin. Those arising in the stomach may be less aggressive than those arising in the intestine, but most are KIT positive and amenable to tyrosine kinase inhibitor therapy. Some GISTs may have mutations in platelet-derived growth factor receptor-α(PDGFR-α). Antibiotic therapy to obliterate Helicobacter pylori infection may be useful in treating MALTomas. Azathioprine and corticosteroids may be employed in treating inflammatory bowel disease, but the greatest risk for malignancy with inflammatory bowel disease is adenocarcinoma, particularly of the colon. Cyclophosphamide is a chemotherapy agent not employed in treating GISTs. Radiotherapy is not generally effective against mesenchymal malignancies.

A 30-year-old female comes to the office with complaints of dyspnea, cough, and wheezing for the past 4 months. She reports that initially the symptoms were related only to exercise, which forced her to stop her aerobic workouts. Now she has daily symptoms even at rest and awakens several nights a week with wheezing. She is a non-smoker. She denies any particular environmental factors and had no respiratory problems until now. On exam, she has a very mild end-expiratory wheeze. Office spirometry reveals a reduced FEV1 and FEV1/FVC at 70% predicted. Which of the following is the best initial treatment plan? A. Low-dose inhaled corticosteroid alone B. Inhaled cromolyn, 4 times a day C. Inhaled beta2-agonist, every 4-6 hours D. Inhaled corticosteroid plus long-acting beta-agonist E. Sustained release theophylline, twice a day

D. Inhaled corticosteroid plus long-acting beta-agonist This patient has asthma that would be characterized as moderate-persistent by the National Asthma Education and Prevention Program Guidelines. As such, she should be on daily medication with an inhaled corticosteroid and a long-acting inhaled beta agonist; alternatively, you could prescribe medium doses of an inhaled corticosteroid alone. The emphasis on utilizing anti-inflammatory therapies (inhaled corticosteroids) was a major focus on the NAEPP guidelines for any asthma patient characterized as mild-persistent or greater. This was to overcome the over-reliance on symptomatic relief with beta2-agonist inhalers and the rising number of asthma deaths (most clutching their beta-agonist inhalers and on no steroids!). A beta2-agonist inhaler must be available for symptomatic relief with the understanding that more frequent use of the beta-agonist inhaler indicates a need for stepped-up anti-inflammatory therapy (higher-dose inhaled steroids, PO prednisone). Sustained release theophylline may help with nocturnal symptoms but has no anti-inflammatory action. Inhaled cromolyn alone can take weeks to start working and is best used in atopic asthma in children. Board Testing Point: Recognize how to implement clinical guidelines for treatment of asthma.

An 18-year-old girl is brought in to the hospital by her parents because of an asthma attack. She normally takes inhaled fluticasone and albuterol; however, she is not currently responding to the albuterol. On examination, she appears anxious and is moderately short of breath. There are loud bilateral wheezes on examination, with a prolonged inspiratory and expiratory phase and use of accessory muscles of respiration. Her temperature is 37.6 C, blood pressure is 118/78 mmHg, heart rate is 106 beats per minute, respiratory rate is 26 breaths per minute, and oxygen saturation is 93% on room air. She is placed on supplemental oxygen and bronchodilators and an arterial blood gas is drawn. Arterial Blood Gas pH 7.40 PaO2 60 mmHg PaCO2 40 mmHg Which of the following is the most appropriate next step in management? A. Increase supplemental oxygen flow rate B. IV corticosteroids C. Azithromycin D. Intubation ​

D. Intubation. The patient in this vignette is experiencing an acute asthma exacerbation with worrisome symptoms/signs: loud wheezes, use of respiratory accessory muscles, tachycardia, tachypnea, and poor oxygen saturation. In addition, the arterial blood gas shows a normal PaCO2, which is not a good sign; hypoxic patients should hyperventilate to maintain oxygenation, which causes hypocapnia and a respiratory alkalosis. When a patient has a normal PaCO2, this is a sign that the patient is tiring and decompensating. Even though intubation should be avoided if possible, there is a low threshold for intubating patients that are showing signs of respiratory fatigue. (A) Increasing the oxygen flow rate might improve oxygenation, but the patient is likely starting to develop hypercapnic respiratory failure, which is due to inadequate ventilation. Once the patient is on a ventilator, ventilation is controlled primarily by adjusting the respiratory rate or tidal volume. Oxygenation can be maintained by adjusting the FiO2 and positive end-expiratory pressure (PEEP). (B) IV corticosteroids are used in acute exacerbations but will not act immediately to prevent this patient from further decompensation. (C) Azithromycin and other antibiotics are used in acute COPD exacerbations, not acute asthma exacerbations.

A 70-year-old man presents to the ED with sudden onset of severe epigastric pain associated with retching but with little vomitus. His blood pressure is 140/90 mm Hg and his heart rate is 90 beats per minute. Attempts by the ED physician to place a nasogastric tube are unsuccessful. Upright chest radiograph reveals a large gas bubble just above the left diaphragm. Which of the following is true regarding this condition? A. The stomach is likely twisted along the axis transecting the lesser and greater curvature. B. In children it is largely due to a paraesophageal hernia. C. It is associated with Bergman's triad. D. It is associated with a wandering spleen. E. It is initially managed conservatively for the majority of patients. ​

D. It is associated with a wandering spleen. Gastric volvulus is associated with Borchardt's triad (sudden onset of severe upper abdominal pain, recurrent retching without vomitus, and an inability to pass a nasogastric tube). Bergman's triad (mental status changes, petechiae, and dyspnea) is seen with fat emboli syndrome (C). The volvulus can be either organoaxial (twisting around the axis between the gastroesophageal junction and pylorus), which is twice as common, or mesenteroaxial (twisting along the axis between the lesser and greater curvature) (A). Gastric volvulus most commonly occurs in association with a diaphragmatic defect. The stomach becomes trapped in the defect and twists. In children, the defect is congenital (such as a Rochdale hernia), whereas in adults, it is more often traumatic or secondary to paraesophageal hernias (B). Gastric volvulus can also occur in the absence of a diaphragmatic defect. In such situations, there is typically a congenital absence of intraperitoneal visceral attachments. It is seen in association with a wandering spleen, a condition in which the spleen also lacks peritoneal attachments and is prone to torsion. Gastric volvulus is a surgical emergency because there is a high risk of gastric necrosis if it is unrecognized (E). On occasion, there is a delay in diagnosis because the nasogastric tube is successfully passed and subsequent imaging is negative. Management consists of detorsion and repair of the diaphragmatic defect. If a paraesophageal hernia is present, consideration is given to performing a fundoplication. If the stomach is compromised, a gastric resection may be needed. If a volvulus is found without necrosis and without a diaphragmatic defect, then detorsion and gastropexy are performed.

A 40-year-old woman is referred to an endocrinologist for evaluation of persistent lethargy, diminished mentation, and poor tolerance of cold. She reports that the symptoms began after she was prescribed lithium for bipolar disorder. A physical examination reveals that her thyroid gland is enlarged and a diagnosis of hypothyroidism is made. A blood test substantiates that her T4 thyroid hormone levels are low, while levels of circulating thyroid stimulating hormone (TSH) are markedly increased. The most likely reason for the increase in TSH is A. the presence of a tumor that secretes TSH in an uncontrolled fashion. B. the presence of a tumor that secretes thyrotropin releasing hormone (TRH, the hormone that stimulates TSH release from the pituitary). C. ingestion of excessive amounts of iodide. D. Lack of the normal suppressive effect of thyroid hormone on TSH and TRH synthesis and secretion. E. a decrease in the levels of plasma proteins that bind thyroid hormone.

D. Lack of the normal suppressive effect of thyroid hormone on TSH and TRH synthesis and secretion. Thyroid hormone is secreted from the thyroid gland in response to TSH coming from the pituitary, which in turn is released in response to TRH from the hypothalamus. In this patient, symptoms of hypothyroidism are likely due to adverse effects of lithium on thyroid hormone synthesis. The lack of circulating thyroid hormone means that the normal negative feedback that suppresses TSH and TRH release as thyroid hormone levels rise is absent (option D). Tumors producing TSH or TRH are unlikely because thyroid hormone levels are low (thus eliminating options A and B). Ingestion of excessive iodide can inhibit organic iodide binding in the thyroid, but the effects are mild and transient and unlikely to be associated with persistent symptoms or a substantial rise in TSH (eliminating option C). Finally, if plasma binding proteins are decreased, levels of free thyroid hormone would be expected to increase, in turn suppressing TSH. Thus, option E is not consistent with the clinical picture or the blood test findings.

A 55-year-old man is evaluated in follow-up for severe COPD, which was diagnosed 2 years ago. He has had two exacerbations in the past year requiring hospitalization, and his baseline exercise tolerance is low. He completed pulmonary rehabilitation 3 months ago without much improvement in exercise capacity. He quit smoking 1 year ago. His medications are tiotropium, fluticasone/salmeterol, daily roflumilast, and albuterol as needed. On physical examination, vital signs are normal; BMI is 22. Oxygen saturation is 92% breathing ambient air. Scattered wheezing is noted bilaterally. Chest radiograph and CT scan both show emphysematous changes in the upper lobes. Spirometry shows an FEV1 of 40% of predicted and a DLCO of 25% of predicted. His 6-minute walking distance is 240 meters (787 feet), consistent with decreased exercise tolerance. Which of the following is most likely to benefit this patient? A. Change fluticasone/salmeterol to fluticasone/vilanterol B. Daily prednisone C. Lung transplantation D. Lung volume reduction surgery

D. Lung volume reduction surgery Educational Objective: Treat severe COPD with lung volume reduction surgery. Key Point: In patients with severe COPD and predominantly upper-lobe emphysema and low post-rehabilitation exercise capacity, lung volume reduction surgery results in improved survival compared with medical treatment. The most appropriate next step in management is to evaluate for lung volume reduction surgery (LVRS). To be eligible for LVRS, patients must meet the following criteria: (1) severe COPD; (2) symptomatic despite maximal pharmacologic therapy; (3) completed pulmonary rehabilitation; (4) evidence of bilateral predominant upper-lobe emphysema on CT; (5) postbronchodilator total lung capacity greater than 100% and residual lung volume greater than 150% of predicted; (6) maximum FEV1 greater than 20% and less than or equal to 45% of predicted and DLCO greater than or equal to 20% of predicted; and (7) ambient air arterial PCO2 less than or equal to 60 mm Hg (8.0 kPa) and arterial PO2 greater than or equal to 45 mm Hg (6.0 kPa). In patients with severe COPD and predominantly upper-lobe emphysema and low post-rehabilitation exercise capacity, LVRS results in improved survival compared with medical treatment. However, higher mortality is seen in patients with severe emphysema with an FEV1 of less than or equal to 20% of predicted and a DLCO less than or equal to 20% of predicted or homogeneous emphysema on high-resolution CT scan. Because of this patient's severe disability associated with his COPD and available parameters suggesting he may be a candidate for LVRS, further evaluation of this option would be appropriate. This patient is on appropriate medical treatment, and switching from one combination medication to another medication in the same class is not likely to be helpful. Vilanterol is a newer ultra-long-acting 02-agonist that may be used once daily. Although a once-daily inhaler regimen may be more convenient for some patients than a twice-daily regimen, these formulations tend to be of significantly higher cost, and there are no evidence-based clinical benefits for these newer ultra-long-acting agents. Therefore, making this change in medication would not be expected to have a significant effect in this patient. Systemic glucocorticoids are recommended for the short-term treatment of acute exacerbations of COPD. However, continuous therapy with systemic glucocorticoids is associated with significant side effects and is not recommended for the chronic management of COPD. Criteria for referral for lung transplant evaluation include one of the following: pulmonary hypertension, cor pulmonale, or both despite oxygen therapy; history of exacerbation associated with acute hypercapnia; and FEV1 less than 20% of predicted with DLCO less than 20% of predicted or homogeneous distribution of emphysema. This patient has an FEV1 greater than 20% of predicted, a DLCO greater than 20% of predicted, and emphysema in the upper lobes; therefore, he is not a candidate for lung transplantation.

All of the following are improved after gastric bypass except: A. Diabetes B. Hypertension C. GERD D. Marginal ulcers ​

D. Marginal ulcers Marginal ulcers are a complication of gastric bypass. They occur after any gastro-jejunostomy as the jejunum lacks Brunner's glands (alkaline mucus) which usually protect the duodenum from acidic gastric secretions.

A 62-year-old woman is evaluated in the hospital after being admitted 4 days ago for an acute exacerbation of COPD. She has responded well to treatment and is ready to be discharged today. Her medical history is notable for moderate-severity COPD, heart failure, depression, osteoporosis, hypertension, and hyperlipidemia. Her discharge medications are lisinopril, carvedilol, simvastatin, sertraline, alendronate, vitamin D, calcium, tiotropium, levofloxacin, prednisone, and albuterol. On physical examination, temperature is normal, blood pressure is 120/84 mm Hg, pulse rate is normal, and respiration rate is 18/min. Oxygen saturation is 93% breathing ambient air. No jugular venous distention is noted. Pulmonary examination reveals a few scattered wheezes and a few basal crackles. She is not using accessory muscles of breathing. Trace pedal edema is noted. She ambulates well without oxygen. Which of the following factors increases this patient's risk for early hospital readmission for COPD? A. Adequacy of discharge medications for COPD B. Female gender C. Length of hospitalization D. Multiple comorbid conditions

D. Multiple comorbid conditions Educational Objective: Identify risk factors for early readmission in a patient hospitalized for COPD exacerbation. Key Point: The rates of early hospital readmission for COPD exacerbation increase as the number of comorbidities increases. Comorbidities such as heart failure, lung cancer, anxiety, depression, and osteoporosis are important causes of increased early hospital readmission in patients with COPD exacerbation. The rates of early readmission for COPD exacerbation increase as the number of comorbidities increases. Higher rates of readmissions were noted in patients with COPD and combinations of comorbidities like heart failure and osteoporosis (21%), heart failure and anxiety (18.2%), heart failure and depression (15.9%), and heart failure and alcohol abuse (14.4%). Inadequate discharge medications for COPD exacerbation were also associated with early readmission rates, specifically the failure to prescribe a short-acting bronchodilator, oral glucocorticoid, and an antibiotic. This patient was discharged on all of these medications; therefore, inadequate discharge medications should not be a cause of early readmission. Another patient factor associated with early readmission is male gender. Female gender does not increase the risk for early readmission. Patients who were discharged less than 2 days or more than 5 days after admission are at risk for early readmission. This patient's hospital stay was 4 days, so this is unlikely to contribute to her risk of early readmission.

A 38-year-old woman comes to the office with progressive muscle weakness over the last 6 months. She finds It difficult to climb stairs and comb her hair, and she is unable to walk even short distances without rest. There is no associated muscle pain. The patient was diagnosed with hypothyroidism a year ago when she started gaining weight and has been taking levothyroxine since then. Four months ago, she developed acute back pain. X-ray at that time revealed demineralization of the vertebral bones. Temperature is 37 C (98.6 F), blood pressure Is 155/100 mm Hg, pulse is 76/min; and respirations are 15/min. BMI is 32 kg/m2. Physical examination shows facial hirsutism and mild proximal muscle weakness in the extremities. The patient's muscles are not tender on palpation, and no fasciculations are noted. Deep tendon reflexes and sensory examination are normal. TSH is normal. Which of the following is most likely responsible for this patient's muscle weakness? A. Electrolyte abnormality B. Mitochondrial Dysfunction C. Motor neuron disease D. Muscle aatrophy E. Muscle inflammation F. Peripheral nerve demyelination

D. Muscle aatrophy This patient has painless muscle weakness associated with weight gain; bone loss; hypertension, and hirsutism, which Is consistent with hypercortisolism (Cushing syndrome}. Myopathy in Cushing syndrome is characterized by weakness predominantly involving the proximal muscles and may be severe enough to interfere with daily activities. It is due to the direct catabolic effects of cortisol on skeletal muscle, which leads to muscle atrophy. Educational objective: Myopathy in Cushing syndrome is characterized by progressive painless muscle weakness predominantly involving the proximal muscles. It is due to the direct catabolic effects of cortisol on skeletal muscle, leading to muscle atrophy.

A 28-year-old man is evaluated for shortness of breath, fatigue, and nighttime cough. He has been employed in an auto body shop for the past 4 years and notes that his symptoms began about 6 months ago. He believes his symptoms may be associated with his work. He is being evaluated after several days off from work and currently has no symptoms or medical concerns. Medical history is unremarkable and he takes no medications. He is a never-smoker. On physical examination, vital signs are normal. BMI is 25; his weight has remained the same for 2 years. Oxygen saturation breathing ambient air is 99%. The sinus examination is normal and the lungs are clear, with no wheezing or crackles noted. The remainder of the examination is unremarkable. A chest radiograph is normal, and spirometry results are normal. Which of the following is the most appropriate next step in management? A. Advise him to switch employment B. High-resolution chest CT C. Inhaled glucocorticoid daily D. Repeat spirometry after workplace exposure

D. Repeat spirometry after workplace exposure Educational Objective: Diagnose suspected occupational asthma. Key Point: Spirometry before and after rechallenge with workplace exposures is helpful to confirm the diagnosis of occupational asthma. The most appropriate next step in management is to repeat spirometry after workplace exposure. Occupational asthma, related to workplace exposures to agents associated with airway hyperreactivity, should be suspected in all adults with a diagnosis of asthma because it may be a primary target for treatment. It should also be suspected in patients with asthma-like symptoms that vary with exposure to the workplace, as in this patient. This patient has likely been exposed to diisocyanates from spray painting in the auto body shop where he works, which are associated with triggering bronchospasm. Early recognition of an association of asthma symptoms with potential workplace exposure, and testing if indicated, is important for diagnosis and to guide therapy. Serial monitoring of peak flows throughout the workday, with comparison to a baseline time period away from exposures, can be helpful to support the diagnosis. Similarly, spirometry before and after rechallenge with workplace exposures is helpful to confirm the diagnosis. Because of the possibility that this patient's occupational exposure may be responsible for his asthma symptoms, it is appropriate to perform spirometry before and after exposure to the potential causative agent. If documented, treatment of occupational asthma should follow guidelines for typical asthma, and allergen exposure should be controlled or eliminated if possible. If significant changes in spirometry are not seen post workplace exposure and a suspicion for any underlying asthma remains, a methacholine challenge test may be appropriate. Advising a change in workplace venue to avoid allergen exposure for occupational asthma would be premature because the diagnosis has not been confirmed. High-resolution chest CT is indicated for evaluation of suspected parenchymal lung disease. However, this patient's symptoms are most consistent with airways disease, and he has no clinical evidence of a parenchymal process. Therefore, this testing would not be expected to be of benefit. Empiric therapy for asthma without documentation of the diagnosis or an association of his asthma symptoms with workplace exposure would not be appropriate.

A 56-year-old man is evaluated in the emergency department because of a 3-day history of increasing dyspnea, fever, and cough with purulent sputum. He has severe chronic obstructive pulmonary disease with a history of exacerbations requiring hospitalization. Medications are ipratropium, salmeterol-fluticasone, and albuterol. On physical examination, temperature is 38.0°C (100.4°F), blood pressure is 134/84 mm Hg, pulse rate is 88/min, and respiration rate is 30/min. He is awake and alert but is dyspneic and uses the accessory muscles to breathe. Pulmonary examination shows bilateral expiratory wheezes but no crackles. An arterial blood gas study performed while breathing 2L of oxygen via nasal cannula shows pH of 7.31, PCO2 of 53 mm Hg (7.0 kPa), and PO2 of 55 mm Hg (7.3 kPa). Oxygen saturation is 89%. Chest radiograph shows hyperinflation but no infiltrates. In addition to antibiotics, glucocorticoids, and bronchodilators, which of the following is the most appropriate management? A. Continuous positive airway pressure B. Increase in nasal oxygen C. Intubation and mechanical ventilation D. Noninvasive positive pressure ventilation

D. Noninvasive positive pressure ventilation Educational Objective: Manage an exacerbation of chronic obstructive pulmonary disease (COPD) with non-invasive positive pressure ventilation. The most appropriate additional management is noninvasive positive pressure ventilation (NPPV). This patient's acute exacerbation of COPD warrants a trial of NPPV. This treatment reduces the mortality rate, the need for intubation, and the length of hospital stay, and also improves respiratory acidosis and decreases the respiration rate and the severity of breathlessness in patients who are candidates for therapy. Patients with moderate to severe dyspnea, moderate to severe acidosis (pH <7.35) or hypercapnia, and respiration rate greater than 25/min benefit from NPPV. Exclusion criteria for NPPV are respiratory arrest, cardiovascular instability (hypotension, arrhythmias, and myocardial infarction), change in mental status, uncooperativeness, high aspiration risk, viscous or copious secretions, recent facial or gastroesophageal surgery, craniofacial trauma, fixed nasopharyngeal abnormalities, burns, and extreme obesity. Severe acidosis (pH <7.25) and respiration rate greater than 35/min are indications for intubation rather than NPPV. Intubation is also indicated in patients who do not benefit from an initial trial of NPPV. Positive airway pressure is the treatment of choice for most patients with obstructive sleep apnea. Continuous positive airway pressure (CPAP), which provides a constant fixed-level pressure throughout the respiratory cycle, is used for most patients. However, CPAP does not provide ventilatory support and would not be helpful in treating hypoxic ventilatory failure, as in this patient. Increasing the administration of oxygen alone is not an appropriate treatment option because the patient's oxygen saturation is adequate. Oxygen therapy is indicated for patients who have hypoxemia, arterial PO2 of less than 55 mmHg (7.3 kPa), or oxygen saturation of 88% or less. KEY POINT: In patients with exacerbation of COPD characterized by acute hypercapnic respiratory failure, noninvasive positive pressure ventilation reduces the mortality rate, the need for intubation, and the length of hospital stay.

Which of the following is the most effective treatment for intractable dumping syndrome? A. Low-fat, lactose-free diet B. Serotonin antagonists C. Low-carbohydrate, high-fat diet D. Octreotide E. Creation of a reversed jejunal segment ​

D. Octreotide Dumping syndrome can occur after any type of gastric surgery and is divided into early and late forms. Early dumping (20-30 minutes after eating) is more common and has both GI (nausea, vomiting, crampy abdominal pain, and explosive diarrhea) and cardiovascular (diaphoresis, dizziness, flushing) symptoms. The symptoms are related to the release of various substances including serotonin, neurotensin, bradykinins, and enteroglucagon. Late dumping (2-3 hours after eating) is the result of a reaction to a large amount of carbohydrates suddenly reaching the small intestine. This leads to a sudden release of large amounts of insulin with subsequent hypoglycemia with resultant diaphoresis, confusion, and tachycardia. Some patients only have early dumping, some only have late dumping, and some have both. The initial therapy for dumping syndrome is quite effective and consists of dietary modification (A-C). This includes eating frequent small meals, avoiding large amounts of carbohydrates and instead eating food high in protein and fat, and avoiding large amounts of fluids with meals. Octreotide, a long-acting somatostatin analogue, is the most effective treatment. It is effective against both the GI and cardiovascular symptoms. Long-term use of octreotide is associated with steatorrhea and cholelithiasis. Serotonin antagonists (B) have been used for dumping syndrome but have not been routinely effective. Creation of a reversed jejunal segment (E) can be considered in the management of postvagotomy diarrhea.

The best test for localization of a gastrinoma is: A. MRI B. CT C. Abdominal ultrasound D. Octreotide scan E. Selective angiography ​

D. Octreotide scan More than 90% of gastrinomas have receptors for somatostatin. Octreotide scanning (somatostatin receptor scintigraphy) has been shown to be the most sensitive test for localization of gastrinomas. However, successful localization depends on size and location. Somatostatin receptor scintigraphy is poor for very small (<1.1 cm) tumors and for small primary duodenal tumors. Duodenal gastrinomas are best localized by endoscopic ultrasonography. Abdominal ultrasound is not helpful (C). Failure to detect the tumor preopergatively should not preempt surgical exploration because an additional 33% will be found at surgery. CT and angiography may also be useful adjuncts in detecting gastrinoma (B-E). Aside from MRI's utility in detecting liver metastasis, it is not often employed in the workup for a presumed gastrinoma (A).

A 62-year-old woman is evaluated in follow-up after her first hospitalization for an acute exacerbation of COPD. She is currently completing a 5-day course of oral glucocorticoids. She has not had fever, and her cough and sputum production have improved. Although she feels much better, she notes that her exercise capacity has not returned to its preadmission baseline. Medical history is otherwise unremarkable. She has a 30-pack-year history of smoking but stopped smoking 3 years ago. Her medications are tiotropium, mometasone/formoterol, and as-needed albuterol. On physical examination, vital signs are normal. Oxygen saturation breathing ambient air is 92%. Lung examination shows mildly diminished airflow but is otherwise clear. The remainder of the examination is unremarkable. Pulmonary function testing performed 5 months ago showed an FEV1 of 55% of predicted. Which of the following is the most appropriate treatment? A. Add roflumilast B. Extend glucocorticoid taper for an additional 5 days C. Start 2 L of oxygen by nasal cannula D. Start pulmonary rehabilitation

D. Start pulmonary rehabilitation Educational Objective: Treat a patient with a recent COPD exacerbation with pulmonary rehabilitation. Key Point: Pulmonary rehabilitation is recommended for all symptomatic patients with COPD who have an FEV1 less than 50% of predicted and specifically for those hospitalized with an acute COPD exacerbation; it may also be considered for symptomatic or exercise-limited patients with an FEV1 greater than or equal to 50% of predicted. The most appropriate treatment is to start pulmonary rehabilitation. Pulmonary rehabilitation is recommended for all symptomatic patients with COPD and an FEV1 less than 50% of predicted and specifically for those hospitalized with an acute exacerbation of COPD. Pulmonary rehabilitation may also be considered in symptomatic or exercise-limited patients with an FEV1 greater than or equal to 50% of predicted, such as this patient. This patient has had a recent acute exacerbation, has diminished exercise capacity, and is on maximal medical treatment; therefore, she would benefit from pulmonary rehabilitation. Roflumilast is indicated only in patients with severe or very severe COPD with recurrent exacerbations. This is this patient's first exacerbation, and this treatment is therefore not indicated. Long-term glucocorticoid therapy is not indicated in patients with COPD owing to serious side effects; however, short-term use is helpful during an acute exacerbation. A recent study showed that, in addition to optimizing COPD therapy, a 5-day course of an oral glucocorticoid is adequate for most patients with COPD exacerbation. Increasing the duration of therapy by an additional 5 days would not likely be of benefit because this patient is completing a 5-day course of glucocorticoids, her symptoms have already improved, and the anti-inflammatory effect of glucocorticoids extends beyond completion of the course of medication. Long-term oxygen therapy is indicated in patients with an arterial PO2 at or below 55 mm Hg (7.3 kPa) or oxygen saturation breathing ambient air at or below 88% (confirmed twice over a 3-week period), with or without hypercapnia. Other indications are evidence of pulmonary hypertension, peripheral edema suggesting right-sided heart failure, or polycythemia, in combination with an arterial PO2 less than 60 mm Hg (8.0 kPa), or oxygen saturation less than 88% breathing ambient air. This patient's oxygen saturation is 92% breathing ambient air; therefore, oxygen supplementation is not currently indicated.

A 44-year-old woman is evaluated in follow-up after multiple recent emergency department visits for worsening asthma. She has had asthma since childhood, but her asthma symptoms have progressively worsened recently. Over the past 2 years, she has had twice-yearly visits to the emergency department requiring treatment with prolonged glucocorticoid tapers. She has required hospitalization twice within the last 4 months. She has no symptoms of gastroesophageal reflux, sinus disease, or other symptoms, and she diligently avoids environmental exposures and likely triggers. Medical history is significant for multiple allergies; skin testing has been positive for allergy to dust mites, cats, and ragweed. She is a never-smoker. Medications are a high-dose inhaled glucocorticoid, a long-acting β2-agonist, a leukotriene antagonist, a long-acting anticholinergic agent, and an as-needed short-acting β2-agonist. On physical examination, vital signs are normal. BMI is 26. Slightly puffy facies are noted. The lungs reveal decreased air movement and mild, diffuse wheezes. Skin fragility is observed on the arms. The remainder of the examination is unremarkable. Laboratory studies show a serum IgE level of 362 U/mL (362 kU/L) and 6% eosinophils on peripheral blood smear. Which of the following is the most appropriate treatment? A. Allergen immunotherapy B. Daily prednisone C. Infliximab D. Omalizumab

D. Omalizumab Educational Objective: Treat severe asthma with omalizumab. Key Point: Omalizumab is indicated in patients with moderate to severe persistent asthma with the following characteristics: (1) symptoms inadequately controlled with inhaled glucocorticoids, (2) evidence of allergies to perennial aeroallergens, and (3) serum IgE levels between 30 and 700 U/mL (30-700 kU/L) (normal range, 0-90 U/mL [0-90 kU/L]). The most appropriate treatment is omalizumab. Severe asthma occurs in 10% of patients with asthma and is associated with long-term treatment with high-dose glucocorticoids, frequent exacerbations, and high health care utilization, as in this patient. This patient's asthma has not been controlled despite aggressive nonpharmacologic interventions and treatment with multiple drugs. Omalizumab, a humanized monoclonal antibody directed at IgE, is the first biologic agent approved by the FDA for use in asthma. Administered subcutaneously every 2 to 4 weeks, omalizumab is indicated in patients with moderate to severe persistent asthma with the following characteristics: (1) symptoms inadequately controlled with inhaled glucocorticoids, (2) evidence of allergies to perennial aeroallergens, and (3) serum IgE levels between 30 and 700 U/mL (30-700 kU/L) (normal range, 0-90 U/mL [0-90 kU/L]). Although it is very expensive, omalizumab has been shown to reduce emergency department visits and appears to be cost effective in appropriately selected patients; it is not indicated for use in patients other than those meeting these treatment parameters. Allergen immunotherapy (allergy shots) can be useful when a single trigger is identified or when a trigger is consistently the source of asthma instability. It is a reasonable longer-term plan in this patient; however, immunotherapy may take 3 to 5 years to achieve a noticeable therapeutic effect. Omalizumab would be expected to provide more immediate control of her asthma than is possible with allergen immunotherapy. This patient has required frequent and high doses of systemic glucocorticoids and is exhibiting side effects (skin thinning and cushingoid facies). Because of the long-term detrimental effects of prolonged systemic glucocorticoid therapy, particularly in this younger patient, daily glucocorticoid therapy would not be appropriate. Other forms of immune modulation, such as with tumor necrosis factor-α inhibitors (such as infliximab) or other immunosuppressive agents (such as cyclophosphamide and azathioprine), have not been shown to be effective in controlling severe asthma and are associated with a high level of adverse effects in these patients.

A 43-year-oid woman comes to the physician complaining of bilateral joint pain and swelling in her hands for the past few months. She reports easy fatigability and loss of energy that has worsened insidiously. It is especially difficult for her to perform daily activities in the morning due to prolonged stiffness. The patient also describes frequent knee pain accompanied by a low-grade fever. She takes over-the-counter Ibuprofen to relieve her symptoms. Her hematocrit is 33% and creatinine is normal. This patient is at greatest risk for which of the following? A. Avascular bone necrosis B. Osteitis deformans C. Osteitis fibrosa cystica D. Osteoporosis E. Osteosarcoma ​

D. Osteoporosis This patient has a characteristic presentation of rheumatoid arthritis (RA). RA affects women more often than men and has a typical onset at age 30-50. Morning stiffness for >1 hour is typical of RA and would explain this patient's difficulty with morning activities. Symmetric joint swelling is characteristic. The most commonly affected joints include the metacarpal-phalangeal joints, proximal interphalangeal joints, wrists, and knees. Due to the autoimmune nature of the disease, systemic symptoms such as fatigue, fever, and anemia often occur. Patients are started on treatment (including disease-modifying agents) based on the severity of their symptoms. Increased levels of proinflammatory cytokines, corticosteroid therapy, and lack of physical activity may contribute to local (around inflamed joints) or generalized loss of bone mass in RA patients. There is an increased risk of osteopenia, osteoporosis, and bone fractures, especially if other risk factors (eg, low body weight, female sex, family history of osteoporosis, cigarette smoking, postmenopausal state, excessive alcohol use, other comorbidities) are present. The degree of bone loss generally correlates with disease activity. Strategies to prevent bone loss (adequate physical activity, optimization of vitamin D and calcium intake, minimization of glucocorticoid dose) should be implemented. Most experts suggest a low threshold for starting bisphosphonate therapy in RA patients. Although there may be associations between RA and some of the other answer options, the risk of osteoporosis is the greatest (up to a 2-fold increase, with one study showing a prevalence of 22%). Educational objective: Patients with rheumatoid arthritis are at increased risk of developing osteopenia, osteoporosis, and bone fractures, especially if additional risk factors for osteoporosis are present. Management includes adequate physical activity, optimization of calcium and vitamin D intake, minimization of corticosteroid therapy, and consideration for bisphosphonate treatment.

A 35-year-old man comes to the office due to worsening shortness of breath. The patient is an avid hiker; he began to have shortness of breath 18 months ago on challenging hikes, but it now occurs even while he is walking. He has also had mild wheezing and sputum production recently. The patient takes an antihistamine as needed without much improvement. He has no other medical problems. The patient has smoked half a pack of cigarettes daily for the past 5 years. His father died from lung and liver disease at a young age. Spirometry shows decreased forced expiratory volume in 1 second (FEV1), decreased forced vital capacity (FVC), and a decreased FEV1/FVC ratio. This patient's lower lung lobes are most likely to demonstrate which of the following findings? A. Centriacinar emphysema B. Colonization with pathogenic bacteria C. Compensatory hyperinflation D. Panacinar emphysema E. Subpleural blebs ​

D. Panacinar emphysema This patient with chronic progressive dyspnea has spirometry findings (reduced forced expiratory volume in 1 second [FEV1]/forced vital capacity [FVC] ratio) that are consistent with obstructive lung disease. His young age and family history of lung and liver disease suggest emphysema due to alpha-1 antitrypsin deficiency. The diagnosis is further suggested by his minimal smoking history; chronic obstructive pulmonary disease due to tobacco exposure alone is most commonly seen in those with a >30-pack-year smoking history. Alpha-1 antitrypsin is the major inhibitor of neutrophil elastase and deficiency leads to unchecked elastase-mediated tissue damage in the lungs. Because alpha-1 antitrypsin is deficient throughout the acinus, the entirety of the acinus is affected, resulting in panacinar emphysema. The lower lung lobes are predominantly affected, likely due to relatively greater perfusion compared to the upper lung lobes, allowing for an increased rate of neutrophil infiltration. Educational objective: Alpha-1 antitrypsin is the major serum inhibitor of neutrophil elastase. Alpha-1 antitrypsin deficiency typically causes early-onset panacinar emphysema, predominantly affecting the lower lung lobes.

A 45-year-old-man is evaluated in the emergency department for 1¬day history of coffee-ground emesis and a 2-day history of melena. He last vomited 18 hours ago. The patient has been having epigastric pain for several months. He drinks 3 to 4 cans of beer weekly and takes ibuprofen daily for chronic low back pain. Medical history is otherwise unremarkable. On physical examination, temperature is normal, blood pressure is 118/75 mm Hg, pulse rate is 100/min, and respiration rate is 18/min. There are no stigmata of chronic liver disease. The epigastrium is tender to palpation without rebound or guarding. A stool sample is positive for occult blood. The remainder of the examination is normal. Intravenous fluids are initiated and upper endoscopy is scheduled. Which of the following is the most appropriate treatment at this time? A. Erythromycin B. Nasogastric tube placement C. Octreotide D. Pantoprazole E. Ranitidine ​

D. Pantoprazole Educational Objective: Treat acute upper gastrointestinal bleeding. This patient with acute upper gastrointestinal bleeding would benefit most from a proton pump inhibitor (PPI), such as pantoprazole. Helicobacter pylori infection and nonsteroidal anti-inflammatory drags (NSAIDs) are the two most common causes of upper gastrointestinal bleeding. Ibuprofen and other NSA!Ds block the enzyme cyclooxygenase and interfere with production of prostaglandins that play a central role in the defense and repair of gastric epithelium. In patients with peptic ulcer bleeding, beginning a PP! before undergoing upper endoscopy has been shown to decrease the likelihood of high-risk stigmata on subsequent upper endoscopy and to reduce the likelihood of requiring an intervention during the procedure. In patients with peptic ulcer disease, the use of intravenous PPI therapy has also been shown to reduce the risk of recurrent hemorrhage following endoscopic hemostasis. KEY POINT: In patients with upper gastrointestinal bleeding due to peptic ulcer disease, proton pump inhibitors can decrease the potential need for intervention during upper endoscopy and can reduce the risk of recurrent hemorrhage.

A 75-year old man presents with pain of moderate severity affecting the base of both thumbs. Hand X-rays shows evidence of osteoarthritis at both first carpometacarpal (CMC) joints. Local application of gel containing diclofenac has not helped his symptoms. What would be the next most appropriate drug treatment for his pain? A. Co-codamol B. Diclofenac C. Gabapentin D. Paracetamol E. Tramadol ​

D. Paracetamol Paracetamol is the first-line systemic analgesic for mild to moderate pain. Non-steroidal anti-inflammatory drugs (NSAIDs) such as diclofenac should be used with great caution in the elderly and there would be a higher risk of adverse effects with co-codamol, gabapentin and tramadol. ​

A 32-year-old woman presents with a 3-month history of tiredness, shortness of breath and rash. She admits to passing dark urine but denies any other urinary symptoms. There is no previous medical history of note other than a 6-month course of oral anticoagulants for a spontaneous deep vein thrombosis (DVT) 2 years previously. On examination she has a petechial rash around her ankles and some bruises on her forearms. The full blood count demonstrates haemoglobin 61 g/1, white blood cell count 1.2 x 109/1, platelets 10 x 109/1, mean cell volume 105 fl and reticulocytes 4%. Dipstick analysis of the urine was positive for 'blood', but the microscopy showed no red cells. Which one of the following is the most likely diagnosis? A. Acute myeloid leukaemia B. Aplastic anaemia C. Megaloblastic anaemia D. Paroxysmal nocturnal haemoglobinuria E. Systemic lupus erythematosus

D. Paroxysmal nocturnal haemoglobinuria (PNH) The combination of thromboembolic events with a pancytopenia blood picture does suggest this rare diagnosis. In particular, dipstick analysis of the urine was positive for 'blood', but the microscopy showed no red blood cells. This final statement is because there is intravascular haemolysis, with intravascular release of haemoglobin. This then passes through the renal tubules, ending up in the urine, and turning the dipstick analysis positive. However, because there are no actual red blood cells in the urine, the microscopy is negative.

A 32-year-old man with a history of Wolff-Parkinson-White syndrome presents to his primary care physician because of a rash on his face and joint aches. These complaints started about 1 month ago but got much worse after spending a long day at the beach a few days ago. He also has had pain, warmth, and swelling of his knees for the past few weeks, as well as daily low-grade fevers to 37.9°-38.3°C (100.2°-100.9°F) for the past 2 weeks. He is taking a medication to prevent atrial fibrillation. Family history is negative for any rheumatologic disorders or malignancy. At present, he has a temperature of 38.3°C (100.9°F), heart rate of 84/min, and blood pressure of 118/76 mm Hg. On physical examination, he has a rash on his cheeks bilaterally that extends across his nasal bridge. His knees are edematous, red, and warm bilaterally with decreased flexion. Neurologic examination is non-focal. Laboratory evaluation reveals a positive antinuclear antibody titer. Which of the following medications is the most likely cause of this patient's symptoms? A. Amiodarone B. Hydralazine C. Isoniazid D. Procainamide E. Sotalol

D. Procainamide This patient is presenting with a lupus-like syndrome. He has malar rash, fevers, and arthritis, ail signs that are seen in patients with lupus. Patients, with drug-induced lupus usually have positive antinuclear antibodies, but tend to have a less severe presentation compared to patients with systemic lupus erythematosus. The syndrome usually resolves several weeks after discontinuing the medication. Procainamide is a class IA antiarrhythmic drug that is used in some patients with Wolff- Parkinson-White (WPW) syndrome to prevent atrial fibrillation and can cause a drug-induced lupus-like syndrome. Other drugs that cause it include hydralazine, isoniazid, minocycline, propylthiouracil, lithium, carbamazepine, and phenytoin. Bottom Line: Drug-induced lupus can be induced by procainamide, hydralazine, isoniazid, minocycline, propylthiouracil, lithium, carbamazepine, and phenytoin. ​

A 62-year-old man is evaluated for declining exercise capacity over the past year. He was diagnosed with moderate COPD 3 years ago. His symptoms had previously been well controlled with tiotropium and as- needed albuterol. He has not had any hospitalizations. He is adherent to his medication regimen, and his inhaler technique is good. He quit smoking 2 years ago. All immunizations are up to date, including influenza and pneumococcal vaccination. A chest radiograph performed 3 months ago for increased cough and sputum production was normal. Pulmonary function testing performed 3 years ago showed an FEV1 of 58% of predicted, an FEV1/FVC ratio of 60%, and a DLCO of 85% of predicted. On physical examination, vital signs are normal. Oxygen saturation is 93% breathing ambient air. No jugular venous distention is noted. The lungs are clear. Cardiac examination reveals normal heart sounds. There are no murmurs. No edema is noted. Which of the following is the most appropriate management? A. Add roflumilast B. Obtain complete pulmonary function tests C. Perform chest CT D. Repeat spirometry E. Start oxygen therapy

D. Repeat spirometry Educational Objective: Evaluate worsening COPD symptoms with repeat spirometry. Key Point: Spirometry is indicated when patients with COPD experience a change in symptoms; annual spirometry can help determine which patients have rapid decline in lung function. The most appropriate management is to repeat spirometry. Lung function can worsen over time in patients with COPD. During each visit, new or worsening symptoms (cough, sputum, dyspnea, fatigue), smoking status, adherence to and effectiveness of the medication regimen, adverse effects of treatment, and inhaler technique should be assessed. The frequency, severity, and causes of exacerbations should be evaluated. Comorbidities should be identified and managed. Spirometry is indicated when patients with COPD experience a change in symptoms. Annual spirometry can help determine which patients have rapid decline in lung function. Because this patient had pulmonary function testing done 3 years ago and has since had a decrease in his exercise capacity, spirometry is indicated. If spirometry shows worsening airflow obstruction in this patient, addition of a long- acting p2-agonist and an inhaled glucocorticoid may help with symptom control. If spirometry does not show significant worsening of airflow, other comorbidities such as cardiovascular disease should be considered. Roflumilast is a phosphodiesterase-4 inhibitor that is indicated in patients with severe and very severe COPD with recurrent exacerbations. This patient does not meet these criteria for use of this medication. Monitoring patients with COPD using complete pulmonary function testing (with lung volumes and diffusing capacity) rather than spirometry is not cost effective and does not change management. Complete pulmonary function testing is not required unless lung volume reduction surgery (LVRS) or lung transplantation is being considered. CT of the chest is not routinely recommended in the monitoring of COPD. This patient had a recent chest radiograph, which was normal, and there are no symptoms or signs to suggest a tumor that would warrant a CT scan at this time. Chest CT would be useful if this patient was being evaluated for LVRS or lung transplantation. Oxygen therapy is not indicated because an oxygen saturation greater than 88% is adequate. If oxygen saturation is less than 92%, arterial blood gas studies should be performed.

A 52-year-old man is evaluated for a 3-month history of fatigue and pain of the hands and knees. The pain has progressively worsened and is accompanied by 1 hour of morning stiffness. He takes ibuprofen as needed, which provides minimal pain relief. On physical examination, vital signs are normal. Synovitis of the proximal interphalangeal joints, elbows, left knee, and ankles is noted. Radiographs of the hands and knees are normal. Aspiration of the left knee reveals a synovial fluid leukocyte count of 12,000/µL (12 x 109/L) . Which of the following is the most likely diagnosis? A. Fibromyalgia B. Osteoarthritis C. Polymyalgia rheumatica D. Rheumatoid arthritis ​

D. Rheumatoid arthritis The presence of symmetric swelling of the proximal interphalangeal joints, elbows, and ankles in this patient is strongly suggestive of an inflammatory arthritis, most likely rheumatoid arthritis. Rheumatoid arthritis can affect most joints; however, the lumbar spine, thoracic spine. and distal interphalangeal joints are spared. Rheumatoid arthritis typically results in prolonged morning stiffness and inflammatory joint fluid, as noted in this patient. Fibromyalgia is characterized by chronic widespread musculoskeletal pain for at least 3 months and is more common in female patients. The physical examination in patients with fibromyalgia usually is normal except for widespread pain and tenderness. Active synovitis is not consistent with a diagnosis of fibromyalgia. Osteoarthritis can affect the hips, knees, lumbar and cervical spine, and proximal and distal interphalangeal joints; however, involvement typically is asymmetric, and the pain does not begin in multiple joints abruptly and simultaneously, as noted in this patient. Polymyalgia rheumatica is characterized by aching in the shoulders, neck, and hip girdle region, as well as fatigue and malaise that develop over weeks to months. This patient's symptoms and the objective findings of synovitis are not consistent with polymyalgia rheumatica. Educational Objective: Distinguish between inflammatory and noninflammatory arthritis. Morning stiffness lasting more than 1 hour and a synovial fluid leukocyte count greater than 5000/µL (5.0 x 109/L) are associated with inflammatory arthritis

An older man with a history of COPD complains of worsening dyspnea and exercise intolerance, even after smoking cessation and an evidence-based pharmacologic regimen. The physician discusses the possibility of adding theophylline to the patient's chronic regimen. What potential adverse reaction should the patient be counseled about before starting this medication? A. Bradycardia B. Pulmonary fibrosis C. Hypocalcemia D. Seizures ​

D. Seizures Theophylline is a methylxanthine drug that acts as a phosphodiesterase inhibitor and increases bronchodilation. It is not a first line treatment for COPD or asthma, but it may be an adjunctive treatment in select patients for chronic COPD or asthma. This medication has a narrow therapeutic index and may cause arrhythmias, seizures, and persistent vomiting. It should be used with caution in patients with cardiac disease, hyperthyroidism, peptic ulcer disease, and a seizure disorder, since it can exacerbate these conditions. (A, C) Theophylline may cause tachycardia and hypercalcemia. (B) It does not cause pulmonary fibrosis.

A 32-year-old woman is evaluated after missing several menstrual periods. She also reports vaginal dryness and several episodes of non-bloody bilateral breast discharge. The patient otherwise feels well. She has not had heat or cold intolerance or changes in appetite but does note decreased libido over the past several months. Medical history is unremarkable, and she takes no prescription or over-the counter medications. On physical examination, the patient appears healthy. Vital signs are normal. Body mass index is 25, which is a stable measurement. No skin rash or visual field defects are present. There is an expressible non-bloody discharge from both nipples. The remainder of the examination is normal. A urine pregnancy test is negative. Which of the following is the most appropriate next diagnostic study? A. Magnetic resonance imaging of the pituitary gland B. Serum estrogen and progesterone measurement C. Serum follicle-stimulating hormone-luteinizing hormone measurement D. Serum prolactin measurement

D. Serum prolactin measurement Educational Objective: Diagnose central (secondary) hypogonadism secondary to a prolactinoma. Measurement of the serum prolactin level is the most appropriate next diagnostic step. Galactorrhea without evidence of an elevated serum human chorionic gonadotropin level is extremely suggestive of a prolactinoma. Missed menstrual periods, vaginal dryness, and decreased libido also suggest hypogonadism, which is likely due to decreased gonadotropin hormone secretion from the anterior pituitary gland (a frequent occurrence with a prolactinoma) . Although prolactinomas may lead to classic bitemporal hemianopia, this finding only occurs if they enlarge adequately to compress the optic chiasm. An elevated serum prolactin level would confirm the clinical suspicion that a prolactinoma is inducing central secondary hypogonadism in this patient. Magnetic resonance imaging of the pituitary gland would be a confirmatory test if the serum prolactin level is elevated; however, performing imaging studies before a biochemical diagnosis is established is not indicated and would not be a high-value initial intervention in the evaluation of this patient. Serum estrogen and progesterone and follicle-stimulating hormone-luteinizing hormone measurements would likely be abnormal in this patient but would not explain the galactorrhea and would not identify the precise cause of her hypogonadism. K E Y P O I N T Because prolactinomas in female patients often present with galactorrhea and signs and symptoms of secondary amenorrhea, a serum prolactin measurement is an appropriate initial study.

A 35 year old woman with well-controlled RA states that she wishes to become pregnant. Her medication consists of methotrexate 20 mg weekly, folic acid 5 mg weekly and ibuprofen 400 mg 3 times daily. What advice would you give with regard to her plans to conceive and her medication? A. She can go ahead and try to conceive so long as she reduces the dose of methotrexate to 10 mg weekly and ibuprofen to 200 mg 3 times daily B. She should stop the ibuprofen for at least 3 months before trying to conceive but can continue the methotrexate C. She should stop the methotrexate and the ibuprofen and then go ahead and try to conceive D. She should stop the methotrexate for at least 12 months before trying to conceive but can continue the ibuprofen E. She should stop the methotrexate for at least 3 months before trying to conceive but can continue the ibuprofen ​

E. She should stop the methotrexate for at least 3 months before trying to conceive but can continue the ibuprofen Methotrexate is teratogenic and must be stopped completely at least 3 months before attempting to become pregnant. It is not necessary to stop ibuprofen before becoming pregnant but NSAIDs are contraindicated after 20 weeks. ​

A 78 year old woman with osteoarthritis and long-term indometacin therapy presents as an emergency with a short history of dizziness and passage of black stools over 2 days. Clinically, she has signs of anemia, tachycardia and hypotension. Her hemoglobin is 55 g/L and her urea is 21 mmol/L (126 mg/ dL) with a creatinine of 85 gmol/L (0.96 mg/ dL). What is the most likely diagnosis? A. She has acute on chronic renal failure due to long-term NSAID therapy B. She has bled from a gastric Dieulafoy lesion C. She has had an acute lower GI bleed from an NSAID-induced ulcer D. She has had an acute upper GI bleed from an NSAID-induced ulcer E. She has suffered a silent myocardial infarction, which is often asymptomatic in older patients ​

D. She has had an acute upper GI bleed from an NSAID-induced uulcer She has a high urea but her creatinine level is within the upper range of normal for women. NSAIDs cause renal damage but the most serious abnormality in this case is the suspected GI bleed. A silent myocardial infarction is a possible complication of an acute and significant GI bleed but it is not the most likely diagnosis here. NSAIDs cause ulceration throughout the GI tract but lesions that bleed are most likely in the upper part, including gastric and duodenal ulcers. Melaena (black tarry stool) is characteristic of an upper GI bleed. Lower GI bleeds present with fresh blood. Dieulafoy lesions are rare causes of upper GI bleeding and are caused by a single tortuous small artery in the submucosa that may erode through the mucosa and cause significant bleeding. These lesions are diagnosed endoscopically.

A 15-year-old girl from Nigeria who recently immigrated to England has been referred by her GP because she looks anaemic. On examination you notice frontal bossing of the skull and chronic leg ulcers. Her Hb is 70 g/l (120-160 g/l) and Howell-Jolly bodies are seen. Which one of the following is the most likely diagnosis? A. Acute lymphocytic leukaemia B. Aplastic anaemia C. Myeloma D. Sickle-eelI disease E. Thalassaemia ​

D. Sickle-cell disease We are told that the girl is anaemic with Howell-Jolly bodies seen on the peripheral blood film. Howell- Jolly bodies are a sign of hyposplenism, which is found in children and adolescents with sickle-cell anaemia. The sickle cells cause splenic infarction, which results in a hyposplenic state. Leg ulcers are also associated with sickle-cell disease. ​

A 72-year old white male presents to his physician's office with a 9-month history of wheezing, dyspnea on exertion, and daily sputum production. He is a 60-pack-year smoker. Examination shows markedly decreased breath sounds with mild wheezing at the end of expiration. Spirometry is consistent with a diagnosis of COPD. Which of the following interventions will be most effective for improving this patient's long-term survival? A. Inhaled ipratropium B. Long-term oral corticosteroids C. Inhaled corticosteroids D. Smoking cessation

D. Smoking cessation Given that cigarette smoking is a risk factor for disease development, it is logical to assume that smoking cessation will have a favorable impact on disease course. This hypothesis is supported by data from the Lung Health Study, which demonstrate that individuals with early airflow obstruction who are able to cease smoking experience an improvement in the rate of decline of lung function back to normal rates for age and reduced mortality in 15-year follow-up.

A 42-year-old woman comes to the office with a 4-month history of heartburn. She describes a periodic "sticking sensation" In her chest during meals. In addition, the patient has recently been unable to participate in her normal exercise routine due to dyspnea on exertion and joint pain in her hands and feet. She does not use tobacco, alcohol or illicit drugs. Lung examination reveals bilateral end-inspiratory crackles. Endoscopic evaluation shows mild hyperemia in the distal esophagus. Esophageal manometry shows lack of peristaltic waves in the lower two-thirds of the esophagus and a significant decrease in lower esophageal sphincter tone. Which of the following is the most likely mechanism responsible for this patient's manometric findings? A. Dysfunction of inhibitory neurons B. Eosinophilic infiltration of esophageal mucosa C. Loss of intramural neurons D. Smooth muscle atrophy and fibrosis E. Striated muscle inflammation

D. Smooth muscle atrophy and fibrosis This patient has a number of findings (eg. esophageal dysmotility, fibrotic lung disease, arthralgias) consistent with extradermal manifestations of systemic sclerosis (SSc). Classic early skin manifestations of SSc include thickening or hardening, edema, and pruritus. However, if skin symptoms are mild, patients may first seek attention due to gastrointestinal (Gl), joint, or respiratory disease. Gl complications are common in SSc and primarily affect the esophagus. SSc causes smooth muscle atrophy and fibrosis in the lower esophagus. Common symptoms include dysphagia, choking, heartburn, and hoarseness. Esophageal manometry in affected patients typically shows hypomotility and incompetence of the lower esophageal sphincter (LES). Educational objective: Systemic sclerosis can cause atrophy and fibrosis of the smooth muscle in the lower esophagus. This leads to decreased peristalsis and decreased tone in the lower esophageal sphincter. Typical symptoms include heartburn and dysphagia.

A 22-year-old woman without prior history of asthma complains of intermittent cough and chest tightness over the past 6 months. About 1 year ago she moved home to live with her parents, who own two cats. You suspect possible asthma. To evaluate her for asthma, you would order which of the following diagnostic tests: A. Measurement of serum IgE B. Allergy skin tests C. Blood tests for speciic IgE to common aeroallergens D. Spirometry prebronchodilator and postbronchodilator administration E. Chest radiograph

D. Spirometry prebronchodilator and postbronchodilator administration

A 65-year-old male returns to your office for follow-up regarding his COPD. His last visit to you was about 9 months ago. He was diagnosed with COPD about 6 years ago. He has a 50-pack-year history of smoking and stopped smoking about 2 years ago. PFTs reveal an FEV1 that is 40% predicted. Current pulmonary medications include tiotropium capsule via inhalation once daily along with fluticasone and salmeterol combination diskus twice daily. He uses 2 L oxygen by nasal canula 24 hours/day. He also is currently engaged in pulmonary rehabilitation. He has been compliant with your recommendations for vaccinations for influenza and pneumonia. As you review his chart, you note he has been admitted on 3 different occasions over the last 9 months for respiratory related issues. He has gained about 20 lbs over that time period related to the frequent burst of systemic steroids. He is frustrated by his poor quality of life and frequent hospitalizations. An ABG reveals pH: 7.37, pCO2 41, PO2 76 on 2 L NC. Which of the following treatments should be considered next? A. Increase his oxygen to 4 L. B. Start nocturnal non-invasive ventilation. C. Start daily prednisone at 40 mg indefinitely. D. Start azithromycin at 250 mg PO daily. E. Refer for lung transplantation.

D. Start azithromycin at 250 mg PO daily. This patient has high-risk COPD based on the severity of his FEV1 predicted and the number of exacerbations per year. He has already been treated as per guidelines with anti-inflammatories and long-acting bronchodilators. He has quit smoking, receives indicated vaccinations, has engaged in pulmonary rehabilitation, and has been assessed for oxygen treatment. Azithromycin has been evaluated in patients with severe COPD with frequent exacerbations. When given at a dose of 250 mg PO daily, it was associated with reductions in exacerbation and increased the time to first exacerbation. It would be a reasonable treatment option for this patient assuming relative contraindications do not exist. Caution must be used given the potential of adverse reactions related to hearing impairment and QT prolongation. Based on his testing, neither increasing his oxygen level nor initiating non-invasive ventilation in the absence of hypercarbia will prove beneficial. Chronic systemic steroids are associated with many adverse effects and have been associated with increased morbidity and mortality and thus are not recommended. Lung transplantation could be considered, although, in the setting of COPD, is a complex algorithm. There are many variables to consider, but these include an FEV1 < 25% predicted, evidence of hypercarbia, and resting hypoxemia. This patient does not meet those criteria. Board Testing Point: Know indications for azithromycin in a patient with severe COPD with frequent exacerbations.

The above patient eventually recovers. On POD #10, however, she develops sudden severe dyspnea. ABG values are: pO2 50 and pCO2 25. The most appropriate next step is: A. Re-exploration B. Chest CT C. Upper GI series D. Heparin E. WBC scan ​

D. Start heparin. This patient has likely suffered a PE (dyspnea, hypoxia, hypocarbia). Starting heparin before CT angio is indicated if the clinical suspicion is high. Massive PE is the MCC of death following gastric bypass.

A 55-year-old man is evaluated for abdominal fullness and nausea of 2 weeks' duration. He has no vomiting or fever. One month ago, he was diagnosed with type 2 diabetes mellitus. He reports an unintentional weight loss of 5 kg (11 lb) over the past month, generalized weakness, and poor appetite. Metformin is his only medication. On physical examination, blood pressure is 158/90 mm Hg and pulse rate is 90/min. BMI is 29. His face is round and red. A dorsocervical fat pad is present. His abdomen is distended, but nontender. Violaceous striae measuring 8 to 12 mm wide are noted on his upper arms and abdomen. There is 1+ bilateral lower extremity edema. Multiple ecchymoses and acanthosis nigricans are present. Laboratory studies: Adrenocorticotropic hormone <5 pg/mL (1.1 pmol/L) 24-Hour urine cortisol excretion Initial measurement 280 µg/24 h (771.6 nmol/24 h) Repeat measurement 300 µg/24 h (826.7 nmol/24 h) Cortisol, serum 46 µg/dL (1269.6 nmol/L) Urine Catecholamines 40 µg/m2/24 h (236.4 nmol/m2/24 h) Metanephrines 1000 µg/24 h (5070 nmol/24 h) CT scan of the abdomen with and without contrast reveals a 5.6-cm heterogeneous right adrenal mass with focal areas of calcifications and hemorrhage. The density of the mass is 50 Hounsfield units, and the contrast washout at 10 minutes is 20%. Which of the following is the most appropriate next step in the management of this patient's adrenal mass? A. Chemotherapy B. Fine-needle bbiopsy C. Radiation tttherapy D. Surgical excision

D. Surgical excision Educational Objective: Treat adrenocortical carcinoma. Key Point: Surgical removal after appropriate biochemical assessment is the most appropriate treatment for adrenocortical carcinoma, especially in patients with early disease. Adrenocortical carcinoma (ACC) is the most likely cause of this patient's Cushing syndrome, and surgical excision is the most appropriate management. The patient has classic clinical manifestations of Cushing syndrome (CS), and the 24-hour urine cortisol is markedly elevated on repeated measurements. Plasma adrenocorticotropic hormone (ACTH) is suppressed consistent with an ACTH-independent cause. The imaging characteristics of ACC include a large mass with irregular borders or shape, calcification, high attenuation (high Hounsfield units) on CT, and delay in contrast medium washout (less than 50% at 10 minutes), findings all present in this patient. The treatment of ACC depends on the extent of disease at presentation. Surgical removal after appropriate biochemical assessment remains the best option, especially in patients with early disease. Even after apparent complete resection, adjuvant therapy with mitotane, a known adrenal cytotoxic drug, may be beneficial. Treatment with mitotane is recommended for patients with persistent disease and others with known metastases and is associated with objective remissions in approximately 25% of patients. The main factors limiting the use of mitotane include nausea, vomiting, lethargy, and neurologic side effects. Experience with other cytotoxic chemotherapy is limited, but has usually been ineffective. A poorer prognosis is associated with advanced stages of the disease, the presence of metastasis at diagnosis, an older age, and cortisol hypersecretion by the tumor. In patients without clinically evident disease after initial surgery, the median survival rate is 60% at 5 years. Fine-needle biopsy cannot distinguish a benign adenoma from a carcinoma and is not used in the evaluation of ACC. It is sometimes used to distinguish ACC from metastatic disease. Its use in this patient is both unnecessary and inappropriate. Radiation therapy is not used as the principal initial treatment in ACC; however, it may be used as adjuvant therapy after surgery to prevent tumor recurrence. Radiation therapy can also be used to treat areas of metastasis, such as to the bones or brain.

A 37-year-old man presents to the emergency department complaining of dizziness and confusion. He works part-time as a mechanic to finance his nursing school education. He states that over the past month he has had episodes of dizziness associated with tremulousness that are relieved by eating and/or lying down, He is otherwise healthy. His temperature is 37.1°C (98.7°F), heart rate is 78/min, and blood pressure is 124/82 mm Hg. On physical examination the patient is diaphoretic and his hands are tremulous. He mentions that he did not eat breakfast this morning. Laboratory studies show: Na+: 144 mEq/L K+: 3.8 mEq/L Cl-: 110 mEq/L HCO3-: 22 mEq/L Glucose: 50 mg/dL C-peptide: 0.4 ng/mL (normal: 0.5-2.0 ng/mL) Which of the following is the most likely diagnosis? A. Insulinoma B. Pheochromocytoma C. Surreptitious glyburide use D. Surreptitious insulin use E. Surreptitious repaglinide use

D. Surreptitious insulin use This patient has factitious hypoglycemia. Factitious hypoglycemia is fasting hypoglycemia resulting from the administration of either insulin, sulfonylurea, or meglitinide by the patient or another individual. It commonly occurs in patients in the health care professions, individuals whose family members have diabetes, and those with a history of factitious disorders. The key to making the diagnosis in this case relies on C-peptide levels. C-peptide is a peptide chain on proinsulin that is cleaved during the conversion of proinsulin to insulin. C-peptide levels rise with increased production of endogenous insulin. The triad characteristic of exogenous insulin administration is fasting hypoglycemia, hyperinsulinism, and suppressed C-peptide levels. In the case of sulfonylurea or meglitinide ingestion, C-peptide level measurements are not helpful, as both classes of medication stimulate release of endogenous insulin and C-peptide. A psychiatry consult and behavioral therapy would be the most appropriate long-term management for the patient. Bottom Line: Patients with hypoglycemia and low C-peptide level must be evaluated for factitious hypoglycemia due to exogenous insulin administration. Long-term management involves behavioral therapy and follow up with a psychiatrist.

A 30-year-old man presents with worsening back pain. The history reveals that this has been present for the past 3 years and is increasing in severity. It is characterized by early-morning stiffness, nocturnal pain, and modest improvement with exercise. He initially noted an improvement with an NSAID, but subsequently that effect was lost, and he has tried two other NSAIDs without effect. Physical examination reveals limitation in forward flexion of the lumbar spine. An AP x-ray of the pelvis reveals bilateral erosive changes in the sacroiliac joints. A lateral x-ray of the lumbar spine reveals squaring of several vertebral bodies. The next step in pharmacologic treatment of this condition is which of the following? A. Methotrexate B. Leflunomide C. Sulfasalazine D. TNF inhibitor biologic agents E. Azathioprine ​

D. TNF inhibitor biologic agents The patient meets the diagnostic criteria for ankylosing spondylitis (AS). He has long-standing back pain that has features of inflammatory back pain and has limitation in spinal mobility. The presence of bilateral erosive sacroiliitis provides imaging support for the diagnosis of AS. He has had an adequate trial of NSAIDs, and his back pain has not responded adequately to these agents. The classical DMARDs used in rheumatoid arthritis—methotrexate, leflunomide, sulfasalazine, and azathioprine—have proved ineffective in controlling the spinal inflammation seen in AS. On the other hand, the TNF inhibitors (infliximab, etanercept, adalimumab, golimumab) have established efficacy in controlling the signs and symptoms of AS

In investigating the cause of an acute monoarthritis in a 50 year old man in a non-tuberculosis (TB) endemic region, synovial fluid from the swollen joint is sent for Gram stain, culture and polarised microscopy. The laboratory staff call with the results: they say 'Gram stain negative; culture results are not available for another 48 hours. There are many neutrophils and there are negatively birefringent crystals in the fluid.' What is the correct management? A. Antibiotics and steroids should be used together B. Intra-articular steroids can be given straight away because the diagnosis is reactive arthritis C. The patient can be treated for gout because infection has been excluded D. The joint should be drained and analgesia given but steroids should be withheld until the culture result is available E. The results favour a diagnosis of pseudogout ​

D. The joint should be drained and analgesia given but steroids should be withheld until the culture result is available The available results are consistent with acute gout (urate crystals are negatively birefringent) but do not exclude the possibility that infection may also be present. In sepsis, often bacterial identification is not possible until culture is available. A negative Gram stain suggests there might not be infection but does not rule it out. The cause of pseudogout is calcium-containing crystals (usually pyrophosphate) and these crystals are positively birefringent. Reactive arthritis is certainly possible but intervention should be delayed until it is clear it is not septic arthritis. ​

A 58 year old man with haemochromatosis is referred to the rheumatology clinic with a 3-year history of pain mainly affecting the small joints of the hands and wrists. Four weeks previously he had developed acute pain, swelling and redness of the right wrist, which had responded to treatment with naproxen 500 mg 3 times daily. He has a history of type 2 diabetes treated with diet and metformin and has been treated with regular venesection for the previous 3 years. Clinical examination of his hands is unremarkable with no evidence of synovitis. Laboratory investigations are as follows: haemoglobin 115 g/L, white cell count 8.2 x 109/L, platelets 345 x 109/L, ESR 20 mm/ hr, serum iron 100 gmol/L (558 gg/dL), AST 35 U/L, bilirubin 15 gmol/L (0.88 mg/dL) and ALP 150 U/L. Radiographs of the hands and wrists show joint space narrowing and subchondral cysts affecting the MCP and the radiocarpal joints with no osteophyte formation. Which of the following statements is true? A. The arthritis is an incidental finding unrelated to the diagnosis of haemochromatosis B. The clinical picture is consistent with rheumatoid arthritis C. The most likely cause for his joint symptoms is diabetic cheiroarthropathy D. The most likely explanation for the acute flare in his joint symptoms is calcium pyrophosphate deposition disease E. The risk of further flares in symptoms can be reduced by continued venesection and restoration of serum iron levels to normal ​

D. The most likely explanation for the acute flare in his joint symptoms is calcium pyrophosphate deposition disease Calcium pyrophosphate deposition disease (CPPD) is a common feature of the arthropathy associated with haemochromatosis and the presentation with acute pain and swelling that settles with NSAID treatment is consistent with CPPD-associated arthropathy (pseudogout). Diabetes can cause an arthropathy but has different features and RA is unlikely as symptoms have settled and there is no synovitis. Although venesection should be continued as treatment for the haemochromatosis, there is no evidence that this will improve the arthritis. Option A is incorrect since an arthropathy is associated with haemochromatosis in up to 50% of cases. ​

Which of the following is true regarding gastrointestinal stromal tumor (GIST)? A. They rarely present with GI bleeding. B. They arise from smooth muscle cells. C. Malignant potential is readily determined by histologic features. D. They can be managed by laparoscopic wedge resection. E. The extent of the tumor is best determined preoperatively by endoscopy. ​

D. They can be managed by laparoscopic wedge resection. GISTs were previously called leiomyomas or leiomyosarcomas because they were thought to arise from smooth muscle cells, but they in fact originate from mesenchymal components (from Cajal cells) (B). They stain positive for CD117 (c-kit). They are most commonly found in the stomach and, although rare, it is the most common mesenchymal tumor of the intestinal tract. Because they are not epithelial tumors and grow in the wall of the stomach, they tend to be large at the time of presentation. They cause mucosal ulceration and frequently present with GI bleeding (A). Large tumors may also produce symptoms of weight loss, abdominal pain, and fullness and early satiety. An abdominal mass may be palpable. An endoscopic biopsy specimen may be negative in as many as one- half of cases due to sampling error because most of the tumor is submucosal (E). A CT scan provides a better assessment of the extent of the tumor. Determining whether a GIST is malignant is not straightforward because there are no discriminating cellular features (C). The malignant potential is determined by mitotic activity (> 5 mitoses/50 high power field) with 1 cm. Lymph node dissection is not necessary because tumors spread hematogenously and lymph node metastasis is extremely rare. Wedge resection with 1 cm margins is adequate treatment in most cases. This can be performed laparoscopically. However, microscopically positive margins have not been demonstrated to affect survival.

A 50 year old man presents with epigastric pain unrelieved with PPIs. You perform EGD and a mass is found in the stomach mucosa only. You biopsy it and it comes back mucosa associated lymphoid tissue. The next appropriate Tx is: A. Total gastrectomy B. Partial gastrectomy C. Chemo-XRT D. Tx for H pylori ​

D. Tx for H pylori Mucosa associated lymphoproliferative tissue lymphoma (MALT lymphoma) is related to H pylori infection and is considered a low grade B cell NHL. Initial Tx is with 3 pronged Tx for H. pylori (amoxicillin, clarithromycin and PPI) which cures 90% of patients.

A 32-year-old man is evaluated because of a 5-week history of mid- epigastric abdominal pain. He also notes progressive nausea, vomiting, early satiety, and a documented weight loss of 2.7 kg (6 lb.) during this time. Over the past 10 days, he has been using an over-the-counter proton pump inhibitor (PP!) once daily with only minimal improvement in his pain. Medical history is unremarkable, and he takes no prescription medications. He does not use tobacco, alcohol, or recreational drugs. On physical examination, vital signs are normal. The abdomen is soft with mild tenderness in the epigastric area with deep palpation. The remainder of the examination is unremarkable. Which of the following is the most appropriate next step in management? A. A PPI twice daily for 4 weeks B. Omeprazole, clarithromycin, and amoxicillin C. Upper gastrointestinal barium study D. Upper endoscopy ​

D. Upper endoscopy Educational Objective: Manage peptic ulcer disease. This patient should undergo upper endoscopy. His constellation of symptoms raises concern for peptic ulcer disease, particularly in the distal stomach or duodenum. Upper endoscopy is the preferred diagnostic study, as it allows direct visualization of the upper gastrointestinal tract and provides the opportunity for Helicobacter pylori testing of biopsy specimens as needed. KEY POINT: Upper endoscopy is indicated for diagnosis in patients of any age with mid-epigastric abdominal pain and associated symptoms, including nausea, vomiting, early satiety, or weight loss. ​

A 54-year-old man complains of burning epigastric pain that usually improves after a meal, and is occasionally relieved with antacids. On examination, he appears well and besides some epigastric tenderness on palpation, the rest of the examination is normal. Upper endoscopy confirms a duodenal ulcer. Which of the following statements concerning PUD is most likely correct? A. duodenal ulcer is seen more often in older people than is gastric ulcer B. clinically, gastric ulcers are more common than duodenal ulcers C. duodenal ulcers can frequently be malignant D. infection can cause both types of peptic ulcer E. peptic gastric ulcers are usually quite proximal in the stomach ​

D. infection can cause both types of peptic ulcer Helicobacter pylori infection is the cause of most peptic ulcers, and the usual route of infection is via the water supply. Duodenal ulcer is clinically more common, although the prevalence on autopsy series is similar. Duodenal ulcer does not represent a malignant potential. Gastric ulcers are seen in an older population and can be malignant. Benign peptic gastric ulcers tend to be more distal.

If the turnover number of all enzymes involved in glycogen metabolic regulation and activity is 100 reactions per second, how many glucose molecules could be removed from glycogen in 1s upon activation of one molecule of protein kinase A (PKA)? A. 100 B. 1,000 C. 10,000 D. 100,000 E. 1,000,000

E. 1,000,000 One active PKA can activate in 1s 100 molecules of phosphorylase kinase. Each phosphorylase kinase can, in 1s activate 100 molecules of glycogen phosphorylase (so at this point we have 100 times 100 active molecules of phosphorylase, or 10,000 active phosphorylase molecules). Each active phosphorylase molecule can release 100 glucose residues per second from glycogen, and since there are 10,000 active phosphorylase molecules, 1,000,000 molecules of glucose are released per second once a single molecule of PKA has been activated. This is an example of cascade amplification, in which an increase in activity of just one molecule at the top of the cascade can result in a large response further down the cascade.

A 67-year-old woman is evaluated for muscle weakness that has developed over the past 6 months. The patient has gained significant weight and developed hypertension and type 2 diabetes mellitus 2 years ago. Her diabetes is not well controlled. Medications are metformin, hydrochlorothiazide, lisinopril, amlodipine, and metoprolol. On physical examination she appears chronically ill. Blood pressure is 154/92 mm Hg, and other vital signs are normal. Body mass index is 40. Skin examination is notable for facial hirsutism. Central obesity, mild proximal muscle weakness, and 2+ peripheral edema are noted. Results of recent laboratory studies show a serum creatinine level of 1.3 mg/dL (115 µmol/L) , a fasting plasma glucose level of l44 mg/dL (8.0 mmol/L) , and a serum potassium level of 2 . 9 mEq/L (2.9 mmol/L) . Which of the following tests should be performed to establish the cause of this patient's diabetes? A. Computed tomography of the adrenal glands B. Magnetic resonance imaging of the pancreas C. Serum C-peptide measurement D. Serum glutamic acid decarboxylase antibody titer E. 24-Hour urine free cortisol excretion

E. 24-Hour urine free cortisol excretion Educational Objective: Diagnose hypercortisolism (Cushing syndrome) as a secondary cause of diabetes mellitus. Measurement of the 24-hour excretion of urine free cortisol is the most appropriate next test in this patient to determine the cause of her diabetes mellitus. Various secondary causes of diabetes exist, most involving other endocrinopathies, effects of medications, pancreatic diseases, or genetic conditions. Hypercortisolism (Cushing syndrome) is one of these secondary causes. The most common cause of hypercortisolism is glucocorticoid therapy, followed by the secretion of adrenocorticotropic hormone (ACTH) by a pituitary adenoma (Cushing disease) and the hyperfunctioning of an adrenocortical adenoma. In this patient, the combination of diabetes, hypertension, central obesity, hypokalemia, proximal muscle weakness, and edema strongly suggests the presence of hypercortisolism. The diagnosis can be confirmed by several tests, including measurement of24-hour excretion of urine free cortisol, an overnight dexamethasone suppression test, or a midnight salivary cortisol measurement. Computed tomography of the adrenal glands is appropriate after hypercortisolism is diagnosed, especially when it is non-ACTH dependent, to identify the type of adrenal condition responsible. This test would be premature in this patient in whom the diagnosis has not been confirmed. Pancreatic imaging could be considered when signs and symptoms (such as abdominal or back pain, jaundice, or chronic diarrhea) suggest that an underlying pancreatic disorder is the cause of diabetes. This patient has none o f these signs or symptoms, and thus magnetic resonance imaging of the pancreas is unlikely to be revealing. Residual β-cell function can be assessed by measuring the serum C peptide level, which is often high-normal in patients with early type 2 diabetes because of insulin resistance. Similarly, measuring the serum glutamic acid decarboxylase antibody titer is useful to confirm the presence of autoimmune (type 1) diabetes when no other evidence exists. However, the serum C-peptide level will not indicate the cause of diabetes in this patient, and measuring the serum glutamic acid decarboxylase antibody titer also is unlikely to be helpful because she does not have type 1 diabetes. K E Y P O I N T: Hypercortisolism (Cushing syndrome) is a likely cause of secondary diabetes mellitus in a patient with hypertension, central obesity, and hypokalemia.

A 6-year-old boy is brought to the physician by his mother for a follow-up examination. He has persistent bedwetting. Over the past year, his parents have attempted various methods to prevent him from wetting his bed, including fluid restriction in the evenings, sticker rewards, and bedwetting alarms, with no improvement. The patient wets his bed 2-3 times a week. He does not have problems going to the bathroom during the day. The physician prescribes an oral medication that successfully controls his symptoms. The most likely effect of this drug on the principal cells of the kidney is increased activity of which of the following? A. Guanylate cyclase B. Tyrosine kinase C. Phospholipase C D. Steroid hormone response element E. Adenylate cyclase ​

E. Adenylate cyclase Desmopressin, a synthetic vasopressin (antidiuretic hormone) analog, is used to control nighttime enuresis if first-line treatments (e.g., fluid restriction, bedwetting alarm) are not effective.

A 25-year-old woman is seen for follow-up of persistent asthma symptoms despite treatment with inhaled fluticasone 88 µg twice daily for the past 3 months. According to the National Asthma Education and Prevention Program guidelines endorsed by the National Institutes of Health, which of the following changes in therapy can be considered? A. Addition of a leukotriene antagonist. B. Addition of a long-acting beta-agonist. C. Addition of low-dose theophylline. D. Increase the dosage of inhaled corticosteroid. E. Any of the above can be considered.

E. Any of the above can be considered. A step up in asthma therapy should be considered when a patient continues to have symptoms after 3 months on appropriate therapy. Symptoms to consider when determining whether asthma therapy should be increased include presence of daily symptoms, nocturnal awakening more than once weekly, and limitations in daily activity. Physicians should also review the use of rescue inhalers and lung function when making decisions to step up therapy. In addition, the use of standard asthma severity questionnaires such as the Asthma Control Score may be helpful. When stepping up therapy in mild persistent asthma, the preferred next step in therapy is increasing to medium-dose inhaled corticosteroids or adding a long-acting beta-agonist. However, an alternate therapy that can be considered is adding a leukotriene antagonist, low-dose theophylline, or the leukotriene synthesis inhibitor zileuton to low-dose inhaled corticosteroids.

A 56-year old man presents with acute pain, redness and swelling of the first metatarsophalangeal (MTP) joint of the right foot. Investigations show an elevated ESR (35 mm/hr), CRP of 56 mg/L, a mild neutrophilia (12.1 x 109/L), serum creatinine 75 gmol/L (0.85 mg/dL), estimated glomerular filtration rate (eGFR) > 60 mL/min/1.73 m2 and a serum uric acid level of 450 gmol/L (7.6 mg/ dL). Radiographs reveal evidence of erosions in the affected joint. What would be the most appropriate treatment? A. Allopurinol 100 mg daily initially gradually increasing in dose until uric acid falls below 360 gmol/L (6.1 mg/dL) B. Colchicine 500 mg 3 times daily until symptoms settle followed by colchicine 500 mg daily on a long-term basis C. Colchicine 500 mg 3 times daily until symptoms settle followed by long-term diclofenac 75 mg twice daily D. Diclofenac 75 mg twice daily followed by long-term low-dose aspirin 75 mg/day E. Etoricoxib 60 mg daily followed by allopurinol starting at 100 mg daily, gradually increasing in dose until uric acid falls below 360 gmol/L (6.1 mg/dL) ​

E. Etoricoxib 60 mg daily followed by allopurinol starting at 100 mg daily, gradually increasing in dose until uric acid falls below 360 gmol/L (6.1 mg/dL) Acute gout can be managed with either NSAID therapy or colchicine, but urate-lowering therapy with allopurinol is indicated to control hyperuricaemia in the long term, to reduce the risk of recurrence and prevent long-term joint damage. Allopurinol alone would not be appropriate since it may cause a further flare in acute gout due to the change in uric acid levels. ​

A 30-year-old woman comes to your office complaining of shortness of breath. She states that she first noticed the shortness of breath 6 months ago. She is fine at rest but cannot go jogging any longer due to shortness of breath, and she is awakened at night once or twice a week with a feeling that she can't catch her breath. She denies fever, cough, sputum production, orthopnea, or chest pain. On exam, she is a well-appearing female in no apparent distress. The cardiovascular exam is normal, and auscultation of the lungs is normal. There is no leg edema, clubbing, or cyanosis. You perform routine blood work that indicates a normal CBC and chemistries. A chest x-ray is normal, and spirometry done in the office is normal. An arterial blood gas on room air shows pH 7.41, PCO2 40 mmHg, and PO2 of 90 mmHg. Which of the following diagnostic tests would you perform next? A. High-resolution CT (HRCT) of the chest B. Echocardiography C. Full pulmonary function tests with diffusing capacity determination D. Flow-volume loop E. Exercise challenge test

E. Exercise challenge test With a normal cardiovascular and pulmonary exam, the most likely cause of the episodic dyspnea in this patient under 40 years of age is exercise-induced bronchospasm (EIB). Because this bronchospasm is episodic and a reversible airways obstruction, spirometry performed while the patient is asymptomatic may well be normal. An exercise challenge test is the most direct way to establish a diagnosis of EIB. This usually involves 6 to 8 minutes of ergometer or treadmill exercise, sufficient to raise the heart rate to 85 percent of the predicted maximum. A test is generally considered positive if the FEV1 falls by 10 percent or more, although a fall of 15 percent is more diagnostic. Alternatively, surrogate tests to assess bronchial hyperresponsiveness (e.g., cold air hyperventilation, methacholine or histamine inhalation challenge) may be performed in specialized laboratories, but do not always correlate with the presence of EIB. If the patient had rales on the exam and was older, the possibility of an interstitial lung disease would have to be entertained, and an HRCT of the chest would be indicated. Remember: 10% of patients with interstitial lung disease will have a normal chest x-ray at the time of presentation. If the patient gave a history of chest pain or syncope with exertion, and physical exam revealed a right parasternal heave and an accentuated P2, then the diagnosis of pulmonary hypertension by echocardiography would be warranted. If an upper airway obstruction was suggested by the history—or the patient was found to have stridor—a flow-volume loop would be a good screening test for upper airway obstruction. A normal ABG with a normal A-a gradient would make a diffusing capacity limitation unlikely. Board Testing Point: Know how to diagnose exercise-induced bronchospasm.

A 16-year-old boy presents with shortness of breath after his family adopts a new pet. His pulmonologist suspects an underlying allergy-induced asthma and orders a pulmonary function test. Which of following is most likely to have decreased in this boy? A. Tidal volume B. Expiratory reserve volume C. Forced vital capacity D. Inspiratory capacity E. FEV1 (forced expiratory volume in 1 second) ​

E. FEV1 (forced expiratory volume in 1 second) Allergy-induced asthma is associated with airway narrowing and obstruction, which impairs the volume of air than can be forcibly expired per unit time. In most cases, forced vital capacity would be unaffected because this parameter is not time dependent. Tidal volume is similarly unaffected. Static lung volumes and capacities (expiratory reserve volume and inspiratory capacity) do not change appreciably with airflow obstruction, although residual volume may be increased by obstructive physiology when air-trapping occurs.

Which volume remains in the lungs after a tidal volume (VT) is expired? A. Tidal volume (VT) B. Vital capacity (VC) C. Expiratory reserve volume (ERV) D. Residual volume (RV) E. Functional residual capacity (FRC) F. Inspiratory capacity G. Total lung capacity

E. Functional residual capacity (FRC) During normal breathing, the volume inspired and then expired is a tidal volume (VT). The volume remaining in the lungs after expiration of a VT is the functional residual capacity (FRC).

A 75 year old woman with established osteoporosis, with previous forearm fracture, known stable chronic kidney disease (CKD) stage 3 (eGFR approx. 48 mL/min/1.73 m2) and no history of TB (she was immunised against TB as a child and has had no recent TB contacts), presents with an acutely swollen very painful knee. She feels generally unwell, had dysuria a month previously treated with antibiotics but has had no rash or diarrhoea. Investigations show haemoglobin 122 g/L, white cell count/neutrophils slightly raised, platelet count 450 x 109/L and normal lymphocytes with ESR 45 mm/hr. Liver and thyroid function tests are normal. Which of the following statements is correct? A. Background renal impairment has no relevance to the current presentation B. Blood cultures are likely to give the diagnosis C. Septic arthritis is unlikely D. The dysuria has no relevance to the current presentation E. Joint aspiration should be done. ​

E. Joint aspiration should be done. The clinical features are fairly non-specific and gout, septic arthritis and pseudogout are possible diagnoses. Indeed, severe gout or pseudogout can cause systemic symptoms identical to those caused by infection. The diagnosis is made on knee fluid aspiration and then Gram stain and culture of the fluid, but also polarised light microscopy of joint fluid examining for crystals (urate-causing gout or calcium-containing causing pseudogout). CKD stage 3b-5 is associated with urate and calcium-containing crystal-induced musculoskeletal disease. Secondary joint infection following incompletely treated urine infection is possible but also previous infection can trigger subsequent bouts of crystal arthritis. ​

A 29-year-old woman presents to the clinic for assessment of on-going symptoms of shortness of breath and wheezing. She has a long history of mild asthma that is well controlled on inhaled corticosteroids. She recently experienced a flare that is characterized by recurrent episodes of bronchial obstruction, fever, malaise, and expectoration of brownish mucous plugs. On physical examination, the heart exam is normal, and there is bilateral wheezing on expiration. A CXR reveals upper lobe pulmonary infiltrates, and on the complete blood count the eosinophil count is 2000/mL. Special testing reveals that the serum precipitating antibodies to Aspergillus are positive. Which of the following is the most appropriate next step in management? A. antihelminthic therapy B. a short course of systemic glucocorticoid therapy C. desensitization treatment D. high-dose glucocorticoids by puffer E. Long-term systemic glucocorticoid therapy

E. Long-term systemic glucocorticoid therapy Allergic bronchopulmonary aspergillus usually requires long-term treatment with glucocorticoids. The major diagnostic criteria are bronchial asthma, pulmonary infiltrates, eosinophilia greater than 1000, immediate wheal and flare response to Aspergillus fumigans, serum precipitins to A fumigans, elevated serum IgE, and central bronchiectasis. Allergic bronchopulmonary aspergillosis (ABPA): This is an uncommon condition which may complicate asthma, in which a hypersensitivity reaction to the colonizing organism Aspergillus fumigatus results in cough, dyspnea and, frequently, worsening asthma control. There is predominantly central upper lobe bronchiectasis on high-resolution CT scan. Diagnostic criteria Asthma Pulmonary infiltrates Peripheral eosinophilia Immediate wheal-and-flare response to Aspergillus fumigatus Serum precipitins to A. fumigatus Total serum IgE elevated, Aspergillus specific IgE elevated Central bronchiectasis Treatment: The mainstay of treatment for ABPA is systemic corticosteroid medication. Ideally this can be administered in limited courses, but in some patients long-term treatment is necessary. Anti-fungal medication is generally ineffective, presumably due to the nature of the condition (a hypersensitivity disorder), and the difficulty in maintaining clearance from colonization of the airways by Aspergillus.

A 55 year old man presents with progressive anorexia, weight loss, diarrhea, nausea and vomiting, and profound peripheral oedema. Blood tests show evidence of anemia and hypoalbuminemia. Upper GI endoscopy shows enlarged, nodular and coarse gastric folds. What is the most likely diagnosis? A. Classic NSAID gastropathy B. Crohn's disease of the stomach C. Cronkhite-Canada syndrome D. GI manifestations of thyrotoxicosis E. Menetrier's disease ​

E. Menetrier's disease These features are very consistent with Menetrier's disease, which is a rare condition of unknown a etiology characterized by excessive production of transforming growth factor-alpha (TGF-α). As a result, the mucosal folds of the body and fundus are greatly enlarged. Whilst some patients have upper gastrointestinal symptoms, the majority present in middle or old age with protein-losing enteropathy due to exudation from the gastric mucosa. Endoscopy shows enlarged, nodular and coarse folds. Crohn's disease of the stomach usually presents with deep ulcers. Hyperthyroidism is associated with gastrointestinal symptoms but does not usually cause any endoscopic features. Cronkhite-Canada syndrome may present with generalized gastrointestinal polyps, cutaneous pigmentation, alopecia and onychodystrophy. NSAID gastropathy presents with gastritis, erosions or single/multiple superficial ulcers. ​

A 60-year-old man comes to the urgent care center after being awakened by severe pain in his right great toe, which is suddenly swollen and very tender to the touch. He has also had occasional headaches and "unbearable'1 pruritus after a hot bath over the past several weeks. The patient does not use alcohol, tobacco, or illicit drugs. His temperature is 36.8 C (98.3 F), blood pressure is 140/90 mm Hg, and pulse Is 90/min. Cardiopulmonary examination Is normal. The abdomen is soft and nontender, and there is no appreciable ascites. The liver span is 10 cm at the mid-clavicular line, and the spleen is palpable 2 cm below the costal margin on full inspiration. His right great toe is swollen and erythematous, with severe pain on any movement. Aspiration of the affected metatarsophalangeal joint shows negatively birefringent crystals and many leukocytes but no organisms. Which of the following is most likely responsible for this patient's symptoms? A. Chronic kidney disease B. Hemochromatosis C. Hyperparathyroidism D. Inherited enzyme deficiency E. Myeloproliferative disorder F. Pernicious anemia G. Portal hypertension ​

E. Myeloproliferative disorder This patient has podagra from an acute attack of gout. Gout occurs as a result of overproduction or underexcretion of uric acid. Overproduction of uric acid is typically related to consumption of a high-urate diets but any condition that increases catabolism and turnover of purines can raise uric acid levels and trigger a gout attack. Common causes include hematologic malignancies, tumor lysis syndrome, and psoriasis. Myeloproliferative disorders are common secondary causes of gout. This patient has several clinical features suggesting polycythemia vera (PV), including pruritus triggered by hot baths (aquagenic pruritus), headaches, and hepatosplenomegaly. PV is characterized by increased cell turnover due to clonal hyperproliferation in all 3 primary bone marrow lineages (ie, red cells, white cells, platelets). Up to 40% of patients with PV have gout. Allopurinol inhibits uric acid formation and is used to prevent gout attacks in patients with hyperuricemia due to PV. Educational objective: Gout is a common complication of myeloproliferative disorders due to excessive turnover of purines and the resulting increase in uric acid production.

A 22-year-old woman presents to the hospital with severe dyspnea. She has a history of atopic dermatitis and recently started a topical corticosteroid. She lives with her parents and two younger siblings, one of which recently had an upper respiratory infection. She does not smoke or use any illicit drugs. On examination, her temperature is 37.8 C, blood pressure is 108/72 mmHg, heart rate is 112 beats per minute, respiratory rate is 26 breaths per minute, and oxygen saturation is 88% on room air. Her systolic blood pressure decreases by more than 10 mmHg during inspiration. She is having difficulty speaking in full sentences and is diaphoretic. There are bilateral inspiratory and expiratory wheezes on examination with no dullness to percussion or tactile fremitus over the lung fields. There is no urticaria or angioedema on skin examination. Which of the following treatments should be given for this patient's current condition? A. Cautious oxygen administration, targeting an SaO2 <94% to prevent hypercapnia B. IV ceftriaxone and azithromycin C. Subcutaneous epinephrine for bronchodilation D. Nebulized ipratropium for bronchodilation E. Nebulized albuterol for bronchodilation F. Immediate pericardiocentesis

E. Nebulized albuterol for bronchodilation Nebulized albuterol for bronchodilation. This patient is suffering an acute asthma exacerbation without a previous asthma diagnosis. She has another component of the atopic triad (eczema), which indicates an increased risk for asthma. She is likely suffering from an asthma exacerbation caused in part by her recent exposure to a viral illness. There were wheezes on examination with no urticaria or angioedema, making an anaphylactic reaction unlikely. Once the reader can guess that the diagnosis is asthma, the next step is knowing how to manage an acute exacerbation. Management of an acute asthma exacerbation involves oxygen administration as well as intermittent or continuous nebulized albuterol, which is the first-line treatment. If the patient fails to respond to albuterol, then ipratropium and magnesium are additional options to promote bronchodilation. Oral corticosteroids should also be given to reduce airway inflammation during and after the exacerbation. Response to therapy can be monitored by following the SaO2 as well as either the FEV1 or the peak expiratory flow (PEF). Arterial blood gases may also be useful; be concerned about the finding of a normal PaCO2, which is often indicative of respiratory fatigue leading to the requirement of intubation (hypoxemia should cause hyperventilation and hypocapnia). There are important differences between the management of asthma and COPD exacerbations, which is summarized in Box. (A) Oxygen should be administered to asthmatics with a target SaO2 >90%. In COPD, the target is 90% to 94% due to the concern for the development of hypercapnia; however, this is not seen in asthma. (B) This patient is unlikely to have pneumonia given that she is afebrile and has no suggestive findings of pneumonia on lung examination. (C) Subcutaneous epinephrine is useful in anaphylaxis, but has no benefit over inhaled β2 agonists in asthma for bronchodilation. (D) Ipratropium is an anticholinergic and is the first-line treatment for COPD exacerbation (although albuterol is often used too); it may be used as an adjunctive therapy in asthma exacerbation, but albuterol is the first-line therapy. (F) Pulsus paradoxus is not specific to cardiac tamponade and can result from airway obstruction, which causes air trapping and hyperinflation of the lungs leading to an increase in intrathoracic pressure and therefore an increase in external pressure around the heart. The rest of the patient's presentation is not consistent with tamponade.

A 65-year-old woman comes to the office with progressively worsening right knee pain. She has had this pain for 2 years but had not sought medical attention and last saw a physician 5 years ago. There is no history of trauma to the joint. The patient initially treated the pain with over-the-counter medications. However, lately the pain has been worse, especially in the evening. Medical history is unremarkable. She recently retired from her occupation as an elementary school janitor and does not use tobacco, alcohol, or illicit drugs. Blood pressure is 160/100 mm Hg and pulse is 70/min. BMI is 34 kg/m2. Knee examination shows tenderness over the medial tibial condyle and a small joint effusion. Range of motion of the knee elicits bony crepitus. Which of the following is the most likely diagnosis? A. Anserine bursitis B. Iliotibial band syndrome C. Lateral collateral ligament injury D. Medial meniscal injury E. Osteoarthritis F. Patellofemoral pain syndrome ​

E. Osteoarthritis This patient has typical characteristics of osteoarthritis (OA) including age >50h obesity, and chronic joint pain that is worse after activity at the end of the day. OA predominantly affects the large weight-bearing joints of the lower extremities (ie, hips, knees), the small peripheral joints in the hands, and the cervical and lumbar spine. OA is due to degeneration of the articular cartilage and is usually an idiopathic phenomenon of aging, although it can also occur due to acute or chronic joint injury and certain systemic diseases (eg, hemochromatosis). Examination findings suggesting OA in the knee include: « Periarticular bony hypertrophy and tenderness « Limited range of motion « Crepitus and pain with motion « Small joint effusion without erythema or warmth « Varus or valgus angulation of the tibia « Popliteal (Baker) cyst behind the joint X-rays can confirm the diagnosis. Arthrocentesis is occasionally helpful and shows ciear fluid with few inflammatory cells. Laboratory testing including inflammatory (eg, erythrocyte sedimentation rate, C-reactive protein) and serologic (eg, rheumatoid factor) markers will be normal. Educational objective: Osteoarthritis causes chronic joint pain and is most common with advancing age, obesity, and prior joint injury. Examination findings include bony enlargement and tenderness, crepitus with movement, and painful or decreased range of motion.

A 46-year-old woman presents to her primary care physician because of weakness. She says that for the past month she has had difficulty brushing her hair and climbing stairs. She also says that her arms and legs feel sore. On physical examination cranial nerves II through XII are intact. She has 2/5 strength in the upper arms bilaterally but 5/5 strength in the hands. She has 3/5 strength in the quadriceps but 5/5 strength in the lower legs bilaterally. Her muscles are diffusely tender. She has normal sensation and 2+ reflexes throughout. Laboratory tests show: Creatine kinase: 3200 U/L (normal 26-140 U/L) Erythrocyte sedimentation rate: 18 mm/hr. WBC count: 8400/mm3 Hemoglobin: 13.2 g/dL Alanine aminotransferase: 84 U/L Aspartate aminotransferase: 92 U/L Electromyography shows myopathic changes. Which of the following is the most likely diagnosis? A. Acute viral infection causing rhabdomyolysis B. Amyotrophic lateral sclerosis C. Myasthenia gravis D. Polymyalgia rheumatica E. Polymyositis

E. Polymyositis Polymyositis causes a progressive symmetric weakness, predominantly affecting the proximal muscles and sparing distal muscles and fine motor movements. Patients often present with difficulty in everyday activities, such as getting dressed, brushing their hair, getting out of a chair, or climbing stairs. On examination, patients may have muscle tenderness and diminished strength, but sensation and reflexes are normal. Laboratory evaluation shows elevated muscle enzymes, including creatine kinase, lactate dehydrogenase, aldolase, alanine aminotransferase, and aspartate aminotransferase. Bottom Line: Polymyositis causes a progressive symmetric weakness, predominantly affecting proximal muscles and sparing distal muscles and fine motor movements. Lab evaluation shows elevated levels of muscle enzymes including creatine kinase, lactate dehydrogenase, aldolase, alanine aminotransferase, and aspartate aminotransferase. ​

A 19-year-old man had his spleen removed 1 year ago after a motorcycle accident. He returns for routine follow-up and has fully recovered after his accident. A routine complete blood count is performed. (See Figure below.) Which of the following findings on the blood film are most consistent with a previous history of splenectomy? A. Increase in macrophages B. Leukopenia C. Polycythemia D. Increased reticulocytes E. Red cells with nuclear

E. Red cells with nuclear The spleen normally functions to pit nuclei and their fragments from red cells. These nuclear fragments are called Howell-Jolly bodies. The spleen has immune functions and filter functions. It also enhances iron reutilization, and acts as a reservoir and blood-volume regulator. In disease states, it can be a site of extramedullary hematopoiesis.

The most sensitive and specific diagnostic test for gastrinoma is: A. Basal and stimulated gastric acid outputs B. Octreotide scan C. Fasting serum gastrin D. Calcium stimulation test E. Secretin stimulation test ​

E. Secretin stimulation test The most sensitive and specific test for gastrinoma (ZES) is the secretin stimulation test. An IV bolus of secretin is administered, and gastrin levels are checked before and after injection. An increase in serum gastrin of 120 pg/mL or greater has the highest sensitivity and specificity for gastrinoma. There are numerous other causes of hypergastrinemia. They can be divided into those associated with an increased acid production and those with a decreased acid production (A). In the latter situation, the hypergastrinemia is reactive due to hypo- or achlorhydria. In addition to ZES, G-cell hyperplasia, gastric outlet obstruction, and retained antrum after Billroth II reconstruction are associated with increased acid production. Reactive hypergastrinemia is seen with atrophic gastritis, pernicious anemia, and gastric cancer; in patients receiving H2-receptor antagonists and PPIs; and after vagotomy. Hypergastrinemia is also seen in chronic renal failure due to decreased catabolism. Given this broad differential, fasting serum gastrin levels (C) are not sufficiently specific to establish the diagnosis of ZES in the majority of patients unless gastrin levels are extremely high (>1000 pg/mL). The secretin stimulation test has higher sensitivity and specificity than the calcium stimulation test (D). The calcium stimulation test is used if the secretin test result is negative and there is a high suspicion for ZES in the presence of hypergastrinemia. Once the diagnosis of ZES is established, a nuclear octreotide scan (B) seems to be the most sensitive test to localize the tumor.

A 23-year-old woman has symptoms of fatigue and not feeling well. She appears well and the physical examination is normal, but her hemoglobin is low at 9.8 g/dL with a mean corpuscular volume (MVC) of 76 fL. Her family history is positive for thalassemia. She also mentions a history of heavy menses in the past year. Which of the following would be most helpful in distinguishing thalassemia from one of pure iron deficiency anemia? A. Peripheral blood smear B. Osmotic fragility test C. Ham test D. Hb electrophoresis E. Serum ferritin level

E. Serum ferritin level A serum ferritin determination would be most helpful; in iron deficiency anemia the ferritin level is very low (< 20 ng/ml). Iron stores in thalassemia are usually normal or increased and the ferritin level is normal or increased. The blood film, Ham test, and osmotic fragility test will not confirm the diagnosis of thalassemia or iron deficiency. While Hb electrophoresis can be useful in the diagnosis of some thalassemias it will not determine if the patient is iron deficient.

A 35 year old man develops low back, posterior heel pain and a swollen knee and has a pustular skin rash on the soles of his feet. There are no preceding illnesses, no previous psoriasis or family history of it. What is the most likely diagnosis? A. Ankylosing spondylitis B. Gout C. Post-streptococcal arthritis D. Psoriatic arthritis E. Sexually acquired reactive arthritis ​

E. Sexually acquired reactive arthritis A pustular plantar foot rash occurring simultaneously with inflammatory back pain, enthesitis and synovitis does suggest reactive arthritis. A sexual history may not be volunteered but should be sought - with direct questions about new recent sexual encounters, penile discharge, dysuria and other genital symptoms, if necessary. ​

A 57 year old man who is a heavy smoker presents to his family physician with epigastric pain, occasional vomiting, tiredness and easy fatigability. Clinical examination reveals signs of anemia and epigastric tenderness but no masses or organomegaly. Routine blood tests confirm mild iron deficiency anemia but no other abnormalities. Which action by the family physician is the most appropriate? A. She should arrange for him to have an urgent barium meal B. She should check his H. pylori serology and start him on eradication therapy if positive. C. She should start him immediately on H. pylori eradication therapy D. She should start him on a course of PPIs and review him in 2 months for repeat blood tests E. She should start him on PPIs and refer him for an urgent upper GI endoscopy ​

E. She should start him on PPIs and refer him for an urgent upper GI endoscopy This patient clearly requires urgent upper GI endoscopy to rule out significant pathology, particularly gastric neoplasia. The absence of weight loss and persistent vomiting is reassuring but the presence of anemia is alarming and should trigger urgent referral for endoscopy. Barium meal is rarely used. The other options are inappropriate because urgent endoscopy is mandatory in this situation and over-rides the other suggestions. ​

A frail 85-year-old female presents to the ED with severe dehydration and ketoacidosis due to persistent vomiting. She is not a diabetic. She has had significant weight loss. Workup eventually demonstrates gastroparesis. After this initial hospitalization, which of the following long-term treatment strategies should be initiated? A. Cisapride B. Metoclopramide C. Botulinum toxin injection at the pylorus D. Gastric electrical stimulation device E. Six low-fat, low-fiber meals per day

E. Six low-fat, low-fiber meals per day The etiology of gastroparesis is most commonly idiopathic. Diabetes is the next most common etiology. The first step in the management of gastroparesis is fluid hydration and nutritional support. Agents to prevent nausea and vomiting are also important. Long-term management begins with dietary modification. Symptoms can be controlled with eating six small, low-fat, low-fiber meals throughout the day, which slows down gastric emptying. This may require additional nutritional supplementation with high-caloric shakes. Promotility agents such as metoclopramide or erythromycin can be helpful as well but have additional side effects restricting their widespread use; metoclopramide can lead to tardive dyskinesia and has a black box warning for this (B). Botulinum toxin injection at the pylorus has not been shown to have effective results (C). Cisapride, a cholinergic agonist, has been banned in the United States secondary to its multiple drug interactions (A). Rarely, and in severe refractory cases of gastroparesis, a gastric cardiac pacemaker can be implanted, but this needs to be done at laparoscopy under general anesthesia and lead placement must be confirmed by intraoperative endoscopy. This, however, would not be appropriate for an elderly and frail patient (D).

A 54-year-old, previously healthy man sustained an extensive thermal burn injury involving 70% of the total body surface area of his skin. He was hospitalized in stable condition. Three weeks after the initial burn injury, he developed melanotic stools. His blood pressure dropped to 80/40 mm Hg, and his hematocrit declined to 18%. Where are gastrointestinal ulcerations most likely to be found in this man? A. Colon B. Duodenum C. Esophagus D. Ileum E. Stomach ​

E. Stomach So-called stress ulcers, also known as Curling ulcers, can occur in patients with burn injuries. The ulcers are often small (<1 cm) and shallow, never penetrating the muscularis propria, but they can bleed profusely. Similar lesions can occur after traumatic or surgical injury to the central nervous system (Cushing ulcers). Duodenal ulcers are typically peptic ulcers in individuals with Helicobacter pylori infection. Esophageal varices can cause massive hematemesis, but they occur in patients with portal hypertension, caused most commonly by cirrhosis. Metaplastic columnar epithelium at the lower end of the esophagus is present in Barrett esophagus, resulting from chronic gastroesophageal reflux disease. Ileal ulcerations and colonic ulcerations are often due to inflammatory bowel disease that can be from infections such as shigellosis, or they may be idiopathic, as in Crohn disease.

A 73-year-old woman presents to the emergency room complaining of black tarry stools for the previous 2 days and now symptoms of lightheadedness when standing up. On physical examination, she has a postural drop in her blood pressure and increase in heart rate. The abdomen is soft and nontender, but digital rectal examination confirms the presence of melena. She recently started using ibuprofen for hip discomfort. She is admitted to the hospital for treatment and upper endoscopy confirms the diagnosis of a gastric ulcer. Which of the following is the most likely explanation for the gastric ulcer? A. increasing acid production B. causing direct epithelial cell death C. promoting replication of Helicobacter pylori D. an antiplatelet effect E. inhibiting mucosal repair ​

E. inhibiting mucosal repair NSAIDs inhibit prostaglandins, which play an important role in maintaining gastroduodenal mucosal integrity and repair.

A 9-year-old boy with homozygous sickle cell anaemia is brought into the Emergency Department. He had complained of chest pain in the previous 48 hours with no cough. He was pyrexial, temperature 38°C, and shocked and clammy in the extremities. Compliance with his prophylactic medications had been poor recent ly because of his relocation to live with his grandmother. There was no record of childhood vaccinations. Which organism is most likely to be causative? A. Escherichia coli B. Methicillin-resistant Staphylococcus aureus C. Mycoplasma hominis D. Salmonella spp E. Streptococcus pneumoniae

E. Streptococcus pneumoniae We are told that this is a 9-year-old boy who has significant homozygous sickle cell anaemia, who has not been on prophylactic antibiotics such as penicillin. He also has not had his vaccinations. Patients with sickle cell disease are prone to infections within encapsulated organisms because of their asplenic state. These include Srreprococcus pneumoniae, Haemophilus influenzae and Neisseria meningitidis. To combat these infections, patients with homozygous sickle cell disease should be on lifelong penicillin and be vaccinated against these organisms.

A 64-year-old man comes to the emergency department with a 3-day history of progressively increasing pain; swelling, and redness of his right knee. The pain is severe with weight bearing, and he is unable to ambulate without assistance. The patient reports no recent trauma to the knee but has had subjective fever. He was diagnosed with gout 10 years ago and had several recurrences of acute gout Involving multiple joints, but has had good control since he started taking allopurinol. Six weeks ago, he went on a camping trip in the mountains of New Hampshire. Medical history is notable for type 2 diabetes mellitus and hypertension. Current medications include lisinopril and metformin. The patient does not use tobacco, alcohol, or illicit drugs. His temperature is 38.3 C (100.9 F), blood pressure is 110/65 mm Hg, and pulse is 110/min. Examination shows a tender, swollen, erythematous right knee with markedly decreased range of motion. The remainder of the examination is unremarkable. Which of the following is the most appropriate next step in management? A. Lyme serology B. MRI of the knee C. Naproxen and outpatient follow-up D. Serum uric acid level E. Synovial fluid analysis F. X-ray of the knee

E. Synovial fluid analysis This patient's presentation with fever and acute monoarticular arthritis warrants urgent synovial fluid analysis (cell count. Gram stain, culture) to exclude septic arthritis. Patients with an established diagnosis of gout who have typical symptoms of a flare can be managed without joint aspiration. Although gout can cause low-grade fever, the progressive nature of this patient's symptoms over 3 days (gout flares typically have an abrupt onset with maximal symptoms within 12-24 hours) increases the likelihood of a septic joint, particularly in the setting of diabetes. Underlying joint disorders (eg, gout, pseudogout, osteoarthritis) increase the risk for secondary joint infection, in patients with crystal-induced arthritis (eg, gout), the presence of crystals alone does not rule out septic arthritis as these can be present in synovial fluid between attacks. If the Gram stain is positive and joint fluid white cell count is >50,000/mm;, the patient should be started on empiric antibiotics until culture results are known, if crystals are present, the fluid is nonpurulent, and Gram stain is negative, the patient may be managed as for a standard gout flare (Choice C). Educational objective: Synovial fluid should be obtained urgently for cell count, Gram stain, and culture in any patient with possible septic arthritis. Preexisting joint disorders can increase the risk for secondary Infection of the joint. In patients with crystal-induced arthritis (eg, gout), the presence of crystals in the synovial fluid does not rule out septic arthritis as these will be present between attacks.

A 55-year-old woman comes to the office with a one-week history of pain in multiple joints. She has achy pain and stiffness in both wrists and multiple metacarpophalangeal and proximal interphalangeal joints in both hands. The patient's symptoms are worst in the morning and partially improve over 10-15 minutes of normal activity. There is no associated fever, chills, rash, or weight loss. She works in a day care center and does not use tobacco, alcohol, or illicit drugs. Vital signs are normal. On examination, there is mild swelling with no redness or tenderness of the involved joints. Laboratory studies show normal blood counts and serum chemistries. Erythrocyte sedimentation rate is 12 mm/hr. Which of the following is the most likely diagnosis in this patient? A. Acute rheumatic fever B. Fibromyalgia C. Polymyalgia rheumatica D. Rheumatoid arthritis E. Systemic lupus erythematosus F. Viral arthritis ​

E. Systemic lupus erythematosus This patient has acute, symmetrical arthralgias with mild synovitis primarily involving the small joints of the hands. In light of her brief morning stiffness, normal erythrocyte sedimentation rate (ESR), and exposure to small children, this most likely represents viral arthritis due to parvovirus B19. Children with parvovirus often develop fever and the "slapped cheek" facial rash of erythema infectiosum. However, adults are more likely to develop joint symptoms; skin symptoms, if present, are mild and nonspecific. Parvovirus B19 is a nonenveloped, single-stranded DNA virus that primarily infects red cell precursors. Viral arthritis due to parvovirus B19 is typically polyarticular and symmetric. It is a common, self-limited condition that usually resolves within 1-2 months. The precise pathophysiology is unclear, but clinical manifestations correlate with anti-parvovirus lgM levels, which may be assayed for diagnostic purposes. Treatment is symptomatic, primarily with nonsteroidal anti-inflammatory drugs. Educational objective: Viral arthritis due to parvovirus B19 is characterized by acute, symmetric, small-joint arthralgias, mild joint swelling, and a benign, self-limited course. It is usually seen in adults, whereas erythema infectiosum due to parvovirus B19 is more common in children.

A 66-year old woman presents with pain and stiffness affecting the wrists, proximal interphalangeal (PIP) and metacarpophalangeal (MCP) joints of the hands, gradually worsening over a period of 6-8 weeks. On examination, there is symmetrical swelling and tenderness of both wrists and the MCP and PIP joints of the hands. Investigations show that anti-citrullinated peptide antibodies (ACPAs) and rheumatoid factor are negative, but that she has an elevated ESR (25 mm/hr) and a raised CRP (65 mg/L). X-rays of the hands are normal. Which of the following statements is correct? A. Magnetic resonance imaging (MRI) of the hands should be done to clarify the diagnosis B. Rheumatoid arthritis is excluded by the negative ACPA test and normal radiographs C. The joint pain and swelling is most likely due to generalised osteoarthritis D. The presentation is consistent with polymyalgia rheumatica (PMR) E. The presentation is typical of seronegative rheumatoid arthritis ​

E. The presentation is typical of seronegative rheumatoid arthritis The negative ACPA and normal radiograph does not exclude RA since ACPA and rheumatoid factor are negative in about one-third of patients and radiographs are normal in early RA. The presentation would not be consistent with PMR and the distribution of involvement (wrists, metacarpophalangeal joint; MCPJ) excludes OA. MRI would not be necessary since the patient has typical signs of synovitis.

A 35-year-old woman comes to the physician with pain and stiffness of her wrists and hand joints for the last several months. Her morning stiffness lasts for more than an hour. She also complains of joint swelling. Her past medical history is significant only for a similar episode a year ago; which resolved with over-the-counter ibuprofen. Joint examination shows mild redness, warmth, swelling, and tenderness in the proximal interphalangeal! and metacarpophalangeal joints and wrists. X-rays show periarticular osteopenia and erosions of the proximal interphalangeal! and metacarpophalangeal joints. The patient began taking indomethacin, which provides good relief, but symptoms recur if she skips a dose. Which of the following is the most appropriate next step in management of this patient? A. Azathioprine B. Celecoxib C. Etanercept D. Glucocorticoids E. Infliximab F. Methotrexate G. No additional treatment

F. Methotrexate All patients diagnosed with RA should be started on disease-modifying antirheumatic agents (DMARDs) as soon as possible as joint damage begins early in its course (Choice G). Nonsteroidal anti-inflammatory drugs and COX-2 inhibitors (eg, celecoxib) are adjunctive therapies for symptomatic relief but do not reduce disease progression. Glucocorticoids can also relieve symptoms and short-term radiographic progression but are also not effective in preventing eventual joint destruction. In fact, they can result in generalized bone loss (ie, osteoporosis). DMARDs include nonbiologic agents (eg, methotrexate, hydroxychloroquine, sulfasalazine, leflunomide, azathioprine) and biologic agents (eg, etanercept, infliximab, adalimumab, tocilizumab, rituximab). Methotrexate is the preferred initial DMARD in patients with moderately to severely active RA due to its efficacy and long-term safety profile. Patients should be tested for hepatitis B and C and tuberculosis before starting therapy. Methotrexate should not be used In patients who are pregnant or are planning to become pregnant in the near future and those with severe renal insufficiency, liver disease, or excessive alcohol intake. Patients who do not respond after 6 months may require biologic DMARDs such as tumor necrosis factor-alpha inhibitors (eg, etanercept, infliximab) as step-up therapy (Choices C and E). Educational objective: All rheumatoid arthritis (RA) patients should receive disease-modifying antirheumatic drugs (DMARDs) as early as possible in the disease course. Methotrexate is the initial DMARD of choice in most patients with active RA. Nonsteroidal anti-inflammatory drugs or glucocorticoids should be used for initial temporary symptomatic relief while awaiting response to DMARD therapy.

A 68-year-old man is seen in follow-up for a recent diagnosis of acromegaly. He presented with chronic fatigue, joint and back pain, and an increase in his shoe size over the past 2 years. Medical history is significant for hypertension and type 2 diabetes mellitus. Current medications are lisinopril, metformin, and as-needed acetaminophen. On physical examination, blood pressure is 146/88 mm Hg and pulse rate is 90/min. BMI is 29. He has a prominent brow. Macroglossia is present. Lung and heart examinations are unremarkable. Musculoskeletal examination reveals large hands and knees with bone swelling and crepitus. Skin is thickened, and there is excessive perspiration. On neurologic examination, bitemporal hemianopsia is noted. Laboratory studies are significant for an elevated serum insulin-like growth factor 1 level of 996 ng/mL (996 µg/L) and serum prolactin level of 42 ng/mL (42 µg/L). MRI shows a 2.5 x 1.8-cm pituitary macroadenoma that elevates the optic chiasm and appears to envelop the left carotid artery and invade the left cavernous sinus. The optic chiasm is mildly atrophied. Which of the following is the most appropriate next step in therapy for this patient? A. Dopamine agonist B. Growth hormone receptor blockade C. Somatostatin analogue D. Stereotactic radiation therapy E. Transsphenoidal pituitary surgery

E. Transsphenoidal pituitary surgery Educational Objective: Treat acromegaly with transsphenoidal pituitary surgery. Key Point: The primary therapy for acromegaly is transsphenoidal surgery to remove the causative growth hormone-secreting pituitary adenoma. Transsphenoidal resection of the pituitary adenoma is the initial treatment of choice in patients with acromegaly. It is also the only treatment that is potentially curative. Because this patient's tumor is invading the left cavernous sinus and compressing the optic chiasm, complete resection will likely not be possible; however, surgery can effectively debulk the tumor and preserve vision in addition to significantly decreasing growth hormone (GH) secretion as measured by insulin-like growth factor 1 (IGF-1) levels. In patients in whom complete resection is not possible, such as this patient, additional therapy may be required such as stereotactic radiation therapy or medical therapy to inhibit GH secretion or block its effect on the tissues. However, surgical resection remains an essential first step in the treatment of acromegaly. A small number of GH-secreting pituitary adenomas co-secrete prolactin. Although dopamine agonist therapy with agents such as bromocriptine would treat the associated prolactin elevation, it is minimally effective in acromegaly and would not adequately treat GH secretion or address the mass effect of a GH-secreting adenoma. A GH receptor blocker, pegvisomant, is available. Pegvisomant works in the peripheral tissues as an antagonist to GH but does not decrease its production by the tumor. This patient needs intervention to treat mass effect at this time because the tumor is damaging the optic chiasm and the patient's vision, and this treatment would not be expected to decrease the tumor size. Somatostatin analogues, such as octreotide and lanreotide, inhibit GH secretion and are helpful in treating some patients with acromegaly. They are used primarily in patients with unresectable tumors without significant mass effect or those with a contraindication to surgery. They may also be used in patients with continued GH secretion following incomplete transsphenoidal resection. However, they would not be an appropriate treatment in this patient with a large, invasive, vision threatening pituitary tumor. Radiation therapy may be added to surgical or medical therapy to help increase the chance for remission or cure. Radiation to the pituitary carries a high risk of causing pituitary insufficiency and damage to surrounding tissues (particularly the optic nerves); therefore, it is not usually an initial treatment for acromegaly in most patients. In those in whom it is used, stereotactic surgery (gamma knife) is the preferred approach to minimize potential complications.

A 56-year-old man presents with epigastric pain, diarrhea, and weight loss. Upper endoscopy reveals giant gastric folds in the proximal stomach. A biopsy specimen demonstrates diffuse foveolar hyperplasia with no evidence of malignancy. Twenty-four-hour gastric pH levels are consistent with achlorhydria. Which of the following is true regarding the most likely condition? A. There is no risk for the development of cancer. B. It is associated with cytomegalovirus in adults. C. Medical therapy is highly effective. D. It is an inherited condition. E. Treatment includes a high-protein diet. ​

E. Treatment includes a high-protein diet. The patient has Menetrier disease. Menetrier disease is an acquired disease with no family predisposition (D). The etiology is unknown, but it is associated with an increase in transforming growth factor alpha. Giant rugal folds, particularly in the fundus and body, and a loss of parietal cells develop. Thus, patients have hypo- or achlorhydria. Patients lose a large amount of protein in the stomach due to the hypersecretion of mucus. This results in weight loss and peripheral edema. Giant rugal folds are also seen with ZES; however, the latter can be ruled out by the demonstration of achlorhydria. Mucosal biopsy shows diffuse hyperplasia of the surface mucus-secreting cells as well as loss of parietal cells. There seems to be an increased risk of gastric cancer, although it is not high (A). It is associated with cytomegalovirus infection in children and with H. pylori infection in adults (B). Most patients with Menetrier disease are middle-aged men. Symptoms include epigastric pain, weight loss, diarrhea, and hypoproteinemia. Treatment includes a high-protein diet, anticholinergic agents, and H. pylori eradication if the patient tests positive. Medical treatment has not been particularly effective, although there are cases of spontaneous resolution (C). For patients who do not respond to medical therapy, gastric resection may be indicated, particularly for intractable severe hypoproteinemia or if dysplasia or malignancy develops. Recently, Erbitux (cetuximab), which blocks the action of transforming growth factor alpha, has been used as a potential new treatment.

A 52-year-old man is evaluated for a 3-month history of postprandial fullness, early satiety vomiting, and epigastric discomfort. He has had these symptoms intermittently for at least 6 months, but for the past 3 months they have occurred with nearly every meal. The symptoms tend to be much worse with large meals, especially dinner. The discomfort is described as a burning sensation that is located in the epigastrium. The patient does not have recent weight loss, dysphagia, odynophagia, or blood in his stool, although he has had several episodes of vomiting associated with his symptoms. He otherwise feels well. Medical history is unremarkable and he currently takes no medications. Physical examination findings, including vital signs, are normal. A complete blood count is normal. Which of the following is the most appropriate management of this patient's symptoms? A. Abdominal Computed ttomography B. Abdominal uultrasonography C. Empiric proton pump inhibitor ttherapy D. Helicobacter pylori fecal antigen ttesting E. Upper endoscopy ​

E. Upper endoscopy Educational Objective: Evaluate dyspepsia with alarm features. The most appropriate management of this patient's symptoms is to perform upper endoscopy. Nearly 70% of patients with dyspepsia do not have a physiologic explanation for their symptoms. According to the Rome III criteria, functional dyspepsia includes one or more of the following: (1) bothersome postprandial fullness, (2) early satiety, (3) epigastric burning, and (4) epigastric pain with lack of structural disease on upper endoscopy. To establish the diagnosis, these criteria should be met for 3 months, with symptom onset at least 6 months prior to diagnosis. Guidelines recommend upper endoscopy for patients with dyspepsia and alarm features. Alarm features for dyspepsia include onset after age 50 years; anemia; dysphagia; odynophagia; vomiting; weight loss; family history of upper gastrointestinal malignancy; personal history of peptic ulcer disease, gastric surgery, or gastrointestinal malignancy; and abdominal mass or lymphadenopathy on examination. Because this patient is 52 years old and has had vomiting, upper endoscopy should be performed to evaluate for a serious underlying cause of his symptoms. KEY POINT: Upper endoscopy is indicated for patients with dyspepsia and alarm features, including symptom onset after age 50 years; anemia; dysphagia; odynophagia; vomiting; weight loss; family history of upper gastrointestinal malignancy; personal history of peptic ulcer disease, gastric surgery, or gastrointestinal malignancy; and abdominal mass or lymphadenopathy on examination.

A 24-year-old woman comes to the office with a 4-week history of joint pain. She has had moderate, achy pain and swelling in multiple metacarpophalangeal joints of both hands, with lesser pain in the wrists, knees, and elbows. In addition, the patient experienced intermittent fevers at the onset of her pain, but has been afebrile since then. She has had relief of her symptoms with over-the-counter ibuprofen, but in the week since she scheduled her appointment, she has not required medication as the pain has resolved. The patient is married and has 2 children. Vital signs are normal. Examination shows normal range of motion in all joints tested, with no redness, warmth, or swelling in any hand joints. X-rays of the hands are normal. Which of the following is the most likely diagnosis? A. Gonococcal arthritis B. Lyme disease C. Rheumatoid arthritis D. Systemic lupus erythematosus E. Viral arthritis ​

E. Viral arthritis This patient has symmetric polyarticular arthritis with a brief, self-limited course. In light of her exposure to young children, this is likely viral arthritis due to parvovirus B19. Viral arthritis is often associated with joint swelling and tenderness, but objective findings may be subtle or absent. Whereas children with parvovirus B19 typically present with the characteristic "slapped cheek" rash, adults are more likely to develop a nonspecific morbilliform exanthem and may have no skin manifestations at all. A similar arthritis may also be seen with HIV, mumps, rubella, and other viruses. The arthritis of parvovirus B19 may resemble early rheumatoid arthritis (RA) or systemic lupus erythematosus (SLE), and may be associated with a weakly positive rheumatoid factor or antinuclear antibody titer. However, RA and SLE are significantly less common, usually produce more obvious synovitis, and typically follow a more chronic, protracted course (Choices F and H). In addition, SLE is usually associated with manifestations in multiple systems (eg, malar rash, oral ulcers, serositis) and is less likely to resolve spontaneously. if there is doubt regarding the diagnosis, parvovirus 819 infection can be confirmed with assay for anti-parvovirus lgM or nucleic acid amplification testing. Educational objective: The self-limited arthritis of parvovirus B19 may resemble early rheumatoid arthritis (RA) or systemic lupus erythematosus (SLE), and may be associated with a weakly positive rheumatoid factor or antinuclear antibody titer. However, RA and SLE are significantly less common, typically produce more obvious synovitis, and usually follow a chronic, protracted course.

Which of the following lung volumes or capacities can be measured by spirometry? A. Functional residual capacity (FRC) B. Physiologic dead space C. Residual volume (RV) D. Total lung capacity (TLC) E. Vital capacity (VC) ​

E. Vital capacity (VC) Residual volume (RV) cannot be measured by spirometry. Therefore, any lung volume or capacity that includes the RV cannot be measured by spirometry. Measurements that include RV are functional residual capacity (FRC) and total lung capacity (TLC). Vital capacity (VC) does not include RV and is, therefore, measurable by spirometry. Physiologic dead space is not measurable by spirometry and requires sampling of arterial PCO2 and expired CO2.

A 65-year old woman with osteoarthritis attends her family physician because of knee pain worse on going up- and downstairs. She is a non-smoker, does not drink alcohol, but is taking simvastatin 20 mg daily for high cholesterol and lisinopril 10 mg daily for hypertension. On examination her height is 154 cm and weight is 95 kg. What would be the most appropriate treatment for her knee pain? A. Arthroplasty B. Cognitive behavioural therapy (CBT) C. Regular weight-bearing exercise D. Surgical synovectomy E. Weight loss ​

E. Weight loss Obesity aggravates joint pain in osteoarthritis and weight loss is one of the most effective therapies for osteoarthritis (OA) of the lower limbs. Weight-bearing exercise is unlikely to help and may worsen symptoms. Surgical synovectomy is not indicated in OA and arthroplasty would only be indicated for advanced OA resistant to medical therapy. Cognitive behavioral therapy would be unlikely to help. ​

A 70-year-old woman is seen for follow-up evaluation for possible Cushing syndrome. She presented with new-onset diabetes mellitus and a 9.1-kg (20-lb) weight gain over the last 6 months. Medical history is otherwise unremarkable, and she is currently taking no medications and has had no exposure to exogenous glucocorticoids in the past year. On physical examination, blood pressure is 160/90 mm Hg, pulse rate is 80/min, and respiration rate is 12/min. BMI is 30. Facial plethora, central obesity, and bilateral supraclavicular fat pads are noted. There are violaceous abdominal striae measuring 1 cm wide and multiple ecchymoses on the extremities. Initial laboratory studies show a serum cortisol level of 9 µg/dL (248.4 nmol/L) following a 1-mg dose of dexamethasone the night before, and a 24-hour urine free cortisol level that is greater than 3 times the upper limit of normal, which is confirmed on a second measurement. A plasma adrenocorticotropic hormone (ACTH) level is undetectable. Which of the following is the most appropriate diagnostic test to perform next? A. CT scan of the adrenal glands B. Inferior petrosal sinus sampling C. Late night salivary cortisol measurement D. MRI of the pituitary gland

Educational Objective: Evaluate the cause of Cushing syndrome. Key Point: CT of the adrenal glands should be performed in patients with ACTH-independent Cushing syndrome because adenomas and carcinomas of the adrenal cortex are common causes. The most appropriate diagnostic test to perform next is a CT scan of the adrenal glands. This patient was suspected to have Cushing syndrome (CS) based on pathognomonic clinical findings and is not taking exogenous glucocorticoids. Two screening tests for CS are abnormal, which is adequate to establish the diagnosis. The next step in evaluation is measurement of the plasma adrenocorticotropic hormone (ACTH) level to differentiate between ACTH-dependent and ACTH-independent CS. With ACTH-dependent causes, the plasma ACTH is usually greater than 20 pg/mL (4.4 pmol/L). However, the low plasma ACTH reported in this patient is consistent with ACTH-independent CS. A CT scan of the abdomen to evaluate the adrenal glands is therefore indicated because the most common causes of ACTH-independent CS are adenomas and carcinomas of the adrenal cortex. Inferior petrosal sinus sampling and MRI of the pituitary gland are used to confirm the presence of a corticotroph adenoma of the pituitary gland in ACTH-dependent CS, which is the most common cause of CS overall. However, neither is indicated in this patient because biochemical testing has not revealed ACTH-dependent CS. The diagnosis of CS requires that at least two first-line screening tests be abnormal, including the low-dose dexamethasone suppression test (LDST) (both standard and overnight), 24-hour urine free cortisol (UFC), or late-night salivary cortisol. This patient failed to suppress cortisol levels following an overnight LDST and had an elevated 24-hour UFC level confirmed by a repeat collection. Therefore, measurement of late-night salivary cortisol is unnecessary because the diagnosis of CS has already been established.

A 52-year-old man has had a 4-kg weight loss and nausea for the past 6 months. He has no vomiting or diarrhea. On physical examination, there are no remarkable findings. Upper gastrointestinal endoscopy shows a 6-cm area of irregular, pale fundic mucosa and loss of the rugal folds. A biopsy specimen shows a monomorphous infiltrate of lymphoid cells microscopically. Helicobacter pylori organisms are identified in mucus overlying adjacent mucosa. Cytogenetic analysis shows t(11;18)(q21;q21). He receives antibiotic therapy for H. pylori, and the repeat biopsy specimen shows a resolution of the infiltrate. What is the most likely diagnosis? A. Autoimmune gastritis B. Chronic gastritis C. Crohn disease D. Diffuse large B-cell lymphoma E. Gastrointestinal stromal tumor F. Mucosa-associated lymphoid tissue tumor ​

F. Mucosa-associated lymphoid tissue tumor Certain gastrointestinal lymphomas that arise from mucosa-associated lymphoid tissue (MALT) are called MALT lymphomas. Gastric lymphomas that occur in association with Helicobacter pylori infection are composed of monoclonal B cells, whose growth and proliferation depend on cytokines derived from T cells that are sensitized to H. pylori antigens. Treatment with antibiotics eliminates H. pylori and the stimulus for B-cell growth. However, lesions acquiring additional mutations, such as p53, may become more aggressive. MALT lesions can occur anywhere in the gastrointestinal tract, although they are rare in the esophagus and appendix. In H. pylori chronic gastritis, which may precede lymphoma development, there are lymphoplasmacytic mucosal infiltrates. Diffuse large B-cell lymphomas and other non-Hodgkin lymphomas that are not MALT lymphomas do not regress with antibiotic therapy. Autoimmune gastritis is a risk for development of gastric adenocarcinoma. Crohn disease is rare in the stomach and is not related to H. pylori infection. Gastrointestinal stromal tumors are uncommon; these bulky tumors may be proliferations of interstitial cells of Cajal, myenteric plexus cells that are thought to be the pacemaker of the gut.

A 43-year-old man comes to the office with muscle weakness. His symptoms began 6 weeks ago and are worst in the hips and shoulders The weakness has had a progressive course so that he is now having difficulty rising from chairs and combing his hair. The patient has had to reduce his regular exercise regimen due to weakness and has requested a modified work schedule for his job as a building maintenance manager. He has no history of spinal injury and no neck pain. Past medical history is insignificant, and the patient takes no medications. Muscle biopsy reveals major histocompatibility complex class I molecule overexpression on the sarcolemma with CD8+ lymphocyte infiltration. Which of the following is the most likely cause of this patient's condition? A. Diffuse systemic sclerosis B. Eaton-Lambert syndrome C. Myasthenia gravis D. Polyarteritis nodosa E. Polymyalgia rheumatica F. Polymyositis ​

F. Polymyositis This patient, a middle-aged man with progressive, symmetric proximal muscle weakness, has typical symptoms of polymyositis The weakness may be painless or associated with diffuse myalgias. Muscle enzyme levels (e.g., creatine kinase) are invariably elevated, and autoantibodies (e.g. antinuclear antibodies, anti-Jo-1 antibodies) are present in most cases. Polymyositis is similar to dermatomyositis, although it lacks the typical skin findings; both may occur independently or as a paraneoplastic manifestation of an underlying malignancy. Polymyositis is an inflammatory myopathy triggered by unknown, possibly viral, antigens. It likely represents a cell-mediated immune response against myocytes. Increased expression of major histocompatibility complex class I antigens on the sarcolemma has been demonstrated and likely leads to presentation of autoantigens to CD8+ cytotoxic cells that subsequently initiate myocyte destruction. Muscle biopsy in polymyositis reveals inflammation, patchy necrosis, and regeneration and fibrosis of muscle fibers. Infiltration of the endomysium by macrophages and CD8+ lymphocytes is typically seen. Educational objective: Polymyositis causes symmetric proximal muscle weakness. Muscle biopsy reveals inflammation, necrosis, and regeneration of muscle fibers. Over-expression of major histocompatibility complex class I proteins on the sarcolemma leads to infiltration with CD8+ T lymphocytes and myocyte damage.

A 42-year-old man comes to the emergency department due to severe back pain following a fall. He tripped while getting out of his car and landed on his back. The patient has a 20-year history of ankylosing spondylitis, for which he takes prescribed medication regularly, but he has never had pain similar to this. His medical history is also notable for a duodenal ulcer 7 years ago. The patient does not use alcohol, tobacco, or illicit drugs. Vital signs are normal except for a mild, regular tachycardia. He appears to be in pain, with grimacing and diaphoresis. Examination shows midline tenderness over the upper lumbar region. Spinal range of motion is restricted. There is no tenderness over the sacroiliac joints. Cardiac auscultation shows an early diastolic murmur. Which of the following is the most likely cause of his current pain? A. Disc herniation B. Referred visceral pain C. Spinal metastasis D. Spondylitis flare-up E. Spondylolisthesis F. Vertebral fracture ​

F. Vertebral fracture This patient has ankylosing spondylitis (AS) with reduced range of motion in the spine. He also has a murmur suggesting aortic regurgitation, a common finding in chronic AS. He now has new acute pain following a fall. Although a ground-level fall would be unlikely to cause a fracture in an otherwise healthy man of his age, the clinical suspicion for vertebral fracture should be much higher in a patient with AS. Patients with longstanding AS can develop osteopenia/osteoporosis due to increased osteoclast activity in the setting of chronic inflammation (mediated by TNF-α and interleukin-6). In addition, spinal rigidity in these patients can increase the risk of vertebral fracture, which often results from minimal trauma. Associated findings may include thoracic wedging and hyperkyphosis. Educational objective: Patients with long-standing ankylosing spondylitis can develop bone loss due to increased osteoclast activity in the setting of chronic inflammation. In addition, spinal rigidity in these patients can increase the risk of vertebral fracture, which often results from minimal trauma.

A 27-year-old African American woman comes to the office due to 6 weeks of pain and swelling of her hands and wrists. The pain worsens in the morning and is associated with stiffness for 10-15 minutes. She has also had easy fatigability and muscle aches. The patient has no significant past medical history and does not take any medications. She has been sexually active with a new partner. Her temperature is 37.9 C (100.2 F); blood pressure is 140/90 mm Hg, and pulse is 76/min. Physical examination reveals slightly swollen, tender metacarpophalangeal and proximal interphalangeal joints. There is a nontender ulcer on the buccal mucosa. Mild cervical and axillary lymphadenopathy is present. Cardiopulmonary auscultation is normal The abdomen is soft and nontender. The tip of the spleen is palpable with exhalation. Laboratory results are as follows: An x-ray of the hands and wrists reveals no bony erosions. Urinalysis shows 2+ protein and 20-30 red blood cells/hpf. Which of the following is the most likely diagnosis? A. Behcet disease B. Disseminated gonococcal infection C. Felty syndrome D. Hodgkin disease E. Parvovirus B19 infection F. Sarcoidosis G. Systemic lupus erythematosus

G. Systemic lupus erythematosus This patient's presentation strongly suggests systemic lupus erythematosus (SLE). Young African American women are at greatest risk for developing SLE and fatigue, painless oral ulcers, nondeforming arthritis, renal abnormalities (e.g., hematuria, proteinuria, glomerulonephritis), and cytopenias are amongst the most common manifestations of active disease. Arthritis and arthralgias affect 95% of patients with SLE and tend to be migratory, symmetric, polyarticular, and accompanied by brief morning stiffness (much shorter in duration than rheumatoid arthritis [RA]). The knees, carpal joints, and joints of the fingers are most commonly affected. Joint pain often exceeds the objective findings on examination, and x-rays usually show no evidence of joint destruction or erosion (unlike other inflammatory arthritides such as RA). Educational objective: Systemic lupus erythematosus is a multisystem inflammatory disease that often presents with arthritis and arthralgias. Joint involvement tends to be symmetric, migratory, and nonerosive with brief morning stiffness. The knees, carpal joints, and joints of the fingers are most commonly affected.

A 23-year-old woman comes to the office due to progressive low back pain and stiffness. The pain has been present for the last few years and has worsened somewhat in recent weeks. The patient has relief of the pain with physical activity and over-the-counter ibuprofen. Associated symptoms include morning stiffness that usually lasts an hour, but she. Has had no rash, ocular symptoms, urinary symptoms, or diarrhea. Medical history is notable only for an appendectomy at age 11. The patient has taken birth control pills for the past 2 years and is sexually active with one partner. On examination, she is afebrile with normal height and weight. There is reduced forward flexion of the lumbar spine and tenderness over the lumbosacral area. The remainder of the physical examination is normal. Which of the following would be most likely to establish a diagnosis in this patient? A. Antinuclear antibodies B. Erythrocyte sedimentation rate C. HLA-B27 testing D. Nuclear medicine bone scan E. Prednisone therapeutic trial F. Rheumatoid factor G. X-ray of the sacroiliac joints

G. X-ray of the sacroiliac joints This patient has typical features of ankylosing spondylitis (AS), including chronic, progressive back pain and stiffness; pain relief with activity; lumbosacral tenderness; and reduced spinal range of motion. The incidence peaks in individuals age 20-30. Although AS has a male:female ratio of 2:1, it should be considered in any young patient with progressive low back pain and stiffness lasting >3 months. In young patients with characteristic inflammatory back pain, plain x-rays of the pelvis showing sacroiliitis can confirm the diagnosis of AS. However, x-rays may be negative in early stages; MRI can confirm sacroiliitis in such cases. Fusion of the vertebral bodies with ossification of intervertebral discs (bamboo spine) also suggests the diagnosis. There is a strong association between HLA-B27 and AS, but whereas >90% of patients with AS have HLA-B27, only 5% of patients with HLA-B27 have AS. Therefore, HLA-B27 is not specific for AS and testing for it is not necessary for diagnosis. On the other hand, demonstrating radiographic changes of the axial skeleton is necessary for diagnosis (Choice C). Educational objective: Ankylosing spondylitis (AS) is characterized by chronic inflammatory back pain and stiffness, lumbosacral tenderness, and reduced spinal range of motion. In young patients with characteristic pain, plain x-rays showing sacroiliitis can confirm the diagnosis. AS is strongly associated with HLA-B27, but HLA-B27 is not specific for AS and not necessary for diagnosis.

A 39-year-old man with chronic alcoholism is brought to the hospital after a fall, while intoxicated. He has completely recovered except for a bruise on his shoulder. His only abnormality is a low hemoglobin of 9.6 g/dL, the platelets and WBC are normal. Additional tests including ferritin, vitamin B12, and folate are all normal. Which of the following findings is most likely to be seen on his peripheral blood film? A. macrocytosis B. basophilia C. red cell fragments D. increased platelet adhesiveness E. Atypical lymphocytes

(A) Macrocytosis is frequently seen in patients with chronic alcoholism for multiple reasons including vitamin B12 or folate deficiency, chronic liver disease, and also from the direct toxic effects of alcohol on red cells. Because both iron deficiency and folate deficiency are very common in alcoholics, a dimorphic blood film can also be seen (macrocytes, hypersegmented neutrophils, and hypochromic microcytes can be found) on the same slide.

A 36-year-old woman with sickle cell disease presents with increasing pain in her right hip. She has no fever, chills, back or other bone pain, and there is no history of any trauma. On examination, her temperature is 37.3°C, range of motion in the right hip is reduced, she walks with a limp, and the right leg is shorter than the left. Movements of the hip and walking on it are painful. X-rays of the hip are shown in Figure below. Which of the following is the most likely diagnosis? A. avascular necrosis (AVN) of the femoral head B. osteomyelitis C. hip fracture D. septic arthritis E. multiple myeloma ​

(A) avascular necrosis (AVN) of the femoral head The diagnosis is AVN of the femoral head. Bone complications occur frequently in patients with sickle cell anemia. Infarcted areas of bone and bone marrow can become sites of infection (osteomyelitis) with Staphylococcus aureus or salmonella, but the lack of fever and clinical signs in this patient suggest AVN. Infarction of the femoral head nutrient artery leads to AVN and chronic joint pain. The humeral head is also at risk of AVN. Septic arthritis is a less common complication. ​

A 49-year-old man presents with jaundice, nausea, and vomiting. He has a history of chronic alcoholism, and is currently drinking over 1 bottle of red wine a day. On physical examination, he is jaundiced and pale with a large tender liver. Laboratory data include hemoglobin of 9 g/dL, WBC of 4200/mL, and platelet count of 80,000/mL. His liver enzymes and bilirubin are also elevated. Ultrasound of the abdomen reveals liver enlargement with no bile duct obstruction, a normal size spleen, and no ascites. Which of the following cells are affected by the toxicity of chronic alcoholism on the bone marrow? A. developing erythrocytes and myelocytes B. mature polymorphonuclear leukocytes C. mature red cells D. mature platelets E. eosinophils ​

(A) developing erythrocytes and myelocytes Alcohol is directly toxic to dividing and maturing cells, but may also affect neutrophil function. The hematologic effects of alcohol may be direct or indirect, via diet, infection, liver disease, and GI disease. The resulting hematologic abnormalities may be profound. ​

A 25-year-old woman is pregnant for the first time. At prenatal assessment, she is found to have blood group A and Rh(D)-negative. Her husband is tested and he is Rh(D)-positive. Which of the following is the most appropriate initial test to determine her risk for hemolytic disease of the newborn (erythroblastosis fetalis) due to Rh incompatibility? A. anti-Rh(D) antibody in maternal serum B. direct Coombs test of maternal serum anti-Rh C. antibody in baby's serum D. ABO testing of the husband E. direct Coombs test of neonate serum after birth

(A)anti-Rh(D) antibody in maternal serum Testing the mother for the presence of Rh (D) antibodies (alloimmunization) is done by indirect Coombs test of the maternal serum with known Rh(D) positive cells. Prevention of Rh immunization by the administration of Rh antibody (via Rh immune globulin) has been an effective preventive measure for this disorder. This condition is usually associated with a positive Coombs test using the baby's RBCs and a positive indirect Coombs test using the mother's serum. ABO hemolytic disease of the newborn is clinically mild because the antigens are not fully expressed in utero.

An 82-year-old woman is brought to the hospital because of functional decline at home and an inability to care for herself. She has a prior history of hypertension and dyslipidemia, and her medications include hydrochlorothiazide and atorvastatin. She looks disheveled, pale, and has muscle wasting; her heart and lungs are clear and there are no focal neurologic findings. Her hemoglobin is 9 g/dL, MCV 105 fL, WBC 4500/mL, and platelets 100,000/mL. The blood film shows macrocytic RBCs and hyper-segmented neutrophils. Which of the following vitamin deficiencies is most likely responsible for her pancytopenia? A. B12 (cyanocobalamin) B. Folate C. pyridoxine D. Thiamine E. Riboflavin

(B) folate The macrocytosis and hypersegmented neutrophils suggests a megaloblastic anemia due to either folate or vitamin B12 deficiency. Folate deficiency results from decreased intake and malabsorption. Body folate stores are meager (only 3 months on average, compared to a few years for vitamin B12), and therefore easily depleted when intake is poor. Alcohol itself can depress folate levels acutely, and can also cause pancytopenia directly.

A 63-year-old man is involved in a motor vehicle accident and is brought to the hospital. On examination, his blood pressure is 90/60 mm Hg, pulse 110/min, and his abdomen is distended and rigid. He has internal injuries and bleeding on an abdominal CT scan, and requires emergent blood transfusion on his way to the operating room. His blood group is AB. Which of the following statements is not correct? A. he is a universal recipient B. he has anti-A and anti-B in his serum C. if a cross-match is not available; group O RBCs are universal D. if insufficient AB blood is available, type A can be used E. if insufficient AB blood is available, type B can be used ​

(B) he has anti-A and anti-B in his serum Group AB has neither anti-A nor anti-B antibodies, as both A and B antigens are on the red cells. Persons with type AB blood are considered "universal recipients." It is a rare blood group (only 2%-3% of population); if large amounts of blood are required in an AB individual, any type of blood (A, AB, B, O) can be used as long as the plasma is removed from the donor blood (in other words packed RBCs are used)

A 27-year-old female presents with easy fatigue and light-headedness. She also has a dry cough and fever for the past few days. On examination, she is pale, her lungs are clear, and the rest is normal. A chest x-ray (CXR) shows patchy bilateral infiltrates; the hemoglobin is 8.4 g/dL, reticulocyte count of 6%, and many spherocytes on the peripheral blood film. Which of the following is the most likely significance of the spherocytosis on the blood film? A. multiple long bone fracture B. hereditary elliptocytosis C. Coombs-positive hemolytic anemia D. glucose-6-phosphate dehydrogenase (G6PD) deficiency E. Leukemia

(C) Coombs-positive hemolytic anemia Spherocytes are seen on the blood film in patients with moderate to severe hemolytic anemia. This patient has autoimmune hemolytic anemia due to cold agglutinins due to the pneumonia. Spherocytosis is not associated with G6PD deficiency, trauma, or leukemia. The typical changes in G6PD deficiency include the presence of Heinz bodies. The cell morphology is not usually changed unless hemolysis is very severe. In hereditary spherocytosis, the Coombs test is negative.

A 19-year-old man is found to have low hemoglobin on routine screening. He feels well at the present time, but in the past, he has had two presentations to the hospital for severe abdominal and back pain that resolved on its own with no specific diagnosis. His blood film is shown in Figure below; the hemoglobin is 10 mg/dL, mean corpuscular volume (MCV) 80 fL, and reticulocyte count 0.04 (4%). Which of the following is the most likely explanation for his previous episodes of abdominal pain? A. aplastic crisis B. sequestration crisis C. vaso-occlusive crisis D. hemolytic crisis E. adrenal crisis ​

(C) vaso-occlusive crisis The painful crisis seen in patients with sickle cell disease is due to vaso-occlusive disease where microvascular ischemic injury occurs to tissues. Acute illness, dehydration, and infections are common precipitants for this type of crisis. Because sickle cell anemia is a chronic hemolytic anemia, the reticulocyte is chronically elevated, except in aplastic crises, and erythrocyte life span is shortened. Infection is the most common precipitant of an aplastic crisis, particularly those caused by parvovirus B19. Sequestration crisis (hemoglobin level <6 mg/dL) is sudden massive pooling of red cells in the spleen resulting in hemodynamic instability. Hemolytic crisis is the sudden further reduction in red-cell life span marked by falling hemoglobin, jaundice, and increased reticulocyte count. ​

45-year-old woman develops symptoms of shortness of breath on exertion, easy fatigue, and jaundice. On examination she is pale, and there is a palpable spleen, but no lymphadenopathy. Her hemoglobin is 9.0 g/dL, the reticulocyte count 9%, and the direct antibody test (Coombs) is positive. Which of the following bone marrow findings is most likely to be seen in this patient? A. megaloblastic changes B. giant metamyelocytes C. Increased erythroid-to-myeloid ratio D. Increased lymphocytes E. shift to left of the myeloid series

(C)increased erythroid-to-myeloid ratio Bone marrow examination is most likely to show increased erythroid-to-myeloid ratio. Erythroid hyperplasia is common to all hemolytic anemias, and megaloblastic features only develop if they become folate deficient. A left shift only occurs if the bone marrow is under stress, like during a severe infection, and giant metamyelocytes or increased lymphocytes are not features of an autoimmune hemolytic anemia. ​

A 49-year-old man is involved in a motor vehicle accident, resulting in large amounts of blood loss and hypotension. He is initially given normal saline at the accident site, and on arrival at the hospital, the trauma team orders an emergent blood transfusion with type O "universal donor" packed red cells. Which of the following is a possible complication of unmatched type O blood? A. type O donors have a higher incidence of hepatitis C virus B. type O donors have a shorter survival time when transfused than do other cell types C. at times type B blood may be mistyped as type O D. type O donors may have high titers of anti-A and anti-B in their plasma E. conversion of the recipient to type O blood ​

(D) type O donors may have high titers of anti-A and anti-B in their plasma Anti-A and anti-B in the donor plasma are usually absorbed in the recipient's tissues but have the capacity to harm recipient cells. Type O blood donors do not have a higher risk of hepatitis C, nor do type O red cells have a shorter survival time when transfused into non-type O recipients. ​

A 68-year-old man with aplastic anemia is due for another blood transfusion. He has had multiple blood transfusions in the past, but with the last transfusion, he developed fever and chills after the transfusion was started. Repeat cross-matching and testing at the time ruled out an acute hemolytic transfusion reaction. The next transfusion is ordered through a "filter" to prevent or minimize the febrile reaction. Which of the following mechanisms is most likely to explain the effect of the filter? A. reducing bacterial transmission B. reducing viral transmission C. reducing parasite transmission D. reducing leukocyte transfusion E. reducing reticulocyte transfusion ​

(D)reducing leukocyte transfusion Febrile reactions to donor leukocytes may be severe and cause hypotension, especially in repeatedly transfused patients. Antibodies to platelets can also develop. Usually, at least seven transfusions are required to induce sensitization. Occasionally, blood products can be contaminated with bacteria during the blood collection process and result in host infection but this is less likely than febrile reactions to donor leukocytes. Reducing viral and parasitic transmission associated with blood transfusions is an on-going activity and current blood procurement and screening processes make the risk of transfusion associated infections low. ​

A 50-year-old White woman presents with a 3-week history of tiredness and pallor. A family member has noted some yellowness of her eyes, but she denies darkening of the urine. Physical examination reveals only slight jaundice. Laboratory data include hemoglobin of 9 g/dL, reticulocyte count of 8%, a bilirubin in the serum of 2 mg/dL (indirect reacting), and some microspherocytes on peripheral smear. The direct antiglobulin test (Coombs) is positive. Which of the following is the most likely cause for her anemia? A. blood loss externally B. decreased red cell production C. Ineffective erythropoiesis D. Intravascular hemolysis E. extravascular hemolysis

(E) extravascular hemolysis This patient has autoimmune hemolysis and the red cell destruction is extravascular, usually occurring in the liver, spleen, or other reticuloendothelial sites. It liberates unconjugated bilirubin causing jaundice. Intrinsic causes of hemolytic anemia are usually inherited, and are a result of abnormalities of membranes, red cell enzymes, globins, or heme. Extrinsic hemolysis is a result of mechanical forces, chemicals or microorganisms, antibodies, or sequestration in the monocyte-macrophage system.

A 21-year-old woman is suspected of having mycoplasma pneumonia based on symptoms of a dry cough, fever, normal lung examination but abnormal chest x-ray with bilateral infiltrates. She is also anemic with hemoglobin of 10.5 g/dL, reticulocyte count 7%, and WBC 12,000/mL. Hemolytic anemia from cold agglutinins is suspected as the cause. Which of the following tests will confirm an autoimmune cause of the anemia? A. positive antinuclear antibody (ANA) B. positive rheumatoid factor C. polyclonal gammopathy D. presence of Heinz bodies E. positive Coombs test ​

(E) positive Coombs test The Coombs test is an excellent screen for autoimmune hemolytic anemia. The diagnosis of autoimmune hemolytic anemia requires demonstration of immunoglobulin and/or complement on the patient's RBC. The other tests are not helpful in making the diagnosis of autoimmune hemolytic anemia. ​

A 60 year old man undergoes partial gastrectomy and has a Billroth II anastomosis (gastro-jejunostomy}. Two months after the procedure he develops severe abdominal pain and non-bilious vomiting. Eventually he has a bilious emesis and the pain is relieved. Which of the following tests is most diagnostic of this patient's symptoms: A. Abdominal CT B. EGD C. Angiography D. HIDA scan ​

A. Abdominal CT Based on the Sx's, this patient is likely suffering from afferent loop obstruction. The best test is an abdominal CT scan (shows dilated fluid filled limb that does not fill with contrast). EGD balloon dilatation can be used to relieve the obstruction, with surgery if that fails.

A 53-year-old woman has had nausea, vomiting, and mid-epigastric pain for 5 months. On physical examination, there are no significant findings. An abdominal CT scan shows gastric outlet obstruction. Upper gastrointestinal endoscopy shows an ulcerated 2 x 4 cm bulky mass in the antrum at the pylorus. A urease test is positive. Which of the following neoplasms is most likely to be seen in a biopsy specimen of this mass? A. Adenocarcinoma B. Leiomyosarcoma C. Neuroendocrine carcinoma D. Non-Hodgkin lymphoma E. Squamous cell carcinoma ​

A. Adenocarcinoma The most likely cause of a large mass lesion in the stomach is a gastric carcinoma, and this lesion is an adenocarcinoma, likely the intestinal type found in the antral region. Adenocarcinoma is related to Helicobacter pylori infection, with p-catenin mutation. The incidence of this type of gastric cancer has been decreasing for decades in places where food processing methods have improved. Malignant lymphomas and leiomyosarcomas are less common and tend to form bulky masses in the fundus. Neuroendocrine carcinomas are rare. Squamous cell carcinomas typically appear in the esophagus.

A 55 year man develops severe epigastric pain. CT scan shows free air. You explore him and find a perforated gastric ulcer. The most appropriate next step is: A. Antrectomy including the ulcer and vagotomy B. Whipple C. Ligaton of GDA D. Graham patch E. Stent ​

A. Antrectomy including the ulcer and vagotomy. You should resect gastric ulcers when you have to operate as there is a high likelihood that this actually represents gastric CA.

A 38-year-old patient is being investigated for his anaemia. He has had a normal upper and lower GI endoscopy and renal function resting is unremarkable. A bone marrow biopsy shows a reduction of haematopoietic cells and a trephine biopsy shows mainly fatty bone marrow. Which one of the following is the most likely diagnosis? A. Aplasric anaemia B. Chronic myeloid leukaemia C. Erythropoietin deficiency D. Folate deficiency E. Vitamin 812 deficiency

A. Aplastic anaemia This man has a bone marrow biopsy which shows a reduction in haematopoietic cells. Assessment of bone marrow cellularity is best made on trephine biopsy, which often shows replacement of the normal cellular marrow by fatty marrow.

An 80-year-old women regularly attends Hematology Outpatients for her chronic lymphocytic leukemia (CLL). She has been followed up for 5 years, her condition has remained stable and she has required no treatment. At this clinic attendance, she reports that she has been feeling extremely tired for the last 2 weeks. On examination she is clinically anemic and mildly jaundiced. She has a 2-cm splenomegaly. There are no other abnormal physical findings. Her Hb 6.3 g/ dl, MCV 106 fl, MCH 29 pg, MCHC 32 g/ dl, reticulocytes 205 x 109/l (normal range 50-100), wee 85 x 109/ litre, with lymphocytes of 65.5 x 109/litre and platelets 110 x 109/litre. Which one of the following is the main cause of her anemia? A. Autoimmune haemolytic anaemia B. Progression of chronic lymphocytic leukaemia C. Iron deficiency due to blood loss D. Richter's transformation of chronic lymphocytic leukaemia E. Acute folic acid deficiency ​

A. Autoimmune haemolytic anaemia This lady now has a macrocytic anaemia with a raised reticulocyte count. The fact that the reticulocyte count is raised suggests that the bone marrow is working correctly, but that blood is either being lost through blood loss or haemolysis. The fact that she is mildly jaundiced points towards the haemolytic anaemia. CLL has a well-recognized association with autoimmune haemolytic anaemia, as well as autoimmune thrombocytopenia. ​

A 24-year-old Kenyan student presents to the Emergency Department with fevers and rigors. On examination he is found to have massive splenomegaly. He admits to having had malaria in the past and to having taken prophylaxis intermittently. The laboratory staff reports infection with Plasmodium malariae after examining the blood film. Antimalarial treatment is commenced but 2 days later he is acutely unwell. The hemoglobin is 4.6 g/ 1, reticulocytes 40%, creatinine 450 mmol/1 and there is oliguria. What is the likely diagnosis? A. Blackwater fever B. Drug-induced haemolysis C. G6PD deficiency D. Sickle cell disorder E. Tropical splenomegaly syndrome

A. Blackwater fever This man has become very unwell after treatment for malaria has started. He has become acutely anemic. The raised reticulocyte count suggests either bleeding or hemolysis. There is evidence of acute renal impairment. This is consistent with the development of blackwater fever. This is a rare complication consisting of intravascular hemolysis that occurs in the acute phase of malaria treatment. This can cause intense jaundice, and can be associated with renal shutdown.

A 55 year old man with hematemesis has a bleeding posterior duodenal ulcer on EGD. The most appropriate next step is: EGD therapy Embolization PPI only GDA ligation Graham patch ​

A. EGD therapy EGD Tx with hemo-clips. Epinephrine injection and cautery can also be used.

A 72-year-old woman comes to the office due to persistent headaches, fatigue, and diffuse muscular pain. She has a history of hypertension and osteoarthritis. The patient does not use tobacco, alcohol, or illicit drugs. Her father had a subarachnoid hemorrhage from a ruptured berry aneurysm. She is treated with prednisone, leading to marked and rapid improvement in her symptoms. Which of the following pathologic processes is most likely responsible for this patient's condition? A. Granulomatous inflammation of the media B. Homogenous acellular thickening of arteriolar walls C. Medial band-like calcifications D. Onion-like concentric thickening of arteriolar walls E Transmural inflammation with fibrinoid necrosis

A. Granulomatous inflammation of the media This patient, an elderly woman with headaches, muscular pain, and a rapid response to glucocorticoid therapy, has typical features of giant cell arteritis (GCA). GCA is the most common form of vasculitis in persons of northern European descent and occurs almost exclusively in patients age >50. About half of patients with GCA will also have polymyalgia rheumatica, which causes achy pain in the shoulder and hip girdles. GCA also confers a higher risk of thoracic aortic aneurysms, but not berry aneurysms (her father's history is unlikely related to her presentation). GCA is characterized by a T-cell-mediated inflammatory process of medium-to-large arteries. It may occur diffusely but predominantly affects the arteries of the head and neck, especially the temporal artery (i.e., temporal arteritis). Biopsy specimens of the temporal artery will show scattered focal granulomatous inflammation centered on the media with intimal thickening, elastic lamina fragmentation, and giant cell formation (without distinct granulomas). GCA is histologically identical to Takayasu arteritis, which typically involves the aortic arch and affects primarily younger patients. Educational objective: Giant cell arteritis is characterized by granulomatous inflammation of the media and predominantly involves medium to smaller branches of the carotid artery, especially the temporal artery. The resulting symptoms respond promptly to glucocorticoid therapy.

A patient underwent an 80-cm ileum resection for Crohn's disease 4 years ago; he now presents with anaemia. His haemoglobin is 6.8 g/litre and MCH is 38 pg. Which one of the following is the most likely cause? A. Impaired vitamin B12 absorption B. Impaired iron absorption C. Chronic bleeding after surgery D. Haemolysis E. Bacterial infection

A. Impaired vitamin B12 absorption He has had an ileal resection for Crohn's disease and now is anaemic. B12 is absorbed in the terminal ilium, and this is a well- recognised problem when the ilium has been resected.

A 60-year-old man comes to the physician complaining of fatigue and muscle weakness in his extremities. He has also lost 7 kg (15 lb) during the last 3 months. He has no other medical conditions. The patient smokes 2 packs of cigarettes daily and consumes alcohol occasionally. His muscle strength is 3/5 in the proximal muscle groups symmetrically. His reflexes are 2+ bilaterally. No sensory abnormality is present. There are erythematous to violaceous papules involving the dorsum of his fingers. Chest x-ray reveals an ill-defined mass in the right lower lobe. His muscle weakness is most likely due to a lesion involving which of the following? A. Muscle fibers B. Peripheral nerves C. Postsynaptic membrane D. Presynaptic membrane E. Spinal cord

A. Muscle fibers This patient's smoking history, weight loss, and right lower lobe mass are concerning for lung cancer. Patients with suspected malignancy can develop a paraneoplastic syndrome due to the tumor producing hormones and/or cytokines. Paraneoplastic syndromes are not directly related to the cancer's invasion. They are most commonly seen in cancers of the lung, breast, and ovaries, and lymphomas. Symptoms typically resemble those of primary endocrine, metabolic, hematologic, or neuromuscular disorders. Paraneoplastic syndromes affecting the peripheral nerve and/or muscle (e.g., myasthenia gravis, Lambert-Eaton syndrome, dermatomyositis) most commonly present with muscle weakness, as seen in this patient. Dermatomyositis is an idiopathic inflammatory myopathy with immune-mediated muscle injury. Dermatomyositis patients typically present with symmetrical proximal muscle weakness and erythematous rash on the dorsum of the fingers (Gottron's sign) and/or upper eyelids (heliotrope eruption). Diagnosis is usually confirmed by muscle biopsy. Educational objective: Dermatomyositis is an idiopathic inflammatory myopathy with immune-mediated muscle injury that can be due to a paraneoplastic syndrome in malignancy. Patients typically have symmetrical proximal muscle weakness and erythematous rash over the dorsum of the fingers (Gottron's sign) and/or upper eyelids (heliotrope eruption).

A 79-year-old male with chronic back pain and chronic obstructive pulmonary disease (COPD) requiring supplemental oxygen presents to the emergency department (ED) with epigastric abdominal pain that started suddenly 2 days ago. His abdominal examination is significant for epigastric tenderness but is otherwise unremarkable. A computed tomography (CT) scan demonstrates a small amount of free air under the right hemidiaphragm but no contrast extravasation. An upper gastrointestinal (GI) water-soluble contrast study demonstrates a duodenal ulcer but no extravasation. Which of the following is the best management? A. Nasogastric tube decompression, intravenous (IV) antibiotics, and proton pump inhibitor (PPI) B. Exploratory laparotomy C. Diagnostic laparoscopy D. Oral antibiotics, clear liquid diet for 2 weeks, and follow-up in clinic E. Serial abdominal exam in the ED for 6 to 8 hours and if improving, he may be discharged with oral antibiotics ​

A. Nasogastric tube decompression, intravenous (IV) antibiotics, and proton pump inhibitor (PPI) Nonoperative management for perforated peptic ulcer disease is gaining popularity and is now accepted as an appropriate first-line management for poor surgical candidates (e.g., COPD using home oxygen) who are stable, have no evidence of peritonitis, and have no contrast extravasation. Conservative management is also more favorable if the duration of symptoms has been present for more than 24 hours. By this time the perforation has typically been sealed. Self-sealing of the perforation is achieved by either adhesion formation to the caudate lobe, the greater omentum, the gallbladder, or the falciform ligament. In one study, only 3 out of 109 patients managed nonoperatively developed an intra-abdominal abscess (which can be managed with antibiotics and percutaneous drainage). This may speak to the intrinsic immune function of the omentum and the fact that the upper GI tract has a low bacterial load. Eighty percent of nonoperative cases respond favorably, and morbidity is not significantly increased. Patients deemed appropriate candidates for nonoperative management should be admitted, placed NPO (nothing by mouth), and given IV fluid resuscitation, IV antibiotics covering gram-negative and an-aerobic organisms, and PPIs. Nasogastric tube insertion is critical to help decompress the stomach and allow the perforation to heal. CT scan may be considered for patients who fail to improve or those who deteriorate clinically. Surgery is the next step for patients who fail conservative management (B, C). Outpatient follow-up is not appropriate because nonoperative management should be performed in a monitored setting with frequent abdominal exams and follow-up esophagogastroduodenoscopy (EGD) to ensure that the perforation has sealed (D, E).

Which of the following is considered to be a risk factor for gastric cancer? A. Pernicious anemia B. Blood group O C. Carbonated acidic soda D. Female gender E. Diabetes ​

A. Pernicious anemia Risk factors for gastric cancer include dietary factors such as a large consumption of smoked meats, pickled foods, high nitrates, and high salt, whereas a diet high in fruits and vegetables may be protective (D, E). Other risk factors include smoking, low socioeconomic status, black race, H. pylori infection, chronic atrophic gastritis, blood type A, previous partial gastrectomy, achlorhydria, pernicious anemia, polyps (adenomatous and hyperplastic), male gender, and certain familial syndromes such as hereditary nonpolyposis colorectal cancer, Li-Fraumeni syndrome, familial adenomatous polyposis, and Peutz-Jeghers syndrome (B, D). Peutz- Jeghers syndrome is associated with a markedly increased risk of cancer in the esophagus, stomach, small bowel, colon, pancreas, breast, lung, uterus, and ovary, with a cumulative 93% risk of cancer. Carbonated acidic soda has not been shown to increase the risk for cancer (C). Gastric cancer has been categorized by Lauren into intestinal and diffuse types based on histology. The intestinal type is thought to be more related to environmental factors, is associated with chronic gastritis, and is well differentiated. The diffuse type is usually poorly differentiated and associated with signet rings and occurs in younger patients and in association with familial disorders and also with type A blood. The diffuse type has a worse prognosis.

A 75-year-old woman attends her follow-up at the Haematology Clinic. She was initially referred by her GP with anaemia but currently has no symptoms. She had pulmonary TB 15 years ago, which was treated with a 6-month conventional course of chemotherapy. This patient has a long history of alcohol dependence but has not drunk for 8 years. Her family has tried to get her to move out of the old family house due to its poor state of repair. Routine blood tests show Hb 9.3 g/ dl, MCV 83 fl, RDW (red cell distribution width) 23%, bilirubin 49 µmol/1itre, LDH 537 U/ litre with normal renal and liver function. The only significant finding on her bone marrow trephine is an accumulation of iron around the nuclei of developing erythrocytes. What is the likely diagnosis? A. Lsoniazid effect B. Megaloblastic anaemia C. Myelodysplastic syndrome D. Lead poisoning E. Congenital sideroblastic anaemia ​

D. Lead poisoning This can cause a normocytic anaemia with sideroblastic change. This is particularly associated with old houses that still have lead water pipes.

An 18-year-old man is seen in the clinic for routine assessment. He reports no active symptoms, there is no past medical history and he is not taking any medications. He mentions that his brother has a "low blood count" that is hereditary but does not recall the name of the disorder. His physical examination is entirely normal. A complete blood count reveals a hemoglobin of 10.5 g/dL and microcytic red cells on the blood film. A hemoglobin electrophoresis confirms the diagnosis of beta-thalassemia minor. Which of the following findings is characteristic of this condition? A. an increased amount of fetal hemoglobin (HbF) or hemoglobin A2 (HbA2) B. Increased osmotic fragility of the red cells C. absent bone marrow iron D. Increased macroglobulins in the serum E. Small amounts of HbS

A. an increased amount of fetal hemoglobin (HbF) or hemoglobin A2 (HbA2) An increased amount of HbF or HbA2 is present in patients with beta-thalassemia. As the hemoglobin beta-chains are decreased in beta-thalassemia, the excess alpha-chains combine with gamma-and delta-chains to make HbF and HbA2, respectively. Increased osmotic fragility of red cells is a feature hereditary spherocytosis and not thalassemia. Most patients with thalassemia have normal quantities of bone marrow iron, and HbS is not a feature of thalassemia but of sickle cell disease.

A 38-year-old woman is prescribed 3 units of blood after heavy PV loss following the delivery of her fifth child. Twenty minutes into the first unit of blood she spikes a temperature of 39 oC and suffers rigors. Which one of the following is the most likely cause? A. A delayed haemolytic transfusion reaction caused by red cell antibodies in the patient reacting against red cells in the unit of blood B. A reaction between white cell antibodies in the patient and white cells in the unit of blood producing a febrile non-haemolytic transfusion reaction C. Anaphylaxis resulting from a reaction between anti-lgA antibody in the patient, who is lgA deficient, and lgA in the unit of blood D. An immediate haemolytic transfusion reaction caused by administration of an A Rh-positive unit of blood to an O Rh-positive patient E. Septicaemia resulting from transfusion of a unit of blood contaminated by bacteria

B. A reaction between white cell antibodies in the patient and white cells in the unit of blood producing a febrile non-haemolytic transfusion reaction A reaction between white cell antibodies in the patient and white cells in the unit of blood producing a febrile non-haemolytic transfusion reaction is the most likely cause of the presentation. Please note that now all white cells are removed from blood products, which has been mandatory in the UK since 1999 as a measure to potentially reduce the risk of variant CJD transmission by transfusion.

A 52-year-old woman presents with recurrent sinusitis and a chronic cough. In the past week, she became very concerned after she developed hemoptysis. Upon review, she affirms fatigue and generalized arthralgia. Physical examination is remarkable for a saddle nose deformity. A CBC reveals mild leukocytosis and normocytic anemia. Urinalysis shows microhematuria, and a subsequent kidney biopsy demonstrates granulomatous inflammation within the arterial walls. What is the most appropriate diagnosis based on this patient's constellation of findings? A. Churg-Strauss vasculitis B. Granulomatosis with polyangiitis C. Microscopic polyangiitis D. Polyarteritis nodosa E. Thromboangiitis obliterans ​

B. Granulomatosis with polyangiitis Formerly known as Wegener granulomatosis, granulomatosis with polyangiitis (GPA) is caused by inflammatory infiltration of small and medium-size arteries. The disease tends to affect the sinuses, upper and lower airways, and kidneys. Symptoms may include cough, hemoptysis, pleuritis, and dyspnea. Although only a minority of patients present with glomerulonephritis early on, approximately three-fourths of patients eventually develop this complication. Most patients with GPA are positive for circulating antineutrophil cytoplasmic antibody (C-ANCA). Initial treatment is with cyclophosphamide. Bottom Line: Patients with granulomatosis with polyangiitis (GPA) suffer from upper airway symptoms and rapidly progressing crescentic glomerulonephritis. Most patients test positive for C-ANCA.

A 38-year-old man presents with an acute anaemia. Hb 7.2 g/litre, haptoglobin< 0.0 5 g/ litre, reticulocytes 320 x 109/litre (reference 25 x 109/litre to 85 x 109/litre). Which one of the following is the most likely form of the anaemia? A. Iron deficiency anaemia B. Haemolytic anaemia C. Aplastic anaemia D. Pernicious anaemia E. Folic acid deficiency anaemia ​

B. Haemolytic anaemia We have a young man who has an acute anaemia. Reticulocytes are raised. When patients have anaemia with a reticulocytosis, this indicates either bleeding or haemolysis. A raised bilirubin level will point towards haernolysis, whereas a normal bilirubin would point to bleeding. In this situation, we have been told that there is a low haptoglobin level. Haptoglobin is a protein that is found normally in plasma that binds free haemoglobin. The haptoglobin-haemoglobin complex will then be removed by the spleen. Therefore, low haptoglobin levels will signify intravascular haemolytic anaemia. ​

A 22-year-old patient who is a vegan and very active has been complaining of tiredness and shortness of breath for the last 2 months. On examination she looks anaemic. Which one of the following is the most likely vitamin deficiency causing her anaemia? A. Vitamin A B. Vitamin Bl C. Vitamin B6 D. Vitamin B12 E. Vitamin C ​

D. Vitamin B12 Patients who are vegans can become vitamin 812 deficient because of nutritional deficiency. This really does only occur, however, in true vegans and takes many years to develop. This is because of the extensive 812 stores the body normally maintains. ​

A 68-year-old woman comes to the emergency department in acute distress. She complains of the sudden onset of complete vision loss in her right eye. Her vision had been blurry for the past few days and acutely worsened an hour ago. The patient also describes 2 months of right-sided headaches for which she took ibuprofen without much relief. She has no nausea or vomiting but does have malaise and fatigue. Her medical problems include diet-controlled type 2 diabetes mellitus, hypertension, and degenerative joint disease. Her temperature is 37.2 C (98.9 F), blood pressure is 146/86 mm Hg: and heart rate is 78/min. Examination shows complete loss of vision in the right eye. Pupils are 4 mm bilaterally. Fundoscopy shows a swollen pale disc with blurred margins. The heartbeat is regular, and a bruit is heard in the right sub-clavicular area. Motor and sensory examination is within normal limits. Erythrocyte sedimentation rate is 85 mm/h. Which of the following is the most appropriate next step in management of this patient? A. High-dose aacetazolamide B. High-dose intravenous methylprednisolone C. Low-dose oral prednisone D. Methotrexate therapy E. MRI of the brain F. Temporal artery biopsy

B. High-dose intravenous methylprednisolone This patient's clinical presentation, with headaches, sudden loss of vision, abnormal fundoscopic findings, and elevated erythrocyte sedimentation rate, is suggestive of giant cell arteritis (GCA). GCA (also referred to as temporal arteritis) is a chronic vasculitis affecting the medium and large vessels and is characterized by a variety of localized, systemic, and ocular manifestations. Physical examination may show diminished pulses and/or bruits on auscultation of the axillary, brachial or femora! arteries and over the supraclavicular and/or carotid areas. Anterior ischemic optic neuropathy is the most common ocular manifestation and is detected on fundoscopy by the presence of a swollen and pale disc with blurred margins. Visual syndromes with vision loss are the most dreaded complication of GCA, and blindness can develop in up to 25%-50% of untreated patients. In patients with suspected GCA and visual loss, the most appropriate next step is to initiate therapy with high-dose intravenous corticosteroids, followed by oral therapy with a slow taper over several months. Immediate therapy with systemic corticosteroids reduces the progression of visual complications in the affected and unaffected eyes: it should be started promptly while awaiting confirmation of the diagnosis. Educational objective: Giant cell arteritis (temporal arteritis) should be suspected in patients with temporal headaches, jaw claudication, and vision loss. Patients with suspected giant cell arteritis who have visual symptoms should be started immediately on high-dose systemic glucocorticoids to reduce the progression of visual complications.

A 61-year-old man is evaluated for a 10-month history of generalized weakness. He reports no pain or myalgia. History is significant for hypercholesterolemia treated with a stable dose of simvastatin for the past 3 years. On physical examination, temperature is normal, blood pressure is 138/74 mm Hg, pulse rate is 70/min, and respiration rate is 16/min. BMI is 27. There is symmetric weakness of the arm and thigh muscles with slightly reduced grip and power of the finger flexors. No muscle tenderness is noted. There is no rash, skin thickening, or digital ulcers. Reflexes and the remainder of the physical examination are normal. Laboratory studies are notable for a normal complete blood count, an erythrocyte sedimentation rate of 23 mm/h, and a serum creatine kinase level of 365 U/L. Chest radiograph is normal. Electromyogram and nerve conduction studies show myopathic changes in the proximal and distal muscles of the extremities as well as some neurogenic changes. Which of the following is the most likely diagnosis? A. Amyotrophic lateral sclerosis B. Inclusion body myositis C. Myasthenia gravis D. Statin-induced myopathy

B. Inclusion body myositis Educational Objective: Diagnose inclusion body myositis. Inclusion body myositis has an insidious onset, with muscle weakness that may be diffuse and involve both the distal and proximal muscles. The most likely diagnosis is inclusion body myositis (IBM), an insidious and slowly progressive inflammatory myopathy that occurs more commonly in men and in those over the age of 50 years. Muscle weakness may be diffuse and involve both the distal and proximal muscles. Although typically symmetric, IBM muscle involvement may be asymmetric in up to 15% of patients. Skin is generally spared. IBM is rarely associated with extra-muscular manifestations such as rash, fever, or pulmonary involvement. Patients with IBM typically have only mildly elevated (typically <1000 U/L), or even normal, levels of muscle enzymes. The characteristic triad of electromyographic findings for myopathy includes short-duration, small, low-amplitude polyphasic potentials; fibrillation potentials at rest; and bizarre, high-frequency, repetitive discharges. This older male patient has developed slowly progressive weakness affecting both the proximal and distal muscles without any significant pain or stiffness. This presentation suggests a myopathy with weakness based on his history and physical examination, mild elevation of muscle enzymes, and abnormal electromyogram (EMG) results, all of which are most consistent with IBM.

A 35-year-old woman presents to her primary care physician with complaints of progressive weakness over the past month. She says she has noticed an increasing amount of difficulty associated with getting out of chairs, climbing stairs, brushing her hair, and lifting her 3-year-old son out of his playpen, an activity that she was able to do without difficulty prior to the onset of her weakness. She admits to missing 3 days of work approximately 6 weeks ago due to "a bad cold." Physical examination is remarkable for symmetric proximal muscle weakness. Several tests are ordered to establish a diagnosis. Which of the following most strongly supports a diagnosis of polymyositis? A. β-Amyloid deposits noted on muscle biopsy B. Increased creatine kinase levels C. Normal creatine kinase levels D. Perifascicular atrophy noted on muscle biopsy E. Positive rheumatoid factor ​

B. Increased creatine kinase levels Polymyositis, dermatomyositis, and inclusion body myositis are all idiopathic inflammatory arthropathies. Although the etiology behind these disorders is largely unknown, they typically present with progressive bilateral muscle weakness. Diagnostic distinction between the three entities is made on muscle biopsy. In polymyositis, a biopsy would show muscle fiber necrosis, degeneration, regeneration, and an inflammatory cell infiltrate consisting of CD8+ T cells. In addition, creatine kinase levels may provide further distinction between the disorders. Polymyositis is always associated with increased creatine kinase (CK) levels. CK levels in dermatomyositis and inclusion body myositis can be normal, and therefore cannot be used to rule out these two diseases. Bottom Line: Polymyositis is always associated with increased creatine kinase (CK) levels. ​

An 82-year-old woman is evaluated for a 2-week history of left-sided headaches with pain on chewing, accompanied by achiness in the shoulders and hips. She has no other pertinent personal or family history. She takes no medications. On physical examination, temperature is 38.1 °C (100.6 °F), blood pressure is 132/86 mm Hg, pulse rate is 88/min, and respiration rate is 18/min. BMI is 25. Eye examination is normal. There are tenderness and swelling over the left temporal area. Moderate to severe pain on range of motion of the shoulders and hips is noted. There is no pain over the temporomandibular joints on palpation. Laboratory studies, including basic metabolic panel, complete blood count, and liver chemistries, are normal; erythrocyte sedimentation rate is 85 mm/h. Which of the following is the most appropriate immediate next step in management? A. Initiate prednisone, 15 mg/d B. Initiate prednisone, 60 mg/d C. Obtain MRI of the head D. Obtain temporal artery biopsy ​

B. Initiate prednisone, 60 mg/d Educational Objective: Treat giant cell arteritis with high-dose prednisone. Immediate treatment with prednisone, 60 mg/d (or 1 mg/kg/d), is indicated for patients with suspected giant cell arteritis to prevent visual complications. Treatment with prednisone, 60 mg/d (or 1 mg/kg/d), is indicated immediately for this patient. She has temporal artery pain and tenderness, along with jaw claudication in the setting of low-grade fever and a very high erythrocyte sedimentation rate. Given her age, these findings are most consistent with giant cell arteritis (GCA). The presence of shoulder and hip symptoms is consistent with polymyalgia rheumatica (PMR), which commonly co-occurs in patients with GCA (approximately 50% of cases). Despite a lack of visual symptoms to date, the patient is at risk of acute and potentially catastrophic visual loss. Immediate treatment is therefore warranted, the standard regimen being prednisone at a dose of 60 mg/d. (In the setting of severe visual loss, high-dose pulse glucocorticoids might be considered.) The addition of low-dose aspirin has been shown in limited studies to further reduce the risk of visual loss in patients with GCA already receiving prednisone and is favored by some experts. Low-dose prednisone in the range of 10 to 20 mg/d is generally adequate treatment for isolated PMR but has not been shown to adequately treat GCA or to prevent visual complications.

A 72-year-old woman presents to the emergency department because of headache. She says that the pain, which has been present for the past week, is constant, rates 7 of 10 in severity, and is localized to the left side of her forehead and above her left ear. She notes pain in her jaw when she chews, and she also complains of blurry vision. On review of systems she notes a 4.5-kg (10-lb) weight loss over the past month. She denies chest pain, shortness of breath, abdominal pain, change in bowel movements, or weakness or numbness in her extremities. She has a temperature of 37.2°C (99.0°F), heart rate of 86/min, and blood pressure of 116/78 mm Hg. She has tenderness to palpation of the left temporal area. On neurologic examination cranial nerves II through XII are intact, she has normal sensation to light touch and vibration bilaterally, and strength is 5/5 in all four extremities. Laboratory tests show: WBC count: 9S00/mm3 Hemoglobin: 10.1 g/dL Platelet count: 560,000/mm3 Erythrocyte sedimentation rate: 110 mm/hr. C-reactive protein: 3.2 ng/mL (normal 0.7-1.89 ng/mL) Alanine aminotransferase: 38 U/L Aspartate aminotransferase: 62 U/L Alkaline phosphatase: 410 U/L Which of the following is the best first step in management? A. Antineutrophil cytoplasmic antibody titer B. Initiation of glucocorticoid therapy C. Initiation of therapy with nonsteroidal anti-inflammatory drugs D. Magnetic resonance angiography of the brain E. Temporal artery biopsy

B. Initiation of glucocorticoid therapy This patient presents with signs and symptoms suggestive of temporal (or giant cell) arteritis, a chronic vasculitis or large- and medium-sized vessels. Clinical presentation often includes headache that is unilateral and localized to the temporal or frontal regions. The headache of giant cell arteritis has no pathognomonic features, but typically the headache is either new in a patient without a history of headaches or of a new type in a patient with a history of chronic headaches. The temporal artery may appear enlarged and tender to palpation. A bruit may be auscultated over the temporal or carotid artery. Patients may complain of jaw or arm claudication and visual changes. Without appropriate therapy, temporal arteritis can lead to ischemic optic neuropathy and blindness. Patients can also have systemic symptoms including fever, weight loss, and fatigue. A key laboratory finding is an elevated erythrocyte sedimentation rate (ESR), often >100 mm/hr. as seen in this patient. The diagnosis of temporal arteritis should be considered in patients 50 years of age or older presenting with fever, anemia, and elevated ESR. The diagnosis is confirmed by biopsy of the temporal artery as soon as possible. However, initiation of therapy with glucocorticoids should not be delayed while biopsy results are pending, as the risk of permanent vision loss is high. Bottom Line: Temporal arteritis should be considered in any patient 50 years of age or older presenting with fever, anemia, and elevated erythrocyte sedimentation rate. Therapy with glucocorticoids should not be delayed pending biopsy results, as the risk of permanent vision loss is high.

The MC nutritional and metabolic disorder following gastrectomy is: A. Calcium deficiency B. Iron deficiency C. B-12 deficiency D. Folate deficiency E. Fat-soluble vitamin deficiency ​

B. Iron deficiency ​

A 40-year-old man presents with acute weakness and palpitations. He has a history of diarrhoea and weight loss over the past few months and he is opening his bowls up to 6 times per day. Investigations reveal (normal range in brackets): Sodium 143 mmol/1 (157-144) Potassium 5.0 mmol/' (5.5- 4.9) Urea 15 mmol/1 (2.5- 7.5) Creatinine 195 µmol/1 (60-110) Bicarbonate 15 mmol/1 (20- 29) Haemoglobin 9.0 g/d (15.5-18) Mean cell volume (MCV) 122 fl (80- 96) W hite cell count 5.2 x 109/l (4-11) Platelet count 185 x 109/l (150-400) ESR 45 mm/1st hour Anti-endomyseal antibodies positive Which one of the following is the most likely explanation for the clinical picture and blood results seen here? A. Alcoholic liver disease B. Aplastic anaemia C. Folic acid deficiency D. Hypothyroidism E. Vitamin C deficiency ​

C. Folic acid deficiency This man has a macrocytic anaemia with a slightly low white cell count. Anti-endomysial antibodies are positive, indicating coeliac disease. He also has symptoms of malabsorption, namely diarrhoea and weight loss. The renal impairment is because of fluid loss and subsequent dehydration.

A 76-year-old woman is seen in the outpatient clinic for muscle soreness and stiffness. She says she has been having difficulty getting out of chairs and with activities that require lifting her arms above her head secondary to the pain she experiences. Review of systems reveals she has also experienced fever, malaise, and weight loss. On physical examination there is minimal joint swelling, no obvious joint deformity, and no appreciable muscular weakness. Laboratory tests show: Hemoglobin: 9.6 g/dL Erythrocyte sedimentation rate: 85 mm/hr. Creatinine: 0.9 mg/dL Creatine phosphokinase: 56 U/L Which of the following is the most likely diagnosis? A. Fibromyalgia B. Polymyalgia rheumatica C. Polymyositis D. Rheumatoid arthritis E. Systemic sclerosis ​

B. Polymyalgia rheumatica This collection of pain and stiffness in the shoulders and pelvic girdle with fever, malaise, and weight loss is most consistent with polymyalgia rheumatica (PMR), a rheumatologic disorder most commonly seen in elderly women. Anemia and a markedly elevated erythrocyte sedimentation rate are almost always present. Roughly 15% of patients with PMR develop giant cell arteritis, and approximately half of those with giant cell arteritis also have PMR. Treatment for PMR is low-dose prednisone. Steroids generally lead to a complete resolution of symptoms. The cause of PMR is unknown Bottom Line: Polymyalgia rheumatica (PMR) is a rheumatologic disorder characterized by pain and stiffness in the shoulders and pelvic girdle with fever, malaise, and weight loss. Lab tests commonly show anemia and a markedly elevated erythrocyte sedimentation rate. ​

An 18-year-old man, of Italian extraction, is found to have a hypochromic microcytic anemia of 10 g/dL. In addition, there are a fair number of anisocytosis, poikilocytosis, and target cells seen on the blood film. The WBC is 9500/mL, the platelet count is 240,000/mL, and the reticulocyte count is 7%. The spleen is palpated 5 cm below the left costal margin. Which of the following is the most likely diagnosis? A. sickle cell trait B. thalassemia minor C. HbS-C ddisease D. sideroblastic anemia E. hereditary spherocytosis ​

B. thalassemia minor Thalassemia minor usually represents a heterozygous state and is often asymptomatic. Symptoms may develop during periods of stress such as pregnancy or severe infection. Hemoglobin values are usually in the 9-11 g/dL range. The red cells are small and poorly hemoglobinized.

A 72-year-old Caucasian female complains of frequent headaches, fatigue and recent weight loss. Her shoulder muscles feel stiff in the morning. A tender cord is palpated in the right temporal area. Her ESR is 85 mm/hr. Which of the following Is the most likely complication of this patient's condition? A. Chronic lymphocytic leukemia B. Hypothyroidism C. Aortic aneurysm D. Ulcerative colitis E. Membranous glomerulonephritis

C. Aortic aneurysm The patient described in the vignette has typical features of giant cell arteritis. Symptoms include headache, jaw claudication, muscle fatigue, and visual disturbance. On examination, patients have scalp tenderness and a decreased temporal artery pulse. The erythrocyte sedimentation rate (ESR) is generally more than 50 mm/hr. As giant cell arteritis can involve the branches of aorta, an aortic aneurysm is a well-known complication. For this reason, patients should be followed with serial chest x-rays. Educational objective: Aortic aneurysms are well-known complications of giant cell or temporal arteritis due to the involvement of the branches of the aorta. For this reason, patients should be followed with serial chest x-rays.

A 28-year-old woman is evaluated in the emergency department for a 3¬week history of progressively worsening pain in the left arm. The pain worsens with use of the arm. She also notes fatigue, malaise, and the inability to walk long distances due to discomfort in her legs. She reports no cough, nausea, vomiting, or burning on urination. She takes no medications. On physical examination, temperature is 38.1 °C (100.5 °F), blood pressure is 166/95 mm Hg in the right arm and 115/56 mm Hg in the left arm, pulse rate is 72/min, and respiration rate is 14/min. BMI is 27. Pallor of the fingertips and delayed capillary refill of the nail beds are noted in the left hand. A diminished radial pulse of the left arm and decreased dorsalis pedis pulses bilaterally are noted. A bruit is heard over the mid abdomen. There is no rash. Laboratory studies: Erythrocyte sedimentation: rate 115 mm/h Partial thromboplastin time: Normal Prothrombin time: Normal D-dimer: Negative Urinalysis: Normal Which of the following is the most appropriate diagnostic test to perform next? A. Antimyeloperoxidase antibody assay B. Antiphospholipid antibody assay C. Aortic arteriography D. Temporal artery biopsy

C. Aortic arteriography Educational Objective: Diagnose Takayasu arteritis. Arteriography of the aorta and its branches can be used to confirm the diagnosis of Takayasu arteritis. The most appropriate diagnostic test to perform next is aortic arteriography in this patient who most likely has Takayasu arteritis. She presents with arterial compromise in the setting of a systemic febrile illness. Asymmetric blood pressure in the arms suggests arm involvement, and a midabdominal bruit, leg symptoms, and hypertension suggest aortic and renal artery obstruction. The differential diagnosis includes other forms of vasculitis and/or thrombosis. Given her age, sex, and high erythrocyte sedimentation rate, a diagnosis of Takayasu arteritis is likely. Because there are no specific laboratory tests used to diagnose or define Takayasu arteritis, arteriography of the aorta and its branches is used to confirm the diagnosis and define the extent of the problem. Alternative imaging modalities such as CT angiography or MR angiography might also be used for the same purpose. An antimyeloperoxidase antibody assay would be useful if microscopic polyangiitis (MPA) were a diagnostic consideration; however, the kidney disease seen in this patient is due to renal artery obstruction and occurs in the absence of active urine sediment such as would be expected in MPA glomerulonephritis. Moreover, peripheral artery involvement would not be expected in MPA, a small-vessel disease. The presence of antiphospholipid antibodies would be consistent with the antiphospholipid antibody syndrome and with the presence of thrombotic disease potentially occluding the arm, aorta, and renal arteries. However, her normal prothrombin and partial thromboplastin times indicate the absence of a lupus anticoagulant (one criterion for antiphospholipid antibody syndrome), and the lack of elevation in fibrin degradation products (D-dimer) argues against a thrombotic disease. Although Takayasu arteritis and giant cell arteritis (GCA) share remarkably similar pathology, GCA occurs in older patients and is characterized by temporal arteritis, whereas Takayasu arteritis is a disease of the young that rarely, if ever, involves temporal arteries. Thus, a temporal artery biopsy is not indicated.

A 60 year old man undergoes partial gastrectomy and has a Billroth II anastomosis. One year later he develops gastric pain and bilious emesis (which does not relieve the pain). EGD reveals bile in the stomach and biopsy shows gastritis All of the following are true of this condition except: A. The majority of patients with bile reflux do not develop Sx's B. Sx's do not correlate with the amount of bile in the stomach C. Conversion to a Roux-en-Y gastrojejunostomy is a surgical option D. This is more common with a Billroth I compared to Billroth II E. Cholestyramine may help with Sx's ​

C. Conversion to a Roux-en-Y gastrojejunostomy is a surgical option Bile reflux gastritis is more common with a Billroth II compared to Billroth I. Bile reflux gastritis can present late after gastric surgery.

A 60 year old man undergoes partial gastrectomy and has a Billroth II anastomosis. Five days later he develops severe RUQ pain, fever, and hypotension. This is most consistent with: A. Cholecystitis B. Pancreatitis C. Duodenal stump blow-out D. Hepatitis E. Cholangitis ​

C. Duodenal stump blow-out emergent re-exploration, washout, and placement of a lateral duodenostomy tube is indicated for the above patient. ​

A 68-year-old woman comes to the emergency department due to an acute vision disturbance. She had an episode of dimming of vision in the left eye that occurred abruptly and resolved spontaneously in 20 minutes. For the past several weeks, the patient has had a dull ache in the left side of her jaw while chewing that resolves when she stops eating. She has also had malaise and hip muscle aches over the last several months. The patient has a history of hypertension and hypothyroidism. On examination, her blood pressure is 130/70 mm Hg and pulse is 66/min. Neurological examination, including cranial nerves and motor and sensory functions, is unremarkable. Visual acuity, visual fields, and appearance of the ocular fundi are normal. Which of the following is the best initial test for this patient? A. Angiography B. Blood lipid fractions C. Erythrocyte sedimentation rate D. Head CT scan E. Rheumatoid factor F. Transthoracic echocardiography

C. Erythrocyte sedimentation rate This patient, an elderly woman with jaw claudication and an episode of amaurosis fugax (i.e.. transient monocular visual loss), most likely has giant cell (temporal) arteritis (GCA). GCA is the most common form of vasculitis in persons of northern European descent and occurs almost exclusively in patients age >50 About half of patients with GCA will also have polymyalgia rheumatica. which causes achy pain in the shoulder and hip girdles. Headache is the most common presentation of GCA, but jaw or tongue claudication is also common. Ocular manifestations may include amaurosis, diplopia, blurred vision, and ischemic optic neuropathy; untreated GCA may lead to permanent blindness. Physical findings in GCA may be normal, but patients often have tenderness over the course of the temporal artery. If GCA is suspected, a C- reactive protein (CRP) level or erythrocyte sedimentation rate (ESR) should be determined promptly. Although CRP and ESR are nonspecific, they are highly sensitive and almost always significantly elevated in GCA. Patients with characteristic symptoms and elevated CRP or ESR should undergo temporal artery biopsy for definitive diagnosis. Educational objective: Erythrocyte sedimentation rate (ESR) and C-reactive protein (CRP) have very high sensitivity for giant cell (temporal) arteritis. Patients with suspected giant cell arteritis who have an elevated ESR or CRP level should be referred for temporal artery biopsy to confirm the diagnosis.

Gastric cancer is a leading cause of cancer-related mortality worldwide. Which statement about this malignancy is correct? A. Africa has the highest incidence rates due to the poor diet B. Diet is the most important risk factor C. H. pylori infection is the most recognised acquired risk factor D. The incidence has been steadily rising in most parts of the world in the past 3 decades E. The overall 5-year survival rates are now around 50% ​

C. H. pylori infection is the most recognised acquired risk factor The incidence of gastric cancer has, in fact, been falling steadily across the world, although it is still a global killer. H. pylori infection is the most important risk factor and this far outweighs all the other traditional risk factors, such as diet. H. pylori plays a key pathogenic role and the infection has been classified by the International Agency for Research on Cancer (IARC) as a definite human carcinogen. Despite advances in treatment, the overall 5-year survival is still low (< 30%). Africa has a low incidence of gastric cancer (the so-called African enigma) despite having high rates of H. pylori infection.

Which one of the following is the MOST common cause of aplastic crisis in a patient with sickle cell disease? A. Dehydration B. Respiratory syncytial virus infection C. Human parvovirus B19 infection D. Repeated blood transfusion E. Haemophilus influenzae septicaemia ​

C. Human parvovirus B19 infection Human parvovirus B19 infection is the most common cause of an aplastic crisis in patients with sickle cell disease. This usually occurs more frequently in children than adults. Essentially this causes a sudden halt in the production of red blood cells, with a low reticulocyte count. ​

A 45-year-old woman who has been treated in the past for presently clinically inactive systemic lupus erythematosus (SLE) presents with a recent onset of fatigue, lightheadedness, and yellowness of her eyes. Her hemoglobin is found to be 6.3 g/dL (with a baseline of about 12.5-13.0), reticulocytes are 15%, indirect bilirubin is 2.8 IU/L, and LDH is 840 mg/dL. A peripheral blood smear shows a large number of polychromatic red cells and spherocytes. Her direct antiglobulin test (Coombs test) result is positive for IgG. Her treatment at this time should include A. plasmapheresis. B. rituximab plus danazol. C. intravenous methylprednisolone. D. urgent splenectomy. E. restarting her previous SLE treatment with azathioprine.

C. Intravenous methylprednisolone. This patient has severe warm antibody-mediated autoimmune hemolytic anemia (AIHA), which is probably related to her history of SLE. It appears to be sufficiently acute and severe to require transfusions of packed red blood cells, although the blood bank will likely have difficulty with typing and crossmatching. (The latter problem should not deter giving transfusions in a case like this in consultation with the transfusion medicine specialist on call, often having to resort to transfusing with not completely matched but the "best-matched" units of blood available.) High-dose corticosteroids are the mainstay of initial therapy. Rituximab, danazol, and splenectomy are all second-line treatment options. Plasmapheresis is usually ineffective because at least 50% of an individual's IgG (in this case, IgG autoantibody) is distributed into the extravascular space at any given time. Although azathioprine may have been effective in controlling this patient's other manifestations of SLE in the past, it is not good initial therapy for AIHA.

A 67-year-old man comes to the office with a persistent headache and pain in the jaw when chewing food For the past 2 months, he has been unable to eat "tough foods like steak because the pain makes it take too long to chew them." The patient has a history of hypertension, type 2 diabetes mellitus, and hyperlipidemia. His blood pressure is 130/70 mm Hg and pulse is 76/min and regular. Physical examination is unremarkable. The patient is immediately started on the appropriate therapy, and an arterial biopsy is performed. Histopathology shows multinuclear giant cells and internal elastic membrane fragmentation. Prompt institution of therapy in this patient most likely reduces the risk of which of the following complications? A. Glomerulonephritis B. Hepatic necrosis C. Ischemic optic neuropathy D. Myocardial infarction E. Pulmonary hemorrhage F. Pulmonary hypertension ​

C. Ischemic optic neuropathy This patient, with headache and jaw claudication, has typical symptoms of giant cell (temporal) arteritis, the most common systemic vasculitis in the US adult population. Giant cell arteritis (GCA) occurs almost exclusively in patients age >50 and is characterized by granulomatous inflammation of the media with fragmentation of the internal elastic lamina, primarily in the arteries of the head and neck. GCA can cause severe ischemic optic neuropathy due to ophthalmic artery occlusion, which frequently leads to blindness if not treated promptly. Other optic complications of GCA include amaurosis fugax, central or branch retinal artery occlusion, and cerebral infarction leading to central visual field defects. In light of this, GCA is considered a medical emergency, and physicians should not wait for biopsy results before starting high-dose glucocorticoid therapy. Educational objective: Giant cell arteritis (GCA) is characterized by granulomatous inflammation of the media, with fragmentation of the internal elastic lamina of medium and small branches of the carotid artery. Irreversible blindness is a severe complication of GCA. and patients with suspected GCA require immediate glucocorticoid therapy.

A 69-year-old woman presents to her primary care physician complaining of fatigue and achiness in her shoulders and hips. She says that over the past 2 months she has lost 4.5 kg (10 lb.) and feels tired all the time. Her whole body feels stiff in the morning for at least half an hour. She has trouble getting dressed because it is difficult to put on a shirt or pull up her pantyhose. On physical examination she has decreased active range of motion of her shoulders and hips. She has no muscle tenderness. Strength is 5/5 in all four extremities. Neurologic examination is non focal. Laboratory tests show: WBC count: 8600/mm3 Hemoglobin: 10.4 g/dL Platelet count: 610,000/mm3 Erythrocyte sedimentation rate: 120 mm/hr Creatine kinase: 52 U/L A short course of prednisone leads to almost complete resolution of symptoms. Which of the following is the most likely diagnosis? A. Fibromyalgia B. Osteoarthritis C. Polymyalgia rheumatica D. Polymyositis E. Rheumatoid arthritis ​

C. Polymyalgia rheumatica This patient has a clinical presentation suggestive of polymyalgia rheumatica (PMR). PMR is a disorder that is linked to temporal (giant cell) arteritis, and almost exclusively occurs in patients >50 years old. PMR typically presents as subacute or chronic onset of symmetrical pain and morning stiffness in large proximal joints Including the shoulder, hip girdle, and neck. Patients may report trouble getting dressed because of stiffness in these joints. Patients may also have systemic symptoms including weight loss and fatigue. When present, the pain of PMR is due to synovitis and bursitis of the joints, rather than actual muscle tenderness as the name implies. The classic laboratory finding in both PMR and temporal arteritis is elevated erythrocyte sedimentation rate, usually >40 mm/hr. and sometimes exceeding 100 mm/hr. The rapid response to prednisone further confirms the diagnosis, as both PMR and temporal arteritis typically have a dramatic response to even low-dose steroids. Bottom Line: Polymyalgia rheumatica presents as subacute or chronic onset of symmetrical pain and morning stiffness in large proximal joints including the shoulder, hip girdle, and neck. This condition responds rapidly to steroids.

A 55-year-old woman presents to her primary care physician complaining of fever, headache, and general malaise over the past 2 weeks. For the past month she has had recurrent headaches of increasing frequency. She has also felt feverish and generally "not well" for the past 2 weeks. She denies any recent trauma, increase in stress, or known exposure to infection. Her temperature is 38.2°C (100.8°F), heart rate is 92/min, and blood pressure is 114/85 mm Hg. The patient has marked tenderness over her temples. Laboratory tests show the erythrocyte sedimentation rate is markedly increased at 109 mm/hr. Which of the following is the most likely diagnosis? A. Cluster headaches B. Migraine headaches C. Temporal arteritis D. Tension headaches E. Trigeminal neuralgia ​

C. Temporal arteritis This patient has temporal arteritis. Temporal arteritis, also known as giant ceil arteritis, is characterized by inflammation of large- to medium-sized arteries, especially the temporal artery. Patients often present with headaches, fever, anemia, elevated erythrocyte sedimentation rate (ESR), and marked tenderness to palpation over the temporal regions, with a palpably thickened temporal artery noted on occasion. Laboratory abnormalities associated with temporal arteritis include elevated ESR and normochromic or slightly hypochromic anemia. Serum creatine kinase is not elevated in this disorder. Temporal artery biopsy is the gold standard in establishing the diagnosis of temporal arteritis. However, scheduling a biopsy should not delay initiating treatment with glucocorticoids, in order to avoid irreversible sequelae such as anterior ischemic optic neuropathy. Bottom Line: Temporal arteritis is characterized by inflammation of large- to medium-sized arteries, especially the temporal artery. Patients often present with headaches, fever, anemia, elevated erythrocyte sedimentation rate, and marked tenderness to palpation over the temporal regions.

A 50-year old man presents with epigastric pain unrelieved with PPIs. EGD shows a 6 cm intra-mural hypoechoic mass in the stomach. CT scan does not demonstrate any adenopathy. The biopsy comes back as C-kit positive. What is the most appropriate next step: A. H. pylori Tx B. Chemo-XRT C. WLE with 1 cm margin D. Partial gastrectomy E. Total gastrectomy ​

C. WLE with 1 cm margin Resection with 1 cm margin. This is a GIST tumor (gastro-intestinal stromal tumor) based on the C-kit finding. ​

7-year-old boy has severe microcytic anemia due to beta-thalassemia major (homozygous). He requires frequent blood transfusions (once every 6 weeks) to prevent the skeletal and developmental complications of thalassemia. Which of the following medications is also indicated in the treatment of patients requiring frequent blood transfusions? A. oral calcium supplements B. fresh frozen plasma (FFP) C. desferrioxamine D. penicillamine E. Cryoprecipitate ​

C. desferrioxamine Iron chelation with desferrioxamine will reduce the toxicity from iron overload if given regularly in high doses. The most lethal toxicity of iron overload is iron infiltration of the myocardium, with resultant dysfunction and death. Penicillamine has no role in the treatment of thalassemia patients requiring frequent transfusions. As well FFP, and cryoprecipitate are not indicated in the management of patients with thalassemia as there are no defects in thrombosis or coagulation.

A 23-year-old woman of Italian extraction is found to have a hypochromic microcytic anemia of 10 g/dL. In addition, there is a fair degree of anisocytosis, poikilocytosis, and targeting on the blood film. The WBC is 9500/mL, the platelet count is 240,000/mL, and the reticulocyte count is 7%. The spleen is palpated 5 cm below the left costal margin. Which of the following is the most appropriate treatment for her condition? A. ssplenectomy B. removal of the abnormal Hb pigment C. purely supportive plasmapheresis intramuscular iron

C. purely supportive The present treatment of choice for thalassemia minor is purely supportive. Care is taken to watch for anemia during intercurrent illness, due to a regenerative crisis. The therapies that are listed have no role in the management of patients with thalassemia minor.

A 62-year-old woman with chronic kidney disease (CKD) p resents to the Renal Clinic for review. She has been feeling increasingly lethargic over the past few months. As well as CKD she has a history of Type 1 diabetes, although this is currently well managed with insulin pump therapy. Examination reveals a BP of 138/ 82 mmHg, pulse is 80/min and regular. She looks pale and her BMI is 21. Investigations; Hb 7.9 g/ dl MCV 92 fl WCC 5.4 x109/l PLT 191 x109/l Na+ 138 mmol/l K+ 4.9 mmol/l Creatinine 395 micromol/l PO43- 1.7 mmol/l Ca++ 2.21 mmol/l AALP 160 U/l Ferritin 67 microg/l Which of the following is the most appropriate intervention with respect to the anaemia? A. Blood transfusion B. Erythropoietin C.Iron ttransfusion D. Oral iiron E. Upper and lower GI endoscopy ​

C.Iron ttransfusion In chronic renal failure, EPO therapy is only considered in patients where the ferritin is >100 mg/l. As such in this situation iron replacement is the initial intervention of choice. Iron is poorly absorbed from the GI tract in patients with renal failure, as such IV replacement is the modality of choice. In this case the MCV is in the normal range, which suggests that currently, in the absence of symptoms, upper and lower GI endoscopy is not required.

A 60 year old man undergoes partial gastrectomy and has a Billroth II anastomosis. Months later he develops mild abdominal pain and steatorrhea. His peripheral blood smear shows megaloblastic anemia. Which of the following tests is most diagnostic of this patient's symptoms: A. HIDA scan B. CT scan C. Angiography D. EGD aspiration E. MRI ​

D. EGD aspiration. This patient is suffering from afferent loop syndrome (blind loop syndrome) caused by poor motility of the biliary limb and bacterial overgrowth. Bacteria use up B-12 which results in megaloblastic anemia. Steatorrhea can be aggravated but the blind loop due to deconjugation of bile acids, EGD with aspiration is used to make the diagnosis. Tx - tetracycline and metronidazole (Flagyl) to reduce bacteria; metoclopramide (Reglan) or erythromycin to improve afferent limb motility. ​

A 73-year-old man with chronic lymphocytic leukaemia (CLL) is followed up in clinic. He has become increasingly breathless over the last 3 months but has no other symptoms and is on no medication. On examination, he is pale and has bilateral cervical and inguinal lymphadenopathy and a firm 5-cm splenomegaly. Full blood count shows: Hb 7.4 g/dl; wee 25 x 103/mm3; platelets 117 103/mm3; urea 15 mmol/litre; creatinine 140 µmol/litre; bilirubin 49 µmol/litre. Which investigation is most appropriate to demonstrate the likely cause of anaemia? A. Bone marrow aspirate B. Auto-antibody profile C. Erythropoietin level D. Antiglobulin test E. Urinary haemosiderin

D. Antiglobutin rest This man has known CLL and he has become increasingly anaemic. The other key investigation to note there is that his bilirubin is raised. Therefore, Anti-globulin test (also known as the Coombs' test) (Option D) is the correct answer. CLL is associated with auto-immune haemolytic anaemic, which will result in a positive Coombs' test, raised reticulocytes, raised unconjugated bilirubin and a raised LDH level. ​

A 39-year-old man is receiving a cross-matched blood transfusion after being admitted after a motorbike accident and sustaining a pelvic fracture. The nurses call you as he has sustained a spike in temperature to 37.8 oC some 20 min into the transfusion. His vital signs are normal with a stable BP of 115/70 mmHg; his pulse has slowed since starting the transfusion and fluid replacement from 105 bpm to 92 bpm. Which one of the following is the correct course of action? A. Change the unit for a fresh one B. Continue the transfusion and give chlorpheniramine and hydrocortisone C. Continue the transfusion at half rate D. Continue the transfusion at the same rate and give paracetamol E. Stop the transfusion and send the bag for analysis ​

D. Continue the transfusion at the same rate and give paracetamol It is likely that this man has a febrile non-haemolytic transfusion reaction. His vital signs are stable, and he clearly needs to receive his transfusion. There are no other worrying features in this situation, and the transfusion should be continued. Paracetamol to reduce his temperature may be helpful. ​

Which of the following is true regarding postgastrectomy bile reflux? A. Most patients with bile reflux into the stomach will develop symptoms. B. Symptoms usually correlate with the amount of bile entering the stomach. C. In symptomatic patients, medical management is generally effective. D. Creation of a Roux-en-Y gastrojejunostomy is an effective surgical option. E. It is more likely to occur after a Billroth I than a Billroth II reconstruction. ​

D. Creation of a Roux-en-Y gastrojejunostomy is an effective surgical option. Bile reflux into the stomach can occur without previous surgery, but in most instances it follows ablation of the pylorus, such as after gastric resection or pyloroplasty. After such procedures, most patients will have bile in the stomach on endoscopic examination, along with some degree of gross or microscopic gastric inflammation. However, only a small fraction of patients will have a significant degree of symptoms such as nausea, epigastric pain, and bilious vomiting consistent with alkaline (bile) reflux gastritis (A, B). Symptoms often develop months or years after the index operation. The differential diagnosis includes afferent or efferent loop obstruction, gastric stasis, and small bowel obstruction. These other diagnoses can be ruled out using a combination of abdominal radiographs, upper endoscopy, and abdominal CT scan. A hepatoiminodiacetic acid (HIDA) scan is particularly helpful for demonstrating bile reflux. Bile reflux and gastritis are more likely to occur after Billroth II reconstruction (E) than after Billroth I and least likely after vagotomy and pyloroplasty. Medical management of symptomatic patients is not particularly effective (C). The surgical procedure of choice is to convert the Billroth II into a Roux-en-Y gastrojejunostomy with a lengthened jejunal limb (at least 45 cm).

You are asked to see a 25-year-old white man who experienced marked weakness and dyspnoea 4 days after being admitted for a compound arm fracture after falling from a tree. Estimated blood loss from the initial fracture episode was 600 ml and the patient was transfused with one unit of packed erythrocytes. The initial crossmatch was reported as compatible by the transfusion service. The patient has never been transfused before this incident and has no other serious medical illnesses. The patient's arm fracture was treated with surgical pinning and prophylactic antibiotics consisting of a cephalosporin IV every 12 h. On examination, the patient is febrile and mildly tachycardic, with no evidence of wound infection or compartment syndrome. Laboratory data show a haematocrit of 15%, a raised reticulocyte count and total bilirubin of 70 µmol/1with a conjugated bilirubin of 9 µmol/1. The peripheral smear shows many spherocytes. No haemoglobinaemia or haemoglobinuria is seen on visual inspection of the plasma and urine. The transfusion service reports that the direct Coombs' test is now strongly positive using anti-lgG and only weakly positive with anti-C3d antisera. They further report that routine compatibility tests show no new erythrocyte antibodies in the patient's serum and that, when they attempted to elute antibody from the patient's red blood cells (RBCs) and test against normal RBCs, the results were negative.What is the most likely diagnosis? A. Auto-immune haemolytic anaemia of the cold antibody type B. Auto-immune haemolytic anaemia of the warm ant ibody type C. Delayed haemolytic transfusion reaction D. Drug-induced immune haemolytic anaemia E. Haemolytic transfusion reaction caused by an ABO incompatibility

D. Drug-induced haemolytic anaemia This man has evidence ofhaemolysis, with a raised reticulocyte count, as well as a raised level of unconjugated bilirubin. There is blood film evidence of haemolysis, showing spherocytes. There is no haemoglobinaemia or haemoglobinuria, demonstrating that there is no intravascular haemolysis occurring. The last couple of lines of the question essentially can be summarised by saying that the direct Coomb's test is now lgG positive, but the antibody screen is entirely negative. This therefore is against auto-immune haemolytic anaemia, or a transfusion reaction, because in these cases the antibody screen would be expected to be positive. Therefore, the most likely answer is a drug-induced haemolytic anaemia in response to hiscephalosporin antibiotic therapy. The anaemia should begin to improve after drug withdrawal.

The haematology registrar is contacted urgently about a woman who has attended an antenatal clinic that morning. She is 32 weeks' pregnant and has a blood pressure of 140/ 90 mmHg and mild proteinuria. Her pregnancy has been complicated by hyperemesis. At booking at 16-weeks' gestation, her blood results were: Hb 11.2 g/dl, MCV 105 fl, MCH 30 pg (28-32), MCHC 32 g/dl (32-35), WCC 11.3 x 109/litre and platelets 237 x 109/litre. Now at 32-weeks' gestation her blood results are: Hb 5.9 g/ dl, MCV 118 fl, MCH 32 pg, MCHC 32 g/ dl, WCC 6.4 x 109/lire and platelets 154 x 109/ litre. Which one of the following is the most likely cause of her anaemia? A. β-Thalassaemia trait B. Retroplacental haemorrhage C. Vitamin B12 deficiency D. Folic acid deficiency E. Aplastic anaemia

D. Folic acid deficiency This pregnant lady has a macrocytic anaemia, with a normal white cell count and platelet count. As pregnancy causes an increased demand for folate, this is the likely reason that she has become folate deficient. Her folate intake may well have been further compromised by the hyperemesis.

The above patient has continued bleeding, requiring 4 units of blood, despite EGD therapy. The most appropriate next step is: A. EGD therapy B. Embolization C. PPI only D. GDA ligation E. Graham patch ​

D. GDA ligation GDA ligation through a duodenal incision (duodenostomy). If the patient had been previously treated with PPI (history of ulcer Tx) and was stable in the OR, vagotomy and pyloroplasty at the same procedure would also be indicated. ​

A 55 year old man develops severe epigastric pain. CT scan shows free air. You explore him and find a perforated duodenal ulcer. The most appropriate next step is: A. Antrectomy including the ulcer and vagotomy B. Whipple C. Ligaton of GDA D. Graham patch E. Stent ​

D. Graham patch. You do not need to resect duodenal ulcers. Patient's with duodenal ulcers essentially have no increased GI CA risk and have actually been shown in several studies to have a decreased risk of GI CA compared to the general population. The mechanism by which duodenal ulcers decrease the risk of GI cancers is not fully understood. If the patient had been previously treated with PPI (history of ulcer Tx) and was stable in OR, vagotomy and pyloroplasty at the same procedure is indicated. If the patient had been previously treated with PPI (history of ulcer Tx) and was stable in OR, vagotomy and pyloroplasty at the same procedure is indicated.

A 32-year-old Asian woman is referred to the Haematology Clinic with anaemia. She has rheumatoid arthritis and is currently taking prednisolone, but has recently taken NSAIDs for analgesia. She has a palpable splenomegaly. FBC shows Hb 8 .1 g/ dl, MCV 69 fl, platelets 154 x 103/ mm3, WCC 5.2 x 103/mm3. A bone marrow trephine is stained for iron and shows that iron is present in the developing erythroblasts. Which one of the following is the likely cause of her anaemia? A. Iron ddeficiency B. Anaemia of chronic ddisease C. Sideroblastic aanaemia D. Haemoglobinopathy E. Myelodysplasia

D. Haemoglobinopathy The question has specifically stated the ethnic origin of this 32-year-old lady who has a microcytic anaemia associated with palpable splenomegaly. She is not iron deficient, because iron has been demonstrated in the bone marrow trephine.

Which of the following is the procedure of choice for an intractable duodenal ulcer that fails to heal despite maximal medical therapy? A. Truncal vagotomy and pyloroplasty B. Truncal vagotomy and antrectomy with Billroth I reconstruction C. Truncal vagotomy and antrectomy with Billroth II reconstruction D. Highly selective vagotomy (HSV) E. Distal gastrectomy ​

D. Highly selective vagotomy (HSV) An HSV is the procedure of choice for intractable duodenal ulcer in which medical management fails. In the current era of H. pylori treatment, surgery for intractable duodenal ulceration is rare. There are three main surgical options: HSV, vagotomy and pyloroplasty (A), and vagotomy and antrectomy (B, C). The use of vagotomy and antrectomy has the overall lowest rate of ulcer recurrence (<2%). However, because it involves a gastric resection, it has the highest complication rate. The procedure also predisposes the patient to both postvagotomy syndromes and postgastrectomy syndromes. As such, it is the least favored option in this setting. The use of vagotomy and pyloroplasty has a higher ulcer recurrence rate than vagotomy and antrectomy but less morbidity. An HSV has the highest ulcer recurrence rate (=10-15%) but the lowest morbidity rate. It requires more time and skill, so it is not an optimal choice in an emergent setting. It also has the advantage that it can be performed laparoscopically. Distal gastrectomy (E) is not an option for duodenal ulcer because it does not reduce acid secretion.

A 55-year-old man comes to the office for evaluation of chronic muscle weakness. Over the past several months, the patient has had increasing difficulty walking up stairs and lately has noticed difficulty combing his hair and lifting objects overhead. His temperature is 98 F (36.6 C). blood pressure is 125/80 mm Hg, and pulse is 78/min. On examination, the patient has symmetric proximal muscle weakness and mild muscle tenderness. There is no skin rash. Muscle biopsy reveals an endomysial mononuclear infiltrate and patchy muscle fiber necrosis. An autoantibody directed against which of the following antigens is most likely to be seen in this patient? A. Acetylcholine receptor B. Cardiolipin C. Desmoglein D. Histidyl-tRNA synthetase E. Mitochondria F. Presynaptic calcium channel G. Smooth muscle ​

D. Histidyl-tRNA synthetase Polymyositis typically presents around middle age with insidious onset of symmetric proximal muscle weakness affecting the upper and lower extremities. Myalgias may also occur in some patients. Muscle enzyme (e.g., serum creatine kinase, aldolase) levels are invariably elevated. Autoantibodies, including antinuclear antibodies (ANA) and anti-histidyl-tRNA synthetase (anti-Jo-1), are present in most cases. A biopsy is often performed to differentiate polymyositis from other muscle diseases and typically shows an endomysial mononuclear inflammatory infiltrate and patchy muscle fiber necrosis. Polymyositis and dermatomyositis share a number of clinical features, although polymyositis lacks the typical skin findings of dermatomyositis. Both polymyositis and dermatomyositis may occur independently or as a paraneoplastic manifestation of an underlying malignancy (especially adenocarcinoma). Educational objective: Polymyositis is characterized by symmetric proximal muscle weakness. It is associated with antinuclear and anti-Jo-1 autoantibodies. Biopsy shows an endomysial mononuclear inflammatory infiltrate and patchy muscle fiber necrosis. Polymyositis is similar to dermatomyositis but lacks the characteristic skin findings.

A 30-year-old woman presents with a widespread skin rash that she has had for 5 days. She is taking sulfa medication for recurrent cystitis. A skin biopsy shows leukocytoclastic vasculitis involving dermal venules. What is the appropriate diagnosis? A. Buerger disease B. Giant cell granulomatous arteritis C. Henoch-Schönlein purpura D. Hypersensitivity angiitis E. Polyarteritis nodosa

D. Hypersensitivity angiitis Hypersensitivity angiitis. Hypersensitivity angiitis refers to a broad spectrum of inflammatory lesions that represent a reaction to foreign materials (e.g., bacterial products or more commonly drugs). When the vascular lesions are confined to the skin, the terms leukocytoclastic vasculitis, cutaneous vasculitis, or cutaneous necrotizing venulitis are applied. The other choices are not caused by sulfa drugs.

Early gastric cancer is best defined as: A. Lymph node negative B. Limited to the mucosa C. Limited to the mucosa and submucosa with negative nodes D. Limited to the mucosa and submucosa regardless of nodes E. In the muscularis propria but not the serosa ​

D. Limited to the mucosa and submucosa regardless of nodes Early gastric cancers are defined as adenocarcinoma limited to the mucosa and submucosa of the stomach, regardless of lymph node status (A-C, E). In Japan, because of the high prevalence of gastric cancer, aggressive screening programs are used. This has resulted in as many as one-half of gastric cancers being detected early, whereas in the United States, less than one-fourth are considered early gastric cancer. In one large series of nearly 400 patients with early gastric cancer, 11.9% had positive lymph nodes. Risk factors for lymph node metastasis included large tumor size, lymphatic vessel involvement, and invasion of the submucosa. Lymph node status was the most important determinant of survival.

A 55-year-old female presented with abdominal pain and frequency. She admits to symptoms of weight loss and night sweats. Examination demonstrated a splenic tip palpable 12 cm below the costal margin and mild hepatomegaly, but there was no clinical enlargement of the peripheral lymph nodes. The full blood count was haemoglobin 8.9 g/ dl, white blood cell count 5.4 x 109 per litre, platelets 470 x 109 per litre, mean cell volume 85 femtolitre (fl) and reticulocytes 2.4%. The peripheral blood film was reported to show ovalocytes (elliptocytes) and the occasional myelocyte and nucleated red cell. The serum lactate dehydrogenase (LDH) was 1256 U/l. Which one of the following is the most likely diagnosis? A. Chronic myeloid leukaemia B. Essential thrombocyrhaemia C. Megaloblastic anaemia D. Myelofibrosis E. Non-Hodgkin's lymphoma

D. Myelofibrosis This middle-aged lady has very significant splenomegaly with mild hepatomegaly. There is evidence of a normochromic anaemia, and most importantly a white cell count that is not particularly raised. The peripheral blood film shows early white blood cell and red blood cell precursors. This description is known as a leucoerythroblastic blood picture. The significantly enlarged spleen in the presence of anormal white cell count strongly suggests myelofibrosis.

A 22-year-old Caucasian patient p resents to the clinic with jaundice and pallor. He has recently suffered an upper respiratory tract infection. He tells you that his father had a splenectomy. On examination his temperature is 37.S °C, his BP is 122/ 75 mmHg. He looks pale and has jaundiced sclerae. He has splenomegaly on abdominal examination. Investigations: Hb 99 g/1 (uniform spherocytes seen on b loo d film) WCC 5.6 x 109/l PLT 130 x 109/ 1 Na+ - 140 mmol/1 K+- 4.2mmol/l Creatinine 130 µmol/1 Bilibrubin 203 µmol/1 Which one of the following is the best way to confirm the diagnosis? A. Cold agglutinins B. Coombs' test C. Haemoglobin electrophoresis D. No furth er tests E. Osmotic fragility test

D. No further tests This man has enough clinical and laboratory features to indicate that he has hereditary spherocytosis. The question tells us that his father has had a splenectomy, and more information should be gathered about why this was the case. He now has evidence of haemolysis, with spherocytes on the blood film with a raised bilirubin level. This is entirely sufficient to make a diagnosis of hereditary spherocytosis.

A 22-year-old woman comes to the Gastroenterology Clinic. She has had coeliac disease diagnosed some 5 years ago, and has difficulty controlling her symptoms. She opens her bowels some 3-4 rimes per day and complains of intermittent abdominal bloating. There is no other significant past medical history and her only medication is the combined oral contraceptive pill. Examination reveals a BP of 95/ 50 mmHg, pulse is 70/min and regular. Her abdomen is soft and non-tender, her BMI is reduced at 19.5. Investigations; Hb - 9.1 g/dl Blood film - Howell Jolly bodies seen on film WCC - 7.1 x109/l PLT - 157 x109/ I Na+ - 137 mmol/l K+ - 4.3 mmol/l Creatinine - 102 micromol/l Albumin - 29 g/l ALT - 54 U/l Glucose - 5.1 mmol/l Which of the following infections is she shown to be at increased risk of? A. Herpes simplex B. Influenza C. Meningococcus D. Pneumococcus E Tuberculosis

D. Pneumococcus This patient has functional hyposplenism as a result of coeliac disease. Hyposplenism is associated with increased risk of infection, particularly from capsulated bacteria. Whilst increased susceptibility would therefore be expected to pneumococcal and meningococcal infection, cohort data only suggests increased risk of pneumococcal infection. Data from the Swedish registry suggests that people with coeliac disease (n = 15,325) had a twofold increased risk of infection (HR: 1.6 [95% Cl: 1.2-1.9]) and a fourfold increased risk of pneumococcal infection (HR: 2.5 [95% Cl: 1.2-5.1]). No increased risk of meningococcal infection was observed in this population.

A 45-year-old woman comes to the office due to progressive weakness for the past several months. The patient has had difficulty lifting her foster children and getting in and out of her car. She also reports difficulty swallowing and has had several episodes of choking while drinking water. She has no medical problems, but there is a history of breast cancer in her mother. The patient does not use tobacco, alcohol, or Illicit drugs. Vital signs are within normal limits: and she has normal sensation and deep tendon reflexes. Strength is 4/5 in the deltoid and quadriceps muscle groups, and there is no limitation in range of motion. Physical examination is otherwise unremarkable. Erythrocyte sedimentation rate is 50 mm/hr., and serum creatine kinase is elevated. TSH is normal. Which of the following is the best initial treatment for this patient? A. Amitriptyline B. Beta interferon C. Cyclobenzaprine D. Prednisone E. Pyridostigmine F. Radiation therapy G. Riluzole

D. Prednisone This patient has features of polymyositis, including chronic proximal muscle weakness, otherwise normal neurologic findings, and an elevated creatine kinase. Peak incidence occurs at age 40-50, and women are more commonly affected than men. Proximal muscle weakness typically manifests as difficulty climbing stairs, getting into or out of a low chair or car, or working with the arms overhead (e.g., combing hair). Involvement of upper esophageal musculature can cause dysphagia with regurgitation and aspiration. Inflammatory markers (e.g., erythrocyte sedimentation rate) may be elevated, and most patients have detectable autoantibodies (e.g., antinuclear antibody, anti-Jo-1). Definitive diagnosis is established on muscle biopsy. Polymyositis resembles dermatomyositis but without associated skin manifestations. Initial remission can be induced with glucocorticoids (e.g., prednisone), and most patients also receive a glucocorticoid-sparing agent (e.g., methotrexate, azathioprine) to minimize the long-term adverse effects of treatment. Because polymyositis frequently occurs as a paraneoplastic syndrome, patients should also receive age-appropriate cancer screening. Educational objective: Polymyositis is characterized by proximal muscle weakness and elevated creatine kinase. Initial treatment includes systemic glucocorticoids (e.g., prednisone), and most patients also receive a glucocorticoid-sparing agent (e.g., methotrexate, azathioprine) to minimize the adverse effects of treatment.

A 65-year-old man presents to the clinic for assessment of numerous symptoms which are worse in the winter months. He notices diffuse red patches which are not raised or painful and occasionally purple fingertips. Other symptoms include joint discomfort (arthralgia) and muscle weakness. On physical examination, the vital signs are normal, the heart sounds are normal, and the lungs clear. The joints are normal with no active inflammation, and the muscle strength is 4+/5. On his thighs and knees there are fine "lace"-like appearing skin changes that are consistent with livedo reticularis. Laboratory investigations reveal hemoglobin of 125 g/L, a 2-fold elevation in ALT and AST, a positive hepatitis C virus antibody titer, and early renal impairment. Which of the following is the most likely diagnosis? A. cold agglutinin disease B. Henoch-Schönlein purpura C. antiphospholipid antibody syndrome D. cryoglobulinemia E. cholesterol embolic disease

D. cryoglobulinemia The symptoms are typical of cryoglobulinemia. Cryoglobulins are antibodies that precipitate under cold conditions and are associated with several diseases. There are three main types of cryoglobulin syndromes based on the immunoglobulin composition of the precipitating antibody. Most cryoglobulinemic vasculitis is Type II or mixed and seen secondary to hepatitis C virus infection. Cold agglutinin disease would present with hemolytic anemia. The other syndromes would not generally be temperature sensitive, and do not have an association with hepatitis C virus infection

A 16-year-old young man with sickle cell anaemia is admitted with recent breathlessness. He is febrile and has a clear chest with saturations of 98% on air. From his outpatient notes his usual Hb is 9 g/ dl. FBC taken in the Emergency Department shows WCC 2.6 x 103/ mm3, Hb 4.7 g/ dl, platelets 573 x 103/ mm3 with a bilirubin 25 µmol/litre. Which investigation is the most useful with respect to determining the underlying diagnosis? A. Serum haptoglobin B. Urinary haemosiderin C. Parvovirus serology D. Reticulocyte count E. Chest X-ray ​

D. Reticulocyte count This young man has sickle cell anaemia. He normally has a haemoglobin of 9 g/ dl, but he has become profoundly anaemic. This could be a haemolytic crisis, but equally well this could be splenic sequestration or an aplastic crisis. His bilirubin is only a little bit raised. Reticulocyte count (Option D) is the correct answer. This is a very useful investigation in all forms of anaemia. It is particularly useful if the reticulocyte count is raised. Where there is anaemia with reticulocytosis, it suggests that the bone marrow is working correctly, and that the red blood cells are being destroyed outside the bone marrow. This can be attributed to only two conditions - bleeding or haemolysis. If the bilirubin is normal, this effectively rules out significant haemolysis, and can be a useful pointer to occult bleeding. If the reticulocyte count is normal, it does not often help with the further investigation of anaemia. However, if reticulocytes are very, very low, this would point towards an aplastic crisis, which is an important differential.

A 46-year-old woman is managed with long-term haemodialysis. The cause of her chronic renal failure is long-standing type 1diabetes. She is treated with a steady dose of erythropoietin (EPO), yet on recent dialysis sessions you have noticed a decrease in her haemoglobin (Hb) from 11.1g/ dl to 8.4 g/ dl at her last dialysis session. Which one of the following would be the most appropriate investigation? A. Bone marrow aaspiration B. Lower GI eendoscopy C. Measurement of EPO antibodies D. Serum ferritin E. Upper GI endoscopy ​

D. Serum ferritin Essentially this lady is becoming increasingly anaemic despite EPO supplementation. Patients with chronic renal failure often have co-existent low iron stores. Failure to respond to EPO should prompt an assessment of iron stores. Replacement using intravenous iron infusions may be required.

A 16-year-old Italian girl presents with anaemia. Haemoglobin is 70 g/l (120-160 g/1). Her blood film shows marked hypochromia and variation in cell shape and size. Nucleated red cells are also found. Which one of the following is the most likely diagnosis? A. Acute myeloblastic lleukaemia B. Aplastic aanaemia C. Chronic myeloid lleukaemia D. Sickle-cell aanaemia D. Thalassaemia ​

D. Thalassaemia This young girl's Italian origin is very important. She has a marked hypochromia, ie a reduction in the amount of haemoglobin in each red blood cell. This is called a low MCH. Congenital disorders always need to be considered in young patients, whereas congenital disorders presenting in older patients are obviously much rarer. This is a typical presentation of thalassaemia in an Italian girl, and nucleated red blood cells are usually found in the peripheral blood. ​

A 65 year old man presents with a 3-month history of nausea, vomiting, anorexia, 7-kg weight loss and iron deficiency anemia. Abdominal examination shows an epigastric mass and a bulging nodule in the umbilicus. Upper GI endoscopy confirms the presence of a tumor occupying most of the proximal stomach but not involving the gastro-esophageal junction. Biopsies are obtained from the tumor and from the tumor-free distal stomach. Which statement is correct? A. Biopsies from the tumor-free distal gastric mucosa will invariably show evidence of H. pylori infection B. Despite the large size of this tumor, it could still be regarded as an early cancer because the chance of metastasis to distant organs is small C. The bulge in the umbilicus is known as a Krukenberg tumor D. The bulge in the umbilicus is likely a metastatic nodule (Sister Joseph's nodule) E. The most likely histology of this tumor is squamous cell carcinoma because of its proximity to the esophagus ​

D. The bulge in the umbilicus is likely a metastatic nodule (Sister Joseph's nodule) Virtually all gastric tumors are adenocarcinomas arising from mucus-secreting cells in the base of the gastric crypts. Gastric cancers are very aggressive and usually present quite late with large sizes and with distant spread. While most gastric cancers are initiated by H. pylori infection, the organism often disappears in the latter stages with onset of gastric atrophy and achlorhydria. Krukenberg tumors are metastatic deposits on the ovaries. Sister Joseph's nodule is a metastatic deposit in the umbilicus usually from a cancer in the pelvis or abdomen.

A 70-year-old man comes to the physician for pain and stiffness of his neck, shoulders, and hips for the last 3 months. His stiffness is worse in the morning and lasts 1-2 hours. He also complains of general malaise and a recent weight loss of 3.17 kg (7 lb.). The patient has no headache, scalp tenderness visual symptoms, or jaw claudication. Examination shows no overt synovitis at the joints, with normal passive range of motion. The arteries of the scalp, neck and extremities are normal on palpation without tenderness. Laboratory results are as follows: Leukocytes 7500/µL Hematocrit 31%; Platelets 450,000/µL Erythrocyte sedimentation rate 85 mm/h Thyroid-stimulating hormone 1.8 µU/mL Creatine kinase 33 U/L Which of the following is the most appropriate next step in management of this patient? A. Measure antinuclear antibodies and rheumatoid factor levels B. Recommend low-impact aerobic exercise C. Temporal artery biopsy D. Treatment with low-dose prednisone E. Treatment with nonsteroidal anti-inflammatory drug

D. Treatment with low-dose prednisone This patient has the following characteristic features of polymyalgia rheumatica (PMR): « Age >50 « Subacute-to-chronic (>1 month) pain in the shoulder and hip girdles « Morning stiffness lasting >1 hour « Constitutional symptoms « Elevated erythrocyte sedimentation rate >40 mm/h « No other apparent explanation for symptoms In PMR, the physical examination is frequently unremarkable with patients having no focal tenderness or pain with active or passive range of motion. Signs of inflammation in the joints are absent. When asked to identify the location of their pain: patients typically indicate the soft tissues and not the joints. Low-dose glucocorticoids are the treatment of choice for PMR (e.g., prednisone 10-20 mg daily). Rapid and thorough relief of symptoms is expected, and failure to improve rapidly on prednisone should call the diagnosis into question. PMR is frequently associated with giant cell arteritis (GCA), also known as temporal arteritis. Symptoms of GCA include headache, jaw claudication, vision loss, or tenderness over the temporal artery. If GCA is suspected, patients should be considered for an expedited temporal artery biopsy (Choice C) and receive significantly higher doses of glucocorticoids (e.g., prednisone 40-60 mg or higher daily). Educational objective: Polymyalgia rheumatica (PMR) affects patients age >50 and is characterized by pain and stiffness in the neck, shoulders, and pelvic girdle, along with an elevated erythrocyte sedimentation rate. The treatment of choice for uncomplicated PMR is low-dose prednisone, which results in rapid relief of symptoms.

Which of the following describes the association between Sister Mary Joseph's nodule and gastric cancer? A. A metastatic left axillary lymph node B. A metastatic left supraclavicular lymph node C. An ovarian mass from gastric metastasis D. Umbilical metastasis suggesting carcinomatosis E. An anterior nodule palpable on rectal examination suggesting drop metastasis ​

D. Umbilical metastasis suggesting carcinomatosis A metastatic left supraclavicular lymph node is called the Virchow node (Troisier sign) (B). Intra-abdominal cancers tend to metastasize to the left secondary to lymph drainage into the left subclavian vein via the thoracic duct. A metastatic left axillary lymph node from gastric cancer is called an Irish node (A). A Blumer shelf is a palpable nodule on rectal examination suggesting a drop metastasis (E). An ovarian mass from a gastric metastasis is also known as Krukenberg tumor. (C) An umbilical nodule (Sister Mary Joseph node) suggests carcinomatosis. Although associated with gastric cancer, it may represent any metastatic lesion, most commonly from an intra-abdominal cancer. It was named after Dr. William Mayo's surgical assistant, who made the observation while scrubbing patients for gastric surgery that those with umbilical nodules had widely metastatic and unrespectable gastric cancer. Current recommendations are that if such nodules are found on physical examination, the patient should undergo fine-needle aspiration because such umbilical nodules may sometimes represent benign disease.

A 57-year-old man, with a history of chronic alcohol ingestion, is admitted to the hospital with acute alcoholic intoxication and lobar pneumonia. Physical examination reveals pallor; a large tender liver; and consolidation of the right lower lobe. Laboratory data include hemoglobin of 7 g/dL, WBC of 4000/mL, and platelet count of 85,000/mL. Which of the following is the most likely factor for the pancytopenia? A. hemolysis B. hemobilia C. vitamin B12 deficiency D. toxic marrow suppression E. Hemoglobinopathy ​

D. toxic marrow suppression The pancytopenia in this individual is most likely due to the marrow suppressive effects of alcohol. Anemia without white cell or platelet involvement can be multifactorial in alcoholics and includes marrow suppression, GI bleeding, hemorrhagic diathesis, and nutritional deficiency. Hemolysis does not result in pancytopenia and only rarely will severe vitamin B12 deficiency cause pancytopenia. Hemoglobinopathy is not usually a feature of chronic alcoholism: it is usually an inherited condition. ​

A 30-year-old woman presents with increasing fatigue, but no other symptoms. Her past medical history is negative and she is not on any medications. About 3 weeks ago she had a "flu"-like illness that has now resolved. On examination, the conjunctivae are pale, her sclera are icteric, the blood pressure 110/70 mm Hg, pulse 110/min, lungs clear, and heart sounds normal. Castell sign is negative and no spleen is palpable. Her hemoglobin is 9 g/dL, and the rest of the laboratory workup is shown below. Which of the following is the most likely diagnosis? CASE WORKUP Blood film Polychromatophilia, some spherocytes Bilirubin 2 mg/100 mL total 0.3 mg/100 mL direct Haptoglobin 10 mg/100 mL Lactate dehydrogenase 200 IU/L Urine bilirubin Negative A. iron deficiency B. congenital spherocytosis C. liver failure D. splenomegaly and hemolysis E. autoimmune hemolytic anemia

E) autoimmune hemolytic anemia Autoimmune hemolytic anemia is the most likely diagnosis, based on the elevated indirect bilirubin, spherocytes on the blood film, low hemoglobin and low haptoglobin level. Spherocytosis is seen as well in burn victims, in microangiopathic hemolysis, and in congenital spherocytosis. In liver failure, the elevated bilirubin is usually direct, and in splenomegaly causing hemolysis the spleen should be enlarged on clinical exam. Finally, iron deficiency does not cause a hemolysis-type presentation.

A 68-year-old man is admitted to the intensive care unit because of acute respiratory failure secondary to clinically diagnosed community-acquired pneumonia. His temperature on admission is 41.2° C. Chest radiograph shows diffuse bilateral infiltrates. He is intubated and placed on a respirator, cultured, promptly started on broad-spectrum antibiotics, and placed on a cooling blanket. Intravenous hydration is begun. On the second day, the patient's condition deteriorates. Acrocyanosis develops in his fingertips and toes, the sclerae are noted to be icteric, and the urine from his Foley catheter has turned red (testing 4+ positive for heme but with only a small number of red blood cells). The admission hemoglobin of 14.2 has dropped to 7.8. What is the most likely cause of the acute anemia? A. Gross hematuria possibly secondary to ureteral trauma or a previously undiagnosed genitourinary malignancy B. Fulminant alveolar hemorrhage C. Warm antibody-mediated autoimmune hemolytic anemia most likely secondary to previously undiagnosed, widely metastatic lung cancer D. Drug-induced autoimmune hemolytic anemia, possibly caused by one of the antibiotics started on admission, with intravascular hemolysis E. Acute cold antibody-mediated autoimmune hemolytic anemia and agglutination secondary to Mycoplasma pneumoniae infection

E. Acute cold antibody-mediated autoimmune hemolytic anemia and agglutination secondary to Mycoplasma pneumoniae infection Although unusual, community-acquired M. pneumoniae infection can present with bilateral pulmonary infiltrates and acute respiratory failure. M. pneumoniae infection (as well as certain viral infections) are associated with cold-reactive autoantibody-mediated cold agglutinin and simultaneously hemolytic disease. This patient's abrupt onset of massive intravascular hemolysis could have been triggered by artificially cooling his body temperature. The patient's intravascular cold autoimmune hemolytic anemia (AIHA) is manifested by hemoglobinuria and his acute intravascular cold agglutinin disease by the development of acrocyanosis. The patient does not have gross hematuria; he has gross hemoglobinuria, so genitourinary tract disease cannot be the cause. Fulminant alveolar hemorrhage could cause a precipitous decline in hemoglobin but is very unlikely in the absence of bloody airway secretions via his endotracheal tube. Warm antibody-mediated AIHA secondary to cancer would not be expecte

A 74-year-old man underwent surgery for peptic ulcer surgery 35 years ago. He now presents with symptoms that include abdominal pain and bloating about 30-40 minutes after eating, accompanied by nausea. If he vomits, the symptoms are relieved. On physical examination, the vital signs are normal, there is an old mid-line scar on the abdomen, but no focal tenderness, guarding, masses or organomegaly that is palpated. Which of the following is the most likely cause for his symptoms? A. early dumping syndrome B. Late dumping syndrome C. bile reflux gastropathy D. Retained gastric antrum E. Afferent loop syndrome ​

E. Afferent loop syndrome This pattern of symptoms is characteristic of afferent loop syndrome. It is caused by distention and incomplete drainage of the afferent loop and requires surgical correction. Bacterial overgrowth of the afferent loop is more common. Its clinical presentation includes post-prandial abdominal pain, bloating, diarrhea, fat, and vitamin B12 malabsorption. His symptoms are not characteristic of the dumping syndrome (early or late), and bile acid reflux can present with similar symptoms but is much less common than afferent loop syndrome.

Which of the following is true regarding postvagotomy diarrhea? A. It is effectively treated with octreotide. B. It does not improve with oral cholestyramine. C. Cardiovascular manifestations are common. D. Most patients require the creation of a reversed jejunal segment. E. Diarrhea may improve with the administration of codeine. ​

E. Diarrhea may improve with the administration of codeine. Postvagotomy syndromes include diarrhea, gastric atony, and incomplete vagotomy (leading to recurrent ulceration). Diarrhea follows truncal vagotomy and may be confused with dumping syndrome. The diarrhea associated with vagotomy occurs more frequently and is not associated with the other cardiovascular manifestations seen with dumping syndrome (C). The initial treatment is similar to that for dumping syndrome, with dietary modifications such as frequent small meals with decreased fluid intake and an increase in fiber. A proposed mechanism of the diarrhea is an increase in stool bile salts. Oral cholestyramine is often helpful because it binds bile salts (B). Loperamide and codeine have also been shown to delay intestinal transit time and improve symptoms. In the very rare patient who does not respond to medical management, reversal of a segment of jejunum is effective in slowing transit time and improving diarrhea (D). Octreotide is not effective for post vagotomy diarrhea and may make the situation worse by decreasing pancreatic secretions and thus increasing steatorrhea (A).

A 60-year-old man presents with a 12-hour history of worsening epigastric pain. He has a history of duodenal ulcer, and the results of a recent biopsy 2 weeks earlier were negative for H. pylori. Upright chest radiograph demonstrates free air under the diaphragm. The patient is hemodynamically stable. At surgery, a perforated duodenal ulcer is found with mild peritoneal contamination. Which of the following is the best management option? A. Graham patch of duodenal ulcer B. Graham patch of duodenal ulcer with truncal vagotomy and pyloroplasty C. Truncal vagotomy and antrectomy with Billroth I reconstruction D. Truncal vagotomy and antrectomy with Billroth II reconstruction E. Graham patch of duodenal ulcer with a highly selective vagotomy ​

E. Graham patch of duodenal ulcer with a highly selective vagotomy In the majority of patients with a perforated duodenal ulcer, simple closure of the ulcer with an omental (Graham) patch is all that is necessary (A). This is then followed by treatment for H. pylori. In addition, a Graham patch alone should be used if the patient is unstable, if there is extensive exudative peritonitis, or if the perforation is long standing (>24 hours). However, in the setting of a patient with a known ulcer diathesis who has either already been treated for H. pylori or is H. pylori negative, an ulcer surgery should be added to the operation, provided the patient is a good operative risk, is hemodynamically stable, and does not have extensive peritonitis. The options are either to perform a highly selective vagotomy (HSV) or a vagotomy and pyloroplasty (B). An HSV is the preferred approach in the good-risk patient who is stable, provided the surgeon is comfortable with the procedure. Pyloroplasty is typically performed along with a vagotomy because the widened outlet from the stomach to the duodenum helps circumvent any unwanted effects of the decreased gastric peristalsis and overall change in gastric emptying patterns that occur following vagotomy. The entire procedure can be performed laparoscopically in select patients. Truncal vagotomy and antrectomy (C, D) is generally not recommended in the setting of perforation because of the high associated morbidity and mortality rates.

A 52-year-old woman comes to the physician with a rash over her face that began a few weeks ago. Lately, the patient has also been having difficulty rising from a seated position and climbing stairs. On examination, an erythematous rash on the upper chest and violaceous periorbital edema are present. Her vital signs are within normal limits. Examination shows symmetric proximal muscle weakness in the legs. This patient's condition is most often associated with which of the following? A. Alveolar hemorrhage B. Aortic aneurysms C. Carpal tunnel syndrome D. Glomerulonephritis E. Malignancy F. Sudden visual loss

E. Malignancy This woman with proximal muscle weakness and rash has features consistent with dermatomyositis. Dermatomyositis is an autoimmune condition that is 6 times more common in women. In its most classic form, it causes a proximal extensor muscle inflammatory myopathy and characteristic cutaneous findings (e.g., violaceous poikiloderma over various regions of the body). On the face, the eruption is most often accompanied by periorbital edema and is known as the heliotrope sign. On the chest and lateral neck it is called the shawl sign. On the knuckles, elbows, and knees it is referred to as Gottron's sign. Violaceous, slightly scaly papules overlying the joints are known as Gottron's papules; these are pathognomonic for dermatomyositis. Patients have an elevated creatine phosphokinase (CPK) that is generally 10 times the upper limit of normal. The classic autoantibodies associated with dermatomyositis are anti-Jo-1 (antisynthetase antibody) and anti-Mi-2 (against helicase). Internal malignancies are more common in patients with dermatomyositis (>15% of adult patients) compared to the general population. The most common malignancies are ovarian, lung, pancreatic, stomach, or colorectal cancers, and non-Hodgkin lymphoma. Regular, age-appropriate cancer screening is essential in these patients. Educational objective: Dermatomyositis is characterized by classic cutaneous findings accompanied by proximal muscle weakness. Over 15% of adult patients will have or develop an internal malignancy, most commonly ovarian, lung, pancreatic, stomach, or colorectal cancers, or non-Hodgkin lymphoma. Regular, age-appropriate cancer screening is essential in these patients.

A 39-year-old man has type-1diabetes and end-stage renal failure requiring haemodialysis. He has an O positive blood group. You are trying to find an appropriate kidney donor for him. Which one of the following would be the most appropriate blood group of a potential kidney donor for him? A. A+ B. A C. AB+ D. B+ E. O+

E. O+ O positive is the best answer here. We are told that the recipient of the potential kidney donation is O positive. This means he will have anti-A and anti-B antibodies in his blood. As such, transplanting a kidney from an AB, A or B donor is likely to lead to rejection. ​

A 36-year-old white man presents to his primary care physician with a 3-month history of progressive weakness. He complains that it is increasingly difficult to get up from a chair, climb stairs, lift ordinary household objects, or to comb his hair. He also reports a hard time keeping his head up due to weakness of the neck extensors. He denies having episodes like this in the past, a family history of neuromuscular disease, or exposure to mycotoxic drugs; however, he does recall a 2-week period 4 months ago where he had ''flu-like" symptoms. Since then, he is reporting systemic manifestations such as intermittent fevers, chills, fatigue and anorexia. Physical examination reveals no signs of rash and no involvement of the extraocular or facial muscles. He has symmetric 3/5 muscle strength in his upper and lower extremities bilaterally. Muscle biopsy is consistent with polymyositis. Which of the following is the most appropriate initial course of therapy? A. Etanercept B. Intravenous immunoglobulin C. Methotrexate D. Nonsteroidal anti-inflammatory drugs E. Oral prednisone

E. Oral prednisone Polymyositis is an inflammatory myopathy. Although rare, this group of disorders (which also includes dermatomyositis and inclusion body myositis) usually presents with symmetric and progressive muscle weakness. Oral prednisone is the preferred agent for initial treatment of polymyositis. The treatment should be started as high-dose steroids (up to 80 mg/d of prednisone) for 4-6 weeks, and then a gradual taper lasting up to a year, while always monitoring response to the therapy based on improvement in muscle strength, muscle endurance, and decrease in creatine kinase levels. It is also important to monitor for occurrence of corticosteroid myopathy during the course of the treatment and adjust dosage accordingly. Bottom Line: Polymyositis is an inflammatory myopathy that presents with symmetric and progressive muscle weakness. First-line treatment is with high-dose oral steroids to be tapered over several months to a year while monitoring for occurrence of corticosteroid myopathy. Second-line treatment includes immunosuppressants such as azathioprine or methotrexate. Third- line agents include IVIG. Rarely TNF-a inhibitors may be used in refractory cases of polymyositis.

A 25-year-old woman noticed one episode of passing blood instead of urine. Her GP arranged for a urological examination, which proved to be normal. She looks anaemic, but examination is otherwise normal. Which one of the following is the most likely diagnosis? A. Acute lymphocytic anaemia B. Aplasric anaemia C. Fanconi anaemia D. Glucose-6-phosphate dehydrogenase deficiency E. Paroxysmal nocturnal haemoglobinuria ​

E. Paroxysmal nocturnal haemoglobinuria (PNH) This lady seems to have passed haemoglobin in her urine and not blood after all. She is also anaemic, which occurs as a result of intravascular haemolysis. This is also associated with episodes of venous thrombosis and can be associated with aplastic anaemia.

A 67-year-old woman has experienced severe nausea, vomiting, early satiety, and a 9-kg weight loss over the past 4 months. On physical examination, she has muscle wasting. Upper gastrointestinal endoscopy shows that the entire gastric mucosa is eroded and has an erythematous, cobblestone appearance. An abdominal CT scan shows that the stomach is small and shrunken. Which of the following is most likely to be found on histologic examination of a gastric biopsy specimen? A. Chronic atrophic gastritis B. Primary gastric lymphoma C. Gastrointestinal stromal tumor D. Granulomatous inflammation E. Signet ring cell adenocarcinoma ​

E. Signet ring cell adenocarcinoma The endoscopic and radiologic findings describe the linitis plastica ("leather bottle") appearance of diffuse gastric carcinoma. Histologically, these carcinomas are composed of the gastric type of mucus cells that infiltrate the stomach wall diffusely. The individual tumor cells have a signet ring appearance because the cytoplasmic mucin pushes the nucleus to one side. In chronic atrophic gastritis, the rugal folds are lost, but there is no significant scarring or shrinkage. Primary gastric lymphomas are less common than adenocarcinomas; a lymphoma may be large but would not involve the stomach in a diffuse pattern. Gastrointestinal stromal tumors tend to be bulky masses. Granulomas are rare at this site.

A patient with chronic lymphocytic leukaemia (CLL) on combination chemotherapy still has haemolytic anaemia and thrombocytopenia despite fludarabine. His Hb has dropped by 3 g over the past 3 months despite a course of corticosteroids. Which one of the following interventions will most likely be of benefit? A. Bone marrow transplantation B. Erythropoietin C. High-dose dexamethasone D. Iron infusions E. Splenectomy

E. Splenectomy This is a difficult question. Essentially, this man has CLL and he also has auto-immune phenomenon despite fludarabine. Nowadays, single agent fludarabine as a treatment would be uncommon for patients with CLL. It sounds like the patient has ongoing autoimmune haemolytic anaemia despite corticosteroids. In this situation, the anti-CD20 antibody rituximab may be very helpful. Splenectomy is not without risks, and usually patients with CLL are elderly. It remains, however, an effective treatment for autoimmune haemolytic anaemia that has not responded to steroids

A 30 -year-old patient presents with an episode of jaundice associated with feeling tired. On examination there is anaemia, splenomegaly and jaundice. He has had several such episodes in the past. The blood smear shows reticulocytosis and the red cells demonstrate increased osmotic fragility. Given the probable diagnosis, which one of the following is likely to be the most successful treatment? A. Blood transfusion B. Bone marrow transplantation C. Chemotherapy D. Radiotherapy E. Splenectomy ​

E. Splenectomy This man is likely to have the inherited condition hereditary spherocytosis. Red cells are more spherical and, because of this, they are unable to pass through the splenic microcirculation where they are destroyed. Splenectomy therefore is the principal form of treatment, although this should not be performed unless clinically indicated, eg significant, recurrent anaemia or gall stones. This is because of the risk of post-splenectomy sepsis, particularly in early childhood.

A man has had multiple blood transfusions for sideroblastic anaemia. This time, 15 minutes into blood transfusion, he complained of severe breathlessness. Chest X-ray showed diffuse, bilateral infiltrates. What is the diagnosis? A. ABO incompatibility reaction B. Acute anaphylaxis C. Cardiac failure D. Febrile, non-haemolytic blood reaction E. Transfusion-related lung injury

E. Transfusion-related lung injury (TRALI) This man presents with severe breathlessness as a result of a blood transfusion, and the chest X-ray shows bilateral, diffuse infiltrates consistent with non-cardiogenic pulmonary oedema. This often occurs in patients who have had multiple previous transfusions. TRALI is related to leukocyte antibodies which are present in the plasma of the blood donor. Standard leukocyte depletion of all blood donations in the UK has reduced this type of transfusion reaction. It is most common with transfusions containing relatively more plasma, eg platelets and FFP, but can be seen with red cell transfusions also. ​

A 38-year-old man is evaluated for a 3-month history of abdominal bloating and increased frequency of defecation. He has four to six loose bowel movements a day, including nocturnal bowel movements, without abdominal pain. Medical history is significant for a Roux-en-Y gastric bypass procedure 2 years ago for severe obesity; his weight loss stabilized 1 year ago, although he has lost an additional 4.5 kg (10 lb.) in the past few months. Medications are daily iron and vitamin B12 supplements and a multivitamin. On physical examination, vital signs are normal. Cardiopulmonary examination is normal. The abdomen is not distended or tender, and bowel sounds are normal. Laboratory studies are normal. Stool cultures are negative. Upper endoscopy and colonoscopy are unremarkable except for the expected altered anatomy. Which of the following is the most likely diagnosis? Bile salt malabsorption Celiac disease Irritable bowel syndrome Small intestinal bacterial overgrowth ​

Educational Objective: Diagnose small intestinal bacterial overgrowth as a complication of bariatric surgery. Patients who undergo bariatric surgery are at risk for specific early and late complications of the procedure; small intestinal bacterial overgrowth is a late complication that can manifest with bloating, diarrhea, and features of malabsorption. Having segments of small intestine excluded from the usual stream of gastric acid, bile, and proteolytic enzymes, which all act to decrease excess bacterial growth in the small intestine, is a risk factor for bacterial overgrowth. The diarrhea in bacterial overgrowth can be from deconjugation of bile salts from the intestinal bacteria, leading to fat malabsorption, as well as from decreased disaccharidase levels, leading to carbohydrate malabsorption. Culture of the normally sterile small intestine or carbohydrate (lactulose, glucose, or d-xylose) breath testing can be done to substantiate the diagnosis; alternatively in some patients an empiric trial of antibiotic therapy can be considered. KEY POINT: Small intestinal bacterial overgrowth is a late complication of bariatric surgery and should be considered in patients presenting with diarrhea, bloating, and features of malabsorption after such surgery.

A 52-year-old woman comes to the clinic with gradual-onset weakness of the leg muscles. For several months, she has had Increasing difficulty climbing stairs and rising from a chair. The patient initially attributed her symptoms to being "out of shape." She has no problem chewing food, changes in bowel or bladder function, morning stiffness, or joint pain. Medications include lisinopril and amlodipine for hypertension. She does not use tobacco, alcohol, or illicit drugs. Vital signs are normal. Examination shows 4/5 strength in the thigh muscles of both legs, and distal muscle strength is 5/5. Repeated muscle contractions do not alter the weakness. Mild tenderness is noted in the proximal thigh muscles. Deep tendon reflexes and sensory examination are normal, and Babinski sign is absent. Which of the following would establish a diagnosis in this patient? A. Acetylcholine receptor autoantibody test B. Discontinuation of lisinopril C. Electromyogram D. Erythrocyte sedimentation rate E. MRI of the spine F. Muscle biopsy G. Serum electrolyte assay

F. Muscle biopsy Polymyositis Is an inflammatory muscle disease that presents with slowly progressive proximal weakness of the lower extremities (e.g., difficulty negotiating stairs or rising from a seated position). Proximal arm weakness usually follows, leading to difficulty working with the arms overhead. Patients may develop dysphagia due to involvement of the striated muscles of the upper pharynx. Mild pain or tenderness can develop due to inflammation: however, significant pain or a lack of weakness should prompt consideration of other diagnoses. Polymyositis is similar to dermatomyositis but without skin findings. Muscle enzymes (e.g., creatine kinase) are invariably elevated, and autoantibodies (e.g., antinuclear antibodies, anti-Jo-1 antibodies) are present in most cases. A muscle biopsy is the most definitive diagnostic test and shows a mononuclear infiltrate surrounding necrotic and regenerating muscle fibers. Educational objective: Polymyositis is characterized by chronic proximal weakness. Muscle enzymes (e.g., creatine kinase) are elevated, and autoantibodies (e.g., antinuclear antibodies, anti-Jo-1 antibodies) are present in most cases. Muscle biopsy is the most definitive test and shows a mononuclear Infiltrate surrounding necrotic and regenerating muscle fibers.

A 29-year-old woman comes to the office due to a 6 month history of numbness and pain in her upper arms. The pain is worse on the left side and is exacerbated by lifting or other activity. Associated symptoms include fatigue, fleeting joint pains, and a 5-kg (11-lb) weight loss. The patient's medical history is unremarkable, and she does not use tobacco or illicit drugs. She emigrated from Vietnam 5 years ago and works in a furniture repair shop. The patient's blood pressure is 140/90 mm Hg in the right arm and 90/55 mm Hg in the left arm, and her pulse is 78/min and regular. Conjunctival and oral mucosa are moist and without lesions. Cardiopulmonary examination shows clear lung fields and no heart murmurs: however, a bruit is heard in the right supraclavicular fossa, and the left radial and brachial pulses are decreased. Abdominal examination is unremarkable. No skin lesions are present. Complete blood count, electrolytes, and renal function are normal. Erythrocyte sedimentation rate is 40 mm/hr. Which of the following is the most likely diagnosis for this patient? A. Aortic dissection B. Coarctation of the aorta C. Giant ceil arteritis D. Kawasaki disease E. Polymyositis F. Takayasu arteritis G. Thromboangiitis obliterans

F. Takayasu arteritis This patient with exertional arm pain (likely claudication), systemic symptoms, and pulse deficits has typical features of Takayasu arteritis, a chronic large artery vasculitis that predominantly affects Asian women age <40. It primarily involves the aorta and its branches and is characterized by mononuclear infiltrates and granulomatous inflammation of the vascular media, leading to arterial wall thickening with aneurysmal dilation or narrowing and occlusion. Initial symptoms are nonspecific (e.g., fever, arthralgias, weight loss). As the disease progresses, vascular involvement with arterio-occlusive manifestations (e.g., claudication, distal ulcers) may develop, particularly in the upper extremities. Examination findings include blood pressure discrepancies, pulse deficits, and bruits. Patients commonly have anemia and elevated inflammatory markers (e.g., erythrocyte sedimentation rate. C-reactive protein). Chest x-ray can reveal aortic dilation and a widened mediastinum, and CT and MR1 may reveal thickening of large artery walls and luminal narrowing. Initial treatment includes systemic glucocorticoids. Educational objective: Takayasu arteritis is a large artery vasculitis that is most common in young Asian women. Initial symptoms include fever, arthralgias, and weight loss. Later features include arterio-occlusive symptoms, blood pressure discrepancies, and pulse deficits. CT and MRI can reveal aneurysm formation or luminal narrowing. Treatment includes systemic glucocorticoids.


संबंधित स्टडी सेट्स

Chpt 3 Review Questions - Computer Systems Security Part II

View Set